Sie sind auf Seite 1von 453

I.A.

Maron

Problems
in

Calculus
of

One Variabl

H. A.

MAPOH

HHOOEPEHUHAJlbHOE
W MHTErPAJlbHOE

HCMHCJ1EHME
B

nPMMEPAX H 3AiXAHAX
<DyHKUHH

OJ^HOfl

nEPEMEHHOM

H3/lATEJlbCTBO H AY K A

MOCKBA

I.

A.

MA RON

Problems
in

Calculus

of

One

Variable

(With elements of theory)

TRANSLATED

FROM THE RUSSIAN


by

LEONID LEVANT

MIR PUBLISHERS

MOSCOW

UDC 517=20

First published

1973

Revised from the 1970 Russian edition

Ha
0223-200
041(01)-73

aneAUucKOM R3biKe

Contents

From
Chapter

the

Author

Introduction

I.

1.1.

1.2.

to

Mathematical Analysis

Function.

Domain

of

Definition

1.4.

Inverse Functions

1.5.

Graphical Representation

1.6.

Number

1.7.

Evaluation

1.8.

Testing Sequences

1.9.

The Limit

28

of

Limits
for

1.10.

Calculation of Limits

1.11.

Infinitesimal and

1.12.

of

of

Functions
a

30

Sequence

41

Sequences

48

Convergence

50

Function

tion and

22

Sequences. Limit of
of

of

Infinite

55

Functions

60

Functions. Their

Defini-

Comparison

Equivalent

68
Infinitesimals. Application

to

Finding

Limits

71

1.13.

One-Sided Limits

1.14.

Continuity

and

Their

1.15.

of a

75

Function. Points

of

Discontinuity
77

Classification

Arithmetical Operations

on Continuous Functions.

of a Composite Function
The Properties of a Function Continuous on

Continuity

II.

84
a Clo-

sed Interval. Continuity of an Inverse Function

87

Problems

91

Chapter

1.16.

11

15

Investigation of Functions

1.3.

11

Number

Real Numbers. The Absolute Value of a Real

1.17.

Additional

Differentiation

Definition of

2.2.

Differentiation of

2.3. Successive

Leibniz Formula

93

Functions

of

2.1.

the

98

Derivative
Explicit

Differentiation

Functions
of

Explicit

100

Functions.
107

Contents

and

Implicit

2.4.

Differentiation of Inverse,

cally

Represented

2.5.

Applications of the Derivative

2.6.

The

Parametri-

Functions

Ill

115

Differential of a Function. Application to App-

roximate Computations
2.7.

Chapter

III.

122

Additional Problems

Application

126

Calculus

Differential

of

Investigation

to

of

Functions

131

....

3.1.

Basic Theorems on Differentiable Functions

3.2.

Evaluation

3.3.

Taylor's Formula. Application to Approximate Cal-

131

Indeterminate Forms. L'Hospital's Rule

of

culations
3.4.

143

Application

Taylor's Formula

of

Evaluation

to

of

Limits

147

3.5.

Testing a Function

3.6.

Maxima and Minima

Finding
Function

for

Monotonicity

of a

152

Values

Least

the

of

159

3.8.

Solving Problems in Geometry and

3.9.

Convexity

and

148

Function

and

Greatest

the

3.7.

Concavity

of

Physics

Curve.

....

Points

162

of

166

Inflection
3.10.

Asymptotes

3.11.

General Plan

170

Investigating Functions and Sket-

for

ching Graphs

174

3.12.

Approximate Solution

dental

Equations

3.13.

138

of

Algebraic

and

Transcen183

Problems

Additional

190

Chapter IV. Indefinite Integrals. Basic Methods


and

Integration

4.1.

Direct

4.2.

Integration

by

of

....

190

Expansion

195

Integration

Method

the

of

Substitution

199

4.3.

Integration by Parts

202

4.4.

Reduction Formulas

211

Chapter V. Basic Classes

of

Integrable

Functions

214

Rational Functions

5.1.

Integration

5.2.

Integration of Certain

5.3.

of

214

Expressions

219

Euler's Substitutions

222

5.4.

Other Methods

Integrating Irrational Expressions

224

5.5.

Integration of

Aid

of

Binomial

228

Differential

Trigonometric and Hyperbolic Functions


Integration of Certain Irrational Functions with the

230

....

237

5.6. Integration
5.7.

of

Irrational

of

Trigonometric

or

Hyperbolic

Substitutions

Contents

5.8.

Integration of Other Transcendental Functions

5.9.

Methods

of

Integration (List of Basic

Forms

of

242

tegrals)

Chapter VI. The Definite Integral


6.1.

Statement

niz

of

247
the

Problem. The

Lower and Upper

Sums

Integral
6.2.

240

In-

247

Evaluating Definite Integrals

by

Newton-Leib-

the

Formula

256

Estimating an Integral. The Definite Integral as a


Function of Its Limits

6.3.

6.4.

Changing the Variable

6.5.

Simplification of Integrals Based

of

Symmetry

of

in a

Definite

6.7.

Approximating Definite Integrals


Additional Problems

7.1.
finite

7.2.
7.3.
7.4.

the

Computing

7.6.
7.7.

Formulas

....

307

310

Integral

Definite

Sums with

the Limits of

the Aid of De-

310

Integrals

Finding Average Values

of a

312

Function

Computing Areas in Rectangular Coordinates ...


Computing Areas with Parametrically Represented

7.10.

317
327

The Area of a Curvilinear Sector in Polar Coordinates


Computing the Volume of a Solid
The Arc Length of a Plane Curve in Rectangular

331

336

345

The Arc Length

of a

Curve Represented

Parametri-

348

cally
7.9.

294
301

Coordinates
7.8.

275

Properties

the

Boundaries
7.5.

262

288

by Parts. Reduction

Integration

Chapter VII. Applications of

Integrands

6.6.

6.8.

on

Integral

The Arc Length


Area

of

Surface

7.11. Geometrical

of

Curve in Polar
Revolution

of

Applications

Coordinates

351

354
Integral

360

Computing Pressure, Work and Other Physical


Quantities by the Definite Integrals
7.13. Computing Static Moments and Moments of Inertia.

367

....

372

cf

the Definite

7.12.

Determining Coordinates
7.14.

Additional

of the

Centre of Gravity

Problems

Chapter VIII. Improper Integrals


8.1.
8.2.

8.3.

Improper Integrals with Infinite Limits


Improper Integrals of Unbounded Functions ....
Geometric and Physical Applications of Improper

Integrals
8.4.

Answers and Hints

Additional Problems

383

387
387
397

409
415
418

In fond memory

my

of

From

parents

the Author

This textbook on mathematical analysis

is

based on

many

experience of lecturing at a higher technical college. Its aim


train

the

as

done

is

Much

students

active

in

years'
is

to

to mathematical exercises,

approach

a seminar.

at

attention

is

to problems improving the theoretical

given

background. Therefore standard computational exercises are supple-

mented by examples and problems explaining the theory, promoting its deeper understanding and stimulating precise mathematical

thinking.

Some counter-examples explaining

tain conditions

in

the need for cer-

formulation of basic theorems are also

the

in-

cluded.

The book
opens with
cipal

definitions,

solution

of

along

designed

is

a concise

the

theoretical

following

lines.

Each section

introduction containing the prin-

theorems and formulas. Then follows

detailed

one or more typical problems. Finally, problems with-

which are similar to those solved but


Some of them are provided with hints.
Each chapter (except Chap. IV and V) closes with a separate
section of supplementary problems and questions aimed at reviewing
and extending the material of the chapter. These sections should
prove of interest to the inquiring student, and possibly also to
lecturers in selecting material for class work or seminars.
The full solutions developed in the text pursue two aims: (1)
out

solution

are

given,

contain certain peculiarities.

to

provide

lecturers

with

students to the textbook

time-saver,

most

of

the

since

they can refer the

standard exercises of a

character and concentrate mainly on the solution


more sophisticated problems, thus gaining time for more rewar-

computational
of

for

From

10

ding work;

on

their

and

own

(2)

to

the

Author

meet the needs

of those

who

are working

or following correspondence courses, providing a sub-

stitute for the oral explanations given to full-time students.

The student
vely, that

is

will

find

to say,

if

the

book most

useful

if

he uses

it

acti-

he studies the relevant theoretical material

on to the worked-out solutions, and finally


reinforces the newly-acquired knowledge by solving the problems
given for independent work. The best results will be obtained
when the student, having mastered the theoretical part, immediately attacks the unsolved problems without referring to the text
carefully

before going

solutions unless in difficulty.

Isaac

Mar on

Chapter
INTRODUCTION
TO MATHEMATICAL ANALYSIS

1.1. Real Numbers.


The Absolute Value of a Real Number
Any decimal
a real

terminating or nonterminating,

fraction,

is

called

number.

Periodic
rational

decimal

fractions

number may be written

rational

called

are

in the

form

numbers.

of a ratio,

Every
of

two

p and q and vice versa.


Nonperiodic decimal fractions are called irrational numbers.
If
X is a certain set of real numbers, then the notation
means that the number x belongs to X, and the notation
means that the number x does not belong to X.
A set of real numbers x satisfying the inequalities a < x < b
where a and b are fixed numbers, is called an open interval (a, b).
A set of real numbers x satisfying the inequalities
is
integers

xX
x^X
y

a^x^b

called a closed interval


the inequalities
terval

[a,

b)

or

[a,

b].

a^x<b

or

A set of real numbers


a<x^.b, is called a

Open, closed, and

(a, b].

satisfying
half-open inhalf -open intervals are
x,

covered by a single term interval.


Any real number may be depicted as a certain point on the
coordinate axis which is called a proper point. We may also introoo infinitely
duce two more, so-called improper points, + and
removed from the origin of coordinates in the positive and nega-

<

respectively. By definition, the inequalities


oo
oo hold true for any real number x.
interval {a
e, a
s) is called the e-neighbourhood of the

tive directions,

<x<+

The
number a.
The set
of

of

real

numbers x >
+00.
numbers x <

the improper point

The
of

set

of

real

M
M

is

M -neighbourhood

called the

is

called the

M- neighbourhood

improper point -00.

The absolute value

of

number x (denoted

that satisfies the conditions


\x\

\x\

= x
=x

<

if

if

x^O.

0;

\x\)

is

number

Ch.

12

The properties

Introduction to Mathematical Analysis

absolute values are:

of

means

that

a^x^a;

that

x^a

a;
|

(2) the inequality

^a

|x|>a means

the inequality

(1)

(4)

xy\K \x\+\y\;
x y\>\\x\ \y\\;

(5)

xy\=\x\\y\

(3)

or

jc<- a;

x
(6)

1.1.1.

Prove that the number

0.1010010001... 1000... 01...


is

irrational.

To prove this, it is necessary to ascertain that the


decimal fraction is not a periodic one. Indeed, there are n
between the nth and (n+\)th unities, which cannot occur

Solution,

given
zeros

in a periodic fraction.

1.1.2.

mal

Prove

places

1.1.3.

that

numbered

Prove

any number, with zeros standing in all deci10" and only in these places, is irrational.
the

that

number a and

tional

sum

an

of,

difference between, a ra-

or the

number

irrational

is

an

irrational

number.

+ =

Solution. Consider the sum of a and p. Suppose a


y is a
P
a is also a rational number, since
rational number, then P
Y
it
is
the difference between two rational numbers, which contradicts the condition. Hence, the supposition is wrong and the number

a+

is

1.1.4.

rational

irrational.

Prove

product a|J and the quotient a/[i of a


and an irrational number P is an irrational

the

that

number a ^=

number.
1.1.5.
is

(a)

a rational

Find all rational values


number.

(a) Suppose x and


the difference y
x
express x through q

Solution,
bers.

us

now

Vx

x2

direct

check

y=\/ x* +

are rational

q,

+ x + 3 = q* + 2qx + x\
it

is

\-2q

easy to ascertain that

q^

+3

num-

number. Let

also a rational

+ x+3 = q + x,
x

By

is

x=\/ x + x-{-3 x
r

which

y=\/~x <iJr x Jr 3

=q

Then

of x at

/^

LI. Real Numbers. The Absolute Value

Prove the reverse, namely, y


if

where q

Real Number

of

= ]/ x + x + 3
2

is

rational

any rational number not equal

is

13

to

number
1

/2 .

Indeed,

g4-2? + 7g*-6? + 9
(1-2?) 2

K
The

latter expression

is

_ ^_^ + 3

(l-2^)a

~|1-2<7|

Solution.
rational.

the

since

_1_\

q=*=

2J'

/2 .

an irrational number.

is

+ V2

Assume the contrary,


Then the number

also rational,

Whence

rational at any rational q not equal to

Prove that the sum V~3

1.1.6.

is

\/~2

Prove that

(b)

is

(^-^ + 3)2
K

is

it

i.e.

is

an irrational number.

number [/3 + |/*2

that the

the quotient of two rational

numbers.

number

K2 = l[(K3 + K2)-(/3-K2)l
is

which contradicts the irrational nature of the number


Problem 1.1.5). Hence, the supposition is wrong, and the

rational,

V2

(see

number

K3 + 1/2

irrational.

1.1.7. Prove that for every positive rational number r satisfying


the condition r 2
2 one can always find a larger rational number

<
+ h(h>0) for which
Solution. We may
<r + 2r/z + That
2

i.e.

fc(fc>0)

for

(b)
(c)

(d)

h
sufficient to put

is

+ <
+ 2rh + h = 2,

and
r

(r

/i)

>

for every positive rational number s satisfying


2 one can always find a smaller rational number

which

(s

k) >
2

2.

Solve the following inequalities:

1.1.9.

3 <
x + 2x 8<0;
\x 7x+ 12 >x 7x+
\

<

Prove that

1.1.8.

(a)

it

Then h 2

)/(2r+\).

the condition
s

why

2.

h<\.

assume

is

/i.

= (2 r

ft

+ hf <

(r

2x

1;

(x-2) 2 >4;
2

12.

Solution,
inequalities

(a)

The inequality

|2#

3|<1

< 2x 3 <
1

1,

whence
2

<

2*

<

and

<*<

2.

is

eqivalent

to

the

Ch. I

14

(d)

x2

Introduction to Mathematical Analysis

The given inequality


0, whence 3

7x+ 12 <

Find

1.1.10.

valid for those values of x at which

is

<x<

whether

out

4.

the

equations

following

have any

solutions:

= x + Hence, there are no

+ 5, whence x== 5/2. This


Hence, there are no solutions.
have x = x
x = x 5, whence # = 5/2, which contradicts

(a) \x\=x
5?
b]
(b) |x|=x
Solution, (a)
we have x
solutions. At x
we have
x=-x
value satisfies the initial equation.

x>0

At

(b)

Atx^O
<
we

5.

5.

<

we have
x
our supposition (#<0). Thus, the equation has no solution.
At

Determine the values

1.1.11.

x satisfying the following equa-

of

lities:

x\
(a)

x+

5*

(b)

+ 6| = (x 5*+6).
2

Determine the values

1.1.12.

of

x satisfying the following equ-

alities:

+ 4x + 9) + (2x 3) - x + 4x + 9 +
(b) |(x 4) (x + 2)| = |x 4 |x + 2|.
Solution, (a) The equality |a + 6| = |a| +
(x

(a)

2x

|;

|fe|

only

is

valid

if

x2

+ 4*+9:=(.x:+2) + 5 >
2

any values of x the equality is satisfied at those values


which 2x
3>0, i.e. at *>3/2.
\b\ holds true if and only
(b) The equality \a
6| = |a|
and b have the same sign and

at

at

In

our

case

equality

the

which
xx

will

x2
1.1.13. Solve the
|

3*

(b) \x

hold true

4>jc +
2

Whence

(a)

and

both summands have the same sign. Since

if

for the values of

of

if

x at

2.
__

2>1;

\x\^V$-

inequalities:

5| + 3 >
|5x|.
5x| >
|

2JC

|jc

0;

1.1.14. Find the roots of the following equations.


sin x
sin x
1;

(a)

(b)

x2

Solution,
of

x at

2\x\ 3 = 0.
(a)

This equation will hold true only for those values

which sin*<0,

that

is

why we may

rewrite

it

in

the

Domain

Function.

1.2.

15

of Definition

following way:

sinjc = sinA;+
x = nk (l) n/6 (k = 0,

1,

whence

or sin

a:

1, 2,

..).

This equation can be solved in a regular

(b)

x^O

the cases
and
writing it in the form

x^.0.
|a:|

Substituting y for

whence

#,

not

in.

fit

= 3,

y2

1.

We may

/a ;

way by considering

also solve this equation

2|jc| 3 =

re-

0.

we obtain

|,

Since y

= \x\^0,

the value y 2

does

Hence

y=\x\ = 3,

= 3,

i.e.

x 2 ^=3.

Function.

1.2.

Domain of Definition

The independent variable x

is

defined by a set

of its values.

independent variable x(tX there corresponds one definite value of another variable y then t) is called
the function of x with a domain of definition (or domain) X or,
in functional notation, y y(x), or y
f(x), or y = q>(x), and so
forth. The set of values of the function y(x) is called the range
of the given function.
In particular, the functions defined by the set of natural numbers 1, 2, 3,
are called numerical sequences. They are written
in the following way: x Xi x 2y
... or {x n \.
to each value of the

If

1.2.1.

fix

),

Given the function

/ (x)

=(*+

\)/(x

1).

Find f(2x), 2f(x),

[f(x))>.

Solution.

f(20=i;

2f

W _2};

/<>-!; tfW]'-(f{)'.
Given the function

1.2.2.

(a)

Show

that at x lt x 2

( 1,

1)

the following identity holds true:

/(*>+/(*)=/ (t-).

Ch.

16

Introduction to Mathematical Analysis

I.

Solution. At

f(x l)+

On

1,

M=

log

(1

x)/(l + x) >

+ log

and

= log

;g

;;

hence
.

i,

the other hand,

+ ^2

^1

*l

we have

1)

+ *2

\ __
j

\l-jrX1X2J

Qg

~f-

-j-

XXX 2

X X

Qg

^2

-^l ~l~

-\~

x i x 2~i~~ x i ~h x 2

XiX 2

(1 *i) (\x 2
(!+*!) (l+x 2

log

which coincides with the right-hand member


(b) Given the function / (x) = (a x + a' x )/2
f{x
1.2.3. Given the
f(a+\); f(a)+\.

1.2.4.

= (x+

f (x)

=x
3

Given the function f(x)


/(>-/(">

1.2.5.

and

a)

l)/(x

(1).

Show

that

/( 1);

Find

1).

Find

1.

^*.

\<Cx<

3-^1,
(

x/(jc

Find /(-l),
Solution.

Hence
^

/ (ji/2)

The points

/(2ji/3),

f(ji/2),

The point x =

points

= n/2,

1)

= 2n/3

-J

/(6).

within

lies

= 3x

2),

/(4)

0,

0<*<jx,
jt<*<6.

tan(*/2),

>

=
1

belong

the

interval

to

the

interval

- tan (ji/4) / (2n/3) = tan (jt/3) = 4, x = 6 belong to the interval


1

4)

The function

f (x)

is

36-2

defined over

= 17

f(x)-{

if

5,

if

2*

if

jc<2,

2<*<3,
> 3.

[ji,

1,

0).

[0,

ji).

6].

Hence

'

the whole

by the following law:


2x 3 + 1.
l/(*-2),

\^3.

= 16^2 = T

2.

jc

1.2.6.

> 0).

Given the function

The
Hence

(a

+ y) + f(x-y) = 2f(x)f{y).

function

expression

of

number

scale

Find:

f(V2)

Function.

1.2.

/(|/8),

Domain

of Definition

17

(/log, 1024).

AB=2

ABCD

with side
the square
a straight line
is drawn
perpendicularly to AC. Denoting the distance from
the vertex A to the line
as x, express through x the area S
cut off from the square by the straight
of the triangle
In

1.2.7.

MN

MN

AMN

line

MN.

and

at

Find this area

=2

If

S^

x> V2

S(x)

= ]^2/2

AC = 2 V~2
If*<K"2,

that

0<*<2l/T.
S (X)

at

1).

Note

Solution.

hence
then

(Fig.

AMN

2
-

then

=4 (2 K2~

-^)

2
jc

+4* j/y

4.

Thus,

Fig.

S(*)

0<x</2",

V2< x^2\/2.

-x \\x\f2 4,
2

= {V~2 j2) = V Since


S (2) = 4 + 8 V~2 4 = 8 (K~2~ 1).

<

S (V2/2)

Since

|/T/2

1.2.8.

Bring the number a, which

K2~

equal

is

to the

nth

>

2~,

J/

decimal

expansion of V~2 into a decimal fraction, into corplace


respondence with each natural number n. This gives us a certain
in the

an

function

Solution.

Calculate q)(l),

tp (ri).

Extracting

cp

(2),

q)(3),

we

square root,

find

q)(4).

\f2 =1.4142...

Hence

= 4;

q>(l)

Solution. f(x)
9
14
(a')

1.2.10.

known

= 4;

q>(3)

cp

(4)

= 2.

f (x) = 49/x + x at the points for which 7/x + x=3.


= 49/x + x = (7/x + x) \4,but 7/x + x = 3, hence
2

Calculate

1.2.9.

cp(2)-l;
2

= 5.

Find a

that /(0)
Solution.

function of the form

= 5;

=
1)

10;

/ (1)

/ (x)

= ax'

= 6.

= 5=a'0 + b-0 + c,
f(l)= I0^a b + c,
/(l)=6 = a + 6 + c.
2

f(0)

-\-

bx -f

c,

if

it

is

Ch.

18

Introduction to Mathematical Analysis

I.

Determine the coefficients a, b c from the above system.


2
5.
a = 3; b=r.2; c
2x
5\ hence / (x) = 3x
y

1.2.11.

Find

/(0)=15;
1.2.12.

= a + bc*
f(4) = 90.

= 30;

f(2)

Find

have:

function of the form

(c>0),

f(x)
if

We

cp [xp (a:)]

and

[cp (jc)]

=x

cp (jc)

if

and ^(x)

= 2x

Solution.

= 2*

i|)[<p(*)]

1.2.13.

<

= 2*".

Given the function


5x2+1

f(x)~ 2x
Find f(3x);
1.2.14.

(*

3f (x);

);

[f

'

(x)]\

Let

< x<

at

(3*
4

3x

0<*<1,
l<x<3.

at

Find f(2), /(0), /(0.5), /(-0.5),

0,

at

/(3).

for an exponential function y


a x (a>0;
the argument x
x n (fl=l, 2, ...) form an
arithmetic progression, then the corresponding values of the function y n
aXn (n= 1, 2, ...) form a geometric progression.

1.2.15.

Prove that

a=l) the values

if

of

1.2.16.

1.2.17.

+ 6, q)(#) = 5*. Solve

Find /(*)

if

/(*+l)
1.2.18.

=*

the equation

+2
3jc

/ (x)

=
|

q)

(x)

Evaluate the functions


/

for the points at

1.2.19. f (x)

X)

= x + l/x
2

which

l/x

and

(p

(x)

= x* +

/*

+ x = 5.

= x 2; solve the equation


\f(x) + <f(x)\ = \f(x)\ + \<t(x)\.

= x+l;

y(x)

A rectangle with altitude x is inscribed in a triangle


with the base b and altitude h. Express the perimeter P and
area 5 of the rectangle as functions of x.
1.2.20.

ABC

Find the domains

1.2.21.
(a) /

Function.

1.2.

X)

(d) f(x)

Domain

of Definition

of definition of the

19

following functions:

= \/It=l+V6=i;

= Vs\nx

log^^

(e)

= \og x 5\
5x-\-6
/W = log- -4x + 6

(0 f(x)

x'2

(g)

(h) /(*)

(j)

log

= arc sin

(4

x)\

/W = logcosx;

(k)/(x)^arccos

3
4

+ 2 sin;c

(i)

Solution,
consists

values.

of

To

The domain of definition of the given function


those values of x at which both items take on real
ensure this the following two conditions must be satis(a)

fied:
1

By solving

the

inequalities

>0,

x^O.
we obtain x^l;

x^.6.

domain of definition of the function will be the segment


(e) The function is defined for the values of x for which

the

Hence,
[1,6].

log^^->0.
This inequality will be satisfied

-5^1>1,

or x 2

if

5x + 4<0.

we find 1^a:^4. Thus, the segthe domain of definition of the function.


(f) The function is defined for all positive x different from unity,
which means that the domain of definition of the function consists
oo).
of the intervals (0, 1) and (1,
Solving the latter inequality,

ment

[1,4]

is

20

Ch.

(k)

Introduction to Mathematical Analysis

The function

defined for the values of x for which

is

-1
Since 4+2 sin x
the inequality

>0

^ 4+2
A

any

at

sin x

<
^

1.

the problem

x,

+ 2 sin x

is

reduced to solving

Whence

3<4+-2sinx,
By

(1)

|*|

The

function

*>0,

a:

(g)

...).

for
the values of x for which
This inequality is satisfied at x
0.

<

( oo,

0).

of definition of the following functions:

f (x)

= log|4-x
=V

&

2
|;

cos (sin x) +- arc sin

Find the ranges


( )

1, 2,

= j/arc sin (log x)


= log log log x;
= l + 2 a-in, + _J_.
/ W

(d) f (x)
(e)

= 0,

(k

2fcn;

defined in the interval

/ (x)

(b) / (x)
(C)

is

> x.

Find the domains

1.2.22.
(a)

Hence, the function

defined

is

whence

we obtain

solving the latter inequality

_ + 2:ri<x<^ +

sinx>- 7

i.e.

of the following

= 2- cos 3*

y=j^

~
x

functions:

Solution, (a) For the function


inequality must be satisfied

/(x)

arc sin (log 2 x)

to be

defined the following

^ 0,

whence
log 2 x^ 1 and l^x^2.
function log 2 log 3 log 4 x is defined for
(b) The
whence log 4 x>l and x>4. Hence, the domain

log 3 log 4
of

x>0,

definition

is

4<x< +

oo.
the interval
(c) The given function is defined
satisfied simultaneously:

x=^=0;

but the inequalities


is why the function

< x<
1

l^x^l
is

if

the following inequalities are

and

x>2,

x>2

are incompatible, that


and
not defined for any value of x.

(e)

Domain

Function.

1.2.

of Definition

The following inequalities must be

x)^0

cos (sin

and

21

satisfied

x2

2x

<

simultaneously:

1.

The first inequality is satisfied for all values of x, the second,


|*|=1. Hence, the domain of definition of the given function
consists only of two points x=\.
for

(f)

We

have

cos 3x

Since
-1

< cos3x<

whence, taking into account that y

y^2y
(g)

or

Solving with respect to x


x

The range

>

0,

1,

we obtain

1.

we obtain

'

Ty

of the function y will be


1

-<

we have

1,

4*/

determined from the relation

>0.

Whence

1.2.23. Solve the equation

arc

Solution.

tanVx(x

+ 1)+

arc sin l^x 2

+ x+

jx/2.

Let us investigate the domain of definition of the funcThis function will be defined

tion on the left side of the equation.


for

x2

whence x
at

+ *>0, 0< x + x+ <


2

1,

-\-x = 0.

member

Thus, the left


x =0 and # 2

1.

of

By

the equation attains real values


check we ascertain that

a direct

only
they

are the roots of the given equation.

This problem shows that a study of domains of definition of a


function facilitates the solution of equations, inequalities, etc.
1.2.24. Find the domains of definition of the following functions:
,

(a)

y=

(b)

2x 3
r

= \og

sin (a: 3)

+ j/l6 x\

22

Ch.

(c)

I.

Introduction to Mathematical Analysis

= V 3 x + arc

cos^

The function f (x) is defined on the interval [0,


domains of definition of the following functions:

1.2.25.

are the

/(3r>);

(a)

f(x-5);

(b)

(c)

What

1].

f(tan*)?

Solution. The given functions are functions of functions, or superpositions of functions, i. e. composite functions.
a) Let us introduce an intermediate
argument u
3x 2 Then the
2
2
function f(3x )j=f(u) is defined if
i.e.
<l,

whence

1/]/3<*<;

Similarly:

(c)

0<w<l,

0<3x

l/|/3.

O^tanx^l,

fcrt<

whence
(k

= 0, +1, 2,

...).

The function f (x) is defined on the interval


domains of definition of the functions

1.2.26.

are the

(a)

1.3.

(sin*);

(b)

f(2x

[0, 1].

What

+ 3)?

Investigation of Functions

A function f (x) defined on the set X is said to be non-decreasing


on this set (respectively, increasing, non-increasing, decreasing), if
for any numbers x 19 x 2 X, x <x 2y the inequality f (xj^ f (x 2 )
(respectively,

fW<f(4

The function

f (x)

f(*i)>f(* 2 ). /(*i)>/(* 2 ))

satisfied.

is

said to be monotonic on the set


if
it
possesses one of the four indicated properties. The function f (x) is
said to be bounded above (or below) on the set
there exists a
if
(or m) such that
number
for all
(or m^Lf(x)
for all xX). The function f (x) is said to be bounded on the set
is

X
xX

f(x)^M

if

it

is

bounded above and below.

The function

T>0

f (x)

is

called

periodic

f (x)

if

there

exists

number

x belonging to the domain


of definition of the function (together with any point x the point
x-\-T must belong to the domain of definition). The least number T
possessing this property (if such a number exists) is called the
period of the, function f (x). The function f (x) takes on the maximum value at the point x
and
if f(x )^f(x) for all xX,
Q
the minimum value if f(x )^f(x) for all xX. A function f (x)
defined on a set X which is symmetric with respect to the origin
of coordinates is called even if /(
x) = f(x), and odd if f ( x) =
such that

(x

-f-

T)

for all

$X

-/(*).
In analysing the behaviour of a function

termine the following:

it

is

advisable

to

de-

1.

The domain

2.

Is

Investigation of Functions

1.3.

23

of definition of the function.


the function even, odd, periodic?
3. The zeros of the function.
the zeros.
4. The sign of the function in the intervals between
5. Is the function bounded and what are its minimum and maximum values?
The above items do not exhaust the analysis of a function, and
later on their scope will be increased.
1.3.1. Find the intervals of increase and decrease of the funcax 2 -f- bx
c
and its minimum and maximum values.
tion / (x)
Solution. Isolating a perfect square from the square trinomial,

we have

If

> 0,

then the function f(x) will increase at those values of x


the

satisfying

the function

inequality

when x + b/(2a)

decrease

f (x)

<

+ b/(2a) > 0,
i.e.

0,

i.

x<

at

decreases in the interval

reases in the interval

b/(2a),

the function / (x) assumes the

+oo).

minimum

e.

at

>

b/(2a).

oo,

Obviously, at
value

and

b/(2a),

Thus,
)

if

>

and

0,

inc-

x = b/(2a)

>

the function has no maximum value.


At a
the function f (x) will increase in the
Similarly, at a

<

oo, ^ and decrease in


takes
x = b/(2a) the function
val

f (x)
r

whereas

has no

it

1.3.2.

(a)

1.

point x

is

We

at

value

b*

the function

of

maximum
Problem

attained

b*

4a
(b)

maximum

interoo);

4ac

rectangle,

b/(2a),

= 3x + 5x 1.

The minimum value

= 5/6

4ac

minimum value.
minimum value

(b) Find the rectangle with the


rectangles of a given perimeter.
Solution, (a) Apply the results of

interval

on the

b \

Find the

the

by

area from

all

3 0, b=5,
function at the

1.3.1: a

the

among

>

37
12'

denote by 2p the length of the perimeter of the required


and by x the length of one of its sides; then the area 5

Ch.

24

of

Introduction to Mathematical Analysis

J.

be expressed as

the rectangle will

= x(p x)

S = px

or

2
.

Thus, the problem is reduced to the determination of the maximum


value of the function S (x) =
x2
px. Apply the results of Probb
c
lem 1.3.1: a
0. The maximum value is attai1 < 0,
p,
b/(2a)=p/2. Hence, one
ned by the function at the point x
of the sides of the desired rectangle is p/2, the other side being
equal to p
x
p/2, i.e. the required rectangle is a square.

Show that
the function f (x)

1.3.3.
(a)

of

+ 3x + 5

domain

increases in the entire

definition;

its

x2 ) decreases in the interval (1, +oo).


x/(l
(b) the function g(x)
Solution. The function is defined for all points of the number
x 2 on the number
scale. Let us take arbitrary points x x and x29 x x
scale and write the following difference:

<

(x t )-f (x x )

= (4 + 3x + 5) - (x\ + 3x, + 5) =
= (*2 *l) (A + X X + X\ + 3) =
2

>

and the expression in the brackets is positive


Since x 2
xx
i.e. / (* 2 ) >f(Xj), which
x 1 and jc 2 then / (x 2 )
0,
f (xj
means that the function f (x) increases for all values of x.
at all

1.3.4. Find the intervals


wing functions:
(a)

/ (x)

of

>

increase and decrease for

the

follo-

= sin #-|-cos x;

tan(*+n/3).
Solution, (a) Using the familiar trigonometric formulas, we
(b)

find

]/^ 2 cos (x ji/4).


f (x)
It

known

is

that

the

function

2nn
and increases

in the

and the intervals

of

decreases

^.x^(2n+\)

(2n\)ji<^x^2nn

+ 2azji<

a:

the

intervals

(n=0

of decrease of

n/A

+ (2n+

increase of the

1,

2,

the function

1) ji

(ai

...).

= 0, +1,

same function

-J-

are:

/ (x)
.

..).

are:

+ (2n 1) k< x< n/4 + 2nn (n = 0, 1,


Find the minimum and maximum values of
= a cos x-\-b sin x (u
b 2 > 0).
j (x)

n/A
1.3.5.

in

intervals

Hence, the intervals


ji/4

cos

..).

the

function

= Vcl

f (x)

= a/)/ a + b
2

maximum
minimum

the

the
cos

value

sin

value of

(xa) =

1.3.6.

25

The given function can be represented

Solution.

where cos a

Investigation of Functions

1.3.

a),

= 6/ Ka + b'\
2

+\/ a

being

f (x)

a)|^
cos(a: a)-=l),

Since |cos(x

(at

t)

equal

1,

j/a 2

to

(at

at

the

fr

1).

minimum

Find the

value of the function

=3<*'- 2,,+ 8

f(x)

Solution.

-V b cos (x

equals

f (x)

of

as:

We

denote by

(p

the exponent,

(x)

(p(*) = (*

2)

(x)

takes on
The function f(x)=-3w
same point as the function (p (#).
Hence
(x)

cp

=x 6jc 4 + 2x 2

Whence it is clear that the function


value (equal to zero) at x
0. That is
the function f (x) is equal to 3
1.

e.

minimum

[(x
(p

i.

8.

the

=x

+
3)

(x)

why

value

3].

attains the minimum


the minimum value of

1.3.7.

Test the function


f (x)

<x<

where

ji/2,

increase and decrease.

for

of

= tan x + cot x,

1.3.8. Given: n numbers


x at which the function
f (x)

ai )

a2

a,,

+ (x-a +
2

2)

takes on the minimum value.


Solution. Rewrite the function
f(x)

= nx

2(a, + a +
2

an

f (x)

in

Determine the value

+ (x-a n f

the following way:

+a n )x+(a +al+
2

wherefrom it is clear that f (x) is a quadratic trinomial ax 2 + bx


c,
where a = /t>0. Using the results of Problem 1.3.1, we find that
the function assumes the minimum value at x =
b/(2a)
i.
e. at

x=

(aj

+a +
2

Thus, the

sum

+a n )/n.
of

the

squares

of

deviations

of

the value of x
x is the

from n given numbers attains the minimum value when


mean arithmetic value for these numbers.
1.3.9.

which

of

Which
them

is

of

the

given

functions is (are)
nor odd?

(are) neither even,

even,

odd;

and

26

Ch.

= \og(x+V\+x*);

f(x)

(a)

Introduction to Mathematical Analysis

I.

= log^l+x'
f(x) = 2x x+l;
/(x)

(b)

(c)

/W = *S^T-

(d)

Solution,

/(+

(a)

can

It

be

seen that

/(+

+ f ( x) ^=0.

x)

+ f(- ^)-log(^ + KTT^ + log(-^+KTT^ =


2

x)

= log
= / *) for all
= log
m_ ^ = iog

hence, / (x)
(b)

(are)
(a)

= f(x)

Which

( 1

+*

-- x 2 )=0,

which means that the function

v =
1

\+x)

x from

for all
is

is

odd.

io 6
g

1+jc'

domain

the

of

definition

odd.

of the following functions

is

(are)

even and which

odd?

= 4 2x* + sm
f(x) = V \+x + x

f(x)

(b)

a:,

Hence, the function

1).

1.3.10.
is

f(x)

Thus,

( 1,

Indeed,

(d)

f (x)

(e)

f (x)

x\

\r \ x + x

2
;

= sin x + cos x;
= const.
Prove that

1.3.11.

then the function

if

f (x)

+ b),

(ax

a periodic

is

where a

>

function with period T


with period T/a.

0, is periodic

Solution. Firstly,

f[a(x+
since T
positive

is

T/a)

the period

number such

+ b]=f [(ax + b) + T] = f (ax + b)


function f(x). Secondly,

the

of

let

7\ be a

that

f[a(x+T

1)

+ b]=f(ax + b).

Let us take an arbitrary point x from the domain of definition


b)/a. Then
and put x'
(#

of the function f (x)

+ T + b] =
= f(ax' + b + aT = f(x + aT
follows that the period T<a7\, i.e. 7\>7> and
Whence
the period of the function f(ax + b).
T/a
(x) = A sin (ayx +
where
Note. The periodic function
9

+ = / (ai=^+

ft)

=/(x)

=f

[a {x'

l)

l ).

it

is

are constants,

is

cp),

/I,

co,

called a harmonic with amplitude |j4|, frequency

co

and

initial

function

1.3.12.

phase

(p.

sin(o)A:

27

Investigation of Functions

1.3.

Since the function sin* has a period 2k, the


2n/(s>.
has a period T

(p)

Indicate the amplitude \A\, frequency


T of the following harmonics:

initial

co,

phase

(p

and period
(a)

/ (x)

(b)

/(*)
/(x)

(c)

= 5 sin \x\
= 4sin(3* + n/4);
= 3sin(jt/2) + 4cos(x/2).

1.3.13.

Find the period

(a)

= tan 2x;

(b)
(c)

/ (a;)

each

for

f(x)=cot(*/2);
\(x)^s\x\ 2nx.

tanx has

Solution, (a) Since the function


tan 2x has a period n/2.
1.3.14.
(a)

f (x)

(b)

/(a:)

Find the period

= s\n* x + cos
(a)

sin 4

= 1 y sin
whence
(b)

Jt,

the function

each of the following functions:

for

x;

+ cos x= (sin x + cos


2

2x

cos 4x)

x)

2 sin

x cos 2 x =

= -| + -j-sin ^4x + y)

T = 2n/a = 2n/4 = n/2


= |cosjc| = Kcos x^K(1 + cos2x)/2; but
has a period T = n\ hence, the given function

/(x)

2.x;

a period

= |cosa:|.

Solution,

cos

of the following functions:

the function
has the same

period.

Prove that the function / (x) = cos x 2 is not a periodic one.


Solution. Let us prove the contrary. Suppose the function has a
period T\ then the identity cos (x
cos x 2 is valid.
T) 2
1.3.15.

By

the conditions

of

we have
x2

equality of cosines

for a certain integer k

+ 2Tx + T x = 2nk.
2

But this identity is impossible, since k may attain only integral


and the left member contains a linear or quadratic function
the continuous argument x.

values,
of

1.3.16.

Find the greatest value

of the function
2

f(x)= r
1.3.17.

Which

of the following functions are even,

odd:
(a)

/W=^(l-x) + /(l+x)
2

2
;

and which are

Ch.

28

Introduction to Mathematical Analysis

I.

= x*-\x\;
= x sin x x
/(x) = (l + 2*) /2*?

(b)

f(x)

/ (x)

(c)

(d)

1.3.18.

Find the period

(a)

= arc tan (tan x)\

/ (x)

(b) f

each of the following functions:

(jc)-2cos^.
Prove that the functions

1.3.19.
(a)

for

f(x) = x + s\nx;

(b)

f (x)

= cos Vx

are non-periodic.

1.4.

Inverse Functions

=
yY

Let the function y


f(x) be defined on the set X and have a
there exists a single value of x such that
range V\ If for each
f(x)=y then this correspondence defines a certain function x g(y)
called inverse with respect to the given function y
f(x). The sufficient condition for the existence of an inverse function is a strict
monotony of the original function y f{x). If the function increases
(decreases), then the inverse function also increases (decreases).
The graph of the inverse function x g(y) coincides with that of
the function y
f(x) if the independent variable is marked off along
the (/-axis. If the independent variable is laid off along the x-axis,
i.e. if the inverse function is written in the form y = g(x) then
the graph of the inverse function will be symmetric to that of the
function y = / (x) with respect to the bisector of the first and third
quadrants.
y

3x
5.
1.4.1. Find the inverse to the function y
Sx
5 is defined and increases throughSolution. The function y
out the number scale. Hence, an inverse function exists and in5 with respect to x we obtain
3x
creases. Solving the equation y

+
= +

= (y-5)/3.

k/x (k=^=0) is inverse to itself.


1.4.2. Show that the function y
Solution. The function is defined and monotonic throughout the
0. Hence, an inverse function exists.
entire number scale except x
The range of the function is the entire number scale, except # 0.
k/y.
k/x with respect to x, we get x
Solving the equation y

1.4.3.

Find the inverse


y

Solution.
since ]fx

The

+ >
1

of the function

= \ga(x + V*+l) (fl>0,


function y = log a (x + }/~x + l)
9

\x\

and

is

odd

[see

Problem

a=
is

1).

defined

1.3.9 (a)].

for all x,
It

increases

29

Inverse Functions

1.4.

positive values of x, hence, it increases everywhere


inverse function. Solving the equation

for

y=\og a
with respect to

x,

a?

we

(x

and has an

+ Vlfi+l)

find

= x + Vx +
2

a-y=

l;

x+Vx +
2

1,

whence
x

Show

1.4.4.

= y (a^

(*/

lna).

that the functions

f(x)=x 2 x +
are

= sinh

x^

1,

and

1/2

cp

(*)

1/2

+ j/x 3/4

mutually inverse, and solve the equation


x*

x+

=
#=
1

1/2+ Vx 3/4.

The function
a;
x + 1 = (x 1/2) 2 + 3/4 increases
the interval 1/2^a:<cx), and with x varying in the indicated

Solution.
in

we have 3/4^*/<oo.

interval

3/4<#<oo

is

the

inverse

Hence, defined
x = g(y),

function

in

x^

the

interval

which

1/2,

is

found from the equation

x*-x + {l-y) = 0.
Solving the equation with respect to

x=g
Let us

now

(y)

1/2

x,

we obtain

+ K^T? =

q) (*/).

solve the equation


x*

x+l = l/2 + Vx 3/4.

Since the graphs of the original and inverse functions can intersect
2
x, solving the equation x
x+l = x
only on the straight line y

we

find

1.4.5. Find the inverse of y


smx.
Solution. The domain of definition of the function y
s\nx is
the entire number scale, the range of the function is the interval
is
[
1]. But the condition of existence of an inverse function
1,
not fulfilled.
n/2. If n is
Divide the x-axis into intervals tin
even, then the function increases on the intervals tin
n/2; if n is odd, the function decreases on the intervals
n/2. Hence, on each of the indicated intervals
nn ji/2^

n/2^x^nn +
n/2^x^

^nn +

x^nn +

there exists an inverse

function

defined

on the interval [1,

1].

30

Ch.

I.

Introduction to Mathematical Analysis

jt/2 ^

In particular, for an interyal


verse function jt
arcsinr/.
The inverse of the function y

^*^wt+jt/2
x=

(b)
(c)

(d)

arcsin# + wr

Find the inverse

1.4.6.
(a)

l)

there exists an in-

s\x\x on the interval mi


expressed through atcsin y in the following way:

is

ji/2

y = s\n(3x 1)
# = arcsin(*/3)
# = 5 e*;
xix '
y=2

of the

(ai

= 0, +1, 2,

...).

given functions:

at (jt/6+ l/3)<
at 3<x<

(jt/6+

1/3);

3;

lo

1}

Prove that

1.4.7.

the

function y

= (l x)/(l+x)

is

inverse

to

itself.

1.5.

Graphical Representation of Functions


Sketch the graph

1.5.1.

2x +

(a)

f( x )=x*

(*>)

f(*)=TT#>

(c)

(d)
(e)
(f)

of

each of the following functions:

3;

= s\n x 2sinx;
(x) = arc cos (cos x);
f (x) = Vsm x;
*x.
f( X = X
2

f(x)
/

'

Solution, (a) The


entire number scale.

domain of definition of the function/^) is the


The function f (x) is even, hence its graph is
symmetrical about the ordinate axis and it is
sufficient to investigate the function at

Let us

x^O.

out a perfect square f(x)


2
2.
(x 2
Since
the
first
summand
1)
2
2
(x
1) ^0, the minimum value of the function, equal to 2, is attained at the points
(see Fig. 2).
x
The function f (x) decreases from 3 to 2 on
the closed interval
and increases
oo.
unboundedly on the open interval 1
x

single

= l

O^x^l

< <

The domain

funcfunction / (x) is odd, therefore its graph is symmetrical about the origin
of coordinates and it is sufficient to investigate the function at
(b)

Fig. 2

tion

x ^0.
Since /(0)
that

there

= 0,

are

f (x)

is

of

the entire

definition

number

of

scale.

the

The

the graph passes through the origin. It is obvious


no other points of intersection with the coordinate

Graphical Representation of Functions

1.5.

(1 U|) 2 >0or l+x 2 >2|*|,

Note that |/(jc)|<1. Indeed,


whence
axes.

Since

/ (x)

maximum

>

>

and f (1)
at x
value of the function f

being zero (see Fig.

31

= 1,

the

in

equals

(x)

1,

interval

the

[0,

oo) the

minimum

value

3).

3'

-1

"2

12

-3

-/
Fig. 3

Let us prove that the function increases on the closed interval


Then
Let
2 <1.

fl,)

0^

0^*,<*

<*<1.

2x 2

2x t

2a: 2

Ml, ~l+4

2x 2x^5 "
(l+*S)0 + *i)

-}-

2x 2 x\

^ 2 (x s
2

t )

(1 x^)

(l+*I)(l+*f)
and f(x 2 )

>f(x

).

Similarly, we can show that on the interval


decreases. Finally,
f

(x)

= 2x/( + x )<
2

2xi'x

(1,

oo) the function

= 2/x,

whence it is clear that f (x) tends to zero with an increase in x.


of definition of the function / (x) is the entire
(c) The domain
number scale. The function has a period 2n, that is why it is quite
sufficient to investigate it on the interval [0, 2jx], where it becomes zero at the points x = 0; x = n\ x = 2n.
Writing the given function in the form
f (*)

= (1 sin*)

we note

1,

that it increases with a decrease in the function sin* and


decreases as sin* increases. Hence, the function / (*) decreases on
the intervals [0, n/2] and [3ji/2, 2n]> and increases on the interval
[jx/2, 3jx/2]. Since f(n/2)
1, and /(3ji/2)=3, the range of the
function is
3 (Fig. 4).

-!</(*)<

Ch.

32

I.

The domain

(d)

definition of the function is the entire number


at any x hence, arc cos (cos*) has a
function f (x) is a periodic one with the period 2n

meaning. The
hence,

it

of

|cosx|^l

Indeed,

scale.

Introduction to Mathematical Analysis

to sketch

sufficient

is

its

But

graph on the interval [0, 2jx].


on this interval the following

equality

true:

is

f(x)=

*.

2n

x,

0<x< n
n^x^

2jt.

Indeed, the first assertion follows


the definition of the function
arc cos x while the second one can be
proved in the following way. Let us
put x'
ji
jk
2ji;
then

from

= 2nx
O^x'^in and

/(*)

Taking
(e)

that

= arc
all

cos [cos

(2n x')] =arc

< <

cos (cos x')

= x' = 2ji x.

con sider ation, we draw the graph (see Fig. 5).


]/ s'mx
is
a periodic one with period 2n\
confine ourselves to the interval [0, 2ji]. But

this into

The function y =
is

why we may

-2%

-%

Tt

2lt

Fig. 5

the function is not defined in


defined only in the interval [0,
radicand is negative. The graph

the
ji],

whole interval [0, 2ji], it is


as in the interval (jx, 2n) the

symmetrical about the straight

is

Fig. 6

line

have

a:

= ji/2,
an

as

well as the graph y


s'mx (see Fig. 6). Here we
of a periodic function which does not exist in

example

the infinite set of intervals.


(f) The domain of definition of the function

<x<

and

<

<

oo.

is

Reduce the formula

form

to the

f(x)

x xn *

= x Xo** 10 =

the graph of the given function


the right-hand half plane with the point x
1.5.2. Sketch the graphs of functions defi-

Hence,
in

33

Graphical Representation of Functions

1.5.

jo.
is
1

the half-line

removed

#=10

(see Fig. 7).

ned by different formulas in different intervals (and in those reducible to them):

(a)

sin x

at

at

*/=<

l/(x

(b)0=

1,

at

1/2

at

x=

at

a;

x>0,

(c)

y=x + V#;

(d)

y=2/(x+\f?).

0,

< 0;

Fig. 7

The domain of definition of the function is the


The graph of the function consists of a portion
Jt< x
the sinusoid y = s\x\x on the interval
0, straight line

Solution,
interval
of

1)

10

0< x<
< < 4;

at

2
x

jx<x<0,

(a)

it,

4].

<

9,

2
1/2
{

-1

12
i

.1

-1

4
-2

-i
Fig.

Fig. 9

=2

on the interval (0, 1] and a part of the branch of the hyperon the interval (1,4] (see Fig. 8).
(b) The graph of the function consists of a portion of a cubic
parabola, an isolated point and a half-line (see Fig. 9).
(c) The function may be given by two formulas:

i/

bola

y=\l{x\)

2*.

if

0,

if

y==

x>0,
x<0.

34

Ch.

Introduction to Mathematical Analysis

I.

Thus, the graph of our function


(d)

From

interval (0,
of

(c)

a polygonal line (see Fig. 10).

defined only in the


Thus, the graph
the right-hand part of an equilateral hyperbola

+oo), y being equal

our function

(see Fig.

is

follows that the function

it

is

l/x (x

to

is

> 0).

11).

Fig.

1.5.3.

Sketch the graphs of the following functions:

(a)

^ = cosx+|cosx|;

(b)

y=\x + 2\x.

Solution, (a) cosa;

2cosjc at cos* ^5

cos*|
|

=<

q at COSA;

Doubling the non-negative ordinates


y = cosx (the broken line in Fig.

tion

X
x

-7C

'

Sic
2

JL

the points

cosx<0, we

where

solid line in the

same

The function

12)

graph for the funcat


and assuming y =

2%

~T~

Jax
~7T

12

can sketch the desired graph (the

figure).

+ 2|x may be
(x
2)x at
=/ +

|x

0,

<o.

of the

% s'3n

2
Fig.

(b)

11

(x+2)x

given by two formulas:

at

2,

2.

2
Plotting separately both parabolas: y = (x+2)x = (x+l) l
2
the
parts
corresponding
only
retain
to
l)
and y =
1],
[(*
the above indicated intervals. Drawn in a solid line in Fig. 13 is
the graph of the given function, the broken line showing the de-

leted parts of the constructed parabolas.


1.5.4.

Sketch the graph

of

the function

= 2\x2\ x+
\

l\

+ x.

^2

Solution. At x

y=
At

35

Graphical Representation of Functions

1.5.

2(x2) (x +

l)

+ x = 2x 5.

l<x<2
2(x 2) (x+ + x =

y==

\)

2x

+ 3.

19

\\V//

'2/

-1

'

>

-1

Fig.

Finally,

at

Fig.

13

x<.~

y==

-2(x-2) + (x+\) + x = 5.

Hence, the given function can be rewritten

2x +

Therefore
(see Fig.
1.5.5.

3,

the

graph

is

a polygonal line

14).

Sketch the graph of the function

y^2 x 2~ x
Solution.
2 x and
yt

Draw graphs
y2

= 2~ x

for the functions

(broken

lines

in

and add graphically their ordinates. In doing so bear in mind that y 2 < y <
y 19 and that y 2 tends to zero with an increase
tends to zero with a decrease in x (the solid line
Fig.

15),

<

1.5.6.

way:

x> 2.

2x5,

in the following

*<-l,
1 <x<2,

5,

y=)

14

Sketch the graph of the function


*/

= x sin x.

Fig.

in

15

whereas y

in Fig.

15).

36

Ch.

I.

Introduction to Mathematical Analysis

*/ 2

Solution. Being the product of two odd functions y


x and
x
sin*, the function y is an even one, that is why we shall

analyse

We
Fig.

it

x^O.

for

draw graphs

for

=x

yx

and

y2

=smx

(the broken

lines

in

16).

At the points
points where y 2

where

r/

= smx =

y = y 1 .y 2

= sin# = 0,
1,

y= yi = x.

The

and

latter

at
the
equality

Fig.

16

indicates the expedience of graphing the auxiliary function y 3


x.
Marking the indicated points and joining them into a smooth
curve, we obtain the required graph (the solid line in Fig. 16).

1.5.7. Sketch the graph of the function y


x(x 2
plying the ordinates of the graphs y t
and y 2 z=x 2

=x

(b) y = /arc cos x.


Since the function
odd,
for x ^ 0.

y = x/(x 2

Solution,
tigate

it

4),

(a)

Let us consider

is

it

as the quotient of the

=2

=x

is

it

sufficient to inves-

two functions:

and y = x 4.
y
the denominator y = 0, the
x

Since

by multi-

1.

Graph the following functions:

1.5.8.
(a)

1)

function is not de2


the point 2. In the interval [0, 2) the function y x increases
4
from
to 2, the function y 2 is negative and|r/ 2
x 2 decreases
from 4 to 0; hence, the quotient f(x)=yjy 2 is negative and increases in absolute value, i.e. f(x) decreases in the interval [0,2)
at

fined at

from

to

oo.

In the interval (2, oo) both functions are positive and increasing.
Their quotient decreases since from
it follows that
2
t
x2

xt

2^x <x
x x +

(*!

2 ) (a: 1

4)

Graphical Representation of Functions

1.5.

The indicated quotient tends

to zero as x

37

oo, since y

= _^X
1

The general outline

the

of

graph

is

presented in Fig.

~^ 0-

17 (three

solid lines).

Denote y = arc cos x. The domain of definition of this funcAt x=\ we have y l = 0, hence, y=\/y l * oo at

(b)

tion

x+

1,

i.

e.

x=

on the entire interval

of

definition

dingly,
in Fig.

I.

hence

Simple Transformations

The graph

graph

1),

1,

y=\/y

incre-

n is attained at *--= 1. Accorvalue y 1


value of the function is 1/jx. The solid line
18 represents the general outline of the graph.

maximum
the minimum

The

ases.

18

The function y decreases

a vertical asymptote.

is

Fig.

17

Fig.

the x-axis by
a (see Fig.

Graphs

a) is obtained from the


function y
f(x
by translating the latter graph along
scale units in the direction opposite to the sign

of

the

of the function

of

of

= f(x)

19).

b is obtained from the graph of the


II. The graph
f(x)
y
function y
f(x) by translating the latter graph along the y-ax\s
by \b\ scale units in the direction opposite to the sign of b (see

Fig. 20).

f(kx)(k>0) is obtained from


III. The graph of the function y
the latter graph
the graph of the function y
f (x) by "compressing"
1 and
against the *y-axis in the horizontal direction k times at k
by "stretching" it in the horizontal direction from the r/-axis \\k
(see Fig. 21).
1
times at k

>

<

Ch.

38

J.

Introduction to Mathematical Analysis

>

IV. The graph of the function y


0) is obtained from
kf (x) (k
the graph of function y
f(x) by "stretching" it in the horizontal
1
and "compressing" it against the #-axis
direction k times at k
(see Fig. 21).
1
\/k times at k
(i. e. vertically)

=
>

<

vr

u>-0 /

,o yuf(x)
y=f(x)+l,l<0

Fig. 20

y=

V. The graph of the function


f(x) is symmetrical to that
the function y
f(x) about the x-axis, while the graph of the
function y
x) is symmetrical to that of the function y
f(
f (x)
about the y-axis.
of

Fig. 21

Fig.

22

VI. The graph of the function y = f(\x\) is obtained from the


graph of the function y = f(x) in the following way: for jc>0 the
graph of the function y
f(x) is retained, then this retained part
of the graph is reflected symmetrically about the #-axis, thus determining the graph of the function for jc<0 (see Fig. 22).
VII. The graph of the function y = \f(x)\ is constructed from
the graph y
f (x) in the following way: the portion of the graph

1.5.

Graphical Representation of Functions

39

y
f(x) lying above the x-axis remains unchanged,
other portion located below the x-axis being transformed symmetrically about the x-axis (see Fig. 23).
VIII. The graphs of the more complicated functions
of the function
its

y^'kf (kx+a)
are

+b

drawn from the graph

of

= f(x)

plying consecutively transformations

ap-

to V.

Graph the function

1.5.9.

= 3J/

-2(a;

+ 2.5) 0.8

by transforming the graph y = ]fx.


Solution. Sketch the graph of the function

y=z\fx (which

is the upper
branch of the
parabola y 2
x) (Fig. 24, a), and transform
it in the following sequence.

Sketch the graph

by enlarging

of the

function y

= 3]/2x

the points on the graph of the function


abscissas unchanged (see Fig. 24, b).

Then sketch the graph


be the

mirror

Fig. 24,

Fig. 23

2 times the ordinates of

image

of

y= \

3V 2x which will
graph about the (/-axis (see

the function y

of the preceding

x and leaving their

c).

Fig. 24

By
0.8

shifting the obtained graph 2.5 scale units leftward and then
downwar d draw the desired graph of the function

u nit

y=3V 2(jc + 2.5) 0.8


1.5.10.

(see Fig.

Graph the function #

the cosine curve.

24, d).

= 3cos# j/3sin;t

by transforming

40

Ch.

Introduction to Mathematical Analysis

I.

Solution. Transform the given function

y = 3 cos

x V 3

si

n x

Q
= 2 j/3~(
'

j-cosx

y sin

jc

= 2 K3
Thus, we have to sketch the graph

#=

which

cos

of the function

2VTcos (x + ji/6),

the graph of the function */


2 K 3cosjc translated by ji/6
The function has a period of 2ji, hence it is sufficient to

is

leftward.

draw

its

graph

for

n ^Zx ^

ji

ZVT

\
//
/

\
1

/'
//

\\

/ fa/2
I

>

f\\7t/2

\\

>^

-2V3
Fig. 25

Fig. 26

The graph of any function of the form */ = acosx


a and b are constants, is sketched in a similar way.

+ frsin x,

1.5.11.
(a)

<

b>

Graph the following

y^i
f-JTZ^

(c)

x2

^
s

+ a:+

if

(x-l)/(x+l),

if

(g)

(h)

j/=3s in

(e)
(f)

(i)

if l<x<0,

1,

sin 2 x

= x+ \/x;
=
x x
y
=
sin
=
1/cos*;
y

(d)

functions:

ft

(2jc

4);

t/=2K 3(x+

1.5)

1.2;

< x < k,
<x^
5;

where

y=

(j)

2x

of

a Sequence

41

1|;

y=\\x\-l\;

(k)

j/

(1)

|x

Number Sequences. Limit

1.6.

= cos(sinjc);

y=\s'mx\ + s\nx on

(m)

the interval [0,3k];


f

y = x sign x
2

(n)

where

signx

at

x> 0,

at

x=

at

0,

< 0.

1.5.12. The function y


f(x) is given graphically
Sketch the graphs of the following functions:

(b)
(c)

(d)
(e)

= f(x+

(a)

(Fig.

26).

1);

y = f(x/2);
y=\f(x)\;
= (|/(*)|/(*))/2;
y = \f(x)\/f(x).

1.6.

Number Sequences. Limit of a Sequence

The number a is called the limit of a sequence x u x2y


xn
oo, a lim x n if for any e>0 there exists a number
as n

...

N(e)>0

-+ co

such that the

inequality

a\<&

|jt

holds

true for

all

n> Af(e).
A
A

sequence which has a finite limit is said to be convergent.


sequence {x n } is called infinitely small if lim.*^
0, and infi-

nitely large
1.6.1.

if

lim^^oo.

Given the general term

sequence {xn }:

of the

sin (nn/2)

Write the first five terms of this sequence.


Solution. Putting consecutively
2, 3, 4, 5 in

n=\

term xni we obtain


sin (jt/2)

3
sin (4it/2)
:

4
sin(5jt/2)

~~

~
"" U;

_
~~

1
*

the general

42

Ch.

Knowing

1.6.2.

Introduction to Mathematical Analysis

I.

the

several terms of the sequence, write one

first

of the possible expressions for the general term:


(3 '

I'

(b) 1, -g-,
Afofe.

13

2,

18

-g-

'

23

3,

knowledge

-g-.

4,

terms of a sequence

of the first several

That

sufficient to define this sequence.

is

why

this

is

not

problem should

be understood as one of finding a certain simple inductive regulacompatible with the given terms.
Solution, (a) Note that the numerator of each of the given terms
of the sequence equals the square of the number of this term plus
unity, i.e. n 2 +\, while the denominators form the arithmetic progand the comression 3, 8, 13, 18, ... with the first term
rity

mon

thus

^=3

d = 5. Hence,

difference

= a + d(n = 3 H- 5

(az

1 )

= 5n
l)

2,

we have

_ n* +
2
l

*~~5/i

'

(b) Here the general term of the sequence can be written with
the aid of two formulas: one for the terms standing in odd places,
the other for those in even places:

xn
It

at

1/(6+1)

also possible to express

is

which

j k

=\

the

more complicated,

will be

at

= 2kl
=
2k.
n
n

general term by one formula,

for instance,

^=^[i-(-i)i+^- [i+(-i)"]2

is

Find the first several terms of the sequence


given by one of the following formulas:

jc

= sin (nn/3)\

1.6.3.

term
(a)

(b)
(c)

(b)

is

Using the definition

lim x n

limx n =

if

the inequality

3/5 < 0.01

is

for

1).

a sequence, prove that

Beginning with which

fulfilled?

Solution, (a) For any e

l|<e

of the limit of

x n = (2n l)/(2n+ 1),


2
l)/(5n 2
3/5 if x n = (3n

=l

Af(e) such that

\x n

the general

x n = 2~ n cos nn\
l/n)\
x n =(l

1.6.4.
(a)

if

any

fulfilled.

>

let

natural

us try to find a natural number


n>N(z) the inequality

number

For

Number

1.6.

purpose

this

43

Sequences. Limit of a Sequence

us find the absolute value of the difference

let

2/11
-1

2/i+

2/1+1

Thus, the inequality

jc
|

>

1/e V 2

Hence the

be taken as jV(e),
So, for each e
inequality n

>

Let

Let e

us

>

if

<8

^qrf

number

1/e

whence

V,

may

<
1

f^ =

the

l.

absolute value of the difference

3/z 2

+l

5/z 2

3/5|:

8
5(5/i 2

1)'

be given. Choose n so that the inequality


5(5/i 2

is

satisfied

we can find a number A/ such that from the


e, which means that
will follow that \x n
1

it

find

2
is

integral part of the

lim
(b)

W = (1/8 7,).

i.e.

>N

<e

2n+

-l)

fulfilled.

we

Solving this inequality,

find

.1 t/8 + 5b

Putting

N = (i/?5),
we conclude

n> N

that at

K-3/5|<e,
which completes the proof.
If

= 0.01,

then

*-*({and
ned

all

1.6.5.

beginning with the 6th, are contai0.01).


(3/50.01; 3/5

terras of the sequence,

in the interval

known

-5.

Given a sequence with the general term x n


that

lim

x=

side the open interval

1/3.

Find the number

of points

= 9n _^ 4

It

is

xn lying out-

44

Ch.

Solution.

Introduction to Mathematical Analysis

I.

The distance from the point x n

to

the

point

1/3

is

equal to

19

3(9/z

+ 4)

19

3(9/z

+ 4)

Outside the interval L there will appear those terms


quence for which this distance exceeds 0.001, i.e.

of

the se-

19

3(9/z

+ 4) ^

000

whence
1

Hence, 703 points


val

<; n

(x lt

x2

<

18988

?n

x 103 ) are found outside the

inter-

L.

1.6.6. Prove that the number 1


2
ce with the general term xn
(n
Solution. Estimate from below

rence
n2

2n 2

not the limit of a sequen2

9).

the absolute value of the diffe-

1^-2

-0

is

2)/(2n

|2/2

3 the absolute value of the difference remains greater


At n
than the constant number 1 /2 hence, there exists such e > 0, say,
;

e=

/ 29

that the inequality


2ril

>

holds true for any n^3.


The obtained inequality proves that
given sequence.
1.6.7.

is

not the limit of the

Prove that the sequence

112

1,

with the general term


\/n,

n/(n

+ 2),

if

/2-2/e

if

1,

= 2k,

has no limit.
Solution. It is easy to show that the points x n with odd numbers concentrate about the point 0, and the points x n with even
numbers, about the point 1. Hence, any neighbourhood of the point 0,
as well as any neighbourhood of the point 1, contains an infinite
set of points x n Let a be an arbitrary real number. We can always
choose such a small e
that the e-neighbourhood of the point a will
.

>

Number Sequences. Limit

1.6.

Sequence

of a

45

not contain at least a certain neighbourhood of either point


or
point 1. Then an infinite set of numbers x n will be found outside
this neighbourhood, and that is why one cannot assert that all the
numbers x m beginning with a certain one, will enter the e-neighbourhood of the number a. This means, by definition, that the
number a is not the limit of the given sequence. But a is an arbitrary number, hence no number is the limit of this sequence.

Prove

1.6.9.

number
(2n
|

=l

Prove that lim* n

1.6.8.

+ 3)/(/2+

1)

= (3 n +

l)/3

n
.

limx = 2 if x n = (2n
3)/(n
Find the
1).
term
the
inequality
beginning with which
e,
where 8
2
0.1; 0.01; 0.001, is fulfilled.

that

the

of

xn

if

<

Prove that the sequence

1.6.10.

J_
2

_L

i.

'

'

_L
4

'

'

with the general term

-^rm

*=f

if

is

if

is

odd

j77
n/2

even,

has no limit.

Prove that

1.6.11.
xn

any

at

arbitrarily

large

a>0

limA;

if

= a n /nl
number

Solution. Let a natural


an

> 2a.

f a

a \

k )

\k+l

n\~

Then

at

a
k

>k
a
^
n

+2

<a.(i)--W.(^"
Since lim(l/2)"
have:

V 2 J

and,

it!),

hence,

(2a) k

xn

then at a
a

/n\

sufficiently

< e,

which

Test the following sequences for limits:

x=1/(2jx);
i

(b)

(prove

= 0.

Y\m(a n /n\)

(a)

<

(4-)

1.6.12.

for

an even

(C)

Xn

= ~ C0S nn
~2"

(d)

ai

n,

<j

[!-(-!)].

large n

means

we
that

46

Ch.

Introduction to Mathematical Analysis

I.

Prove that the sequence with the general term

1.6.13.

(k>0)

x n =l/n*
is

an

infinitely small sequence.

Solution.
pro\e that

To prove
-*

Take an

sequence xn

that the

= 0.
n
arbitrary e > 0.

infinitely small

is

is

to

lim x n
oo

Since \x n

l/n k

we have

to solve the

inequality

<

l/n*

whence

n>

j/T/e,

of

i.

N may

Hence

y^l/s.

e,

be expressed as the integral part

= E(^1/S).

e.

1.6.14. Prove that the sequences with the general terms

xn

(a)

are infinitely small


1.6.15.

xn

l)

as

Show that
2/(5f/n+l)

= sin

xn

(b)

[(2/i

1)

oo.
the

with

sequence

infinitely small

is

general

the

n>oo. Find

as

beginning with which the points x n belong to the


(-1/10, 1/10).
Solution. Take an arbitrary e>0 and estimate \x n
ber

term

num-

interval

\:

JC

That

is

why

l-

l""

3/

|xj<e

^
^

3/-

a; soon as n

>

3/1/e

3/-'

Hence limxn

the sequence

is

= 0,

i.e.

oo

infinitely small.

We take now 8=1/10. Since |^n |< l/\/n x n will necessarily


1000. Hence N may
be smaller than 1/10 if \lV~n< 1/10 or n
be taken equal to 1 000. But we can obtain a more accurate result
by solving the inequality
y

>

It holds true
taken equal to

1.6.16.

simal as

It

is

at

X n\

< TO

3/-

> (19/5) = 3. = 54.872.


3

Hence

N may

be

54^1000.
known

n^00,

then

= +

a n where a n is an infinitethat if x n
a
lim x n
Taking advantage of this rule,
a.

oo

find the limits:

3"+!

+ sin (wi/4)

2*

+ ( 1)"

Number

1.6.

c
Solution, (a) x n
<

=3

w+1 -f- sin (nn/4)


"s^

an infinitesimal as

Sequences. Limit of a Sequence

3+
n

1.6.17.

Prove that

is

= 3.

-+ oo

-+

<x>

Yn

Let us prove that the variable

Solution.

= sin (nn/A)

y^n=l.

lim
n

where a n

hence lim x n

oo,

47

can be represented

oo.
sum l+a, where a n is an infinitesimal as n
us put \/n= 1 + a n
Raising to the nth power we obtain

as the

Let

rc=

+aj n =

(1

+Aia +

^=^a^+

wherefrom we arrive

at the conclusion that


lowing inequality holds true:

for

+a"ny

any n

>

the

fol-

n>l + njn^l) a%
(since all the terms on the right are non-negative). Transposing the
we obtain
1
unity to the left and reducing the inequality by n

whence

it

lim
n

V 2/n =

-* oo

Hence

it

an

lim

2/n

that

follows

> a n > 0.

or

also equals zero,

an

e.

i.

is

Since

an infinitesimal.

oo

follows that

yn=l.

lim
n

1.6.18.

>a

-*

oo

Prove that the sequence with the general term

>oo.
infinitely large as
Solution. Let us take an arbitrary positive
the inequality
is

number

and solve

> AJ.

Taking the logarithm, we obtain

Vn> log, /W, n> (log


If

lity

we now take
|x n

|>A4

infinitely

1.6.19.

will

/V

= E (log

be

A4) 3

fulfilled,

then

M) 3

for

Prove that
lim
-* oo

y/a

all

n> N

the inequa-

which means that the sequence

large.

(a

>

0).

is

48

Ch.

Introduction to Mathematical Analysis

Evaluation of Limits of Sequences

1.7.

the sequences {x n \ and {y n \ are convergent, then


Km (x n y n ) liin x n lim
lim xn lim
(2) lim
If

(1)

If

1.7.1.

Find

xvn~

lim x

3n 2 +5n

+n

if

+4

W
w

-4n

2/i3

3/i

4n

n.

qo

'

+ +
+ 4'
1+2
*n =
+r + "

(e)

^ lim y

then lim x n

*/

^
W

5b

+ 2-3/i + 7

4n 3 -2n+ll

'

l+2+ ...+

n_

5/i3

+ n+l

3+A+l
n

Solution, (a)

2_

S+

n*

lim

lim x

(3

5//z

+ 4//z

7-o

-3.
)

Recall that

(d)

l*

+2 +3
2

+...

+n

(2/i+l)

Hence
/i(/i+1)(2/i+1)_ 2n 3 + 3n 2 + Ai
6(5n3 + n+l)
/i+1)
6(5/i*

2
^ A-f-l
n^ n
_ +

30 _L
1

lim
1.7.2.

Find

lim x ny
ft

W
,

(c)

u3

n1

1/15.

if

=yn>;

Solution, (a)

^6

-+ 00

\3
/ 3n + n
^42 + 2n + 7j
xn = ybn;

{t)xn

# =

'

\
2n 3 + 2n +l
U" + 7 " + 3n + 4j

W
(d)

xn

pi*;

=^6n + 3.

(l)xn

(^^r)' =

Urn

_ m 3n + n \ 3n + n \ 3n + n __
i U + 2n+7j U + 2 + 7j U/t + + 7j

\
3_\
= lira 3+1/rt
4+2/rt
,.

/'

2/l

2//i 2

+ 7/rc

'

,4 J

_27
-64

17. Evaluation of Limits

49

of Sequences

In solving this example, and also the rest of the examples


Problem 1.7.2, take advantage of the following equalities (see
Problems 1.6.17 and 1.6.19):
(c)

of

i/n=l

lim

We

and

lim

\/a=\.

(1)

have

lim xn
n

from

but

lim xn

1.7.3.

-+

lim
n

<x>

\/n,

lim
n

-* co

-* co

limv^5=l and

that

follows

it

(1)

i/5n

lim
n

co

lim y^/i

hence

Find
2ft 3

'

Summing

Solution.

we obtain

the fractions,

xn

2ft
i

Oft 3

1 5ft 2 \

13ft 2

2ft 2

+3
+3

15ft

Whence
lim x n
n

Note.

If

_
-

2ft 3

r
iirn^

13ft 2

10n3+ 2 n 2 +

-+ oo

+3 _
+3-

i5 n

we put

2ft
2ft

_ 1
^ 5ft+
\im{y n + z n = 1/5,
5ft 2

+3

'

though each of the


their sum
)
an infinitely large quantity. Thus, from the convergence of a sum of sequences it does not, generally speaking, follow
that the summands converge too.
then the limit of

summands

1.7.4.

is

lim xn

Find

(a)

-*

if

co

= V2n+3 Vril'
xn = Vn + n+ ]/ n n+ 1;
xn = n (nVn + \y,
x

(b)
(c)

(d)

xn

=n n + n\

(ej

xn

]Tn*+\
4

+ Vn

12 + 3 4 + 5 6+

(h)

xn =

...

2ft

+ + +...+.
1-2

2-3

3-4

'

'

ft(ft+l)

50

Ch.

Introduction to Mathematical Analysis

I.

= Vn(V2 +

Solution, (a) xn
3/nVl l/n)-+ -f oo as n
since the second multiplier has a positive limit.
(c)

Xn

_ n*(n-VlF+\ _

-n*

=n
xn

=
(n 2

oo

n+K/iHl

(d)

3 2/3
)

oo

n y n n* + n*

as

oo.

means, x n

It

>

1/3.

Factoring out the terms of the highest power in the numerator and denominator, we have:
(e)

1/4

\\mx n

Find

1.7.5.

>

+ oo

as

oo.

if

n-*<x>

(c)

xn

I
(e)

(f)

=Y n

xn

2n

+n;

C0S
1

= -^ cos

n( 1)"

l2n

n 2 +l

(d)

n+l
2

j-

n3

.
*

1+T+T+-+*

1.8.

Testing Sequences for Convergence

Bolzano-Weierstrass* theorem.
a

finite

Theorem on passing
and

liin
n -y cc

-{ 00 or

A monotonic bounded

sequence has

limit.

x^lim

00

zn

to the limit in inequalities.


too (c
then \imy n
c

= c,

-> cc

but not oo).

-* cc

If
is

xn

^y ^z n

number,

sequence with the


an increasing one.
have to prove that x n+1

Prove

1.8.1.

We

51

xn

term

general

the

that

= (2n l)/(3n+ 1)
Solution.

Testing Sequences for Convergence

1.8.

is

>x n

any

for

n,

i.e.

to

prove that

2n

2/z-j-l
3/Z

The

inequality

latter

e>n

Hence, x n+l

> xn

+ 4 > 3rc+

'
1

equivalent

is

to

obvious

the

inequality

+ 5n+\ > 6m + 5n 4.
2

Given a sequence with the general term

1.8.2.

Xn

'

n\

Prove that this sequence decreases

at

n^lO.

Solution,

Xn+1 ~~

<

Since

at

ft

10"+

(n+

_ 10"

n\

1)!

^10,

then

10
'

/i+

x n+1

10

__

n+

<x n

'

beginning with

number, which means that the sequence decreases

10.

(b)

= ^j'>
n
# = (-l) ~sinft;

(c)

zn

a ) Xn

= n cos nn.

Solution, (a)

(b)

The sequence

The sequence {y n }

is

{x n \

is

(c)

The sequence

{z n \

1,

is

obvious that

|sin|<^T <2.

= \ncosnn\ = n.

Prove that the sequence


Xn

>

it

not bounded, since

is

\z n

1.8.4.

bounded, since

bounded:

l^l = l(-D"|-^T

(a

Test the following sequences for boundedness:

1.8.3.
(

at

this

Xi

> 0)

Xn

Xo

converges.

Solution. Let us prove that


bounded. Firstly, x n
x n _ l as

<

this

sequence

is

monotonic

and

52

Ch.

I.

Introduction to Mathematical Analysis

Hence, the given sequence is a decreasing one. Secondly, all its


terms are positive (by condition a
and x
0), which means
that the sequence is bounded below. Thus, the given sequence is
monotonic and bounded, hence it has a limit.
1.8.5. Prove that the sequence with the general term

>

11

5+1
x1

^i.e.

b+{

52

+l ^5 3 +l

1,1
~ 5+1
+ 5 +1

x2

>

x2

'

5H-1

1,1,1

5+1+

5H-1 ^

5a

+l

'

converges.
4-1
xn
1/(5"
Solution. The sequence \x n ) increases, since x n+1
x n Besides, it is bounded above, since 1/(5"+
and, hence, # +1

>

<

5+1

1)

1)

<

any n and

1/5" at
1

"

+ _L_
+ _^
^3 +l +
5 +l
5
*22

I
1

/
^

I
1

iii
^ ^ili^
52

53

'
'

'
'

I1

'
'

+-J
^
Kn
5+l
_L_

I
1

1/5-1/5" + *

'

'

'

^ 5"

1/5

_ W,

1\

5" J

'

Hence, the sequence converges.

Taking advantage of
monotonic bounded

1.8.6.

limit of a

theorem on the existence

the

of a

the following

sequence, prove that

sequences are convergent:


(a)

= r* B = 2 + l + l+...+I.

xn

(b)

1.8.7.

Prove that the following sequences converge and

find their

limits:
(a)

xx

= V~%

*2

= ^2+l/"2;
n radicals

2n

(b)

*
(/i

(c)

(d)

the

1.41;
(e)

+ 2)!

sequence of
...

1.414;

xn

= n\/n n

Solution,

<* + <

it

decimal

number

obvious
sequence
bounded.

is

It

i.e.

successive

of the irrational

approximations

1;

1.4;

j/~2;

(a)

prove that

'

is

is

the

<

that
x2
xx
increasing.

<x <
3

It

< xn <

now remains

to

xt

Testing Sequences for Convergence

1.8.

53

= V 2 + x n _ n = 2 3,
Sinc e x = V~2<2
<V2 + 2 = 2 x, = V~2+r x <V2 + 2 = 2, .... Let it
that x n _ <2. Then x n =
2 + x n _ < V2 + 2 = 2. Thus,

We have
= V2 + x

xn

l9

be proved
]/
1
1
with the aid of mathematical induction we have proved that x n
2,
i.e. the
sequence is bounded. Hence, it has a finite limit. Let us

<

find

it.

Denote
lim x n
n

Then, x a

= V~2 Jr x n-v

raising

we can

Passing to the limit,


lim x\
n

The roots

lim (2
n

-* oo

= y.

-> oo

power,

second

the

to

rewrite this equality as follows

+ ,^),

or y 2

= 2 + y.

-* cc

obtained quadratic equation

of the

ft

The negative root does not

= 2;

y2

suit here, since

are:

1-

x>0. Hence, lim xn =y


n

(c)

We

sequences

why

have ny

j#

< E (ny) ^ ny
1

and

-^-j

we obtain

yl^tL^^y.

or

{y} converge,

their

= 2.

-* oo

Bui the

being y, that

limit

is

lim x n =

This sequence is non-decreasing, since each following term


obtained from the preceding one x n by adding one more
significant digit to the decimal fraction. The sequence is bounded
above, say, by the number 1.5. Hence, the sequence converges, its
(d)

x n+1

is

limit being
(e)

2.

The sequence decreases monotonically. Indeed,


(fl-f-l)!

Xn+1

Since

(n+\)*

<

Then, since xn

X + 1

l
>

>

_
~~

0, the

_n\

n\

nn

~~nP

<X

'

nn

__

(n+l)

~~~

(n+\) n Xn

'

sequence

bounded below, hence

is

lim x n
n

exists.

Let us denote

it

/.

Obviously,

/=

lim xn
n

show that

= 0.
nn

^0. Now

-+ cd

Indeed,

n J

-+ cc

let

us

54

Ch.

Hence,

^n

I.

<

Introduction to Mathematical Analysis

<-^x n

and xn+l

-y

Passing over to the limit,

we obtain

/^0,

which, together with

brings us to the conclusion:


/

1.8.8.
ral

Find the limits

= 0.
sequences with the following gene-

of the

terms:
n
zn
1

r
-

V n*+\
Solution.

]fn*+2

n
-

*-...+

lim#=l.

Let us prove that


n

l*-l|

Indeed,

-* oo

n Vn + n
2

=
}Tn?

+n

V n 2 + n(n+ V n 2 + n)

We

Vn* + n

2n'

can prove similarly that


lim z n

1.

Then,

= 2M

+
On

the other hand,


l

VrP + n

V~n*

+n

Vri*

+n

}^n 2

+n

=x

ri

Thus,

x n<y n

<

Urn *

z n,

-* cc

lim z
n

-* oo

and according to the theorem on passing to the limit


lim
1.8.9.

Using the theorem on passing to the limit

prove
lim

in inequalities

yn =\.

JX'a=l (a>0).

in inequalities

The Limit

1.9.

Prove the existence


and calculate it.

>

1)

55

sequence y n

of the limit of the

1.8.10.
(a

Function

of a

1.8.11. Taking advantage of the theorem


monotonic sequence, prove the existence of

= a 1/2n

on the limit
a

of

finite limit of the

sequence
Xn

22

Taking advantage

1.8.12.
in

"1"

inequalities,

32

/I

the theorem on passing to the limit

of

prove that
lim x n
n

=\

xn

if

= 2n{Vn +ln).
2

-* qo

Prove that the sequence

1.8.13.

x1

= Va\

xa

=----j/

xs

= V<* + V a + Va;

xn

--=

+ V a\
]f a + Va+

+ Va

n radicals

(a>0)
has the limit b
1.8.14.

= {VAa+\ +

l)/2.

Prove that the sequence with the general term

*n

~3+l

l_

+ 3 +2 +
2

'

'

'

^~?> n

+n

has a finite limit.


1.8.15. Prove that a sequence of lengths of perimeters of regular
2 w -gons inscribed in a circle tends to a limit (called the length of
circumference).

1.9.

The Limit of a Function

point a on the real axis is called the limit point of a set X


any neighbourhood of the point a contains points belonging to X
which are different from a (a may be either a proper or an improif

per point).

Let

the

point a

be the limit point of the domain of definition


The number A is called the limit of the

of the function f(x).


function f (x) as xa,

A=

lim
x

-*

/(.*:),

if

for

any neighbourhood V

the number A there exists a neighbourhood u of the number a


such that for all
lying in a, f(x)V (the definition of the
limit of a function after Cauchy). The number A may be either
finite or infinite. In particular, if the numbers A and a are finite
we obtain the following definition.

of

xX

Ch.

56

A number A
A=

lim
x

-+

f (x),

Introduction to Mathematical Analysis

I.

any

for

if

the

limit of

>

there exists a

called

is

f(x) as

function

number

8(e)

x-+a

>

such

0<\x a|<8

and belonthat for all x satisfying the inequality


ging to the domain of definition of the function f (x) the inequality
holds true (the "e-8 definition").
\f(x)
the definition is as follows. A number A is called
If
lim f (x), if for any
the limit of a function f (x) as x
>, A

A\<s
a=+oo,

-*

00

>

>

there exists a number M(e)


such that for all x satisfying
M(e) and belonging to the domain of definition
the inequality
A\
holds true (the
of the function f (x) the inequality \f(x)
e

"e-M definition").
The notation lim
x

-+

/ (x)

= 00

means

that

<e

lim
x

-*

f (x)

= + 00.

The

considered similarly.
of a function after Heine. The notafor any sequence of values of x con-

rest of the cases are

The definition of the limit


tion limf(x) = A means that
x

-+

verging to the number a


-^2>

^l>

(belonging to the domain of definition of the function and differing


a) the corresponding sequence of values of y

from

0i

= /(*i);

has a limit, which


1.9.1.

Heine

Taking

(i.e.

is

0i

= /(*i);

advantage

of

the

definition

and

of the

the

of

limit

after

theorems on the limits

prove that

3*+l

,.

Solution. Let us consider any


the following two conditions: (1)
to the domain of definition of the
(i.e. x n =
4/5); (2) the sequence
2.
lim x n
i.e.

sequence x l9 x 2 ... satisfying


the numbers x lt x2y ... belong
function f (x) =(3x+ l)/(5*
4)
{x n } converges to the number 2,
,

-+ 00

To the sequence
of

number A.

the

in terms of sequences)

of sequences,

= /(*n).

0n

corresponds the

{x n } there

sequence

of

the function

3^+1
5*1

+4

3;c a

+l

5* 2

+4

'

'

proceeding from the theorem on the limits


l

im

fi x

T.'

)= Hm

3*+l_ lim

'

1.7),

0^l) a ji a l

ii.5^ + 4- lim (5* + 4)

10

+ 4-

values

The Limit

1.9.

Fund ion

of

57

Thus, independently of the choice of a sequence {xn \ which


converges to the number 2(x n ^=
4/5), the corresponding sequences
of values of the function f (x n ) converge to the number 1/2, which,
according to the definition of the limit of a function, means that

3x+l

5^+4=
The

Heine

conveniently
no limit.
For this it is sufficient to show that there exist two sequences {xn }
lim x'n = a, but the corresponding
and {xn } such that lim xn
Note.
applied

definition

the

of

when we have

limit

after

to prove that a function

is

(x) has

sequences {f(x'n )\ and

qo

-*

oo

do not have identical limits.

{f (x)\

Prove that the following limits do not

1.9.2.

^m
3
737;
Y
1 C )X

lim sin

(a)

-*

lim sin x.

(c)

K *

-+

exist:

CO

Solution, (a) Choose two sequences

xn

xn =

and

nn

j-t
+ (4n+l)ji
2

Ta

(n
v

2,

),
/

which

for

lim x n
n

-+

lim
n

<x>

The corresponding sequences

of

a:^

1.

-* oo

values of the function are:

Kx n = s\n + l/{ nn) _ =smnn =


l

and
f

) - sin + g/[(4w ^
{

n]

t)

_ = sin f+i = sin


ji

2/m +

f) =

Hence,
lim

1. e.

it

the

(x n )

and

sequences {/(*)} and

follows

lim sin

that
x

(c)

lim

f (x'n )

{/)} have
does not

different

exist.

Choose two sequences, x n


nn and x'n
which lim jc n = lim a^ = oo. Since
n

00

lim

= 2nn + n/2

-*

cc

sinx=

lim sin

jt/z

= 0,

and
lim
-*

whence

-*

2, ...), for

limits,

sin<=

cp

lim sin * does not exist.

lim sin (2jw


n

-* 00

+ ji/2) =

1,

(n

58

Ch.

Introduction to Mathematical Analysis

Note. The above examples show that one cannot draw the conclusion about the existence of the limit of a function proceeding
from the sequence of values of x of a particular form (for example,
proceeding from x n
1 +2/((4n+ 1) n)
in the item (a) of this problem), but it is necessary to consider an arbitrary sequence x 19

x 2) ...,#, ...

limit.

Proceeding from the definition of the limit of a function


u
(i.e. in
the terms of e-5"; "e-M", etc.), prove that

1.9.3.

Cauchy

after

Km (3x 8)- 5;

(a)

(e)

having a given

lim arc tan x

sin* =1/2.

lim

(f)

-*

= ji/2;

Jt/6

Solution, (a)
that for any e

According to the "e-8" definition we are to prove


there exists 8 >
such that from the inequality

>

I* 1|<8

<

it follows that
5
e.
(
5)
f (x)
f (x)
In other words, it is necessary to solve the inequality
|

|3a:

8 + 5| = 3|a: <
1

e.

The
is

latter inequality

fulfilled

shows that the required inequality /(jt)-f5 |<e


< e/3 = 6. Hence, lim (3x 8) = 5.
1

as soon as \x

According to the "e-M" definition of


for any e >
is possible to
it
such that for all x >
the inequality
(b)

show that

the

Transforming

this inequality,

5* -j-l

+9

3x

Since x

> 0,

it

remains

we obtain
14

|3*

limit one

find a

will be fulfilled.

+ 9| <8.

to solve the inequality

number

has to

M>

1.9.

The Limit

of a

Function

59

whence

^ 149e
hence

M=

Thus,

14 7" 98
.

3e

values of x

>M

M = ^g
14

>

for

we have found
the inequality (*)

Let, for example, 8

= 0.01;

is fulfilled,

such that for

and

this

M=^ ^ ^- = 463

then

(c) We have to prove that


such that from the inequality

for

K>

any

means

all

that

-|-.

there exists 8

>

|x-l|<8
there always follows the inequality
1

(I-*) 2

(i-*) 2

Let us choose an arbitrary

number

>K.

K>

and solve the inequality

(l-*) 2

(**)

K,

whence

|l_x|<-L: (K>0).
Thus,

if

soon as \x

we put

<
1

6,

= -J=-,
VK

then the inequality (**) holds true as

which means that

lirn

__

+oo.

M>

K>

there exists
(d) We have to prove that for any
there always follows the inesuch that from the inequality x
and
Let us choose an arbitrary number
quality \og a
M, then at
consider the inequality \og a
K. If we put a K
the inequality \oga
holds true. Hence,

>M

x>K.

x>M

x>K

lim
X

1.9.4.

x>

Prove that

+ 0O

log a

*=

K>

+oo.

lim cosjc does not exist.


X

-+ 00

Using the sequences of the roots of the equations sin (\/x)=l


and sin (l/x)=
1, show that the function / (x) = sin (l/x) has no
limit as x
>0.
1.9.5.

Ch.

60

Introduction to Mathematical Analysis

from Cauchy's definition

Proceeding

1.9.6.

the

of

limit

of

function prove that:


(a)

lim

(c)

(3a:

2) =

lim

(f)

2x

,.

00

lim

V X

=2;
1

1;

1);

0.

Calculation of Limits of Functions

1.10.
I.

cosx=

lim

(d)

.'^.s+s'tJ
lim a x = +oo (a >
*-*

(g)

= 0;

sin.x:

x -* o

(e)

lim
X -

(b)

1;

and limy

\\mu{x)

the limits

If

(a;)

exist,

then the following

x-+a

theorems hold true:


(1)

lim [u (x)
x

(2)

(3)

II.

main

-+

v (x)} = lim u (x) lim v

lim [u (x)-v(x)]
x^ a

^
For
of

lim U

-*

= lim u (x)
x

-*

x -+

lim v
x

-*

(x)\

(x)

= W^)

(Hm(x)^0).

main elementary functions

all

(x);

definition the equality lim / (x)


x

-+

at

any point

= / (lim x) = f (a)

of their do-

holds true.

x-+a

III. If for all values of a; in a certain neighbourhood of a point a


a) the functions f(x) and q) (x) are equal
(except for, perhaps, x
and one of them has a limit as x approaches a, then the other one
has the same limit.
IV. The following limits are frequently used:

(1)

hm

x-+0
(2)

lim(l
x

-*

lo

lim

...

,.

(4)

x^Q

hm
x^O

(5)

lim

+ l/jc)* =alim(l +a) '* = e = 2. 71828.


^ il+x) = log a e (a>0;a^l);
l

-*

<x>

(3)

=1;

/+ =
x

In (1

a:)
'

1;

^=^ = lna

(a

> 0).

.;

1.10. Calculation
1.10.1.

ixb

(C)

(e)

i;

9x

'

J/"

2*

>

x a:+
2

*3_ x _ 6

,^_J

P and q

integers);

(f)lim^^;

2o 3/0=
r/ 3x 5
5
j/Ff6-2j/3^5

+ 3x* 9x2

m
il

>

(d)

3/-Xft
*-+22 r/
x+6

xs

r1

/u\

(b

^+^|/*Fp7

(g)

^VeZU^'
Hm

61

Find the limits:

+
W Im 3*.+-f*r+T
i

Limits of Functions

of

(h)

f\

*-3 L
*-*3|_

'

l0g

r r
v

VHT+6-3J'
K8a:+

Solution, (a) Since there exist limits of the numerator and denominator and the limit of the denominator is different from zero,
we can use the theorem on the limit of a quotient:
lim V(4* 5

+ 9* + 7 ^ ^i
^-13^ + ^+1
lim

lim 4*

(b)

(3jc

t 9x+7)
+
t
+x +
;

The above theorem cannot be

limit of the denominator equals


the numerator also equals zero

indeterminate form

A;3

-jj-

For

directly

zero

+9+7

used here, since the


Here the limit of
Hence, we have the

x-2.

as

as

^^2

3+1+1

1)

2.

we have

+ 3x2_9 ._2 _ (x2) (x + 5x + = + 5a:+


~~ (x2) (x + 2x + 3) --jc + 2*+3"
x x 6
2

1)

jc

Thus, in any domain which does not contain the point x

=2

the

functions
r,

+ 3x 9x 2
2

equal; hence, their limits are


function y(x) is found directly:

are

/vi- * + 5x+ = 15
^2- + 2*+: n

cp(^)^ im

x -.2

hence,
/

X-+2

x2

x3

X-+2

+ 3x 9x 2
2

+ 5x+l

equal.

also

hm

15
11

The

limit

of the

Ch.

62

Just

(c)

as

Introduction to Mathematical Analysis

I.

in

we remove

(b),

indeterminate

the

form

by

transforming

lim

<->-!

-*

0/2x 2 3 5x);

x(Vx +l
2

V2x^f3

S^** 3

lim

(g)

x)\

QO

,.

/f ,

00

lim

(e)

+ 3x);

V 3* 2+ 3/2x

(d) lim

X2

2^+3^/7+5^/7

v^ + oo
* -*

00

lim

(c)

3a:

(j/9je 2

lim
-*

i>

4 3x + 2

X3

(a) lim

(1 *)

Find the limits

1.10.2.

(b)

(*+0(^^3-3,) =

lim

]/~2x 2

+3

*.

-* OO

So/on.

(^-^)

(a) lim

Here we have the indeterminate form oo

oo;

us subtract the

let

fractions

lim
jc^oo

f^_^)=lim

+
\3* 2

3x

\2x 8
= lim 9

2x3+4x2

3
^ 9x

2 y

6a: 2

^.Too

Note.
ratio

We
the

of

(}^9x

,.

lim
+

(c)

In

of the
2

+i 3x) =
'

handling such

2
8/a:

'

is equal to the
the superior power of x (provided the

at

same

-J-

12/a: 2

such examples the limit

in

coefficients

polynomials are
/ux
(b)

that

see

2 + 4/x

+ 6/a:

degree).
,.

-+

n
= 0.

lim
x

cc

J^-j- 1+3*

examples

bear

in

mind

that the function

where p n (x) is a polynomial ofjdegree n, tending


n
This allows us
to infinity in the same way as the function "j/
to single out the superior power of x and divide both the numerator and denominator by this power of x. In the given example

f{x)= \/p n

(x),

the divisor

is

[/ x\ then

we

j/T

2J/T+3

obtain:

j/3x 2 + l/2x3

63

Calculation of Limits of Functions

1. 10.

x^ + oo

|/"3

+ 3/*/T+5/ff^

2/x+ /4/x 12/x + 9/x


2

Since the

(d)

sum

two positive

of

(1/2j?^3 5*)=

Urn

At

(f)

lim

- cc

X ->

>

a;

we have |/#2

we have \^x2
,

im

is

From

/Vote.

= x

this

+ 2/x)

*V~2 + 37F
*(4

_
+ 2/*) ~

'

and, hence,

= Hm

follows,

it

3 + (5*)] -

therefore

ao

y* a (2 + 3/**)
*(4

.x;,

"","

<

^*(2 + 3/*)

a:

[V2x*

X-*-oo

At

large quantities

infinitely

an infinitely large quantity, then

also

^2 + 3/^ _
*(4

]/"2~

+ 2/*)

incidentally, that lim

*x

3
\

~t

does

not exist.
(g)

lim5 2 ^* +3
*

-*

1.10.3.
(a)

(c)

2x/(* + 3)

lim
)

= 5*-

= 5 = 25.
2

fl0

oo

Find the limits:

Hm^T&l
hm ,^5 X -

(b)

_*I

H ra

(k positive in-

(d) lim-*x-+0

<

teger);
,

,.

sin

(x ji/6)

^3 2cosx'
,

x
x 3sinx+l

2 sin 2 x-f sin

2
*i/6 2sin

'

(method of substitution), (a) Let


* 1; hence
Then x=^z* 26 and 2^3 as
2

3/ ^Z_

= lim

'

Solution

lim

sin*) 2

*-jt/2

us

put

26

+x= z

3
.

- lim ^-3)(^+3*+9) _
= lim2(z + 3z + 9) = 54.
2^3
2

Ch.

64

l+x=z k

Let us put

(d)

Hence,

^T^~

lim

Introduction to Mathematical Analysis

I.

-i; m _z

X-y

2 -*

Let

(e)

us

On

"K"

2 cos

2-0

a:

x=

/"3

o 2

hm

(a)

x^O

a:-

cos x

-5

/L

tan x

(b) lim

as

()

-f ji/6)

(z

+ 2 sin (z/2) cos (z/2)

(z/2)

CQS(2/2)

yA 3

sin (2/2)

+ cos (z/2)

=1.

sin

a:

X3

x-0

cos^2)

lim

(c)

and

Find the limits:

1.10.4.
,.

1.10.1 (d)).

+ n/S

2 cos

/" 3 sin 2

0.

sin2

= lim
,

x +

as

2 sin (z/2) cos (2/2)

= lim

2+ sin 2

3 cos

Problem

ee

y^U
lim

sin z

J^~3

(S

t-jT/6 l/"3

>-o

2^1

and

ji/6--=2;
then
put x
substituting we obtain

lim

lim

x=z k

then

/vi-

Solution, (a) lim


a:

2 sin 2 (x/2)

a:

x5

*-o
tan
/ux ,.
(b) lim

cos

- sinA; = lim

lim

sin

a:

(1

l Hm

x r^'-

*-o

cos

a:)
'

sin(^/2)
x/2

==1
y
2
J

'

C0S x x *
'

*-*

sin*

= lim- cos x
1

Let
Hence,

us

(c)

put

x=

z.

x=l z

Then
Jl

TC

COS -y X

lim

cos

a;

x2

and

as

1.

\
sin

- = lim
1 X
2-0

-7- z

= lim

3T

2-

Note. For a simpler method of solving similar problems see 1.12.


1.10.5.
(a)

Find the limits:

+ l/*)

lim(l

(b) lim(l

*;

+x)

^\

x-
(c

^ .fe)*
ra

In

(e)

lim
x-+ o

(d)

lim(l+/A:) m *;

(f)

lim

e4

(\+x)
t

3*

*-*

In (a+x) In a

(g)
x

->

...

In

(0 llm

-r=r

(h) lim

tan

a:

H y

Solution, (a) lim

= Hni

\*

lim

u"

-*

In

= lim

lim

(e)

+x)

(1

-v

= J = lim

z*0

ln(x/ e)

,,-ilin,
_
e

1)

1.10.6.

x*e. On

as

substi-

1^1+^1
e

Find

(1+^

im
x

(l+l) =lim

Solution, lim

1.10.7.

= e'\

ln3'

Put jt/e l=z; then x--=e(z+l);


tuting we obtain
lni

3*

(i)

Hm

65

Calculation of Limits of Functions

1.10.

|^1

+1^*]

1.

Find the limits:

(a)

lim/l-MV

(b)

lim

-^

x 2Jr 2 X

2x 2

Solution,

V
1

-^.

\(2* +

>/(

at 1)

3a:

Denote:

(a)

fW = (l+x)/(2 + x);
Hm^(x)
lim

(p

(x)

= Hml^ = -|;
= lim

x-*\

But

at

finite

x->\

limits lim

^*

/ (x)

^~ x

^~
l

= A > 0,

= ir
1

lim

q) (.*:)

=5

the following

relation holds true:


lim

lim

[/

{x)

= ex

(p

(x) In

(x)

=e B\nA

~+ a

== j[B

Hence,
lim

/Vo/e.

out that

3 31

43

If

in

- r ""'"-"-(!)" !

=/!

handling examples of the form lim

Vunf(x)=l

and

lim

cp (jc)

= oo,

[/

(x)]v

{x)

it

turns

x-+a

then

the

following

66

Ch.

transformation

Introduction to Mathematical Analysis

I.

may

be recommended:

lim [f(x)]*<*>

lim

x^-a

= lim

[ 1

+ (/ (x)

1.10.8. Find the


(a)

lim(

(c)

lim

^r
+sin

}}*

{x)

ln(p(x)L/U)-l]
1

<*>

)]

+3
;

cot

:rix)

/U
(d) lim

(*)

[/

<*>

>}*

<*>

= e**a

limits:

(1

{1

x -a

(b)

li.n(4^)

,/,m
*;

Q)

(a=kn> with & an

(f|^~y

integer).

Solution, (a) Let us denote:

Use the formula

q) (jc)

<P(*)

= lim^=l;

lim/(x)
lim

= 8* + 3;
a

^^StT'
=

lim

(8a:

-f 3)

= oo.

(*):
8a: 2

lim

,im

+3

(*>

(*>-!]

(ttt)

/W I=|S4
[/(*)-!] = - lim
1

lira q> (x)

gf +

3)

= - 8.

Therefore
1;

1.10.9.

The function

/2xH-3\8x + 3

/(#)

is

given with the aid of the limit

/?

-> cc

Investigate this function and graph it.


Solution. Consider three cases:
2n --=
00, then
(1) \x\> 1. Since in this case lim x

(2)

|jc|<1. In

this case

lim

a,,

Jt

= 0;

therefore /(*)

= !.

(*)

1.10. Calculation

=\ at any n and therefore /


consideration can be written

In this case x 2n

jt-=l.

(3)

if

f(x)=\-l

if

M>
\x\<l

if

briefly,

or,

The

/ (x)

graph

by

increases

2%

By how many times

in

= 0.
the

= 1

x
1) (see Problem 1.5.11 (n)).
function is shown in Fig. 27.

(|

this

The population

1.10.10.

ntry

= sign

of

(jc)

under

Thus, the function


following way:

67

Limits of Functions

of

does

of a

per
it

couyear.

increase

century?

in a

Solution.

we denote

If

number

the

ini-

inhabitants of a
given country as A, then after a
year the total population will amount to
tial

of

Fig. 27

After two

100

years the population

years

will

it

reach

the

will
total

amount
of

to

A[
1

+ p-q)

After

100

I+j-q)

i-

it

will

\ 50

have increased

lim

times.

Taking

into

account

that

=e> we can approximately consider that

e.

Hence, after 100 years the population of the country


7.39 times.
this estimation is very approximate, but

increased e'1
Of course,

idea as to the order of the


/

tity

^1

(a)

i-

lim-

(b)

(c)

=7.245

Find the
1

x 2x*

z /__ x

_2

lirn^E^;
2x-

(d) lim
5a-

5x + 4
2x 3

to within three

limits:

cos x-\~A tan x


2

it

have

gives an

^the

increase in the population;

\ioo

50/

1.10.11.

will

decimal places).

quan-

Ch.

68

(e)

Find the limits:

lim

~~

tan 3 x

..

a
a "' n ,
'".,

gc-ji

(e)

0;

QO

1.10.12.

(c)

Introduction to Mathematical Analysis

lini(j/>+
X->

x);

*-*ji/4

3 tan x
*

cos (x+n/6)

lim tan 2x tan (n/4

(d)

1.10.13. Find the limits:


(a) lim (1

(c)

+ 4/x)* +3

*"* -1
(b) lim

lim^=^;

(d) lim (1

x->0

x-y

lim (tan

;c)

tan 2

hm
x

cd

lg-r\)
zx
\

-r

x) cot2

*;

lim (sin x) ian x

(h)

*;

X-yJt/2

X-+JI/2

(k)

lim

(f)

x-yn/4

(g)

+ 3 tan

*->0

22
lim (sin2*)t- *;

(e)

+0

1.10.14. Find the limits:


.

,-

nn

(a)

(c)

lim
o

arc cos

(1^)
(b)

In (1 + a sin
sin x

hm

In tan

a:
;

a:).

I. II. Infinitesimal and Infinite Functions.


Their Definition and Comparison

if

The function a(x) is called


lim a (x) =
or lima (*) = 0.
x-*a

X-y

The function
lim/(A;)

= oo

infinitesimal

or as .v-^ oo

as

CD

f (x)

or lim /

is
(a;)

called

= oo.

infinite

as

^->a

or

as

x-oo

if

1.11. Infinitesimal and Infinite Functions

69

A quantity inverse to an infinite quantity is called an infinitesimal.


Infinitesimal functions possess the following properties:
(1) The sum and the product of any definite number of infinitesimal functions as x
a are also infinitesimals as x -> a.
The product of an infinitesimal function by a bounded function
an infinitesimal.
Comparison of Infinitesimals. Let the functions a(x) and P (x) be
infinitesimal as x
a. If
a (x)
(2)

is

, .

lim q-K*-*a P to

where c

c=l,

If

the

then

a(x)

function

the

to

is

P(jc)

called

called an infinitesimal

is

of a

is

(x).

If

equivalent;

o (P (x)),
written thus: a (x)
of a /oayer order with respect

to P (*), which
an infinitesimal

/ug/ier order relative

and

from zero, then the


same order.

different

are called infinitesimals of the


functions a(x) and P (x) are called
(x)

a(x)~$(x).

= 0,

number

finite

and P

(x)

then

notation:
If

a certain

is

functions

lim R

0<|c|< + oo,

where

/f

then

the function

(x)

nth order as compared with the


is called an infinitesimal of the
function P(x). The concept of infinite functions of various orders
is introduced similarly.

Prove that the functions

1.11.1.
(

4
X
as
= 2Trrr
5
f(x) = (x l) sin

a ) f( x )

(b)

Solution, (a)

It

is

2
3

as

indeed, lim
X

>

(a:

/ (x)

the function cp(x)

l)

0.

are infinitesimals.

= lim

->

2x

~\ = 0.
r o

= (x l)

is

infinitesimal as x

1;

Secondly, the function

- sin

\J)(a:)

is

a:

to find the limit

sufficient

lim

(b) Firstly,

>

1,

bounded:
I

Hence,

the

given

bounded function
f(x)

is

sin 3

function

<

f (x)

represents

1.

by the infinitesimal
an infinitesimal function as x -+ 1.
\p(x)

the

(p(x),

product

of

the

which means that

70

Ch.

Prove that the functions

1.11.2.
(a)

Introduction to Mathematical Analysis

1.

/(*)=

(b) f (x)

x -+ 4

as

^-yi

as

>

oo

are infinitesimal.

Find

1.11.3.

lim x sin (l/x).


x

Solution.
sin (l/x) is
means that

-*

Compare

(c)

following infinitesimal functions

the

with the infinitesimal


(a)

(p(x)

= tan x (b)
U*) = K9 + x-3.
/,

(x)

Solution,
l

im

Since x is an infinitesimal as x
and the function
bounded, the product xs\n(\/x) is an infinitesimal, which
lim x sin (l/x) = 0.
*

1.11.4.

We

(a)

Hence, tanx 3
(b) We have
*

li

x
is

tan

())

a:

x,1 =
2

lim

/ 2 (x)

l
J

3
tan*
^

,.

li
lim

liin

a:

= 0.

an infinitesimal of a higher order relative to

Hence, jj/sin 2 *
with A'.
(c) We have

(asx

A;:

have

ifE^ =

x ->

-*

X2

J/

x.

3/^

an infinitesimal of a lower order as compared

is

x-> o

li

in

x-+o

Hence, the infinitesimals V~9-{-x

= ir.
6

and x are

of

same

the

order.

1.11.5. Determine the order of smallness of the quantity p with


respect to the infinitesimal a.
(a)

p=rcosa

Solution, (a)

cos2a;

= tana sina.
p^=cosa cos 2a = 2 sin -| a sin
(b)

Whence
|in]
o
a

J^2 =lim

a -

2sin(3a/2)sin a/2)
(

Hence, P is an infinitesimal
second one with respect to a.

of

the

same order

3_
2

as

a2

i.

e. of

the

1.12. Equivalent Infinitesimals

1.11.6. Assuming *
quantities:
(a)

(b)
(c)

compare the following

oo,

= 3* + 2* + 5
f(x) = 2x + 3x
/ (x) = Y x + a
2

/ (*)

and

= 2*

+ 2x

cp

(x)

and

cp

(*) = (* + 2)

an d

<p(x)=l/x.

2* 3

oo

+ 2x

The infinite function 3* 2 + 2* + 5


compared with the infinite function 2* 3 + 2*
*->

large

infinitely

Solution, (a)
as

71

is

+ 2/x 1/x
2

of a
1,

lower order

since

Prove that the infinitesimalsa=* and P=*cos(l/*)(as*^0)


their ratio has no limit.
i. e.

1.11.7.

are not comparable,

Indeed,

Solution.

lim
x

it!),

X cos

1 x\
-L-L=
(
y

->

which means that

these

lim cos (l/x) does not exist (prove


x

->

infinitesimal

functions are not com-

parable.
1.11.8.

If

0,

higher
same order as *?
(are)

(a)

of

100*;

*2

(b)

then which of the following infinitesimals is


order than x\ of a lower order than x; of the

(c)

6 sin*;

sin 3 *;

(d)

(e)

j/tan 3

*.

1.11.9. Let x
O. Determine the orders of the following
tesimal functions with respect to x:
(a)

2 sin 4

(c)

Vl

(e)

a;* 5

+x

1;

2cos(x+);

(g)

TZTf;

(i)

cos*

(b)

[/sin 2 *

(d)

sin 2*

(f)

l/cosx;

4
;

2 sin*;

2 |/sin*;

h ) tan*

(j)

+*

infini-

+*

2
;

cos*.

1.11.10. Assuming the side of a cube to be an infinitesimal, dethe order of smallness of the diagonal of the cube (d), of

termine

the area of

surface (S); of

its

its

volume

(V).

1.12. Equivalent Infinitesimals.


Application to Finding Limits

If

a (x)

the functions

~y

x )> P

lim -77t4-=

Hm

(*)

(*)

and P (*) are infinitesimal


then

~ 6 (*),

as

and

-^y- (replacing an infinitesimal by an equivalent

if

one).

Ch.

72

Introduction to Mathematical Analysis

I.

If

lim f(x)
x

=k

<

<

|*|

oo,

-*

then

/ (x)

~ ka (x).

(x)

If

a.(x)~y(x)

P(*)~Y(*),
then

~ P (x).

(x)

For two infinitesimal functions to be equivalent it


and sufficient that their difference be an infinitesimal
order as compared with each of the two.
Listed below are infinitesimal functions:
(a(x)

~ a (x);

(1)

sin

(3)

1 cosa(x) ~

(4)

arc sin

tan

(2)

(6)

>

(8)

l
>

(c)

~ a (x);

as

~YTTi~'*

(7)

a{x}

(x)

~ a (x);

~ a (x) \na
1

~a
1

arc tan

(5)

- Pa(x),

Prove that

(x);

in particular, (V

l+aW-l- a^.

x-^0
(b)

x;

1_

TT7~*

sin

Solution, (a)

By formula

(8) at
1

X
(c)

(x)

By formula

(1)

P=l/2 we

y
whence

1.12.2.

+ Vx =X
3

y xV x ~y

sin

X.

we have
'

have

o/T+^-D^i.

sin]/ xV~x '~y x)/"x

Replace each

'*V

'

+X

=x
1/2

<\

~X

'\

x*+yx*of

the following infinitesimals with an equi-

valent one:
(a)

necessary
a higher

[a(Jc)] 2 /2;

particular, e* U)

[l+a(x)] p

1.12.1.
a

in

0),

of

x+0)

an infinitesimal as

a (x) ~ a (x)\
ln[l+a(x)] ~a(x);

(a

<

(x)

is

is

3sina

5a

3
;

(b) (1

cosa) +
2

16a 8

+ 5a

-f

6a 5

73

1.12. Equivalent Infinitesimals

Solution,

Note

(a)

sum

the

that

of

two infinitesimals a and

|3

equivalent to the summand of the lower order,


since the replacement of an infinitesimal with one equivalent to it
is tantamount to the rejection of an infinitesimal of a higher order.
In our example the quantity 3 sin a has the order of smallness 1,
( 5a 3 ) the order of smallness 3, hence
of different orders

is

3 sin
(b)

(1 cosa)

a+

With the

1.12.3.

a)

llm

r
hm

,.

hm

(e)
.pv

/i\

cos

In
4

|
a:

-f- a:

~~ cos 2 x )*-r xb

* In

(1

+3*)
,

|/*

(*

/ *

cos *+2sin x sin x * + 3*


tan * 6 sin- x
* 5*
3

'

16a 3

X
sin j/

^(arctan
'

+
+
+
sm x ~ tan ^ ~^

j.

7 tan 7 x-\- sin 6 x-j-2 sin 5 x

lim

(h)

* arc tan 2 *

+ 5a + 6a

16a 3

a;

V^" 1 -f-

hm
1;

(d)

*
*
3 sin*
|.
]" n tan* 2sin 2 * 5* 4

Hm

cos

,_ cos

,,v

a:

(g)

i-

'

2*4- arc sin 2


3-

sin

+ 5a + 6a ~

16a 3

l"n

(b)

1" 0+4*)'
.0 mna-.,^

(c)

-|

aid of the principle of substitution of equivathe limits:

sin 5*

I-

the lower order, therefore

(1 cosa) 2 +
lent quantities find

of

is

+ 5a + 6a = 4siii

16a 3

The summand 16a 3

5a ~ 3 sin a ~ 3a.
3

Solution,

(a)

We

-}-

4x) 4x (see the


have sin5A:^5x; In (^1
page 72). Therefore

list

infinitesimals on

of equivalent

5*
sin 5*
5
=
JlF(T+W , i o-4F = T*2JL- = lim ln " +
S*-' =
,.

Il ,1

(c)

)1

lim

= 4 hm

a:

(d)

From
V

the

list

of

7i

= 4 hm
I

->

+x + x 1

(x

-\-

)/2

~ x/2,

,.

x =

x -> o

equivalent infinitesimals

cos*

we

sin \x

find:

\x.

/2

2.

Ch.

74

I.

introduction to Mathematical

Analysis

Therefore


vT+7+x
r-j
5

lllTl

the list
(e) Using
page 72 we obtain
sin 2x

of

r
lUTl

x/2
=
I

equivalent infinitesimal functions given on

+ arc sin

arc tan x ~ sin 2x ~ 2x.


2

Hence,
sin 2x-\- arc sin 2 x

,.

hm

<h)sin>/7~ Y~x\

+ 3x) -

In (1

'

(arc tan

Find

the

Ilrn

35

^* 5
1

sin^ln(l+3,_)

lim

2x

= t2

3x;
5

K7;

arc tan

1.12.4.

arc tan

Yx

J^jc;

^.3,_3

V~xY
approximate

values

of

the

roots

(/

1.02 and

1^0.994. Estimate the absolute error.


Solution. Use the approximate formula

~l+x/2

]/"l+x
(for

(*)

x sufficiently close to zero). In our case

+ 0.02- 1+^= 1.01;


= 0.997.
V 10.006 ~
J/1

To estimate the

we note

error

that

|_(Kr+7- 1) =! (*- 2 ^1+^+ 2) =


= j(*+i- 2 K^+i) = l(^^
Hence, the absolute error

mated by
Using

the
this

quantity
estimate

1.01
j/0994 ^ 0.997
1^1762

(a)

approximate formula

(*)

is

esti-

y.
we

is

1.12.5. Prove

of the

X2

amounts

= 0.00005, and the absolute


to -^^ 0.000005.
o

that, as x

f/T+xl~jx;

that the absolute error of the root

find

0,

error

of


(b)

arc tan

(c)

cos

~ sin

.x;~

x.

determine the order of smallness, relative


$(x)
x, of the following infinitesimals:

infinitesimal

Vs\n 2 x + x*;

(a)

x '^

x+2

For

^ 2*

determine the order of smallness, relative to


of the following infinitesimals:

$(x)=x 2,

the infinitesimal

2) + 2(jk
2

3(x

(a)

Making

1.12.8.

(b)
1

1.12.7.

75

x-^0

For

1.12.6.
to the

One-Sided Limits

~ mx\

mx
3

13.

J.

use

the

of

j/sin~^.

(b)

4);

method

replacing an infinitesimal

of

with an equivalent one, find the following limits:

<

sin3x

,.

>

/vi-

(1

+ tan 2x)

In (2

cos 2x)

In

"i

1.13.

In

d>

7jc)

'

approximate

4a:)

aTclI^r
V\-\-

+ sin

arc tan 3x

/fx

(1

"J"

sin

3x

0+ tan 2.)
/T+^-l

'

in

1-cosx

value of the root

3,

j/ 1042.

One-Sided Limits

A number A

>x

as x

<

'

(1+2.-3^ + 4*3)

Find an

1.12.9.

In (1

,.

e 'i"3*_l
In

ln*(8in3x+l)

/U

()

called the limit to the right of the function f (x)


lim / (#)
if
for any
there exists
0))
/ (x

is

(A=
x

-*

8>0

* +

such that for all x satisfying the inequality


6 (e) >
< x x Q < 6 (e)
and belonging to the domain of definition of the function f (x) the

4
inequality \f(x)
function / (*
0) as
then we write simply

and

<e

>

The limit to the left of the


defined in a similar way. If *
0,
-
or
and, respectively, /(
0)

holds

x* +

true.

=
+

is

f( 0).

1.13.1.

Find the one-sided limits of the functions:


|

(a)

/(*)={
{

b)

/ (*)

2x + 3
q
3x 5

= 77=777

if

as

1,
.

if

>

asx->l;

Ch.

76

(c)

f(x)=

(d) f(*)

r{

-r

= 3+

(f)

s2x

0;

as x

as

'

(e)

Introduction to Mathematical Analysis

I.

+ 7l /(] _

f (x)

=cos(n/x)

f(x)

=5/(x-2)* asx-+2.

Solution,

(a)

as

1;

0;

Letx<l. Then / (x) =

lim f(x)
*- -0

+ 3.

2x
\

If

#>1,

then

=2

lim f(x)
x - 1+0
to the right (see Fig.

/(*)

= 3x 5;

f (x)

f(l+0)=

(c)

f(l0)

Hence,

the limit to the

is

1^1-cos

is

left.

hence,

the

limit

28).

Y'l sin 2 #

2*

V2

sin x|

but
sin x

0)= Hm /(*)=

Hm

+ 0)=

AT-

(d)

less

than

lim 7'/<i-*>
*

-> 1

as
Further,
x
lim 7i/< 1 -*>=0,
+0
x
-*

/ (*)

if

sin

a:,

if

< < n/2


-ji/2 < x < 0.
a:

|/2

Urn

+0

The expression

remaining

a:,

Hence,

Fig. 28

f(

sin

(1/2-

lim

*-> +

1/(1 a:) tends

+oo, when

to

x tends to

1,

therefore

1,

=+

lim

oo,
X

^1+0

we

y)

= 0,

(1-0)^3.

'

have

a:)

1/(1

oo.

Therefore

" ,+0 =

(e)

! t1.(

3+

H^')" 3+, =

Let us choose two sequences, {xn \ and

{a:;},

respectively.

and X

"-2^TT

(n=l,

with the general

terms
Tn

2, ...)

J. 14.

Then lim x n

Continuity

lim x'n

and

(#)

lim cos

limf

(*;)

limcos(2n+ l)f = 0.

lim

77

a Function. Points of Discontinuity

of

2nn

1;

Hence, the function /(*) has no limit to the right at the point 0;
taking into account that f (x) is an even function, we conclude that
it has no limit to the left either (see Fig. 29).

^^^^

ii

-/I

Fig. 29

^1,

Prove that, as x

1.13.2.

'

W"
j

has a limit to the

Find

1.13.3.

the

+2

at

<x<3

equal to 2 and a limit to the right equal to

one-sided

limits

5.

of the following functions as

0:

(b)
i

left

3x

the function

(c)

= ei>*;
sin x
-L.
^(^) = i- r
f{x)

Continuity of a Function.
Points of Discontinuity and Their Classification

Let the function y


f(x) be defined on the set X and let the
be the limit point of this set. The function f (x) is said
point x
to be continuous at the point x if lim f (x)
f (x ). The latter con-

dition

is

Ax -

-/(*.)]

&y(x

equivalent to the condition lim

<>.

)=^ lim [/(x


a*

-*

+ Ax)

Ch.

78

The function

Introduction to Mathematical Analysis

f (x)

continuous

is

point x

the

at

if
()

and only

if

0) = f(x o ).
f{x o
f(x o -0)
The function f (x) is continuous on the set X if it is continuous
at every point of this set.
Points of Discontinuity of the First Kind. Let the point x be the

(]

point of the domain of definition X of the function f(x).


The point x is called a discontinuity of the first kind of the function f (x) if there exist the limits to the right and to the left and
limit

they are finite. If f (x u


then x u is called
0) = / (x -\-0)
f (x ),
a removable discontinuity. Further, if f (x
0), then x u
0)
f (#
is a non-removable discontinuity of the first kind, and the d-ifference
0)
0) is called a jump discontinuity of the function
f(x
/(*
{)

at the point

f (x)

{)

x, r

Points of Discontinuity of the Second Kind. If at least one of


the limits of f (x
0) and f(x
0) is non-existent and infinite,
then point x is called a discontinuity of the second kind of the function f(x).

Using only the definition prove discontinuity


+ 5r + 2jc 2 + 3jc + 4 at any x.
be an arbitrary point on the number

1.14.1.

ction f(x) - 3jc 4


Solution. Let x
find lim f (x):

Urn

f (x)

Urn (3x 4

the fun-

scale. First

+ 5x + 2x + 3x + 4) = 3jcJ + 5* + 24 +
2

Then compute the value


=/ (*)

Comparing the

of

3a

of
4

the function at the point

a'

3.v

+ 4.

+ 5x + 2x1 + 3x + 4.
{)

results thus obtained,

lin f(x )^=f(x


()

we

see that

).
{

Hence, the function f (x) is continuous at the point x by definition.


Since ,v is an arbitrary point on the number scale, we have proved
continuity of the function for all values of x.
1.14.2.

Given the functions:


for

(a)

f(x)

(b) fix)

(c) / (*)

2x 3

oo < x ^
<

for

for

for

x<3,

for

<

3,

^ x < oo;
>

3;

1,

1.14.

Continuity of a Function. Points of Discontinuity

79

Find the points of discontinuity (if any). Determine the jump


discontinuities of the functions at the points of discontinuity of the
first kind.
Solution, (a) The domain of definition of the function is the entire
number scale ( oo, oo). In the open intervals ( 00, 1), (1, 3),
(3, 00) the function is continuous. Therefore discontinuities are possible only at the points x=l, x
3, at which analytic representa-

tion of the function is changed.


Let us find the one-sided limits of the function at the point

lim l(2* a
-0 b

/(1_0)=

x-+

+ 3)--=

lim (6
+0

5*)=1.

x=

The value

of the function at the point


analytic representation, i. e. /(l)
(2

/<l-0)

is

determined by the
1. Since

x=l.

is continuous at the point


Consider the point x
3\

0)=

(6 5x) =

lim

3-0

x->

/(3

+ 3)/5 =

= /(l+0)=/(l),

the function

/(3

1;

/(l+0)=

first

1:

+ 0)==

lim

(x

3)

9;

= 0.

*-* 3 +

We

see that the right-hand

and the left-hand

though

limits,

finite,

each other, therefore the function has a discontinuity of the first kind at the point x = 3.
The |ump of the function at the point of discontinuity is
are not equal

f(3

to

+ 0)-f(3-0)=0-(-9) = 9.

The function is
number scale., except
(c)

>

3/2 and

defined
at the

2x- 3 <

for

and continuous throughout the entire


point

<

#=3/2. Since 2x

3>0

for

3/2,

>

at

at

*<3/2.

3/2,

Hence,
/

+ 0) = / (3/2-0) = -1.
point x = 3/2 the function has

(3/2

Therefore, at the
tinuity of the first kind.
/

(3/2

+ 0) / (3/2 0)

1.14.3. Test
(

(a)

l(x)=\
[

The jump
equal

is

to

=
(

the following functions for


sin x

a
1

furx=-0;

function

the

of

1)

finite

at

2.

continuity:

this

disconpoint

80

Ch.

J.

Introduction to Mathematical Analysis

= sin(l/x);
x sin (\/x)
/(*) =

(b)

/(*)

(C)

4-3*

for

0;

(e)

<

for

=
2a + x
for
/(x) = arctan(l/*); (f)

(d)

=^= 0,

for

0,

(x)

0;

= (** +

/ (x)

Solution, (a) The function


we have
the point x

l)/(*

continuous

is

at

1).

points ,v=^Q.

all

At

/(0)=1;
Hence,

means

at

that

this point
it

is

liin

the function

continuous

for

sin x

liin

1.

is

continuous

values of

all

as

well,

which

x.

Fig. 30

x^0. There
is defined and continuous for all
(cf.
Problem 1.13.1
limits at the point x
the function suffers a disconti(e)). Therefore, at the point x
nuity of the second kind (see Fig. 30).
2a; the equality / (0)
(d) / (0)
0)
4, and / (+0)
/ (0)
f (
will be fulfilled, i. e. the function f (x) will be continuous at the
(b)

3re

The function

no one-sided

point x
(f)

if

we put 2a =

4,

(_i_0) = /( 1+0)=

= 2.

x->

sided limits are finite and

(x 2

lirn

*+l) = 3,

+ both

i.e.

coincide.

But

at

the

point

one1

the

1.14.

Continuity

of

a Function. Points of Discontinuity

81

function is not defined and, therefore, is not continuous. The graph


1
x
with the point
function is the parabola y
x2
( 1, 3)
1)=^3, then it
removed. If we redefine the function putting / (
1
the function has a remowill become continuous. Thus, at x==
vable discontinuity.

of the

the following functions for

continuity:
should be borne in mind that the function
E(x) is defined as the maximum integer n contained in the number x, i. e. as a number satisfying the inequality n^x.
1.14.4. Test
/ (x)

(a)

= E (x).

It

(b)
|

X(x)

is

if

a;

is

if

is

rational,
irrational.

called the Dirichlet function. For instance, X (0)

X (|/""2)

= 0;

*(jx)

h( 1/2) =

1;

1;

etc.

Solution, (a) The function E (x) is defined throughout the entire


scale and takes on only integral values. This function is
discontinuous at every integral value n of the independent va-

number
riable,

n
(

E (n 0) = n

since

+ 0) = n

1;

(see Fig. 31).

9>

Let us choose an arbitrary


point x on the x-axis; two cases
are possible: (1) the number x
is rational;
(2) the number x is
(b)

3
2
i

-3

irrational.

case X(x )=l. In


any vicinity of a rational point
there are irrational points, where
0. Hence, in any vicinity
X(x)
of x there are points x for which
In the

r**>

first

-2

-1
!

i
i

12

'

H- -1

fJ

-z

i-*J

-3

|A*/|HM*o)-M*)Hi-

Fig. 31

0.
X (x )
the second case
In any vicinity of an irrational point there are rational points
at which X(x)=l. Hence, it is possible to find the values of x for

In

which
\Ay\

= \X(x

)-X(x)\

1.

both cases the difference A*/ does not tend to zero as


Therefore, x is a discontinuity. Since x is an arbitraryX(x) is discontinuous at each point.
point, the Dirichlet function
The graph of this function consists of a set of points with irrational abscissas on the x-axis and of a set of points with rational
that is why it is imposline y
1,
abscissas on the straight
Thus,

in

Ax>0.

sible to sketch

it.

Ch.

82

Introduction to Mathematical Analysis

I.

1.14.5. Using the


6", test
the

of "e

(a)

f(x)

(b)

following

= ax + b

(*)

function in terms
continuity:

definition of continuity of a

functions

for

(a^=0);

x2

if

is

x2

jj

rational,

rra tional

8"
Solution, (a) Choose an arbitrary point x
According to the "e
definition it is necessary to show that for any preassigned, arbitrait
is possible to find a number
rily small number
such
x
6 the inequality / (x)
e holds true.
that at a:
f (x )
Consider the absolute value of the difference
.

f(x)

Let

us

fulfilled

e>0

(x

for

\a

that \f(x)
/(* )|< e This
x satisfying the inequality

\x

x <
\

e or

a:

x < e/|a|

(b)

|<8

a:

is

Choose an arbitrary point x

irrational

the inequality

proved

thus

to x

then

numbers tending

lim

{x n \

If

f(x n )

is

= xl.

sequence

If

to x 0y then lim f(x'n )

{a:^}

any

of rational

= x%.

At x =Q

that x 2

<e

value

of

= \tf-0\=.x*.

\f(x)-f{0)\
obvious

discon-

is

is

for

a sequence

is

the indicated limits are different and hence the function


tinuous at all points x=?^0.
On the other hand, let now x 0. Find the absolute
the difference \f(x)
f(0)\:

It

be

will

numbers tending
of

(a^=0).

=x

requirement

Hence, if we take 8^e/|a|, then at \x


/(*o)l< e is fulfilled. Continuity
\f( x )
point x

<

+ b) (ax + b)\ = \ax + b ax b\ = \a\\ xx

(ax

all

require

S>0

<

<j/"e.

|x|

at

If

e>0

is

given, then,

2
putting 6<j/e and \x
0| = |x]< 6, we obtain A/(0) = x < e.
the function is continuous. And so, the
Hence, ?t the point x
is the only point at which the function is continuous. Note
point x =
that the function under consideration can be expressed through the
x 2 [2k (x) 1]
Dirichlet function (see Problem 1.14.4 (b)): / (x)
|

1.14.6.

Determine

which

functions have at the point x


.

(a

(x)

x+2

<

for

x<2

for

x>2;
o

discontinuity

of

the

following

= arc tan ^-5 x = 5;


/(A') = tanx; x = n/2'

(b) / (x)
(d)

kind
x

= 2;
(c)

f (x)

= j^H*

= 0;

Continuity of a Function. Points of Discontinuity

1.14.

83

= Vx E{V

x)\ x -----n\ where n is a natural number.


f(x)
Solution, (a) Find the one-sided limits at the point x Q -=2\
(e)

f(2

0)=

lim (*
X-+ 2

+ 0) = X*lim+

f(2

(x 2

+ 2) = 4;
1) = 3.

Here the limits to the right and to the left exist, are finite but
do not coincide, therefore the function has a discontinuity of the
first kind at the point x = 2.

E {V

The function

(e)

every

point

1.14.4

=n

has discontinuities of the first kind at


is
a natural number (see Problem

x)

where n

whereas the function |/ x is__continuous

(a)),

Therefore the function f(x)


the first kind at the points

= V x E (V x)
1,

4,

9,

n2

has
,

Test the following functions for continuity

(a)

ex

of

'-

X
/ ex

(b) /(*)

x^O.

...

1.14.7.

f(x)

at all

discontinuities

<
I

for

= 0;

K/Mor^O,
(d)
(f)

/(*)--

for

= 0;

(smxr;

lim

(e)

f(*)

= ii|l

= E(x) + E(-x).

f(x)

1.14.8. For each of

the

following

functions find the points of


of the function at these

and determine the jumps

discontinuity
points:
(a)

/W = F=^T/W = * + r^!|;

(b)

(d)

f(x)=\

_
2

for
.

for

1,

X>L

Redefine the following functions at the point x

1.14.9.

make them continuous:

to
/

(a)

/(*)

tan x
= _;

so as

84

Ch.

(b) f(x)

(c)

f(x)

Introduction to Mathematical Analysis

I.

= 5x2 2x 3 *
= v~n~x
X
sin'2 x

id) f(x)

= cos x

'

Arithmetical Operations on Continuous


Functions. Continuity of a Composite Function

1.15.

If the functions
then the functions

(1)

(x)

and g(x) are continuous

f{x)g(x);

(2)

f(x).g(x)\

at the point x

=x

0f

l^l(g(x )^0)

(3)

are also continuous at this point.


If the function u
(p(x) is continuous at the point x
x and
the function y
tp(x ) then
f(u) is continuous at the point u Q
the composite function y
x
f [<p(x)] is continuous at the point x

1.15.1.

Test the following functions for continuity:


2x b
(a)

f(x)-.

'

x4

+ 4x

3 sin 3

(b) f(x)-

(c)

f(x)

8x

#+

4 cos

=x

+8a; 2

+ll

+ +4
8a:

cos 2 x-\a:

1
#

cos x^r x 2 sin *

cos

/sin x)

Solution, (a) A function representing a ratio of two continuous


functions (polynomials in this case) is discontinuous only at points
for which the denominator becomes zero. But in our case

+ 4x + 8x + 8x + 4 = (x + 2x + 2)
+ 2x + 2 = (x + + >0 at any x, the
3

x*

2
1
and since x 2
1)
never becomes zero. Hence, the function

f (x)

is

denominator
continuous through-

out the entire number scale.


(b) The function f(x) suffers discontinuities only at points for
which the denominator equals zero, i.e. at points which are the
roots of the equation

4cos#

2 =

or

cosa:=1/2,

whence
x

= x n = n/3 + 2nn

Thus, the function


point x n

f (x)

is

(n

= 0,

continuous

+2,

).

everywhere, except at the

Operations on Continuous Functions

1.15.

85

(c) Just as in the preceding example, the numerator is continuous throughout the entire number scale. As far as the denominator
is concerned, according to the theorem on continuity of a composite
function, it is continuous at points where the function u=l/s\nx
since the function cos u is continuous everywhere.
is continuous,
Hence, the denominator is continuous everywhere, except at the
points x
kn (k an integer). Besides, we must exclude the points
which cos(l/sinx)
i.e.
the points at which l/sinx
at
0,
(2p+\)n/2 (pan integer), or sin x 2/[(2p
Thus, the
1) n].
function f (x) is continuous everywhere except at the points x = kn

and x

1.15.2.

arc sin

1)"

^n

+ nn

(& P

= 0

2,

...

).

Test the following composite functions for continuity:

= cosx n

(a)

(b)

y=^ cos log

(c)

=V

1/2

where n

number;

a natural

is

x\

cos

x.

have a composite function y = cosu, where


= cosu is continuous at any point u, and
the function u = x n is continuous at any value of x. Therefore, the
function y = cosx n is continuous throughout the entire number scale.
2
Here y =
(c)
where u = cosx. The function Vl/2 u 2
1/2
Solution,

= xn

We

(a)

The function y

and continuous on the interval


J/ 2/2, ]/~2/2|, the
function u = cosx is continuous throughout the entire number scale.
is

defined

Therefore, the function y


of x for which

= Vl/2 cos
n/4

|cos*|<l/

i.e.

2/2,

5n/4

is

continuous

= -TTT-

(b)

= u\

where

where u

y= j-j-^-^
l

(c)

Solution,

(a)

= 7
x for
{

where w

0,

x+l

fof

X<Q

= tan*.

The function
,

all

values

+ 2nn<^x<^3ji/4 + 2nn,
+ 2 nn^x<^7n/4 + 2nn.

1.15.3. For each of the following functions find


discontinuity and determine their character:
(a)

at

the points of

Ch.

86

Introduction to Mathematical Analysis

I.

x=\. The

suffers a

discontinuity at the point

suffers a

discontinuity at points where u 2

and w 2 =l. Using these values


of x by solving the equations:

whence x =1/2 and x =

of

w,

function

+ u 2 = 0,

i.e.

ul

=2

find the corresponding values

2.

Hence, the composite function is discontinuous at three points:


Xj--=l/2, x 2 =l, x 3 = 2. Let us find out the character of discontinuities at these points.

Jim y
X

therefore x 2

=l

lim y
x

-*

= 0,

lim y
U

-*

cc

removable discontinuity.

is

-y

lim
u

\/2

hence, the points


kind.

-+

= oo;

lim
x

1/2,

#,

=2

are

->

*/

lim y
u

-*

discontinuities

l/(lx). Find
1.15.4. Given the function f(x)
discontinuity of the composite function
y

The point

Solution.

If

of

the second

the

points of

= f{f[f(x)]\.

is

discontinuity of the function

x^=\, then
"

Hence,

^ oo\

the

=f

point

[/(*)]

=
is

-i/'d-v,
a

"T

discontinuity

of

the

function

= /[/(*)]
If

jc=^0,

1,

then
*

= /</[/<*>]Htz^^ = *

continuous everywhere.
Thus, the points of discontinuity of this composite function are
x=l, both of them being removable.
x
is

1.16. Fund. Cont. on Closed Interval: Properties

87

1.16. The Properties of a Function Continuous on a


Closed Interval. Continuity of an Inverse Function
The function

I.

f (x)

on the interval

continuous

[a,

6],

possess-

es the following properties:

bounded on

(1)

f (x)

is

(2)

f (x)

has the

m=

(3) If

b]\

[a,

minimum and maximum values on [a, b]\


max f(x), then for any A satisfymin f (x),

M=

A^.M

ing the inequalities


A.
)

there exists a point x [a, b]

for

which f(x

<

then we can find a point


if
0,
f(a)-f (b)
such that f(c)
0.
II. Continuity of an Inverse Function. If the function y = f(x) is
defined, continuous and strictly monotonic on the interval X, then
there exists a single-valued inverse function x = y(y) defined, continuous and also strictly monotonic in the range of the function
particular,

In

c (a

<c<fr)

y=f(x).
Does the equation

1.16.1.

Solution.

sin* x+

=0

have

a root?

The function
/ (x)

sin x * +

entire number scale. Besides, this function


is continuous over the
changes sign, since /(0)=1, and / (3ji/2) =
3n/2. Hence, by property (3) within the interval [0, 3jt/2] there is at least one root
of the given equation.

Has the equation x b

1.16.2.

interval

1.16.3.
real

18*-}- 2 =

roots

belonging to the

1]?

1,

Prove that any algebraic equation

an odd power with

of

coefficients

a * 2w +

+ a,* +
2

"

+ a 2n x + a 2n =

(*)

, 1

has at Least one real root.


Solution. Consider the function

=a

/ (*)

which

is

x*"

* 1

+ a lX +...+ a
*n

2n x

-f a 2n+v

continuous throughout the number scale.

Let, for determinacy sake, a


11

f(x)=-\-

rn

oo,

>

Then

0.

and Urn f(x)


x

oo

-*

oo.

crj

Hence, we can find numbers a, b a<b such that /(a)<0;


/(6)>0. By property (3), between a and b there exists a number
y

c such
least

that

one

real

Q,
f(c)
root.

which

proves

that

the

equation

(*)

has at

88

Ch.

Introduction to Mathematical Analysis

I.

1.16.4. Let the function f (x) be continuous on [a, b\ and let


have a finite number of roots on the interthe equation f(x)
val [a, b\. Arrange them in the ascending order:

<

Prove that

<

<

x2

<

x,

<

<

xn

b.

each of the intervals

in

x 2 ), (x 2

xj,

(a,

...

a:,),

(* M

b)

retains the same sign.


the function
Solution. If the function changed its sign on a certain interval,
then we could find one more root of the function, which contradicts
the condition. To determine the sign of the function on any of the
indicated intervals it is sufficient to compute the value of the function at an arbitrary point of the appropriate interval.

Given

1.16.5.

on the interval

a function
/

f(x)

x2

=\

if

(x + 2)
2

if

2,

2 < x <
0<x<C

+2]:

0,

2.

0?
Is there a point on this closed interval at which f(x)
Solution. At the end-points of the interval [
2, +2] the given
function has different signs:

f(_2)
But
the

at

any

f(+2)

= -6.

easy to notice that it does not become zero at any point


2
2
(x
and
2) <0
[2, +2]. Indeed, jk
this is due to the fact that f (x) has a discontinuity at

is

it

of

= +6;

x;

the point x
1.16.6.

+ 2>0

interval

= 0.

Does the function


(x)

take on the

value

2y

= x /4 sin nx + 3
3

within the interval [2, 2]?

is
sin:ru;
continuous
3
The function f(x) = x 3 /4
Solution.
within the interval [2, 2]. Furthermore, at the end-points of this
interval it attains the values

/(-2)=1.; /(2)=5.
Since

[2,

<

2]

1.16.7.

2-^-

<

5,

by

then,

property

(3),

withi-n

the

there exists at least one point x such that f(x)

Show

that the function


(

f(x)=

2x
2

+\ for-l<x<0,

2*

= 0,

for

for

0<*<

1,

interval

= 2-y.

89

1.16. Funct. Cont. on Closed Interval: Properties

defined and bounded on the interval


1, 1], has neither maximum,
nor minimum values.
Solution. In the interval [1, 0) the function increases from 3/2
to 1, it does not attain either
to 2 and in (0, 1] it increases from
the value 2 or 0. Therefore the function is bounded but never reaches
its upper and lower bounds. This is because there is a discontinuity
0.
at the point x
[

Show

1.16.8.

on any interval

that

=x E (x)

unity the function f (x)


never reaches its maximum.
Solution.

any

In

interval

length greater than


value but

of

b]

[a,

attains

minimum

its

where n

\n,

given function f (x) increases from


for any
mum. Hence, 0^f(x)<
interval [a, b] we
x. Since on the
can find at least one internal inthen f(n)
and
tegral point n
but f{x)=\ for any
lim /(*)
!,

to

an integer, the

is

never attaining the maxi-

1,

means

x. It

*i

/ / // /

12

-2-10

that the function reaches

minimum value but never


its
reaches its maximum. This is because there is a discontinuity at the point

32

=n

(see Fig.

32),

2n +

Prove that the function y =


\/ x (n a natural number)
is continuous throughout the number scale, considering it as a function
2n+1
inverse to y = x
Solution. The function y = x 2n+i is continuous and increases from
oo to oo over the entire number scale. Hence, the inverse function
1.16.9.

jt

2/ '

j/ y

is

y continuous and increasing. Denoting


again as x we find that the function

defined for
variable

all

the independent

= 2n+ y/ x
y

possesses the required properties.

Prove that

1.16.10.

-a

2 " +1

any function

for

+ a^

"" 1

+a

x*~ 3

form

of the

+ a x + a n+
n

(*)

an
are positive numbers, there exists
where a a iy a 2
an inverse function increasing and continuous throughout the num,

ber scale.
Solution.

As

is

known, the functions

x,

1
,

b
,

.,

x 2n+l increase

throughout the entire number scale. Then, since the coefficients


2n+1
a (i = 0, 1,
.., A2+1) are positive, the function f (x) =a u x
+
2n ~
-\-a x
+
+ a n x-\- a n+l also increases. Furthermore, it is continuous. Therefore, for a function of the form (*) there exists an
inverse function increasing and continuous over the entire number
.

scale.

Ch.

90

I.

Introduction to Mathematical Analysis

Note. This example establishes only the existence of an inverse


function x
g(y), but gives no analytic expression for it. It is not
always possible to express it in radicals. The problems of the existence of an inverse function and of expressing it analytically should
not be confused.

1.16.11.

= x(y)

Prove

one continuous

exists only

there

that

oo < y <

function

which satisfies the Kepler equation:


e$'mx = y (0<e<l).
x
Solution. Let us show that y(x) is an increasing function. Let
x < x 2 be arbitrary points on the number scale. Then

oo)

y (* 2 )

sin x 2

(*,

sin x
<2

Since

xj e (sin x sin x
difference
slnx^ sin x

-x

sin

Since y

(x)

is

<2

\x 2

).

x |:

x x ).

(x 2

sin x < (x x
x e(s\nx smx )=y(x y(x >
l

1,

(x 2

the

of

cos

sin

<e<

whence

value

absolute

the

e sin x x )

= (x

Estimate
|

= (*, e sin

(*i)

sin

x2

1)

2)

l)

0.

continuous function in the interval ( oo, oo), the


inverse function x is a single-valued and
continuous function of y.
1.16.12.

Show

the equation

that

has one root on the interval [1, 2]. Calculate this root approximately to within

two decimal

at

places.

1.16.13. The function f (x) is defined


on the interval [a, b\ and has values of
the same sign on its end-points. Can one
assert that there is no point on [a, b]
which the function becomes zero?
1.16.14. Prove that the function

f{x)

x+

_x

at

at

<x<0,
0<x<
1

and still has the


is discontinuous at the point x
the minimum value on [ 1, 1] (see Fig. 33).

maximum and

1.17. Additional Problems

91

1.17. Additional Problems

Prove the inequalities:

1.17.1.

< {^Y^y for a natural n >


2n ~

< y2n~TT
2n
4
6
2

(a)

(U\
(D)

1;

'

'

'

'

'

'

(a)

Prove the inequalities:


20 2 303 > 303 202

(b)

200!

1.17.2.

<

100 200

1.17.3. Solve the inequalities:

||*|-2|<1;

(a)

(b)

||

3^| J_|_>

1.17.4. Can
numbers be a

1.17.5.
(a)

Do

sin x\

2;

>x + 2.

(* 2)K* +1

(c)

sum,

difference, product

or

quotient of irrational

number?

rational

the equations

=sin x + 3,

tanx| -=tanx-\-3

(b)
|

have any roots?

Prove

1.17.7.

Prove the Bernoulli inequality

(\

the

identity

...

>

+Xl )(\+x 2

x lt x 2

where
(/

f^4^T + -^4^V = *

1.17.6.

... (\

xn

are

+ x )^\+x
n

numbers

of

2
-

+x, + ...+x nt
like sign, and
l+x,->0
}

1, 2,

1.17.8.

Find the domains

(a)

f(x)=]/"^=7*;

(b)

f(x)=}/ s'mVx;

(c) / (x) =-

(d)

V sin

(e)

(a:)

= arc sin

(| a:

g(x)=-=L=.\
V

x \x\

3);

/(A')=arcco SinL.

1.17.9.
(a) /

^b)

following

nx\

/(x)=- 7 =J== and


1/

(f)

of definition of the

Are the following functions identical?

= and cp(x)=
= log x and (a) = 2 log a;
1;

/ (jc)

<p

functions:

92

Ch.

(c)

Introduction to Mathematical Analysis

f(x)=x and q) (x) = (|/ x) 2


f(x)^l and cp (a:) == sin 2 x-\ 2)
f (*) = lqg(* l) + log(x
;

(d)
(e)

1.17.10. In

what

and

cp(*)

= log (* 1) (* 2).

interval are the following functions identical?


10 lQ g*;

(a)

f(x)=x and

(b)

f(x)=VxVx

cp

cos 2 x\

(x)
1

and

<p(*)

= K* (* 1).

1.17.11. An isosceles triangle of a given perimeter 2/? ^=12 revolves about its base. Write the function V (x), where V is the volume
of the solid of revolution thus obtained and x is the length of the
lateral side of the triangle.

1.17.12. Investigating the


solve the inequality

domain

of

definition of functions,

(a)

K* + 2 + Vx5 > l/"5^;


(b) prove that the inequality

log 2

-,(*-3)>-5

has no solutions.
1.17.13. The function y
that

= s\gnx

was defined

in

Problem

1.5.11 (n).

Show

^^signx;

(a)

(b)

(c)

sign (sign x)

x sign

x\

= sign x.

1.17.14. Prove that

for a

if

linear function

= ax + b
x = x n (n =
/ (x)

the values of the argument


) form an arithmetic
2,
1
progression, then the corresponding values of the function

yn

= f(x)

(/i

l,2, ...)

also form an arithmetic progression.

1.17.15. Prove that the product of two even or two odd functions
an even function, whereas the product of an even and an odd
function is an odd function.
is

Prove that if the domain of definition of the function


symmetrical with respect to x=0 then f(x)
f(x) is an
even function and f (x)
x) is an odd one.
/(
1.17.16.

f(x)

is

1.17.17. Prove
interval

/,

/)

that any function f (x) defined in a symmetrical


can be presented as a sum of an even and an odd

93

1.17. Additional Problems

function. Rewrite the following functions


an even and an odd function:

/(*)=-l;

(a)

in the

form

of

sum

of

y=a*.

(b)

Extend the function f(x)=^x 2


x defined on the interthe interval [
onto
3,3] in an even and an odd way,
[0,3]

1.17.18.
val

1.17.19.

number

x.

The function

{x\

Prove that

is

it

= x E (x)

is
a fractional part of
periodic function with period 1.

the graph of a periodic function with


defined on the half-open interval (0, 1] by the formula

1.17.20. Sketch

T=

period

y-x 2

1.17.21. Let us have two periodic functions f (x) and y(x) defined
on a common set. Prove that if the periods of these functions are
commensurate, then their sum and product are also periodic functions.
1.17.22. Prove

1.14.4(b))

is

that the Dirichlet function X(x)


periodic one but has no period.

1.17.23. Prove that

if

periodic, then a

is

-=s\nx-{-coscix

a rational

number.

1.17.24. Test the following functions for


(a)

f{x)

= \x\;

(b)

Problem

the function

f (x)
is

(see

monotony:

f(x)=\x\-x.

1.17.25. Prove that the sum of two functions increasing on a


certain open interval is a function monotonically increasing on this
interval. Will the difference of increasing functions be a monotonic

function?
1.17.26.

Give an example

of

non-monotonic function that has

an inverse.
1.17.27. Determine
finition

the

inverse function and

(x

if
2

1.17.28.
real

domain

its

of

de-

if

Show

that

x'

if

2X

if

oo < x <

1,

<x<4,
< x < oo.

the equation x 2

+ 2jc+

roots.

1.17.29. Construct the graph of the function

= f(x-i) + f(x+0,

==

+ Yx
1

has no

Ch.

94

Introduction to Mathematical Analysis

where

fW =

k(\\x\ll)

at
at

\x\

>

/.

1.17.30. Knowing the graph of the function y


graphs of the following functions:

(z)y

= F(x);

(b)

= VW)\

(c)

= f(x),

= f[f{x)].

that the graphs of the functions y


can be derived from each other by changing

1.17.31. Prove

= \og a nx

in the ratio

sketch the

= log

x and

fl

ordinates

all

l/n.

1.17.32. Prove that if the graph of the function y-=f(x) defined


throughout the number scale, is symmetrical about two vertical
axes x = a and x = b (a<fr), then this function is a periodic one.
y

1.17.33. Let the sequence x n converge and the sequence y n diverge.


be said about convergence of the sequences

What can
(a)

xn

+ yn

(b)

xny n ?

1.17.34. Let the sequences x n and y n diverge.


the sequences x n
y tn x n y n diverge too?

Can one

assert that

Let

1.17.35.
4,

...).

that

Write

\ima n

an

an interior angle of a regular n-gon (ai=3,


several terms of the sequence a n Prove

be

the

first

= n.

\imx n

1.17.36. Prove that from

Is

=a

it

follows that

lim

-> oo

|a:

=|a|.
|

-* oo

the converse true?

sequence has an infinite limit, does it mean that this


a sequence is unbounded, does it
if
mean that it has an infinite limit? Prove that x n = n - l)H is an
unbounded but not an infinite function.
1.17.37.

sequence

If

is

unbounded? And

of

1.17.38. Prove that the sequence {a}, where a n


an arbitrarily chosen irrational number, cannot

1.17.39. Prove that


then the sequence

if

the sequence {ajb n } (b n


fli

+ fl2+---+a

the nth digit


be monotonic.

is

>0)

is

monotonic,

will also be monotonic.

1.17.40. Prove the existence of limits of the following sequences

and
(a)

find

V2

them.

j/ 2 [ 2

|/T

95

1.17. Additional Problems

(b)

xn

= c"lYn\

(c)

xn

= ajn,

Prove

1.17.41.

j^u)j>

(c>0, k>0);

where a n
that

the nth digit of the

is

an

at

chosen

arbitrarily

number
x

jt.

the

sequence

bounded.

is

Prove that the sequence

1.17.42.

(*)

+ (2*)

...+E

1-

(nx)

has the limit x/2.


1.17.43. Prove that

lima*=l
h

(a>0).

->

Given the function

1.17.44.

+#

for

for

0,

= 0.

Prove that

/(*)

li:n
x

1.

Let

1.17.45.

Prove that
if

if

0,

if

oo,

lim P(x)

lim
X

->

a /6

> m,
=m
< m.

()/

> x + ax b) =

Q.

oo

1.17.47. Sketch the graphs of the following

f(x)=

(a)

lim
/Z

/(*)=

(b)

Find the constants a and & from the condition:

1.17.46.

(b)

,V

functions:

0);

-v 00

lim
n

sin 2 "*.

-* go

1.17.48. Prove that

lim

-> GO

[(1

+ x) (1 + x) (1

( 1

-!-

a 2 ")]

(|

<

1).

96

Ch.

Introduction to Mathematical Analysis

I.

Can one replace


computing

1.17.49.

infinitesimals in

infinitesimal
a

Determine the order

1.17.50.

summands by

of

smallness

of

the

chord

infinitely small circular arc relative to the sagitta of the

Determine the order


an inscribed and

1.17.51.

equivalent

limit?

perimeters of

of

same

an

arc.

smallness of the difference of the


circumscribed regular n-gons rela-

of

tive to an infinitely small side of the

inscribed n-gon.

1.17.52. The volumetric expansion coefficient of a body is considered to be approximately equal to the triple coefficient of linear
expansion. On equivalence of what infinitesimals is it based?

Does

1.17.53.

relation

the

log(l+A;)~A

hold

true

as

x-^0?

g(x) be necessarily
1.17.54. Will the sum of two functions f(x)
discontinuous at a given point x if:
(a) the function f (x) is continuous and the function g(x) is disx
continuous at x
(b) both functions are discontinuous at x = x ? Give some examples.

1.17.55. Is the product of two functions f(x)g(x) necessarily


discontinuous at a given point x if:
(a) the function f (x) is continuous and the function g(x) is discontinuous at this point;
(b) both functions f (x) and g(x) are discontinuous at x = x ?
Give some examples.
()

1.17.56. Can one assert that the square of a discontinuous funcexample of a funcis also a discontinuous function? Give an
tion discontinuous everywhere whose square is a continuous function.
tion

1.17.57. Determine the points of discontinuity of the following


functions and investigate the character of these points if:

(b)

/ {x)

(c)

y(x)

= 2^

l/ii

' x)
\

= x[l 2X(x)]

Problem 1.14.4

(b)).

1.17.58. Test

the

where X(x)

following

their graphs:

(b)

= x E{x)\
y = x* + E(x>);

(C)

y={-\)^).

(d)

y=

(a)

1
fl

! L

+ (2sin^

is

functions

the Dirichlet function (see

for

continuity and sketch

1.17.59.

nuity

if

Additional Problems

1.17.

Investigate the functions f[g(x)] and g[f(x)] for contisign x and g(x)
x(l
x 2 ).

/ (x)

1.17.60. Prove that the

function

2x

f(x)

+ V,

0<x<l

at

mum

1.17.61.

1<*<0,

at

discontinuous at the point x


and minimum values on [

is

97

and nonetheless has both maxi1].

1,

Given the function

Ascertain that on the interval [


2, 2] the function takes on
intermediate values from /( 2) to f (2) although it is discontinuous (at what point?).

all

1.17.62. Prove that if the function f (x): (1) is defined and monotonic on the interval [a b]\ (2) traverses all intermediate values
between f (a) and f(b), then it is continuous on the interval [a, b],
y

f(x) be continuous on the interval


y
range being the same interval a^y^ib. Prove that on
this closed interval
there exists at least one point x such that
f(x)=x. Explain this geometrically.
1.17.63. Let the function

[a,

b],

its

1.17.64. Prove

that if the function f (x) is continuous on the


and x19 x 2
x n are any values from this open
then we can find among them a number such that

interval (a, b)
interval,

f(l)

= j[ U(x + f(x )+...+f(xn


1)

1.17.65. Prove that the equation


tive root

which

is

less

2*=1

has

)].

at least

one posi-

than unity.

1.17.66. Prove that

if a polynomial of an even degree attains at


one value the sign of which is opposite to that of the coefficient at the superior power of x of the polynomial, then the latter

least

has at least two real roots.


1.17.67. Prove

= (l-\-x

4-3148

2
)

sign x

that
is

the

inverse

of

the

continuous function.

discontinuous

function

Chapter
DIFFERENTIATION
OF FUNCTIONS

Definition of the Derivative

2.1.

The
is

function y

derivative f (x) of the


defined by the equality
r/

i-

A* -

&y
A*

t-

Ax -

f(x)

at a given

(x+Ax) f
ax

point x

(x)

then the function f (x) is called differentiable at the point x\ and it is infallibly continuous at this point.
Geometrically, the value of the derivative /' (x) represents the
slope of the line tangent to the graph of the function y = f(x) at
the point x.
If

this limit

is

finite,

The number
r
is

\_

Hm

f(x+Ax)f(x)

called the right -side derivative at the point x

The number

is

called the left-side derivative at the point x.

The necessary and

sufficient condition for the existence of the


derivative /' (x) is the existence of the finite right- and left-side
derivatives, and also of the equality fi (x)
f+ (x).
oo, the function f(x) is said to have an infinite deriIf f(x)
vative at the point x. In this case the line tangent to the graph
of the function y
f(x) at the point x is perpendicular to the

x-axis.

2.1.1.

Find the increment Ay and the ratio

functions:
(a)

(b)

= ]/"x

=x

at

x=

+ x _Q

at

and A* = 0.0001;

x=l

and A*

= 0.2.

for the following

2.1. Definition

Ay =

Solution, (a)

of the Derivative

V x + Ax |/"x

99

=--V0. 0001 -=0.01;

^L =10o
^
= 0.0001
Ax
Using the definition

2.1.2.

of

the derivative, find the derivatives

of the following functions:

(a)

= cos ax\

(b)

Ay

Solution, (a)

y = 5x
cosa (x

2x.
+ Ax) cosax =
= 2sin ^ax + Y Ax^ sin y

-2sin(

Ax j

ax-\--^-

Ax

sin

~ Ax

Ax

Hm
^
^Vax^O
CL

Ax

sin

Aa:

2 lim

lx

'

A*

In particular,

-*

sm( ax + .
2

a^=l, then

if

*/

y= ]/ x at
y=\/x
y = 3\x\+l
3

(b)

(c)

Solution,

=;

i.e.

there

(c)

tion

At

Ay

hence,

is

no

Ax>0
at* =

the point

at

the point x

- j/(x+

A#=j/573
v

= sinx.

= 0.

j/x

3
.

'Ax

Ax

,'(0)^^ =

00,

finite derivative.
Fig. 34

the increment of the funcwill be: Ay


3 (0+ Ax)

lim

Ax<0

y'

a sin ax.

functions have no finite derivati-

Ax) 3

Ax - +

At

and

A*

Ax -

x=0;
x=l\

at

Ay

(a)

At x--=0we have

the point

= cosx

2.1.3. Show that the following


ves at the indicated points:
(a)

x\

=
g
ax

=3Ax.

Therefore

3.

the increment of the function

Ay

will

be

Ay = 3(0 + Ax) + 1= 3Ax,


1

hence,
A*

Ay
lim
-> -0
aa:

= 3.

Since the one-sided limits are different,


the point x
(see Fig. 34).

4*

there

is

no derivative

at

Ch. II. Differentiation of Functions

100

2.1.4. Investigate the


1.
the point x
1
Solution. At x

function y

=
Ay = \\n(l + Ax)\

l.

= \\nx\

In

differentiability

for

at

+ Ax)|,

ln(l
|

e.

+ A*) at
ln | + Ax)

Ajk>0,

ln(l
(

Ar/=|ln(l+A^)H|^

at

Ax<0.

Therefore
I

+ Ax)

(1

at

Ax
Ax

ln(1

A*

>

at

Ax<0,

Ajc)

0,

whence
lim

^=+1

and

Since the one-sided limits are


\\nx\
the function y

Hence,

^=

lim

1.

there is no derivative.
differentiate at the point

different,

not

is

x=

(see Fig.

motion

= (f 5^ + 2) m
= 5 sec to = 15 sec.
2

from

tt

35).

Find the average velocity of


by the formula
specified

2.1.5.

2.1.6.

Using the definition

the

of

derivative, find the derivatives


the following functions:
Fig- 35

(a)

2.1.7. Investigate
n/2
the points x

2.2.
I.

(n

an

=x

3
\

= \cosx\

(b)

y-^\/x\

for

differentiability at

integer).

Differentiation of Explicit Functions


Basic Rules of Differentiation

(2)

c'=0;
(uv)'

(3)

(cu)'=cu'\

(4)

(uv)'

(1)

the function y

+ nn

of

= u' v'\

= u'u + uv', the product rule;


(5)
^y = ii!E=ffHl(t,^0) the quotient rule.
Here c = const, and u and v are functions of
f

x which have

deri-

vatives at a corresponding point.


y(x) is differentiate at the point x 0t and
(6) If the function u
the function y
f(u) is differentiate at the point a
<p(*o)> ^en

2.2. Differentiation

the composite function y


y' (x
x

and

= y'u (u

u'x (x

Functions

of Explicit

101

= f(y(x))

is differentiate at the point *


the function of a function, or chain, rule.

),

Differentiation of Basic Elementary Functions

II.

= nu n ~ u'\ (2) (sin u)' = cos u-u';


(cosu)' = s\nu-u';
(tanw)'=-^-;
(5) (cot u)' = -Xsin u
cos w
(u

(I)
(3)
(4)/

n )'

(6)

(lna)'

(7)

(fl

= -^;

=a

)'

lna-'; 8) (<?")'=<?"';

= cosh w';
= sinh h-w';
"
= (arccos^)';
(arcsin a)' =

(9) (sinh */)'


(10) (cosh a)'

(II)

(12) (arc

2.2.1.

tan^)'=

Find y\

= 5*
a
y=

2/3

(a)

*/

(b)

y
Solution,
,/--

lo
15

-^I = (arc cot w)'.

if:

3*

+2*" 3

5/2

b constants).

(a,

xl

(a)

y'

X *'*= 5.1
3

3 x*'*- 2.3*"
1

"1

=
3

a:

'

2.2.2.

Find

*/',

if:

n cos x
o
o
/u\
(a) v = 3cos* + 2sin x\ (b) v = smx cos x
(c) y = (x + 1) arc tan*; (d) */=:* arcsin*.
= 3 (cos*)' + 2 (sin *)' = 3 sin * + 2 cos*;
Solution, (a)
cos x) (sin x cos x)' (sin x-\- cos
(sin x-\- cos x)' (sin
(b) y'
(sin cos x)
(sin x cos x) (cos x-{- sin
(cos a: sin
(sin #+cos x)
(sin x cos
/

'

si

at

/y

*/

'

a:

a:)

a:

jc)

a:)

a:)

(sin

(d)

y'

2.2.3.

= (*

)'

arc sin

*+ (arc sin *)' * = 3x

Find the derivative

pute the particular value of


of the argument:
(a)

/(*)

1/7 + 16/*
2

a: cos a:) 2

3
a:

arcsin *

+ -^-==

of the given function and then comthe derivative at the indicated value

at

= 8;

Ch. II. Differentiation of Functions

102

= (\Vr x) /x at x = 0.01;
at f=n/6.
(c) f(t) = (cos 0/(1 sin
Solution, (a) f (*) = {r" J - 16;r =
Putting * = 8, we obtain
2

(b) f(x)

^7=^.
x

sin

(1

(c) /'(/)

/'(ji/6)

/)

t)

(1

Whence

sin + cos
sin

_
~~

3 1/ x

sin

= 2.

2.2.4. Taking advantage of the differentiation


derivatives of the following functions:
(a)

= 2x + 3x 5;
3

2x 2

+ +
a;

-cos
(g)

(b)

formulas, find the

= V~7+ -^= + 0Ax


x-{-

10
;

V*

(p

ex

= ex (cosx+smx)\

(h)

-j-

sin x

2.2.5. Taking advantage of the rule for differentiation of a composite function find the derivatives of the following functions:
(a)

(d)
(f)

= sin #;
# = lnsin (x +
= In (tan 3*);
3

(b)

(e)

1);

= lntanx;
= arcsin [/l
y = sin |/T/(T=7)

(c)

= 5 C0SX

a;

*/

(g)

*/

Solution, (a) Here the role of the external function is played by


the power function: sin x is raised to the third power. Differentiating
this power function with respect to the intermediate argument
(sin x), we obtain
(sin 3

a:) sin

= 3sin

x\

but the intermediate argument sin a: is a function of an independent


variable x\ therefore we have to multiply the obtained result by
the derivative of sin a: with respect to the independent variable x.
Thus, we obtain

yx =

(sin 3

A:) S in

x (sin x)'x

J^^

x cos

x\

= |2I
= (lntanx) tan ,(tanx); =
x
y' = (5
* In 5 ( sin x) = 5 C0S * sin
(cos x)'x = 5
x

(b) y;
(c)

= 3 sin

C0S

C0S

)cos x

if

.;

a;

In 5;

2.2. Difjerentiadon of Explicit Functions

(d)

= [In sin (x +
3

yx

l)]'sin

= sin
= (arc sin Kl-x

(e)

(JC ,

(J*

(c)

(d)
(e)

1)

[sin (x 3

x2 )

'

( 1

x2

(-2*)=-

);

(x^O).

i^ii^T^j?

the following functions:

of

y = (3 sin x)

)*;

3
;

jc;

+ 2*+l + ln *;
=
sin
3x
#
+ cos(x/5) + tan \f*>\

=
sin(x
5x + 1) + tan (a/x);
#
5

j/2e*

(g)

(h)

*/

= arc cos
= arctan(lnx) + ln(arctan.x:);
;

In 2 arc tan (x/3);

(i)

y=

(j)

+ -\fx + V*(a) y' = 4


+ 3x + 5x

Solution,

( 1

+ 3x+ 5x =
= 4(l+3x + 5x
2

(1

2.2.7.

Find the derivative

)'

2
)

(3+10x:);
1

of the function

2x

= arc sin
1

+ X*

have

+ s 4x _

2
2n(1

l
1

0'

i
k

-i-

2(1

(i+*

2x~y

v
i.

(f)

We
/e

'

2V~T^x*

(b)
y = (\ + 3x + 5x
y= j/sin x + 1/cos
2

Find the derivatives

2.2.6.
(a)

+ 1)], + x *+ l)'x =
C0S
^ + 3x2 = 3 *' C0t + ^
+
([/"TT7) _
-x =
+

1 >

V\ (\

103

2 2

a:

2
)

y-(\-x*)*(\+x

2(1 x 2
|i-* 2 l(i+* 2 r
)

2
)

x-

e.

frb

at

l-TT?
At

|x;|

2.2.8.
(a)

r/

the derivative

is

Find the derivatives

= sinh5xcosh(x/3);

M<i.

"t|*|>i.

non-existent.
of

the following functions:

Ch. II. Differentiation of Functions

104

(b)
(c)

(d)

= coth (tanx) tanh (cot x)\


=
arc cos (tanh x) + sinh (sin 6x);
y
= sinh # + cosh x
*/

(e)

*/

s\nh ax

~ sinh 6a; cosh

6a;

Solution.

= (sinh 5x)' cosh ~ + sinh 5x( cosh

(a)

= 5 cosh 5x cosh y + y sinh 5x sinh y


y'

(c)

= -J= hx)
V tanh
1

+cosh(sin6x)(sin6x)
2

x
1/cosh 2 x

sinh x) /cosh x
+ 6 cos 6x cosh (sin 6x) = c J h + 6 cos 6x cosh (sin 6*).
|^(cosh

2.2.9. Find the derivatives of the following functions:


(a)

4/=-;
lx

x
K~~
(d)

y^(VT^c)

(b)

sin 3

x+

* cos 2

U)

x\

Solution, (a) Apply the method of


Consider, instead of y, the function
z

(u(*)>0);

logarithmic

differentiation.

= ln|y| = Inj/" 10^1 = ln\x\ +\n(x*+ l)-^\n\5-x\.

Taking

into account that (In


Z

But

z'

1.2*.

3(a: 2

+1)

= (In y\Y = y'ly>

u\

)'

15(5

= u'ju we have
24a; +125x
y

\bx(x 2

a:)

-\-

{b

+ 75
x)
14a:

whence

(a:

1)

24x + 125x

Y^=~x

Suppose the functions u

z=\ny = vlnu
also has a derivative in this domain,
z'

14a:

+ 75

15x(x 2 +1)(5-a;)

(x) and v(x) have


given domain of definition. Then the function

(b)

\)

and

= (v In u)' = v' In u + v

derivatives in the

2.2.

Differentiation of Explicit Functions

105

Hence, the function

y = e ln v==e z
domain, and

also has a derivative in the indicated


y'

= ez z' = yz'

Thus,
y'

= u v {v'

Show

2.2.10.

Inu +

the

that

v^^vu"- ^ +u v \nu-v'.
1

function y

= xe~ x ^

xy'=(l x 2
Solution.
y>

the equation

satisfies

y.

= e -x>/2 X e -x*,2 = e -x>/2


2

_X

2).

Xy'= xe -x*/2(l_ x 2),


Hence,

= y(lx

xy'

Show

2.2.11.

xy'=-(\

2.2.12.

(a)

that

the

function

Investigate

the

following

x)y.

= arc sin

(cosx);

/
,
o
Solution, (a) y
,

<

(b)

= ]/~

V 1 cos

).

= xe~ x

Vl x

equation

differentiability:

for
2
.

sin x

V sin

the

satisfies

functions

x)'
= (cos
v

sin*
'

sin x

Hence, y' = at points where sin^>0; y'=\ at points where


sinx<0. At points where sinx^O, i.e. at the points x^=kn
(k =
1, 2, ...) the function, though continuous, is not difI

ferent iable.

(b)

The domain

definition

of

of

function

this

is

the

interval

1.
{

y'

As

,r

7=^ee

X+10

or

l/

(-2x)

1+0

we have

out whether the derivative y' exists


ther

lim

Since

1 Ax

at

x=^0 and *=^

y>+oo.

Let us

= 0,

the point x

1.

i.e.

exists.

x
Ax

Ax->0

at

yAx
1

2
,

then

X=-

as

Ax> + 0,

find

whe-

Ch. II. Differentiation of Functions

106

which means that the function under consithough it is contino derivative at the point x =

Thus, yL

(0) =/= y'+ (0),

deration

has

nuous

at this point.

Note. There are cases of failure of existence of /' (x) and even of
and fL (x) at a given point, i.e. when the graph of the function has neither a right-, nor a left-side tangent at the given point.
For instance, the function

f'+{x)

nx >-\

is

xs\n(\/x) at

continuous at the point x =

derivatives, since -ltl-

Ax

x^=0
x

at
0>

but does not have even one-sided

= sin-r
Ax

2.2.13. Find the derivatives of the following functions:

(b)

= s\nh(x/2) + cosh{x/2);
/(*) = ln[cosh*]; (c) f (x) = 2 j/"cosh x

(d)

f(x)=

(a)

(e)
(f)

f(x)

1;

arc sin [tanh x]\

= Kl+sinh 4.r,
ax {zoshbx+smhbx).
=
e
f(x)
2

f (*)

2.2.14.

Applying logarithmic differentiation

find

the derivatives

of the following functions:


(a)

^(cos^'-^Jb) y=

(c)

f/(x+2)*

Vx^hx

<

V(x+ 3)"

2.2.15.
COS 2 X
4-

sin 2

'

a:

show that
f(ji/4)-3/'(*/4)
2.2.16.

Show

that the function

*
satisfies

2.2.17.

(b)

+ 2y = e-**.

Find the derivatives of the following functions:


2C
(x > 0);
IncosKarcsin 3~

=
y = ^arc tan

2* 2

the differential equation


xy'

(a)

= 3.

j/ cos In 3

x.

2.3.

Formula

Successive Differentiation. Leibniz

107

2.3. Successive Differentiation of Explicit Functions.


Leibniz Formula

If the derivative of the (n


l)th order of a function y
f(x) is
already found, then the derivative of the nth order is determined
by the equality

{n)

(x)

= [y

{n

(x)]

In particular, y" (x)


[y' (*)]', y"' (x)= \y" (x)]\ and so on.
If
u and v are functions differentiate n times, then for their
linear combination c x u-{-c 2 v (c ly c 2 constants) we have the following

formula:

+ c v) (m = c

(c x u

and

+c v

u (n)

(n

product uv the Leibniz formula (or rule)


n(n
X)
u {n '*>xf
nu '- l >v'
u (n) v

for their

(uvy a

>

+ ... +uv = %C
nu
=
n(n ~ " {n ~ k +
= - )!
C =
o
-2-3.
k

(n)

(n

k)

v {k \

=uu,

where
wneie u {0)

v
u {0)

=vu

and
diiu

1)

'

are

fc!(/t

.As

binomial coefficients. Here are the basic formulas:


~
m
(x (n) =m(m\). ..(m
\)x m n
(1)

n+

n
x
(a*Y = a \n a(a>0). In
(In^r =(-1)"-^=^.
= sin(jt + njt/2).
(sinx)
= cos(* + wr/2).
(cos*)
n)

(2)
(3)

particular,

(e

{n)
)

= ex

(n)

(4)

(n>

(5)

Find

the

(a)

y=\nx\

(b)

(e)

(a)

^ = i = x-i; /=(-l)x-;

2.3.1.

the

nth order

derivatives

of

y=

y = s'mx\
3x cos 2 a:;

of

the following

functions:

(4)

= sin xcosx;

--= 1-2-3a:" 4

(c)

y'

=cos a;=

e kx

(c)

sin

(f)

sin (#

if

(a:

we assume that
y

{k)

=sm

given

jt

+ 2ji/2).
for a

= sin 5a: cos 2x;


= In (x + x 2).

*/

+ n/2);

= cos (* + n/2) = sin


In general,

iT'=l-2^- 3

=( l)"" (n 1)

(/2)

*/

(d)
(g)

n=k

(-!)*-! (/I-

1)1

Ch. II. Differentiation of Functions

108

then

will turn out that

it

0<* + i>=COS

Whence, by virtue
any natural n

of

y=- sin 5x cos 2x

(d)

U + k^

(k+l)-

:S1I1

mathematical induction we conclude that

=y

[Vin 7x

+ sin

for

3.x:

Therefore

+ n^+ 3" sin ^3x + n^) ]

'

x2

To simplify
function:

2x+
y

\jn sin (7x

2x+l
+a: 2

y'<

(g)

m = J.

'

x2

computations

the

(jc+2)

+ x 2

+ (jc-1)

transform

us

let

:(^_1)-i

(x l)(* + 2)

-1

+2

the

obtained

+ (x + 2)-

1
.

Whence

/=-l(*-l)- -l(* + 2)i/" = l.2(x 1)~ + 1-2(a: + 2)2

^)

= (_i)-i(n 1)!
= l)"" 1)
1

2.3.2.

#
v

= cx
^4^;

(AZ

(*_l)-

+ (* + 2)-] =

{x-\)

(x

+ 2)

find

-f

Solution. Transform

ax

the

be

-\-b

ex-\-

Whence

expression in the following way:

ad

be

ad
[CX

c(cx-\-d)

ad

y'

= (-i) be

y"

= (-l)(-2) b-^C*(CX + d)-\

y'"

c (cx

+ d)~K

+ d)~

= (- 1) (-2) (-3)

<)= =
/

given

(_l)n!*E_^ C "(

C3

(CJC

w + d)-+i) =

4>

2.3. Successive Differentiation. Leibniz Formula

y = x/(x 2

2.3.3.

find

1);

109

(n)
.

Solution. Transform the given expression

Problem

therefore (see

r_j

i_i

U+l" "^ lj
1

2.3.2):

= (\)

{n)

x
x2

n n\

+1

(x-

2.3.4. Using the Leibniz formula, find the derivatives


indicated orders for the following functions:

y=x

(a)

smx\

find # (25)

=
l); find
x
=
e s'm$x\ find y
y
2
e x (x

(b)
(c)

+ ^^(sin #)

(23)

(24)
;

(n)
.

= (sin x-x

(jk )",

the

*/

Solution, (a) y (2b)

of

2b)

2B)

x2

+ 25 (sin

jk)

subsequent summands

the

since

= (sin *)<

(24)

2
(a: )'

equal zero.

Therefore
(25)
*/

=x

+ 25 y) + 50a: sin

(x

sin

(x

+ 24 ~ ) + 600 sin
=

2.3.5.

Compute
2

at

'

.,

the

value of the

the point x

nih

(a:

+ 23 y ) =
600) cos # + 50* sin x.

derivative of the function

= 0.

2* + 5) = 3x +

Solution. By hypothesis we have y(x)(x 2


us differentiate this identity n times using

then

(x) (x

^ we obtain
2x + 5) + ny< ~

(for
2

2.

Let

the Leibniz formula;

2)

= 0,

Putting #

l)

(x)

(2x2)

+ n(n

{)

y<-

(*). 2

= 0.

we have

n)

(0)

2ny

{n

1)

(n
(0y+ n(nl) y

~ 2)

(0)

- 0.

Whence
y<>

We

(0)=ny<-

(0)-^^^~

2
>

(0 ).

have obtained a recurrence relation for determining the nth


at the point x = 0(n^ 2). The values y (0) and y'(0)
are found immediately:
(0) = 2/5;
derivative

Ch.

110

Differential ion

II.

-4x-f
W= -3x
(*-2*+5)'
2

>i

19

successively putting n
derivatives of higher orders

19

n
(0)==
25'

,
;

= 2,

Then,
the

Fuw turns

of

find the values of

4,

3,

with

the

aid

of

the

recurrence

relation.

For example,
y
y
Find

2.3.6.

"'(C)\

"

'

25

56

32

125

5
19
"

25

125

234
625'

derivatives of the second order of the following

the

functions:
(a)

y = xVT+7*;

(b)

= ^M^;

(c)

y=e~*.

Given the function

2.3.7.

Show

that

= cfi 2x + c xe2x + ex
2

this function satisfies the equation


y"

\y' + \y=ex

2.3.8. Using the Leibniz formula give the derivatives of the


dicated orders for the following functions:
(a)

(b)
(c)

(d)

y=x*s'mx;
x
y = e~ smx;

(n)
;

(2n)
.

a linear combination of simpler


the derivatives of the 100th order of the functions:

find

Show

l+#

/u\

that the function

y
c2

Using the expansion into

find

a:;

2.3.10.

(c l9

find y

2.3.9.

*/

find y"'\

functions

(20)

find

= ex (3x 4);
y= (1 x cos
y

in-

= x n [c

n constants)

cos (In x)

satisfies

x 2 y"

+c

sin (In

the equation

+ - 2n) xy' + +

2.3.11. Prove that


[/

( 1

if

/(a:)

(ax

a-)]

ai

= 0.

has a derivative of the nth order, then

+ b)]

(ni

= a'f n

>

(ax

+ b).

and Parametrically Represented Functions

Implicit

Inverse,

2.4.

2.4. Differentiation of Inverse, Implicit


cally Represented Functions

1.

The Derivative

function y
function x

111

and Parametri-

an

Inverse Function. If a differentiable


has a single- valued continuous inverse
and y'x =0 then there exists also

= f(x)
= g(y)

of

<x<b

Xy

yx

For the derivative of the second order we have


yxx

Y"

2.

The Derivative

function
to

= y(x)

differentiate

with

it

find y"xx

To

respect to
3.

an

Function.
equation F (x, y)

Implicit

respect

If

a differentiable

= 0>

x, considering

to

and solve the obtained equation j^F(x

of x,

to y'x

of

satisfies the

y)

then we have
y as a function

with respect

the equation should be twice differentiated with

and so on.

The Derivative

Function Represented

of a

Parametrically.

If

the system of equations

* = <P(0.
where

and

<p(t)

= *(O. <*<'<P,

if (t)

continuous

single-valued
y
and
exists a derivative
as

defines

The derivatives

q't (t)

of higher orders are

computed successively:

= -^, y = ^>
{

y'

xx

functions and q/(/)=^=0,


then there
function of

differentiable

are

and so on.

Xf

xt

In particular, for the second derivative the following


true:
-

x't

yttx'tt

y't

(xt)*

2.4.1.
(a)

(b)
(c)

For the function

y = 2x' + 3x + x;
b

y = 3x (cos#)/2;
y= x + e x

Solution,

(a)

We

have

find x'y

find x"yy
find

xyy

x = Sx +
2

y'

yx

15x 4

hence,

1,

6x 2

+15x 4 +l

formula

is

Ch. II. Differentiation of Functions

112

+ e*.

#i=l

(c)

= ex

y"xx

hence,

ex

Using the rule for differentiation of an inverse function,


yx for the following functions:
2
(a) y=i/x\ (b) # = arc sin/*; (c) y = In Vl + x
s
Solution, (a) The inverse function x = y
has the derivative
xy = 3y 2 Hence,
2.4.2.

find the derivative

At x

(c)

xy =
f

>

For each

2.4.3.

(c)

(d)

has the derivative

Ve~iy~i

=^

of the following functions represented

rically find the derivative of the

(b)

a:=[/^

the inverse function

y/Ve yl. Hence,

yx =

(a)

paramet-

order of y with respect to

first

x:

y = a(l cos/);
y = kcost+coskt;
= tan/ + cot
=
e~
y

sin/),
x = a(t
sin&/,
x = ks'mt
Jt = 21ncot/,
x = ec\

*/

/;

ct

Solution, (a) Find the derivatives of x and y with respect to the

parameter

/:

x't

= a (I cos

y\

/);

= a sin

/.

Whence
a sin
dy
~
=
cos 7r = cot 172
a
dx
/

(1

2 cosec
4
~ cot!
sin~27
dy
cosec =
-f = sec
2

9
2

(a)

9
2

d#

4 cos 2/ sin 2/

dx

4 sin 2 2t

(c)

Find

for

cos 2t
.

/,

20

sin 2 2/
,

kn
2

are defined parametrically:

= acos
y = bsm*t\
x = a (cos + sin
y = a(s\n /cos/);
t

4-

The functions

(x

ctf

2.4.4.

2fcri).
(/
=7^
7
v
7

/),

(b)

jx=
fx=/ + 3/+l,
t* 3t + U
\y = t*-3t
=
cos/,
e*
fx
=
^sin
I
/

l;

(d)

/.

t/

them the second derivative

of

y with respect

to x.

Implicit and Parametrically Represented Functions

Inverse,

2.4.

Solution,

First find y'x

(a)

= 3b sin
3b sin
^= 2

y'

cos

cos

3acos^sin/

Then we

x\

t\

113

3a cos

= -a
b

ta "^

shall find y"^ using the

sin

/rt

t;

=*<2*+

('

ji

1)

T ).

formula

=^

xt

where

= ~acos

t'

Whence
b

(d)
f

=
=e

^*

cos

sin

cos

cos

sin

2.4.5.

Find

cos

<

3a 2 cos 4

/)

t);

/);

sin

/
#

cos

\cs

e*

x\

3a cos sin
e (cos sin
= e' (cos + sin

1 (

e 1 sin

+ sin
/

a cos 2

(cos

+ sin
sin
/

A'

///

sin

2
e t (cos

/)

'

s in /) 3

y'

xxx :

(a)

(b)

= sec/;

#=tan/.

Solution, (a) First find

=
whence
yx

Then

find the

y't

second derivative
6

3g2 ' [2(/ 2

x*
2

6g3f (/2

+ 3/ +

Find the derivative yx of the following implicit functions:


e-""=c;
y* = 0;
(b) ln*
tfy

2.4.6.

jk

+ 2Q + 2/ + 2] =

e~

xt

(d)

derivative

find the third

(c)

finally,

(a)

= Wle- = 3eH\

xt

And

= 3t

+
+ +
+ # 4x 10*/ + 4-0;
+ tf
2

'

Solution,

(a)

a function of x;

Differentiate

we

with

respect

to

get:

3x 2

+ 2xy + x*y' +2yy = 0.


f

considering y as

Ch. II. Differentiation of Functions

114

Solving this equation with respect to


3x

2.4.7.

Find yxx
y

tan y

arc

(c)

x+y = e*~y.

y'

(b) e

0;

=y x\

ey

respect to x

Solution, (a) Differentiate with


function of x and determine y'\

_X__^ + =0

find

+ 2xy

if:

+x =

(a)

whence

considering y as a

= - = y-> +
X

y'

1.

Differentiate once again with respect to x:

we

Substituting the value of y' thus found,

2(l+y2
yXX

a;

the point

2x + + */ 5-0
2

x=l

Solution. Differentiating with respect to x

3*
Putting x

we

Axy Ax yy' + 5 + y' =


2

and
3

if

and y\ x=x

5.x:

*/

yb

of y" at

Find the value

2.4.8.

finally get

1.

find

that

0.

obtain the value of

_4_4^ + 5 + ^_0;

at

x=\:

= 4/3.

^'

Differentiate once again with respect to x:

6x

4y 8xyy' 8xyy' \x
2

x=l; y^=l and

Putting

a
6

Find

2.4.9.

(c)

e*sin#

(d) ey

= 4/3,

64

4 y64 T

y'

0.

3^/

n
=0,

1:

Q 22
- 827.

for the following implicit functions:

+ xy^e;

\x yy" + y" =

find the value y" at #=-.

+ V xy + y = a;

(a)

t/'

2 y' 2

cos *
find

(b) arc

tan (y/x)

= In V + y
2

2
\

= 0;
at the point (0,

1).

2.4.10. Find y xx of the following implicit functions:


arc tan*/;
(a) y

=x +

(b)

x2

+ 5xy + y

2x + y 6 =

0;

find

at the point (1,

1).

115

Applications of the Derivative

2.5.

2.4.11. For each of the following functions


the indicated derivatives:

represented

parame-

trically find

(i

(a)

sin

'

(d)

= ln(l + f),
X = + 2,
X = e-*\

(e)

(b)

(0 X

(g)

= 4tan

(//2),

2{xy'-y).

2.5.

1);
t;

yxx

find y'x \
find y'x ;
find y'x

find y"xx

that the function y


f(x), defined by the parametric
y e cost, satisfies the relation y"(x
yf

s'mt

Applications of the Derivative

The equation

tangent

of a line

= y(x)

function y

yy

=y'

the

to

M(x 0i

at a point

form

find

Show

/3-t;

= arc sin t,

=e

find y'x

t;

y = a sin t + b cos
y = arc cos 2t;

= arc sin (t* 1),

2.4.12.

y=- arc tan (2/+

equations x

find y'x ;

arc tan

t
t

-f b cos

y=

/*

(c)

COS

=
y=

+6 cos

),

W (*

curve of a differentiate

where y

=y(x

)>

has the

*<>)

straight line passing through

larly to the tangent

equation
point

of

the

line

is

normal

at

the point of contact perpendicucalled the normal to the curve. The


the

will be
1

yyo = 77
y' (*o)
y' (x

(x

),

)>o.

segments AT, AN are


subtangent and the
subnormal, respectively; and the
and
are the
lengths
so-called segment of the tangent
and the segment of the normal,

The

the

called

MN

MT

X
o
Fig. 36

respectively (see Fig. 36). The lengths of the four indicated


by the following formulas:

segments

are expressed

AT =

_y_
y'

MT =

AN:

MN = \y\V\ + {y')
2.5.1.
(a)

Write the equations


3a;
curve y=^x s

to the

of

+2

the
at

2
.

tangent line and the normal:


the point (2,

4);

Ch. II. Differentiation of Functions

116

x + 5
y=x* + 3x 16

(b) to the parabola

curve
with the parabola y
(c)

the

to

= 2x

= 3x

= 0.5;

at

the

at

points of intersection

2
.

Solution, (a) Find the derivative at the point x


y'

The equation

of the

= 3x -3
2

tangent line has the following form:

04= 9{x2)
The equation

of

the normal

9x y 14 = 0.

or

of the form:

is

y 4 = y {x 2)
(c)

= 9.

*/'(2)

= 2:

or

+ 9y 38-0.

Solving the system of equations

= x* + 3jk
y = 3x
y

16,

we

shall

intersection of the curves

find the points of

xt

Now we

find

y'

the

= 2,

x2

= 2,

derivatives

= 4a: + 6x,
3

y'

at

(2)

yx

=y =

12.

=2
y' (2) = 44.

the points x

= 44,

and x

= 2:

Therefore, the equations of the tangent lines have the form

y 12= 44(* + 2), y\2


The equations

of the

tangent

2).

normals have the form

y-\2 = (x + 2),
2.5.2.

-44(^

y -\2

= -(x-2).

Find the points on the curve y


line:

= x'

3x + 5

at

which the

to the straight line y


2x;
perpendicular to the straight line y
#/9;
(c) forms an angle of 45 with the positive direction of the x-axis.
Solution. To find the required points we take into consideration
that at the point of tangency the slope of the tangent is equal to
3x 2
3 computed at this point,
the derivative y'
(a) By the condition of parallelism
(a)

is

(b)

is

parallel

3x 2

whence x t

= l/j/3,

3 =

2,

x 2 =l/\/~3. The required points

^(-1/^3,5 + 8^3/9),

M,

(1/1/3,

are:

5-8^3/9).

(b)

By

the condition of perpendicularity


3a:

whence
2.5.3.

117

Applications of the Derivative

2.5.

^ = 2,

x2

= 2.

3-9,

The required

points:

M 2,
x

Find the angles at which the following

lines

(2, 7).

intersect:

= 4x
= cosx.

= 4x

and the parabola y


the straight line y
s\nx and the cosine curve y
(b) the sinusoid y

(a)

3),

/2\

Solution, (a) Recall that the angle between two curves at the point
is defined as the angle formed by the lines tangent to these curves and drawn at this point. Find the points of
intersection of the curves by solving the system of equations

of their intersection

y=4 x,
y = 4x /2.
2

Whence

AMO,
Determine then the slopes
and
2
l

the points

Af 2

4);

(2,

2).

of the lines tangent to the parabola at

0'(O)=O

y'{2)=-2.

The slope of a straight line is constant


case it equals
1. Finally, determine
the angle between the two straight

for all

its

points;

in

our

lines:

tanq^

^ = 45;

tan

+ 2 ~~
= 1+2

1;
1

q) 2

<P 2

arc tan

1
,

18.5.

that
the segment of
the hyperbola y
c/x
which is contained between the coordinate axes is bisected at the
point of tangency.
have y'
c/x 2 hence, the value of the subtangent
Solution.
for the tangent at the point
(x
y ) will be
2.5.4.

Prove

the tangent

to

We

y_
y'

Ox = x T (Fig. 37), which completes the proof.


Whence follows a simple method of constructing a tangent to the
Then MT will
hyperbola y = c/x: lay off the ^-intercept OT = 2x
i.e.

be the desired tangent.

Ch. II. Differentiation of Functions

118

2.5.5. Prove that the ordinate of the catenary y


acosh (x/a) is
the geometric mean of the length of the normal and the quantity a.
Solution. Compute the length of the normal. Since
y'

= sinh (x/a),

the length of the normal will be

MN = \y\ V l+(y') = y V 1+sinh


whence y =a-MN, and y Y a- MN
%

2.5.6.

(x/a)

/a

which completes the proof.

',

Find the slope

= y cosh (x/a) == y

tangent to the curve

of the

= + 3t 8,
y = 2t 2t 5
2

the point
(2, 1).
Solution. First determine the value of t corresponding to the given values of x and y. This value must simultaneously satisfy the

at

two equations
t*

2t

+ 3t8 = 2

2t 5-

1.

5 the roots of the


The roots of the first equation are t 1 2; t 2
1.
Hence, to the given point there
second equation t t = 2; t 2
2.
Now determine the value of the dericorresponds the value t
vative at the point M:

And

so,

'

'*

=2

==

==

(27

(il)/=2

the slope of the tangent at

+ 3)^2

the

= T'

point

(2,

1)

is

equal

to 6/7.

2.5.7. Prove that the tangent to the lemniscate p


aKcos26at
ji/6 is parallel to the x-axis.
the point corresponding to the value O
Solution. Write in the parametric form the equation of the lem-

niscate:

= p cos = a j/"cos 26 cos 0,


y = p sin = a Y^os 20 sin 0.
x

Whence
xq

{)

,r

a cos 6 sin 20
r

ay

^cos26

a sin 6 sin 26

Xu(n/6)

cos 26

7^
n
cos 20 sin 0,
.

^ cos 0,n
+ a V cos 20

= -aY2,

-./

0e(jx/6)-O.

Applications of the Derivative

2.5.

k-=Q

Thus, the slope

(Ji/b)

= 0.

119

Consequently, the line tangent to

*e (n/6)

lemniscate

the

= a/V

is

with

point

the

at

= ji/6

and p

4a;

(b)

parallel to the x-axis.

and

2.5.8. Find the equations of the tangent


following curves:
(a)

= aV cos 20 =

the normal to the

3xy + Sx 5xy 8y + 9x+ 14=0 at


+ y 2xy = at the point

the point (2,

3);

1).

(1,

Solution, (a) Differentiate the implicit function:


1

5y 5xy 6yy' + 9 =
the point M
the coordinates
48 27 + 36#' 24 5 + 0y'
+9=

2x 2

Substitute

3y 6xyy' +
2

2x

'

of

3):

48*/'

2,

0.

0;

whence
'

Thus the equation

of

= -9/2.

the tangent

line

is

-3=-y(* + 2)
and the equation

of

the normal

y-3-=-(x + 2).
2.5.9.

curve #

Through the point


draw tangents

= Jt\

(2,

which does not belong

0),

to the

to the

latter.

Solution. Let (x Qy x* ) be the point of tangency; then the equation


tangent will be of the form:

of the

y4=y' (x {xx
)

yxi = 4x (xx

By
(2,

).

the desired tangent line passes through the point


hence, the coordinates of this point satisfy the equation of

hypothesis
0),

the tangent line:

%t 4xq (2
= 0;

)\

3x1

= 0,

x
8/3. Thus, there are two points of tangency:
7W 2 (8/3, 4096/81).
Accordingly, the equations of the tangent lines will be

whence x

AM0,

0),

4096

2048

8 \

Ch. II. Differentiation of Functions

120

2.5.10. f (x)
3x b
I5x 3
5x7. Find out at which of the points x
the rate of change of the function is minimal.
Solution. The rate of change of a function at a certain point is
equal to the derivative of the function at this point

/'(*)=

15a:

45a: + 5 = 15 [(x 1/2) + 1/12].


2

The minimum value of f (x)


minimum rate of change of
x

l/j/*2. Hence the


is attained at x
the function f (x) is at the point

/y 2 and equals 5/4.

point is in motion along a cubic parabola I2y=x 3


coordinates changes faster?
Solution. Differentiating both members of the given equation with
respect to t we get the relation between the rates of change of the
coordinates:
2.5.11.

Which

of

its

\2y't

= 3x

-x't

or
yt

_x

2 < x < 2 the ratio y\\x\ is less than unity, i.e. the rate
change of the ordinate is less than that of the abscissa;
(2) at x = 2 the ratio y\\x\ is equal to unity, i.e. at these
points the rates of change of the coordinates are equal;
(3) at jc<-t-2 or x > 2 the ratio y\\x\ is greater than unity, i.e.
the rate of change of the ordinate exceeds that of the abscissa.
(1) at

of

2.5.12. A body of mass 6g is in rectilinear motion according to


3
1
l)
ln(/+ l) (t
(s is in centimetres and /, in
the law s
seconds). Find the kinetic energy (mv 2 /2) of the body one second
after it begins to move.
Solution. The velocity of motion is equal to the time derivative
of the distance:

= +

+ +

(0

= s;= r pr + 3 ('+

a
)

Therefore
*<l)

= 12l

and

^ = |(l2i-)

-=4684(erg).

2.5.13. The velocity of rectilinear motion of a body is proportional to the square root of the distance covered (s), (as, for example,
in free fall of a body). Prove that the body moves under the action
of a constant force.
Solution. By hypothesis we have

= s'i=ay s

(a

= const);

2.5.

Applications

the Derivative

of

121

whence
s"ti

=v'

=a jy=- s = a

12.

But according to Newton's law the force

F = ks

tt

(k

= const).

/2

= const.

Hence,

F^ka
A

pulled to the bank by means of a rope which


at a rate of 3 m/min. Determine the speed
of the raft at the moment when it is 25
distant from the bank
if the drum is situated on the bank 4
above water level.
Solution. Let s denote the length of the rope between the drum
and the raft and x the distance from the raft to the bank. By
hypothesis
2
2
42
s
2.5.14.

is

raft

wound on

is

drum,

=x +

with respect to

Differentiating this relation


ship between their speeds:

find the relation-

/,

= 2xX{,

2ss/

whence
x

t-T

St '

Taking into consideration that


s;

= 3;

x = 25;

^;==

^25

= /25 + 4 25.3,
2

we obtain

2.5.15. (a) Find

y=x*

at the point

the

+4a

25

.3 3.03

(m/min).

slope of the tangent

to the cubic

parabola

= ]/~3/3.
y=

of the tangents to the curve


x2 )
1/(1
intersection with the hyperbola y=l/(x+l).
(c) Write the equation of the normal to the parabola y
#2 -f-4x-f 1
perpendicular to the line joining the origin of coordinates with the
vertex of the parabola.
(b)

Write the equations

at the points of its

(d)

At what angle does the curve y

2.5.16.

The velocity

of a

body

ned by the formula v = 3t-\-t 2


have 4 seconds after the start?

= ex

intersect

in rectilinear

What

the y-ax\s?

motion

acceleration

will

is

determithe body

2.5.17. The law of rectilinear motion of a body with a mass of


2/ 2
100 kg is s
3/+ 1. Determine the kinetic energy (mv 2 [2) of
the body 5 seconds after the start.

Ch. II. Differentiation of Functions

122

2.5.18.

then

its

Show

that
acceleration

the law of motion of a body is s


ae
be
numerically equal to the distance covered.

if

is

2.5.19. A body is thrown vertically with an initial velocity of


a m/sec. What altitude will it reach in t seconds? Find the velocity
of the body. In how many seconds and at what distance from the
ground will the body reach the highest point?
2.5.20.
orbits.

Artificial

move round

the Earth in elliptical


from the centre of the Earth
can be approximately expressed by the fo-

satellites

The distance

r of a satellite

as a function of time

llowing equation:
r

where

=a

M = ^-(t

e cos M y (cos 2M

1)

n)

= time parameter
a = semi-major axis of the orbit
e = eccentricity of the orbit
P = period of orbiting
/ = time of passing the perigee
t

ty the satellite.

Here a, 8, P and / are constants.


Find the rate of change in the distance r from the satellite to the
centre of the Earth (i.e. find the so-called radial velocity of the
satellite).

2.6. The Differential of a Function.


Application to Approximate Computations

If

the increment

Ay

=f

Ay

the function y

of

(x + Ax)

f (x)

= f(x)

can be expressed

= A(x) Ax + a (x,

as:

Ax) Ax,

where

lim
Ax

->

(x,

Ax)

= 0,

such a function is called differentiable at the point x. The


principal linear part of this increment A(x)Ax is called the diffeAx.
rential and is denoted df (x) or dy. By definition, dx
For the differential of the function y
f(x) to exist it is neceA(x).
ssary and sufficient that there exist a finite derivative y'
The differential of a function can be written in the following way:

then

dy
1

llite

The perigee

= y' dx = f

of the satellite

to the centre of the Earth.

orbit

is

{x)dx.

the shortest

distance

from the sate-

Differential of a Function

2.6.

For a composite function


retained in the form

dy

= f(u)

= y(x)

123

the

differential

is

= f (u) da

the form of the differential).


to infinitesimals of a higher order than Ax
takes place. Only for a linear
formula

(the invariance of

With an accuracy up
the

approximate
Ay^dy
= ax b do we have Ay=dy.

function y

orders of the function y


sively determined in the following way:
Differentials of higher

d 2 y=d(dy); d 3 y=d(d 2 y)
If

= f(x)
d

But

and x

is

y = y" {dx)

2
\

an independent variable, then


d3y

where u

if

are succes-

= d(d"~^y).

dny

= f(x)

= y" (dx)\
(x),

cp

.
.

then d 2 y

dny

= y< {dx)\

= f" (u) du

2
-\-f (u) d u, and

so on.
2.6.1.

Find the differential


y = In

Calculate dy at x-=

+e

( 1

i+,io*

and

the function

+ arc tan e bx

10x
)

d^^0.2.

(i-fgiox)^

Substituting *

0;

of

e *xy

_ 5g 5x (2g5-y
1+gio*

-I

l+gio*J
d.x:

= 0.2,

we

1)

get

dtf|x=0; djc=0.2

2.6.2.

= y0.2=0.5.

Find the increment and the differential

at the point

x=

at

of the function

y = 3x* + x\
Ax = 0.1.

Find the absolute and relative errors allowed when replacing the
increment of the function with its differential.
Solution.

Ay =

[3 (x

+ Ax) + (x + Ax)
3

1 ]

(3a:

+ x =
Ax + 9x Ax + 3 Ax + Ax,
1 )

= 9x
dy = (9x +
2

I)

Ax.

Whence

Aydy = 9* A* + 3Ax
2

At

a:

and Ax

= 0.1

we

3
.

get

Aydy = 0.09 + 0.003 - 0.093,


dy=l; Ay

= 1.093.

Ch. II. Differentiation of Functions

124

The absolute

error
|

093

Ay

dy = 0.093, the relative error

&y

0.085 or 8.5%.

1.093'

2.6.3. Calculate

approximately the increment

y
x changes from 5

as

kydy

=x

of

the function

7x + 8
2

to 5.01.

2.6.4. Using the concept of the differential, find the approximate


value of the function

= y(x+Ax) y(x)
y(x + Ax) = y(x) + Ay,
Ay

Solution. Notice that from

putting

or,

Ay

dy,

we

get

y(x+Ax)^y(x) + dy.
In our problem

us put x

let

y'(0)=

i,

d^

=
\2

and A# = 0.15. Then

x)

(2

+ x)

'

= -g-.0.15 = 0.03.

Hence,

The

(0. 15)

y (0) + dy = 0.03 - 0.97.


1

true value of y (0.15)

2.6.5.

= 0.9702

(accurate to

Find the approximate value

10" 4 ).

of:

cos31;
(b) log 10.21;
(c) j/33;
(d) cot4510'.
Solution, (a) In solving this problem we shall use the formula (*)
jt/6,
rc/180, we compute:
of the preceding problem. Putting x
(a)

/x

^(^)

y
cos 3 1
(c)

Put

a;

x)

= cos

= 32; A#=l.

V3
= cos-g-n = -g-;

= _ S in-g=

1 iL = 0.85

+ ^)
By formula

^33^3l + (t/ x); =3 2-l=2 +

(*)

we

get

^= = 2H-l =

2.0125.

Differential of a Function

2.6.

2.6.6. All faces of a copper

125

cube with 5-cm sides were uniformly

the weight of the cube was reduced by


0.96 g. Knowing the specific weight of copper (8) find the reduction
in the cube size, i.e. the amount by which its side was reduced.
Solution. The volume of the cube u==x 3 where x is the length
of the side. The volume is equal to the weight divided by the den0.12 (cm 3 ).
0.96/8
sity: v
p/d\ the change in cube's volume Av
Since Av approximately equals dv and taking into consideration that

down. As

ground

a result

tiv

= 3x

dx we shall have 0.12

Ax

= 3x5

= ^L=

x Ax, whence

0.0016 cm.

Thus, the side of the cube was reduced by 0.0016 cm.


2.6.7.

Find the expressions for determining the absolute errors in


functions through the absolute errors in their argu-

the following

ments:
(a)

(c)

(e)
(f)

= lnx;
= s'mx (0 < < ji/2);
= log(sin#) (0 < x < ji/2);
= log(tanx) (0<x<ji/2).

(b)

= logx;
= tan*

(d)

jk

(0

<x<

ji/2);

Solution. If the function f (x) is differentiate at a point x and


the absolute error of the argument A* is sufficiently small, then
the absolute error in the function y can be expressed by the number

=
(a)

Ay

= \(\nx)'

logarithm
(b)
(e)

is

AX =

i.e.

A*.
I

absolute

the

error

of

a natural

= {\ogx)' A X = A X where M = log e = 0.43429;


A y = [log (sin x)]' A X = M \cotx\ A x
Ay

A^lllogaan^^A^i^A,.

From
is

y'x

equal to the relative error in its argument.

(f)

(e)

and

(f)

it

follows that the absolute error in log tan x


(for the same x and A x ).

always more than that in log sin x


2.6.8.

Find the differentials dy and d 2 y

of the function

7a:
3,
y = 4x
assuming that:
(1) a: is an independent variable;
(2) x is a function of another independent variable.
Solution. By virtue of the invariance of its form the differential
of the first order dy is written identically in both cases:
b

dy

= y' dx = (20x

4x) dx.

Ch.

126

Differentiation of Functions

II.

But in the first case dx is understood as the increment of the


independent variable Ax (dx = Ax), and in the second, as the differential of x as of a function (dx may not be equal to Ax).
Since differentials of higher orders do not possess the property
2
of invariance, to find d y we have to consider the following two
cases.

Let x be an independent variable;

(1)

d y

y"

dx

= (8(k

then

14) dx

2
.

Let x be a function of some other variable. In this case

(2)

d*y

Find

2.6.9.

= (8(k

14) dx + (2(k 14*) d


2

differentials

x.

higher orders (x an independent va-

of

riable):

= 4"* find d y\
find
y = V~\n x 4;
=
sm
find
x\
d
y.
y
2

(a)

*/

(b)

(c)

2.6.10. y
(b)

2.6.11.
4

yjir

d 2 y\

3
.

= \nj^

function
x
tant.

is

d2 y

find

of

if:

another

(a)

is

an independent variable,

variable.

Consider

the particular

when

case

The

Find

volume

the

of

increment

and

sphere

of

differential

radius
of

the

is

equal

to

volume and

explain their geometrical meaning.

gt 2 /2.
2.6.12. The law of the free fall of a material point is s
of
a
moment
the distance at
t
Find the increment and differential
and elucidate their mechanical meaning.

2.7.

Additional Problems

Given the functions: (a) f(x)


\x\ and (b)
derivatives of these functions exist at the point ^

2.7.1.

Do

cp (a;)

x3

[.

= 0?

that the curve y = e\*\ cannot have a tangent line


x = 0. What is the angle between the one-sided tangents to this curve at the indicated point?
2.7.2.

at the

2.7.3.

Show

point

Show

that the function


f(x)

where q(x)

is

tive at the

point

/; (a).

= \xa\y(x),

a continuous function and (p(a)=^=0, has no derivax


a. Find the one-sided derivatives /1(a) and

Additional Problems

2.7.

Given the function

2.7.4.

x2 s\n(l/x)

at

x^=0,

at

Use

127

to show that the derivative


not always a continuous function.

example

this

function

= 0.

is

of

continuous

is

continuous

Let

2.7.5.

-^

\ ax

>

+b

if

a:

^x

if

a:

>

a*

Find the coefficients a and b at which the function


and has a derivative at the point x
.

By

2.7.6.

differentiating the formula cos

deduce the formula sin

From

2.7.7.

(b)

3a:

x sin x

sin

sum

of the

= cos

3 cos x sin

3
a:.

geometric progression

the following sums:

for

+nx n ~
+ 2x + 3x +
+n xn -\
+ 2 x+3 x +
2

\
\

the formula for the

deduce the formulas


(a)

= 3 cos

3a:

2.7.8.

Prove the identity

cosx + cos3*+
and deduce from
sin
2.7.9.

'

Find

+cos(2tt 1)a:=

y'

#-/(sin 2

(c)

{x)

*)

the formula for the

x + 3 sin 3x +

(a)

= logcp

it

(2n

(b)

s'

n2nx

2 sin x

x^-kn
^

sum
1)

sin (2n

l)x.

if:

+ /(cos

(x)

(x)

(cp

x);

> 0;

= f(e*)e';
(x) > 0).
=

2.7.10. Is it reasonable to assert that the product F (x)


f (x) g(x)
has no derivative at the point a:
a:
if:
(a) the function f (x) has a derivative at the point x 0l and the
function <p(x) has no derivative at this point?
(b) neither function has a derivative at the point x ?
Consider the examples: (1) f(x)=x, g(x)
\x\;

(2) /(x)

= |*|.

Is it reasonable to assert that the


derivative at the point x
x if:

g(x)=

sum F

(x)

\x\.

= f (x) + g (x)

has no

Ch. II. Differentiation of Functions

128

has a derivative at the point x


(c) the function f (x)
function g(x) has no derivative at this point?
(d) neither function has a derivative at the point x ?

and the

2.7.11. Prove that the derivative of a differentiate even function


an odd function, and the derivative of an odd function is an
even function. Give a geometric explanation to these facts.
is

2.7.12.

period
2

J A3.

Prove that
is

function with

the derivative of a periodic


periodic function with period T.

Find F'

(x)

if

F(x)

xd

X*

2x 3x2

6x

2.7.14. Find the derivative of the function y


graphs of the given function and its derivative.

= x\x\.
=

Sketch the

2.7.15. Suppose we have a composite function y


f(u) where
u^=(p(x). Among what points should we look for points at which
the composite function may have no derivative?
Does the composite function always have no derivative at these
u2 u
\x\.
points? Consider the function y

2.7.16. Find y" for the following functions:


(a)

= \x*\\

(b)

x 2 sin (l/x), x

at
Is

=7^ 0,

= 0.

there #"(0)?

2.7.17. (a)

f(x)=x n show
;

/'(l)

that
(2)

d)

= 2n

n\

(b) f

(x)=x

n~1

l/x

show that

(n=l,
2.7.18.

y=x e~ x/a
2

...).

show that
n
(\) n(n-

an

i)

~2

atcs\nx satisfies the relation


that the function y
Find y in) (0) (n
2) by applying the Leibniz forto both members of this identity.

2.7.19.

Show

(\x 2 )y" = xy
mula

2,

2.7.20. Prove that the

>

Chebyshev polynomials

Additional Problems

2.7.

129

the equation

satisfy

(1

-x

r; (x)-xT'n

+n

T n (x) = 0.
2

The derivative

2.7.21.

(x)

of the

the form

(e-x

nth order of

the function e'*

has

y>= e - x2 H n (*),

where H n (x) is a polynomial of degree


Hermite polynomial.
Prove that the recurrence relation

the Chebyshev-

called

H n+l (x)-2xH n (x) + 2nH n _ (x)^0

2,

...)

valid.

is

that there exists a single-valued function y=y(x)


x and find its derivative y'x
3y

Show

2.7.22.

defined by the equation y*

Single

2.7.23.

inverse function x

out the single-valued continuous branches of the


4
2x 2
x(y) and find their derivatives if y

^-\
= ^r\n^
r
2
\v

2.7.24. u

check the relation

=
^^
dvdu

1.

2.7.25. Inverse trigonometric functions are continuous at all


points of the domain of definition. Do they have a finite derivative at all points of the domain? Indicate the points at which the
following functions have no finite derivative:
(a)

= arccos^y?-;

(b)

*/

= arc sin
=

Show

that the function y


y(x), defined parametrically:
2
is
differentiate at f
but its derivat
tive cannot be found by the usual formula.
2.7.26.

= 2t \t

= + t\t\

2.7.27. Determine the parameters a, b, c in the equation of the


bx
ax2
c so that it becomes tangent to the straight
1
and passes through the point (
x at the point
line y
1, 0).

parabola y

Prove that the


f(x)sinax, where f (x)
gent to each other at the

Show

>

curves y 1
0) and
f(x) (/ (x)
y2
differentiate function, are tana

2.7.28.

is

common

points.

(x 09 y ) of the equilateral
any point
the segment of the normal from the point
to the point of intersection with the abscissa is equal to the
radius vector of the point M.

2.7.29.

hyperbola

x2

2.7.30.

tangent

Show
line

= a(l cost)

[at,

that

y = a

for

any position of the generating circle the


sin/),
normal to the cycloid x=a(t
pass through the highest (at, 2a) and the lowest
that

and

for

0) points of the circle,

53148

the

respectively.

130

Ch. II. Differentiation of Functions

2.7.31.

(I

Show

coscp)

2.7.32. Let
equality

that

two

cardioids

intersect at

= f(u),

= a (1 -f-coscp)

and

right angles.

where u

= y(x).

Prove the validity

= \"' (u) du + 3/" (u) du d*u + f (u) d*u.


y = f(x), where x = <p(t); the functions

of

the

d*y

2.7.33. Let
cp(0 are twice differentiable and dx=^=0. Prove that

_
~

f (x)

and

d 2 ydxdyd 2 x

dx*

differentials forming the right member of the relation


are differentials with respect to the variable t.

where the

2.7.34.

How

will

the expression

<-*>
be transformed
we introduce a

(where y

is

S- 4?+ y
twice differentiable function of x)
variable
putting x
cost?

2.7.35. In determining an electric current by means


galvanometer use is made of the formula
/

where

=k tan

if

new independent

of a tangent

(p,

= current
k = factor of proportionality (depending
= angle of pointer deflection.
/

on the instrument)

cp

Determine the relative error of the result which depends on the


inaccuracy in reading the angle cp. At what position of the pointer
can one obtain the most reliable results?

Chapter

3
APPLICATION OF DIFFERENTIAL
CALCULUS TO INVESTIGATION
OF FUNCTIONS

3.1.

Basic Theorems on Differ entiable Functions

Fermat's Theorem. Let a function y


f(x) be defined on a certain interval and have a maximum or a minimum value at an interior point x Q of the interval.
If there exists a derivative f (x ) at the point x 0y then f'(x )
0.
Rolled Theorem. If a function f (x) is continuous in the interval
[a, 6], has a finite derivative at all interior points of this interval,
b)
and f(a)
f(b), then inside [a, b] there exists a point

0.
such that f'{l)
Lagrange's Theorem. If a function f(x) is continuous in the interval [a, b] and has a finite derivative at all interior points of
the interval, then there exists a point g(a, b) such that

f(b)-f(a)

= (b-a)f.

Test for the Constancy of a Function. If at all points of a certain interval /'(x)
0, then the function f (x) preserves a constant
value within this interval.

Cauchy's Theorem. Let (p (x) and ip(x) be two functions continuous in the interval [a, b] and have finite derivatives at all interior points of the interval. If these derivatives do not vanish sithen there exists (a, b) such that
multaneously and (p (a)
<p (b)
9

t|)(fr)-t|)(a)

^tK (E)

<p(&)' <p(a)

<p'(g)

'

3x2
Does the function f(x)
satisfy the condition of
1
the Fermat theorem in the interval [1, 2]?
Solution. The given function does not satisfy the condition of the
Fermat theorem, since it increases monotonically on the interval
[1, 2], and, consequently, takes on the minimum value at x = 1 and
the maximum one at x = 2, i. e. not at interior points of the interval. Therefore, the Fermat theorem is not applicable; in other
words, we cannot assert that /'(l) = /'(2) = 0. Indeed, ^ (1)
6,
/'(2)=12.
3.1.1.

5*

Ch. III. Differential Calculus: Investigation of Fund's

132

3.1.2. Do the following


Rolle theorem?
(a)

f(x)

(b) f(x)
(c)

functions

the

satisfy

conditions

of

the

= \-Vtf in [-1, 1];


= In sin x'm [ji/6, 5ji/6];

/(*)= 1-|*|

[-1,

in

1].

they do not, explain why.


Solution, (a) The function is continuous in the interval [
furthermore, / (
0. Thus, two conditions of the
1)
/(1)
If

theorem are

satisfied.

The derivative

f'(x)

# = 0.

= 2/(3 y/ x)

1,

1];

Rolle

exists

at

points except
Since this point is an interior one, the third
condition of the theorem is not satisfied. Therefore, the Rolle theorem is not applicable to the given function. Indeed, /'(jk)^0 in
all

[-1.

!]

Prove that the equation

3.1.3.

3x 6

+ 15* 8-0

has only one real root.


Solution. The existence of at least one real root follows from the
3* 5
15*
that the polynomial f (x)
8 is of an odd power.
Let us prove the uniqueness of such a root by reductio ad absurdum. Suppose there exist two roots x 1
x 2 Then in the interval
3x b
I5x
8 satisfies all conditions of
[*!, x 2 ] the function f (x)
the Rolle theorem: it is continuous, vanishes at the end-points and
has a derivative at all points. Consequently, at some point \,x x
x2
\
But
(x*+
This
contradiction
0.
/'(*)=
0.
15
that
proves
1)
f'(t)
the equation in question has only one real root.

fact

<

< <

>

Does the function f(x) = 3x2


5 satisfy the conditions of
the Lagrange theorem in the interval [
If
it
does, then
2, 0]?
find the point g which figures in the Lagrange formula f (b)
f(a) =
= f(E) (b-a).
Solution. The function satisfies the conditions of the Lagrange
theorem, since it is continuous in the interval [ 2, 0] and has a
finite derivative at all interior points of the interval. The point
is found from the Lagrange formula:
3.1.4.

f> /tx
/

whence
3.1.5.

\)

_
At _ /(Q)-/(-2)
0 ( 2)
ufc

-5-7 _ __ R
~"

'

1.

Apply the Lagrange formula to the function f(x)=\nx


[1, e] and find the corresponding value of .

in

the interval

3.1.6. Ascertain that the functions f(x)


3
7x 2
20x
5 satisfy the conditions

=x

in the interval

[1,

4]

and

find

=x
of

2* + 3
the

and g(x) =
Cauchy theorem

the corresponding value of

3.1. Basic Theorems on Different iable Functions

133

Solution. The given functions f (x) and g(x) are continuous everywhere, and hence, in the interval [1, 4] as well; their derivatives
2
\4x
20 are finite everywhere; in
f (x) 2x 2 and g' (x) 3x
addition, g' (x) does not vanish at any real value of x.
Consequently, the Cauchy formula is applicable to the given

functions:

/(4)-/(l)

= nE)

g'iQ*

S(4)-s(l)
i.

e.

27

14g + 20

3g 2

Solving the latter equation,


i 2 =4.

Do

On

3.1.8.
line

is

= ex

the functions f(x)

ditions of the

is

Cauchy theorem

two values

find

^=2

Of these two values only


3.1.7.

we

^6^

an interior point

and g

in the

(x)

= p-X

interval

of the interval.

satisfy the con-

A(

tangent

the
1,

1) and

1, 2], whose end-points are the abs[


B, the function y
x 3 is continuous and
derivative; therefore the Lagrange theorem is applicable.

In the interval

cissas of the points

has a finite

and

x* find the point at which


the curve y
to the chord through the points

fl(2, 8).

=2

[3, 3]?

parallel

Solution.

:E 1

of

A and

According to this theorem there will be, on the arc AB, at


one point M, at which the tangent is parallel to the chord
Let us write the Lagrange formula for the given function:

= /'(g)[2-(-l)]

/(2)-/(-l)

least

AB.

or

+ = 36
1

3;

whence
1.

Ex

The obtained values

we

I,=

l.

the abscissas of the

of are

desired

points

and S- 2 in
there exist two such points). Substituting
the equation of the curve, we find the corresponding ordinates:

(as

see,

yi

l'i

l;

y2

=U=

l.

Thus, the required points are:


1) and Af 2 ( 1, 1), of which
y (\,
only the former is an interior point on the arc AB.
Note. This problem can be solved without using the Lagrange theorem; write the equation of the chord as a straight line passing
through two given points, and then find the point on the curve at
which the tangent is parallel to the chord.

Ch. III. Differential Calculus: Investigation

134

Fund's

of

Taking advantage of the test for the constancy of a funcdeduce the following formulas known from elementary math-

3.1.9.
tion,

ematics:
(a)

arc

sinx+arc cosx =

2
(b) sin x

(c)

ji/2;

= (l cos2x)/2;

arc cos

-r-,

2 arc tan

a;

O^Zx <oo;

at

defined in the interval f 1, 1]. The derivative


function inside this interval equals zero:

According to the

test

e.

constancy of

for the

C
arccosA:
arcsin A:
To determine the constant
we have jx/2 C, whence
i.

the

of

a function

( 1<#<1).
C let us put,

indicated

/ (x)

for instance,

= const,

x=0;

then

arc sin

The validity

a:

+ arc cos = jt/2


a:

this

of

equality

<x<

the

at

points

1).

a:=1

is

verified

directly.
(b)

Let us take the function


/ (x)

number

defined throughout the


of this function

is

= 2 sin

(a;)

test

for the

sin 2

To determine C

+ y cos

2a;

oo < <

scale:

a;

oo.

The derivative

everywhere equal to zero:

f
According to the

= sin

a:

cos

sin =
2a;

a;

constancy of a function

+ y cos

put, for instance, x

2a:

= C.

= 0;

then

W herefrom
sin 2

0.

x+y cos2x = y

we

get

1/2

= C.

3.1.

Basic Theorems on Differ entiable Functions

or
.

s\n 2 x

2x
= cos
^
1

Let us introduce the function

(c)

== arc cos

/ (x)

2 arc tan

l~*

The derivative
x

4a;

( 1

test

To determine C

2x (

constancy

of a

<

At x

its

for the

us put, say, x

let

1,

proved formula

of the

the function arc cos

derivative

2
x~)

=^

function

>

0.

which gives

C = arccos0

which enables us

formula can
*
I

is

no

has

r-j
r
2
1 -\-x

verified

directly.

derivative.

At

\'

to derive the formula

arccos -j-p^2
latter

at

is
l

that arc cos

0:

at

arc cos

The

~|-

1.

=0.

The validity
Note,

>

all

4x

2 arc tan x = C

arc cos t

2 arc tan

<

According to the

~Y

+7

* a)a

^2

zero for

is

since

scale,

of the function f (x)

~ ~~~j/7

x,

determined along the entire number

135

is

be

arc tan

(x

< 0).

on the strength

obtained

an even function, and 2 arc tan x

is

of

the fact

an odd one.

3.1.10. As is known, (e*)'=e* for all x. Are there anymore


functions that coincide with their derivatives everywhere?
Solution. Let the function f (x) be such that f'(x)^f(x) everywhere.
Let us introduce the function

V(x)

The derivative

of this function equals zero


cp'

By
f(x)

= 1g- = f(x)e-*-

(x)

=f

x
(x) e-

e- x

the test for the constancy of a

= Ce

x.

f {x)

everywhere:

function

0.

f{x)/e

= C,

whence

Ch. 111. Differential Calculus: Investigation of FuncVs

136

And

so,

we have proved

f'(x)^f(x)

group of functions

the

that

f(x)^Ce x

covered by the formula

is

which

for

3.1.11. Prove the inequality

tanx 2

arc

where x 2

>

arc tan x

xr

Solution. To the function


apply the Lagrange formula:

where x x

<<x

<

x2

= arc ianx

/ (x)

arc tan x

arc tan x 2

on the interval

lt

[x l9

x2

i)>

Since

<T^F < and ^2 ^i>0,


arc tan x arc tan x < x x
1

then

putting

In particular,

^=

and x 2

arc tan

<

= x,

(x

we

get

> 0).

3.1.12. Show that the square roots of two successive natural


numbers greater than N 2 differ by less than 1/(2 A/).
Solution. To the function f(x) = Vx on the interval [n, n+1]

apply the Lagrange formula:

/(n+l)-/(n)
where n
If

<| < n+

n>/V

2
,

= /^-|/"n=^,

then

1>N\

<

hence 1/(2

y n+ \y n<

1/(2jV),

whence

\/(2N).

3.1.13. Using the Rolle theorem prove that

the

derivative

(x)

of the function
c

f(*)

x sin

at

at

a:

i
(

>
=

0,

vanishes on an infinite set of points of the interval (0,


Solution. The function f (x) vanishes at points where

n/x^kn,

sin (n/x) ^=0,

k=
Since

the

function

of the interval

[0,

f (x)

1],

the

1,

2,

3,

l).

xl/k,

...

has a derivative
Rolle theorem

at
is

any interior point


applicable to anyone

3.1. Basic Theorems on Differ entiable Functions

of

the

intervals

[1/2,

[1/3,

1],

Consequently, inside each

[l/(k+\)

1/2],

137

...

l/k],

the intervals of the sequence, there is


a point l k
1 /(+')< l k
1/6. at which the derivative f'(l k )=0.
And so we have shown that the derivative vanishes on an infinite
set
points (see Fig. 38).
of
3.1.14. The Legendre polynomial is
a polynomial defined by the following
formula (Rodrigues' formula):
of

<

2nn

(X

dx

(Al

= 0,

...).

2,

1,

1)"

theorem, prove
tneorem,
Using the
Rolle
Koiie
that the Legendre polynomial P n (x)
has n different real roots, all of them

between

found

f(x)

= (x

Consider

Solution.
2

\)

and

-f

1.

the function

= (x

l)

(*+l)".

This function and its n


1
successiFig. 38
ve derivatives vanish at the points
x
(use the Leibniz formula for
higher derivatives of the product of two functions).
It follows from f(l)
that inside the interval
f(l)
1, 1]
a point ^ can be found at which f (l l )
i.e. x
l 1 will be the
root of the first derivative. Now apply the Rolle theorem once again
to the function f'(x) on the intervals [
find
1]. We
1, gj, [| x
that besides +1 and
the function f" (x) has two more roots
on the interval [
1]. Reasoning as before, we will show that,
1,
apart from +1 and
1, fhe (n~l)th derivative has (n 1) more
(n ~ v
roots on the interval [
(x) has all in
1,
1], i.e. the function f
all n+l roots on the interval [
1], which divide this interval
1,
into n parts. Applying the Rolle theorem once again, we ascertain

= \

[.

that the function

in)

(x),

and hence, the function

has n different roots on the interval


3.1.15.

1,

P n (x)= ^jj

in)

(*),

1].

Check whether the Lagrange. formula

is

applicable to the

following functions:
(a)

(b)
(c)

x2

on [3, 4];
on [1, 3];
5x 2 + x
4r*
2 on
f(x)^4x*
f(x)-f(x)--

-\nx

[0,

1];

(d)
If

f(x)=l/x* (x-\) on [-1/2,


it

is,

find

1/2].

the values of appearing in this formula.

3.1.16. Using the Lagrange theorem estimate the value ln(l-j-t?)

Ch. III. Differential Calculus: Investigation of Fund's

[38

3.1.17. Using the Lagrange formula prove the inequality

< \n(l+x)<x

Y^rx

at

>

jc

0.

Evaluation of Indeterminate Forms.

3.2.

L'Hospital's Rule
Indeterminate forms

I.

of the

type

~.

functions

the

If

(x)

and g(x) are differentiate in a certain neighbourhood of the point a,


except, may be, at the point a itself, and g (x)=^0, and if
f

lim
x

/ (x)

lim g(x)
x

-+

-*

lim

or

->

(x)

lim g(x) =- oo,


x

-y

then
(X)

,.

1-

f
lim '-)-{= lim

g(x)
a

f (x)
,

II.

_>

lim
exists (U Hospital's rule). The point a
- a 8
oo.
be either finite or improper -f oo or
oo are reduced
Indeterminate forms of the type O oo or oo

provided the limit

may

(X)'

g'W

to forms of the type

-jj-

or

/
w

by algebraic transformations.

Indeterminate forms of the type l 30 oo or 0 are reduced


forms of the type 0-oo by taking logarithms or by the transfor-

III.

to

mation

[f(x)]

3.2.1.

wing

Applying the L'Hospital

lim
*

pw = evwinfw

_ o

x_ e -x.__2x
xsmx

limits of the follo-

sin
1171

3a:

x^=0

a:

2
)

e~ x
/*

lim

(f)

'

both

/ (x)

~
2

75

functions f(x) = e ax
e~ 2ax and
in the neighbourhood of zero, since

=
1

0;

lim
x

->

Furthermore

infinitesimals

lim
x

lim
Q
os3a:
^ ^ Q C

Solution, (a) Here


are

(d)
\

Incns^-*!

=ln(l+A:)

In (1

point

find the

rule,

functions:

(c)

/'

(x)

and

exist

g (*)

In

= 0.

->

in

any neighbourhood
x- 1, and

that does not contain the point

of

the

Evaluation of Indeterminate Forms

3.2.

139

Finally, there exists a limit of the ratio of the derivatives:


/' (x)

,.

Inn

^ttt

lim

Therefore the L'Hospital rule

(\+x)

r:

x>

oa.

= 3a.

ae ax +2ae~ 2ax
r-Tf

lim

applicable:

is

In

T~n~\

e ax_ e -2ax

lim

ae ax +2ae- 2ax

,.

(*)7

1/(1+*)

When

/Vote.

the limit of the ratio is computed according to the


is usually written directly as shown in (*).
the desired derivatives and limits exist is ascertained in

L'Hospital rule the result

Whether

In case the ratio of the derivatives

the course of calculation.

'

again represents an indeterminate form, the L'Hospital rule should


be applied for a second time, and so on until the indeterminacy is
removed or until it becomes clear that the required limits do not
exist. Therefore, henceforward we write only the necessary transformations, leaving to the reader the task of checking whether the
conditions of their applicability are fulfilled.

j/T
+ 2^+1
v

lim

(b)

In

+ x+x

cos (2x 2

(4a:

limit of the

factor

first

3a:
1)

cos

lim
x - o

3a:

(2a:

cos
A

4a:

x)

x)

(2a:

'

cos (2x 2

sin

computed

is

J^2 + x)+l

6a: cos

,.

= 6n
The

1/(2

x)

-=ir

3/0+25)*)
2/(3 v

lim

2
sin 3x

v/

'

l/2

a:)

directly,

i .

lim
x ^ o

sin

(2a:

x)

the limit of

second one, which represents an indeterminate form of the type

found with the aid

6
~

cos

3a:

lim
x - o

cos
-=

4a:

of

(2a:

the L'Hospital

x)

'

the
-jj-

is

rule:

,.

lim -r
\

sin
x ^
= 6
xlim (4a: n cos
(2a:

a:)

j1

1)

_^

(2a:

9
2

\r =
x) = 6 1-1

x>+oo,

>

6.

the functions x k (k
0);
loga Jc; a* (a
1) are infinitely large quantities. Applying the L'Hospital rule, compare these quantities.
3.2.2.

It

is

known

that,

as

>

,.

2.

lim

1.

=
x rn

lim
lim

lim

mx
.

rn

, .

jloga*

0g x

-^f- =
l

Solution.

_l
,

lim

=
i

= \ogae

lim

^ =

m\
-7-71

kx*
r,

0-

0;

Ch. III. Differential Calculus: Investigation of Fund's

140

function x k (k>0) increases more rapidly than


the logarithmic function \og a x(a> 1), and the exponential function
a x with the base exceeding unity increases more rapidly than the
power function x rn

Hence, the power

Find the

3.2.3.

<

limits:

(cotx-

Hra

(b)

a>

lim

(c)

e*

We

Solution, (a)

Let us reduce

have an indeterminate form

it

apply the L'Hospital


,.

lim

x \J

^^jVlnA:

r
lim

x 1 In* =

1) In

r.

(x

lim
x ^

_^

(a)

lim x
x

<b)

lim x n In

\nx(n

fin (1

>

+ sin
We

(a)

a:

=. lim
x

->

We

lim [in

1/* =
[/x
,

In

*T
+

* In *

lim

^ ^

In

a:

-|-2

2'

limits:
0);
2

x) cot In

(1+ x)].

have an indeterminate form of the type 0-co.

Let us transform

(b)

and then

->

Solution,

at

oo.

->

lim
x

IJ

Find the

rule:

= m
3.2.4.

of the type oo

to an indeterminate form of the type

(1

it

- =

-+

In

to

and then apply

- =\\m
1

lim
x

the L'Hospital

fx

" *

->

have an indeterminate form

+ sin- x) cot ln> + *)] =


(1

-+

the type 0-oo:

of

Jim

sin 2x

2 {1 -{-tan 2 [In 2

ln(l+x)

(1

1+*

sin*
= hm ln(l+x)
=

,.

lim
^ Q

1+a:
3.2.5.

Find the

limits:

sin
(a) lim (l/x)
x -* +

*;

lim

(b)
x

-*

rule:

x n = 0, since rc>0.

x^^^-".

Solution,

We

(a)

Let y -~=(\/xY'mx

Evaluation

3.2.

an

have
then

In

\ny=

lim
x-^-fO

lim \nu

x^ + o
Hence, lim y
->

to

it

the type oo.

In (1/x),

form

the type 0-oo).

of

and apply the L'Hospital

=
In

lim

-;

a:

x^ + oh'smx

= e" =

,.

lim
x

\/x
;

(cos x)/sin 2 x

= hm
x

_^

rule:

sin 2 x

xcosx

= A0.

l.

-I

Find the

3.2.6.

of

->

Let us transform

= s\n x

141

form

indeterminate

lim sinxln(l/x) (indeterminate


x

i-

Indeterminate Forms

of

limits:

lim (sin x)

(a)

tan

lim**.

(b)

^;

X-7C/2

x-0

Compute

3.2.7.

->

cot *.

(tanx)

lim
x

+ Jt/2-0

Solution. Let us take advantage of the identity


co *

(tan

= co

'

an x

but

\anx

cot X In

lim

lim
JC-+jt/2-0

x-*+jt/2-0

" tan x
tan *

]Hl =z Q

ylm
|/=tan *

->

oo

Whence
(ianx) coix

lim
x->

= e =

1.

+jt/2-0

3.2.8. Ascertain the existence of the following limits:

lim

(a)

->

,.

lim
(b)
v

x 2 sin (1/x)
sin

24-2aH-

sin 2x

*-*oo(2x+sin2;t)6>
tan

rj
s,n *

a:

lim

(c)
X

-+

31/

Can the L'Hospital rule be applied

in

computing them?

formal application lead to the correct answer?


Solution, (a) The limit exists and equals zero. Indeed,

Does

its

li

of
i .

lim

jj

smx

x-*0

But the limit

_ m

f__smJJ_/)

x-+0

smx

ii

m^s

n _L
x

x->

0.

the ratio of the derivatives does not exist. Indeed,


2x sin

=
(
K

fx)
1

cos

fx)

lim cos
, .

Ch. III. Differential Calculus: Investigation

142

but lim cos


-

does

(1 /'x)

not

hence

exist,

Fund's

of

L'Hospital rule

the

is

not

applicable here.

The

(b)

limit of the ratio of the functions does not exist:


2

Hm
*->

+ 2,4-sin2,

oc(2*+sin2*)<?

sin

,.x\

2
{

2xy

2.v-|-sin

'

but lim e~ sinx does not exist, since the function e~ Anx traverses the
X

->

GC

values from

Now we

cc

[2

+ 2 cos

+ (2x

2a;

many

infinitely

show that the


2 -f 2 cos

r
lim
x->

1/e to e

will

-|-

2a:

sm

sin 2x) cos x| e


4

lim
x ^ x

times.

limit of the ratio of derivatives exists:

.,

4 cos "

cos- x

/n
#+(2x+sin
.

=
since the function e~* inx

is

2a) cos

a'

* C S *

lim
v - x

+ 4 cos
4

bounded, and

-j- si

A'

n Q
2x

g -s.nx

cos A

x+

2.V+4 cos

= 0>
*

o.

sin 2a-a:-^oo

Here cosx, which vanishes for an infinite set of values of x has


been cancelled out. It is the presence of this multiplier that makes
y

the L'Hospital rule inapplicable in this case, sine? it simultaneously


the functions being compared.

nullifies the derivatives of


tan

lim

(c)
*

-*

jt/2

a-

SeC x

sec 2 x

,.

lim

*-*JT/2

Here application

of the

SCC X tan X

,.

sec a

urn

x-+n/2 tan x

tan a

Inn

SeC x

...

L'Hospital rule gives no useful result, though

there exists a limit:

tan*

hm

sec

x->jt/2

lim
x->n/2

sin* cos
cosx

lim sin x

1.

Jt->.-r/2

Using the L'Hospital rule

3.2.9.

a;

find the limits of the following

functions:
v

In

'

-*

(a:

cin v

lim arc sin

(g)

Hm f-M
+ V * /

(i)

(d)

Hm

/Ux

10

Xnx

'

x
=

i-4s^(^/6)

'

(e)

3)

+ 3*-

<?H1=.

lim

(c)
v

,3

^ cot

(x

a)\

(f)

lim (n

2 arc tan x)

limfa

/*

tdn

*;

lim (cos mx)"'*

(h)
*
2
;

(j)

->

1)* (a

>

In x;

0);

GO

Mm (2

tan (jiA-/(2a))

Approximate Calculations

3.3. Taylor's Formula:

In

cot 2 X

(m) lim

lim^/ |n <^-

(1)

In

a:

>;

143

(n)

lim

(P)

lim

x x

In

+y

->

l/sin a

lim

(o)

2
a;

cosh

\2+

V~9-{-x
J/x*
1

-.0

lim (lncot^) tan

(q)

lim

(r)

*;

2 arc tan x 2

'

3.3. Taylor's Formula. Application to Approximate


Calculations

If the function
through order n
of the nth order

[a,

b]

f(x)=f

is continuous and has continuous derivatives


on the interval [a, 6], and has a finite derivative

f (x)
1

at

every

point of the

interior

the following formula holds

(a)

+
+

(a)

(x-a)+

(a)

interval then at

true:

f" (a)

-^^ + r

where
l

= a + Q(x a)

<9<

and

called Taylor's formula of the function

It

is

If

in this

fw=f (0)

formula we put a

(0)

= 0,

+ r (0) J +

accordingly, the

^ la+

remainder

' ls

in the

&)%,

The last term in the Taylor formula


Lagrange's form and is denoted R n (x):

R.(x)=

f (x).

we obtain Maclaurin's formula:

ln}

1.

(0)

where g=6*,

is

tt)]

jj^L

0<6<

1.

called the remainder in

ix-ar;

Maclaurin formula has the form

=x

+ 2x

b
x* 2
the polynomial P (x)
2x*
1
the binomial x
using the Taylor formula.
1
Solution. To solve the problem it is necessary to find the value
and its derivatives at the point x=\. The
of the polynomial

3.3.1.

in

powers

Expand
of

Ch. III. Differential Calculus: Investigation of Fund's

144

relevant calculations are given below.

P'(1)

P"'(l)

P (l)-72,
P {n) (x) = (n

(4)

any

at

= 0,
= 18,

P(l)--0,

P"(l)-0,

(5)

(l)=120,

> 6)

x.

Substituting the values thus found into the Taylor formula, we get

P(x)^~(x-ir+^(x-iy + ^(x~\r;
P(x)=3(x iy + 3(x 1)
3.3.2.

+ (jc

l)

5
.

Applying the Maclaurin formula, expand

(up to x 9

powers

in

of

inclusive) the function

= \n(\ + x)

f(x)

defined on the
the remainder.
Solution.

interval

/(0)

The derivatives

of

(0 )

to deleting

= lnl=0.

any order

rw=(-ir

Estimate the error due

1],

[0,

of the

given function

(see 2.3):

{f^i.

= (_i)"-i (_i)! (n=

2, 3, ...).

Substituting the derivatives into the Maclaurin formula, we get


In (1

where
as

the

remainder

R 10 (x)

in the

y + ^io W,

Lagrange form

will

be written

follows:
(*)

in

+ *)=* y + y

(10)

()

10!

10

9!

10! (1

10

g)

x U)
10

(1

H- )

Let us estimate the absolute value of the remainder


1
and I
mind that
0, we have

O^a:^

R 1Q (x)\

*)

keeping

10(1

How many

the function
this function on

for

place

<<

>

l*io(*)h
3.3.3.

10

terms in the Maclaurin formula should be taken


x so as to get a polynomial
e
representing
f(x)
the interval [
1], accurate to three decimal
1,

Solution.

The function f(x)=e x has


(n)

(x)

ex .

derivative

of

any order

Taylor's Formula: Approximate Calculations

3.3.

145

Therefore, the Maclaurin formula is applicable to this function. Let


deriva1
us compute the values of the function e x and its first
tives at the point x
0, and the value of the Aith derivative at the
will have
0jc(0<6< 1).
point

We
f(0) = r (0) = r(0) =
fin)
^ =e

...

=/"'- l (0)=--l;

==e Bx

Whence
/(*)

= +TT+ir+

+(f^. + R ^ x ^

where

if

fulfilled,

(7!

0.001

(*)

then the inequality


I

will

"

the inequality

4- <
is

then

1,

3
= -^^<e< _3_
nl

\Rn{x)\

Hence,

0<9<

and

Since, by hypothesis,

< o.ooi

>

7
be fulfilled apriori. To this end it is sufficient to take n
Hence, 7 terms in the Maclaurin formula will suffice.

= 5040).
3.3.4.

At what values

of

cos x

have an error

less

will

the approximate formula

2T

+ TT

than 0.00005?

Solution. The right member of the approximate equation reprethe first six terms in the Maclaurin formula for the function
cos a: (the second, fourth and sixth terms are equal to zero; check it!).
(6)
cos x then
Let us estimate R Q (x). Since (cosa:)

sents

!*.(*)!

For the error to be

less

cos Qx

<

x6

6!

1*1
6!

than 0.00005, choose the values

satisfy the inequality

-LJ1

Solving this inequality,


3.3.5.

Compute

we

< 0.00005.
get

|#|<

0.575.

the approximate values


(a)

cos

5;

of:

(b) sin 20,

of

x that

146

Ch.

accurate to

III. Differential Calculus:

formula

1 -g. + iL

CO sx=

x=n/36\

+( i)-gL + R 2n+2

since
2

lf=2^ =
we

Fund's

decimal places.

five

Solution, (a) Into the Maclaurin

substitute

Investigation of

""'
Tf=l(-f) = 2 410

003808

'

confine ourselves to the following terms:

cos

a:

x 2 /2

the error being estimated at


cos 6x

<-L^<2.5.10-

4T~

And

50,

within the required accuracy

= cos i
=
00

cos 5
3.3.6.

Compute

approximate

the

= 0.99619.

0.00381
value

Y 83

of

accurate to six

decimal places.
3.3.7.

Prove the inequalities:

(a)

xx

(b)

tanx>

<

+ x)< x

> 0;
< x < jx/2;
l+x ^<]/ T+x<l+-jX
2

/2

In

( 1

+ x /3
3

at x

at
r

(c)

at

0<x<oo.

Solution, (a) According to the Maclaurin formula with the rema-

R2

inder

(x)

we have
In (1

+*)

= * 2(1 + 6)"

where
< |< x.
According to the same formula with the remainder
In (1

0<l
Since

+*)

= * 4"

3(1

+ ^)3

>

(x)

we have

whe re

<x.

YiTTW >

and

xx
3.3.8.

Show

that

>
< In (1 +*)<

an+EO 8

sin (a

/2

>

at

>

jt

follows that

a:.

ft)

differs

from

sina

+ /icosa

more than h 2 /2.


Solution.

By

Taylor's formula
sin (a

+ h) = sin a + h cos a 4r- sin 6;

by not

3.4.

Taylor's Formula: Evaluation of Limits

147

whence
h
+ h) (sina + /icosa) =

h2

'sin (a

sinE

3.4. Application of Taylor's Formula to Evaluation


of Limits
The expression
f(x)

= f (a) + Lw

{x

_ a)+ rvi (x _ ay +
-\-o(\x a\

{x

_ a)n H

is

q
er

Taylor

the
(x)

= o [ty (x)]

order

that, as x

smallness

of

the remainder in Peano's form where


>a the function cp (x) has a high-

formula with

means

In particular, at

than

function

the

i.e.

lim ?4-t

= 0.

we have

fw=/(0)+m, +

m,2+

...

/^ xn+o(Uh

Peano's form of the remainder for Taylor's formula shows that,


the Taylor polynomial of degree n for / (x) in the
neighbourhood of the point a, we introduce an error which is an
n
as
+a.
a)
infinitesimal of a higher order than (x
The following five expansions are of greatest importance in solving practical problems:

when substituting

e*=l+x + -g.+
sin x

(1

= x~ + ...+(-

+ *)- = + ax +

*+

yl

In

3.4.1.
gral

(1+*)

Expand

powers

of

with respect to
Solution.

/(*)=

We

...

+ Jnr+o(x);

j^L. + o (**");

{a
+ ~

l)

^^

'

'

nl

Yn

v-3

= *-- + -j- +...+ (-I)""

^+

x+o

o (x n ).

the function / (x)


sin 3 x
x l e~ x in positive intex up to the terms of the fourth order of smallness
x.

have

[x-^+owy-x* \-x + ^- + o(x*)


= x + (* + X + (X = X -| X* +
2

(*");

(X 4 ).

Ch. III. Differential Calculus:

148

Expand

3.4.2.

Investigation of

Fund's

the following functions:

= xV x cos x In (1 + x)\
(*) = ln(l+sin*)

(a)

f(x)

(b)

positive integral powers of x up to the terms of the


of smallness with respect to x.
in

Applying the Taylor formula with the remainder


compute the limits:

3.4.3.
.form,
,

l'i"

<b)

lira
X

i-

hm

(d)

Peano's

in

cosx

cosx

e x sin

- X,L
x 2 /2

(1

-\-x)

'

hm

(e)

_tan**

lim

(c)

V\+x

(*>

order

fifth

^*

->

Solution, (a) Retaining the terms up to the fourth order with


respect to x in the denominator and the numerator, we get
-

lim

Y +
1

X 2 COS X

tan4 *

11

(1

-f-#

1/2

2
)

COS X

* + o(* J
l+y* + 1/2 (1/2)
)|
2
.

d
4

[,
1

x2

--2" +
,

x4
24

+0( *

.
)

, .

hm

'

24

= hm

'

'

x-+

(x4 )
*

L 3

Expand the following functions in positive integral powers


variable x up to the terms of the indicated order, inclusive:

3.4.4.
of the
(a)

(b)

x)

= e2x ~ x2

In cos

x
(c)

eX

3.5.

a:

up to the term containing x b


up to the term containing x 6

up to the term containing

x*.

Testing a Function for Monotonicity

Let a continuous function f (x) be defined on the interval [a, b]


and have a finite derivative inside this segment. Then:
(1) For f (x) to be non- decreasing (non-increasing) on [a, b] it is
(/' (x)^ 0) for all x in (a, b).
necessary and sufficient that /' (x)

>

3.5. Testing a Function for Monotonicity

(2)

For f(x)

cient to

fulfil

to be increasing
the condition f (x)

Determine the intervals

3.5.1.

on

(decreasing)

>

(/' (x)

<

b]

[a,

is

it

for all

0)

monotonicity

of

149

suffi-

in (a, b).

for the following

functions:
(a)

(b)
(c)

(d)
(e)
(f)

= 2x \nx;
f(x) = 2x* 9a;- 24* + 7;
2

f(x)

/(*)
f(x)

= \n\x\= 4x 21 * +
f(x) = e* + bx.
s

20;

The solution of this problem is reduced


which the derivative preserves its sign.

Solution.
intervals in
f (x)
a
it

18*4

f(*)

has a continuous

number

finite

<x <

preserves

derivative in the interval


of stationary points x u x 2i

< xn <

b) y

where

in

each

sign

its

/'

(x k )

the

of

The function is defined at x


Let us find the derivative

(a)

4x

if

>

(&

intervals

2,

b)

(a,

the function
and has in

xn
.

(a,

(a

x x ),

(jc

<x <

then

n),

/' (x)

x a ),

3 ,

0.

= lx\lx.

f'(x)

The function increases

to finding the
If

l/x>0,

i.e.

>

1/2.

The function decreases if Ax


l/x < 0, i.e. x < 1/2.
And so, the function decreases in the interval
<x

<

1/2

and

the interval 1/2<a:<4(b) Evaluate the derivative

increases in

X)

= 6x

18a:

24 = 6 (x 3x
x=
and x=4.

vanishes at the points


1
quadratic trinomial with a coefficient at
in the
then f (x)
intervals
It

its

4).

Since f (x)
highest-power term 6

>

(_

1),

oo,

(4, oo),

and

/'

is

>

a
0,

(x)<0

interval (1, 4). Conthe


sequently, f (x) increases in the
in

first

(1,
(c)

two
4)

in

decreases.

case the derivative

In this

whereas

intervals,
it

x 2 )e~ x vanishes
(2x
f'(x)
the points x
and x=2.

at

In
the intervals ( oo, 0) and (2, oo)
the derivative /' (x)
and the
function decreases; in (0, 2) the derivative
tion increases (see Fig. 39).

Fig. 39

<

3.5.2.

Find

wing functions:

the

intervals of decrease

f'(x)>0 and

and increase

for

the func-

the follo-

Ch. III. Differential Calculus: Investigation

150

= cos (n/x);
f(x) = smx + cosx

(a)

Fund's

of

/ (x)

(b)

Solution, (a)
throughout the

on

= sin
ji

tj

As

obvious, the

is

2jx].

[0,

The function y = cos (n/x) is defined and


number scale, except at the point x =
ji

of y'

sign

differentiable
0\

with that

coincides

the multi-

of

plier sin (n/x).

>
< n/x <
sin (n/x) <

s\n(n/x)

(1)

if

2kn

1)

1) Jt

<

(2k

(k

1, 2,

..);

if

(2)

(2k -f

<

jt/*

1)

n.

,2Jfe+l

in

intervals

in the

Hence, the function increases

and decreases

2(6

2k

'

the intervals

26

+2

2k-\-\

Investigate the behaviour of the function f(x)


3x in the interval ( ji/2, n/2).
Solution. The derivative
3.5.3.

= 2s\nx +

-ftanx
f/

(a:)
v
7

'

= 2 cos # H
ri

o
3

COS

5
2

COS

(1

*) (1

COS X

2 COS

COS X

_ 4 sin

sin(3A:/2)

(jc/2)

cos 2 x
is

positive

only

at

3.5.4.

in

= 0.

the

intervals

Hence, in

Prove that at
x

n/2,

( k/2,

0<#<1

0)

jx/2)

and
the

(0, n/2) and vanishes


function / (a:) increases.

the inequalities

x /3 < arc tan < x x*/6


3

a:

are fulfilled.
Solution.
is

We will prove only the right inequality (the


proved analogously).

The derivative

of

the function
f (x)

is

= arc tan x x + X

-g-

equal to

/'(*>=_!
/

1+JC 2

gl
2

= *2(l

(*

-i)

+x

2
)

left

one

The function
scale,

f(x)

in particular,

side this segment


[0,

1]

Testing a Function for Monotonicity

3.5.

is
it

is

/' (x)

<

continuous throughout the entire number


continuous in the interval [0, 1], and in0. Therefore, f (x) decreases on the interval

and, consequently, for any point x,

/(x)</(0) =

fulfilled,

tanx

the inequality

1,

x + <
a:

~q

whence

< x

arc tanx
3.5.5.

0<x^

or

arc
is

151

-g-.

Prove the inequalities


x

x /6 < sin* < x


3

at

> 0.

O^p^l

3.5.6. Prove that for


and for any positive a and b
the inequality (a
by^.a p bP is valid.
Solution. By dividing both sides of the inequality by bP we get

or

where x

Let

show

us

that the inequality

(*)

holds true at any positive x.

Introduce the function

f(x)=
The derivative
/'

{x)

of

= pxP'

1+xP (\+x)P; x>0.

this function
1

p(l+x)P' = p

-p

(X+'xy-p

p^O

positive everywhere, since, by hypothesis, 1


and x > 0.
Hence, the function increases in the half-open interval [0, oo), i.e.
whence 1 + xp >(1 +x)p, which
f(x)=l+xP (l+x)P>f{0)
completes the prcof. If we put p=\/n then we obtain
is

3.5.7. Prove that the function y = x b


2x*
x increases everywhere,
x* decreases everywhere.
and the function y=l

3.5.8.

Determine the

intervals

of

increase

and decrease

following functions:
(a)
(c)

= x* + 2x 5;
f(x) = cosxx;
f(x)

(b)

(d)

= In (1 x
f(x) = -jx?~;
f (x)

);

for

the

Ch. III. Differential Calculus: Investigation of Fund's

152

(e)/W = H5;

Prove the following inequalities:

3.5.9.

tan*

(a)

(b) e

> x + x*/3,
l+x

>

(c)

3.5.10.

f(x)

=x

at

At

ax

?.5.11.

>

a:

(0

if

jt/2);

x\

1.

what

values of the coefficient a does


increase along the entire number scale?

At what value

decrease along the entire


3.6.

<x<

values of

for all

function

the

of b does the function

f (x)

(0 f{x)

= s'mx bx + c

number

scale?

Maxima and Minima of

a Function

defined on the interval


then an inthe point of maximum of
is called
the function f (x) [the point of minimum of the function f (x)] if
there exists a neighbourhood
of the point x
such that the
inequality f(x)^.f(x ) [f (x)^f(x )] holds true within it.
The generic terms for points of maximum and minimum of a function are the points of extremum.
A Necessary Condition for the Existence of an Extremum. At
points of extremum the derivative /' (x) is equal to zero or does
not exist.
The points at which the derivative f (x)
or does not exist are
called critical points.
Sufficient Conditions for the Existence of an Extremum.
I.
Let the function f (x) be continuous in some neighbourhood of
the point x
If

y=f(x)

function

terior point x

of

this

is

interval

UX

fi

<x

and /'(#)<
at x > x (i.e. if in mothrough the point x the derivative changes
sign from plus to minus), then at the point x the function reaches
1.

If

/'(#)>

ving from

left

at

to right

maximum.
2.

If

/' (x)

<

at

<x

ving through the point x


sign from
a

minus

to plus),

and
from

/' (x)

>

at

>x

(i.e.

if

in

mo-

to right the derivative changes

left

then at the point x

the function reaches

minimum.

the derivative does not change sign in moving through the


then there is no extremum.
II. Let the function f (x) be twice differentiate (that is /' (x )
0)
the function has a
at a critical point x
If f"(x )<0, then at x
the function has a minimum;
maximum; if /" (x )
0, then at x
but if f" (x )
0, then the question of the existence of an extremum
at this point remains open.
3.

If

point x

>

{)

is

{rn

(x

If

of a

Function

153

= r(x )=...=P''-(x = 0, but /<>(x )^0. H


(x
< there a maximum at # and at
f

Let f'(x n )
even, then at

III.

Maxima and Minima

3.6.

>

{)

is

minimum.

0,

(n)

is odd, then there is no extremum at the point x


Let a function y = f(x) be represented parametrically:

/2

IV.

= <p(0

= ty(t),

where the functions <p(t) and ty(t) have derivatives both of the first
and second orders within a certain interval of change of the argument t, and cp' (0=7^0. Further, let, at t = t

(/)

= o.

Then:
(a)

if

(/

<

(b)

i|/ (/

if

= *o =

the

= 0,

the

function

= f(x)

has a

maximum

at x

has a

minimum

at

<p
/r

(c)

> 0,

= f(x)

function y

the

0,

= *o = <P('o);
if

(/

i|5

question of the

existence of an

extremum

remains open.

The points

which q/

at

3.6.1. Using the


wing functions:
(a)

(b)

first

vanishes require a special study.

(/)

derivative,

9x +
/(*)=** 8x* + 22x 24x+
f(x)

= -jx'

1)

Solution, (a)

number

entire

(*-3)

extrema

the

of

the follo-

7;

(c)

find

12;

2
;

The function
scale.

/' (x)

is defined
and differentiate over the
Therefore, only the real roots of the derivative

= 3x* 3x

I8x = 3x (x +

2)

(x3)

expression to zero, we find the


(they should always be arranged in an increasing order). Let us now investigate the sign of
the derivative in the neighbourhood of each of these points. Since
there are no critical points to the left of the point
2, the
derivative at all the points x
2 has one and the same sign: it
in the interval (
is negative. Analogously,
2, 0) the derivative is
positive, in the interval (0, 3) it is negative, at x
3 it is positive. Hence, at the points x
3 we have minima
2 and x 3
are

critical

critical

points.

points: x x

Equating

= 2,

x2

this

= 0,

x3

=3

x=

<
x

/( 2)^ 9 and

mum

/ (0)

= 7.

/ (3)

= 40-^-

=
,

and

at

the

point

>

*2

= 0,

maxi-

Ch. III. Differential Calculus: Investigation

154

(c) Just
derivative

as

item

(a),

since

the

in

/'(#),

(x)

= (x +

l)

- 3) + 3x (x +
X (x 3) = 3 (x +
3

(x

Equating

2
)

x2

(x):

3) + 2x (x +
(x 3) (2x -3x

l)

(x

3
1

we

this expression to zero,

^ = -1,

Fund's

the critical points are the roots of the


function is defined and differentiate

throughout the number scale. Find f


f'

of

^(3-|/T7)/4,

*,

find

1).

the critical points:

= (3 + |/T7)/4,

x4

= 3.

Let us tabulate the signs of the derivative in the intervals between the critical points:

Intervals

Sign of

/'

<

<

<

x2

x2

<

<

x3

:i

<

<

x4

xA

<

(x)

seen from the table, there is no extremum at the point


there is a minimum at the point x 2 a maximum at the
point x 3 and a minimum at the point # 4

As

xt

is

1,

3.6.2. Using the


wing functions:
(a)

/ (x)

(b)

derivative, find

first

=3

(x)='t/(x-iy+V(x+iy.

Solution, (a) The function


the number scale.
Let us find the derivative:

From

x=

the extrema of the follo-

the equation f'(x)

is

defined and continuous throughout

we

the roots of the

find

derivative:

1.

Furthermore, the derivative goes to infinity at the point x = 0.


x 3 =l. The results
0,
1, x 2
Thus, the critical points are * 1=

investigating the sign


in the
neighbourhood of these
points are given in Fig.
40.
The
investigation
shows that the function

of

of the derivative

has

two maxima:

3.6.3.

extrema

/(

1)

= 2;

/(1)

=2

Using the second derivative,


of the following functions:

and

find out

minimum

/ (0)

= 0.

the character of the

3.6.

(a)

(b)

=2

Maxima and Minima

of

a Function

155

sin a;-|-cos2x;

f(x)^2x i

84* +

15x 2

8.

Solution, (a) Since the function is a periodic one we may confine


ourselves to the interval [0, 2n}. Find the first and second derivatives:

= 2 cosx 2 sin 2x== 2 cos x (1 2 sin x)\


= 2sinx 4 cos 2x.
2s\nx) =
From the equation 2 cos (1
y'

y"

a:

points on the interval


x

Now

= jt/6,

find the sign of

[0,

determine

the

critical

2n}\

x 2 =ji/2,

the second

x3

= 5jx/6,

x4

= 3ji/2.

at each

derivative

critical

point:

= 3 < 0; hence, we have a maximum y (n/6) = 3/2 at


the point x = Jt/6;
hence, we have a minimum # (ji/2) =
(jx/2) 2 > 0;
at the
point x = ji/2;
hence, we have a maximum y (5ji/6) = 3/2
(5ji/6) = 3 < 0;
= 5jx/6;
at the point
y" (3jx/2) = 6 > 0; hence, we have a minimum #(3^/2) = 3 at
the point x = 3jt/2 (see Fig. 41).
y" (xc/6)

(/"

a'

:j

Fig. 41

3.6.4.

Investigate the following functions for extrema:

^<Hr +3 ^o
Solution,

(a)

)
)

Though the derivative


/

_2(jr<0),

3(x>0)

exists at all points, except the point


minus to plus when passing through

minimum

here:
/ (0)

= 5 > / (*)

at

x=
the

0,

and changes sign from


= 0, there is no

point x

l<x<0.

Ch. III. Differential Calculus: Investigation

155

This
at

of

explained by the fact that the function

is

Functus

discontinuous

is

0.
the point x
(b) Here the derivative

/' (x)

= 4x(x

0) also exists at

all

points,

except at x
0, and it changes sign from minus to plus when pass0. Nevertheless, we have here a maximum
ing through the point x
but not a minimum, which can readily be checked.
It is explained by the fact that the function is discontinuous at
the point x
0.

3.6.5.
(a)

(b)

Find the extrema

/ (x)

= 3 ^4

+ 8 ^3_
f(x) = Ve*\.

Solution, (a) Here


8x*
3x*
tion f 1 (x)

of the following functions:

18a: 2_|_

is

it

6 o

simpler to find the extrema of the func-

18x + 60. Since


x) = 2x + 24x 36jc=
fi(*) = 12(3* + 4*

f[

2x

(x

+ 2x 3)

3),

the critical

points are:

x1

x2

3,

= 0,

x3

of the extrema is readily determined from the


sign of the second derivative fl(
0; hence, at the point x
3)
the function f 1 (x) has a minimum, and the given function f (x)
hence, at
obviously has a maximum / ( 3)
0;
2/3,
f" (0)
the function f 1 (x) has a maximum, and f(x) a
the point x 2
minimum /(0) 5/6; / 1'(l)>0; hence, at the point x 3 =^\ the
50/53.
function f 1 (x) has a minimum, and f (x) a maximum /(lj
(b) In this case it is easier to find the points of extremum of

and the character

>
=

= 3

<

'

the radicand

which coincide with the points

of

extremum

Let us find the critical points of f 1 (x):


= 2xe x2 f[(x)=0 at the point x
f' (x)
0:
the second derivative at the point x

= 0.

of

the function

/ (x).

Determine the sign

of

f\{x)

= 2e*'{\+2x% fi(0)=2>0.
is
a minimum of the function
x=

Therefore the

point

will also be a

minimum

of the

given function

f (x):

f^{x)\

it

f(0)=0.

3.6.6. Investigate the character of the extremum of the function


0.
cos x at the point x
cosh x
y
Solution. The function y is an even one and apparently has an
extremum at the point x 0. To determine the character of the

3.6.

extremum
point x

Maxima and Minima

us evaluate

let

the

a Function

of

157

function

derivatives of this

at

the

= 0:
y'^smhx sin x,

y'(0)

= cosh^ cos x,
y"' = sinh x + sinx,
# = cosh x + cosx;
y"

y"(0)

y"'(0)=0;

(4)

Since the

non-zero derivative at the

first

= 0;
= 0;

=2 >
point x =

(0)

>

the

following

functions

is

derivative
a mi-

we have

an even order, which takes on a positive value,


nimum y(0) = 2 at this point.

of

3.6.7. Investigate
0:
the point x

0.

an extremum at

for

y=

(a)

cosx

= cos x + -y
j/'(0) = 0;
^"

(b)

= sinx + x
= cosx+1 x; y" (0)=0;
= sinjc y"'(0) = 1=^0.
a:

Solution, (a)

#//
/

1;

And so, the first non-zero derivative at the point x


vative of the third order, i. e. of an odd order; this
0.
there is no extremum at the point x

is

a deri-

means

that

Investigate the following functions for extrema:


2
sin 3x
x*e~*
(b) / (x)
3 sin*.
Solution, (a) The function f (x)
x*e~ x * is continuously differentiate everywhere. Equating the derivative
3.6.8.
(a)

(x)

/' (x)

= 4xe- x2 2x e~ x2 = x e~ x2 (4 2/)
points:

= V2\
Compute

find the critical

to zero,

/" (x)

x2

= 0;

x.,

= V2.

the values of the second derivative at the critical points:

= Wx^-** 8x*e~ x

lOx'e-**

+ 4xe-* =
2

= 2x e- x2 {6 9x -\-2x y
r(0) = 0; r(-K2)<0; f" (V~2 < 0.
Consequently, at the points x = K2 and x =-f-j/2 the function
reaches a maximum
(1^2) = 4e~ = \ As far as the critical
point x =
concerned, nothing definite can be said as yet, we
2

>

is

have

to find derivatives of

order!).

the

first

But

sufficient

f (x)

of

higher orders (up to the fourth

cumbersome, therefore we will turn to


condition of an extremum: let us find the signs

this process

is

Ch. III. Differential Calculus: Investigation of fund's

158

of

the

*a

=0:

derivative

first

neighbourhood

the

in

of the

critical

point

/'(-1)<0; /'(1)>0.
Hence,

at

3.6.9.

= the
y = f(x)

point

the

The function

= 0.

(2 <t <2).

/(0)

represented parametrically:

is

= <p(t) = 5P 20/ + 7,
y== ty(t) = 4P 3t \8t \-3
x

minimum

has a

function

Find the extrema of this function.


have
Solution.

We

<p'(0

(2,

In the interval

Find

2)

it

i|/(0=

(i.

e.

at

(t)

= 24/ 6;

(i.e.

12/

6/ 18 =

*=

(c)

(e)

1 )

y=:f(x)

#=31) and

1033/32).

= x*e-*\

(b)

= - x- V(^W;
= ^/2x + 3jc 36jc;
/ (*)
=
x In *;
/ (*)
f(x)

(c)

the

'<*>=.(>.

(d)

f (3/2) = 30 >

0,

has

f(x)
f (x)

0.

maximum #=14

a
*/

17.25 at

at

= 3/2

of the following functions:

Ax
xa

-|-

'

14

=
a:

(g) f(x)

following

=
= s'm xx\
(b)
f (x)
f(x) = s\nxx + ^ ;
e v*,

<d >

considered interval of va-

8x + 2'
a

3.6.11. Investigate
0:
the point x
(a)

0.

minimum

maxima and minima

(f )

the

of

^ = _ 30 <

function

the
at

3.6.10. Find the


(a) f{x)

20.

Consequently,

/=

to zero:

and f = 3/2.
1
*, =
?
These roots are interior points
riation of the parameter t.
Furthermore:

Whence

15/

y'(t)=0.

and equate

\|/ (/)

= 5f

if

x=0,

if

x=

= x In

functions

f(x)==s'mx

2
a:.

for

x +

an extremum

x*j3;

at

3.7. Finding the Greatest and the Least Values

0/

a Function

59*

3.7. Finding the Greatest and the Least


Values of a Function

The

value of a continuous function f (x) on an


attained either at the critical points, or at the
end-points of the interval. To find the greatest (least) value of the
function we have to compute its values at all the critical points
on the interval [a, b], the values f(a), f (b) of the function at the
end-points of the interval and choose the greatest (least) one out
of the numbers obtained.
If a function is defined and continuous in some interval, and if
this interval is not a closed one, then it can have neither the
greatest nor the least value.
greatest

interval

b]

[a,

(least)

is

3.7.1. Find the greatest and the least


functions on the indicated intervals:
(a)

(b)
(c)

(d)

= 2x 3a: - \2x+ on
f(x) = x \nx on [1, e\\
= xe~ x on [0, +00];
f (x)
f(x) = V(\-x*)(l + 2x*) on
2

f(x)

[2,

5/2];

[-1.

1].

the

following

Solution, (a) Find the derivative


I'

It

values of

vanishes at two

(x)

points:

f (x):

= 6x 6x 12.
x =
and x = 2. They both lie in-2
z
consequently both of them
2

'

side the indicated

interval

>

consideration. To find the extreme values of


the function it is necessary to compute its values at the points
a-j
and x 2 and also at the end-points of the segment:

must be taken

into

f(-2)
Hence, the
(b) Find

= 19, f(A) = _16 2


/2 J
greatest value
/( 1) = 8 and the least f(2) = 19.
the critical points: f (x) = x(l + 2\nx). The derivative

= -3,

f(-l)

= 8;

/(2)

'

is

(x) does not vanish inside the given interval


[1, e]. Therefore
there are no critical points inside the indicated interval. It now
remains to compute the values of the function at the end-points of

the interval

[1,

e]

f(l)

Thus, /(1)

is

the

least

= 0;

f(e)

value

of

= e\

the function and f(e)

=e

the

greatest.

3.7.2. Find the greatest and the


functions on the indicated intervals:
(a)

f/

= sin

a;

sin

2a;

on (00,

00);

least

values of

the following

Ch.

160

III.

Differential Calculus:

= arc cosx on [ V
y = x + ]/ x on [0, 4].
2

(b)
(c)

Represent the

Solution, (a)

whence
riod

Hence,

among

values
vative

function y

= sin

a:

sin 2x in the form

cos 3x
2

an even one and has


greatest and the
on the interval [0, n]. Find the

function

extrema

the

J/2/2];

cos x

is

least

deri-

y'\

= y(3sin 3x sin x).

the derivative vanishes at the points

[0,

n]

x,

= 0,

Compute

x2

= arc cos yL=

x3

= arccos^

= y(n) =

= 4-

arc cos

r_

equal to
3.7.3.

value

least

function

of the

4/(3/3), and

in the interval

oo,

oo)

the greatest to 4/(3 1/3).

The function

f(x)=ax + ^
consists

rz=zjl -

vi

Hence, the

x^

'

the values of the function at these points:

y(0)

is

a pe-

to seek the

sufficient

is

it

y'

In

seen that the

is

it

2jx.

2/2,

Investigation of Fund's

two summands:

of

independent

variable

one

the

x,

(a,

x>0)

b,

summand

other

is

inversely

proportional
proportional

to the

to

it.

Prove that this function takes on the least value at x = ]/~b/a.


Solution. Find the roots of the derivative f (x) in the interval
oo):

(0,

f'(x)
at x

tion

= V'b/a (x>0).
f (x)

reaches

Since

= a- =

f" (x)

minimum

= 2b/x> >
at

this

for

critical

3.7.4. As a result
x the numbers x lt x 2

is required to find at
the errors

It

of

of
,

X)

will be the least.

> 0,

the func-

point. This

extremum (minimum) in the interval (0,


Vb/a the function f (x) attains the least value.

only
x

any x

oo).

is

the

Hence,

at

measurements of an unknown quantity


xn are obtained.
what value of x the sum of the squares

- Xl r + (x-x +
2 )*

+ (*-*B

2
)

Finding the Greatest and the Least Values

3.7.

Compute

Solution.

1'

The only

X)

of

Fund ion

161

the derivative

= 2 (xx + 2(xx )+...+2


t

t )

root of the derivative

(x

n ).

is

Then, for
f(x) has its

all

x we have

minimum

Being the only

f" (x)

> 0.

2n

====

Therefore, the function

the point

at

minimum,

coincides

it

with the

the function (cf. Problem 1.3.8).


And so, the best (in the sense of "the principle

squares") approximate value of an


metic mean of the values x ly x 2
,

3.7.5.

Find the

term

largest

Consider

Solution.
[1,

oo).

the

xn

is

value of

minimum
the arith-

the sequence

+200

function

f (x)

=^

2QQ

in

the interval

Since the derivative


1

is

/i

the

unknown quantity x

in

of

least

positive

at

<x<

W~

jJ/400

(400-^)

(x 3

+ 200)

and

negative

at

x>

j/400,

the

< x < j/^400 and decreases at # > jJ/400.


/
follows that the largest term
inequality
the
From
7<j/ 400 <8
Since a =49/543 > a =8/89,
in the sequence can be either a or a

function

f (x) increases at

it

the largest term in the given sequence

3.7.6. Find the greatest and the


functions on the indicated intervals:
(a)

f(x)

= ]-x*-^x-jX* + 2

(d)

least

on [-2,

= |/4 x on [2, 2];


on
/ (x) = arc tan x y In

(b) f (*)
(c)

jt

/(jc)=2sinjc

6 -3148

+ sin2A'

on

0,

|k

is

values of the following

4];

Ch. III. Differential Calculus: Investigation of Fund's

162

(e)

f(x)

= x 2 In x

(i)f{x)=

2^ 2

on

+|

[1, e]\

for

-2<*<0; 0<*<2,

for

= 0.

Solving Problems in Geometry and Physics

3.8.

3.8.1. The force of a circular electric current acting on a small


magnet with the axis perpendicular to the plane of the circle and
passing through its centre is expressed by the formula

Cx

F=
(a 2

where a
x

= radius of the circle


= distance from the
(0 < x < oo)

+*

centre

3/f
)

the

of

circle

to

the

magnet

C--= constant.

At what x will the value


Solution. The derivative

of

F'(x)

be the greatest?
a2

=C

(a

~ 2x
l
+x v
2

has a single positive root x


alV~2. This solves the problem.
Note. It often happens that reasons of purely physical or geometric character make it unnecessary to resort to the differential
methods in investigating a function for the greatest or the least
value at the point under consideration.

Determine the most economical dimensions of an open-air


pool of volume 32 m 3 with a square bottom so that the
facing of its walls and bottom require the least quantity of ma3.8.2.

swimming
terial.

Solution. Let us denote the side of the bottom by x and the


height by y. Then the volume V of the pool will be

V = x 2 y = 32,

(*)

and the surface S to be faced

S x2

+ 4xy.

Expressing y through x from the relation

S = x2 + \x 1
X

Investigate
interval (0,

the

function

thus

=x +
l

we

(*),

obtained

for

minimum

oo):

S'

= 2jc^;

get

2*-^ = 0;

= 4.

in the

The

3.8.

Solving Problems in Geometry and Physics

163

single point thus found will obviously yield

the least value


function S, since it has no greatest value (it increases un> oo).
O and
boundedly as x
And so, the required dimensions of the pool are: x 4 m, y ^2m.
the

of

Inscribe

3.8.3.

into

a given sphere a cylinder

with the greatest

lateral surface.

3.8.4. 20 m of wire is available for fencing off a flower-bed


which should have the form of a circular sector. What must the
radius of the circle be if we wish to have
y
a flower-bed of the greatest possible surface

area?
Solution.
(see

Let

us

denote

and the
Fig. 42). Then

circle

by x

2O

the radius of the


length of the arc by y

= 2jc + 0,

whence
Fig. 42

= 2(10-*).
The area

of

S = ^xy = x

the circular sector

10
The derivative S'(#)
Since the least value S

(10

x)

(0<#<10).

2x

has a root x--=5.


reached at the end-points of the
interval [0,
10], the obtained value
x
5 yields the greatest surface area S.
is

required

construct
of capaThe thickness of the material
city V
is d. What dimensions (the base radius
and height) should the reservoir have
so as to ensure the least possible
expenditure of the material?
Solution. Figure 43 represents a lonreservoir,
gitudinal
section of the
where the radius of the base of the
inner cylinder is denoted by x and
the
inner cylinder,
the height of
volume of the bottom
by h. The
3.8.5.

It

is

to

an open cylindrical reservoir


.

Fig. 43

and

the

wall

of

the

reservoir

+ df d + n [(x + df x

V=

jx

On

the other hand, by hypothesis

(x

V
whence

2
]

= nd (x + df + nh (2xd + d
we must have

= nx

).

*)

Ch.

164

Substituting into

Investigation of Functus

Differential Calculus:

III.

we

(*),

get

^ + ^.

V^nd (x + dY + ^(2xd + d')^nd(x+dr + 2


Now we have
extremum at x >

We

investigate

to

the obtained function

(x)

for

an

0.

have
2
2
= M<+">y-r.>
V (x) = 2nd(x + d)- -^-^

The only positive root of the derivative


This solves the problem:

3.8.6.

factory

is

to

be

is

connected

the point

x=

y'Vjn.

by a highway with a

The distance DB
factory to the railway is
0
equal to a the segment AB of the
/
railway equals /. Freight charges on
the highway are
times higher than
/
on the railway (m
1).
a
How should the highway
be
connected with the railway so as to
/'
~
ensure the least freight charges from
^
A
x
A
p
factory to town?
b
Solution. First, let us make a drawAA
r is 44
It
ing (see Fig. 44).
absolutely
is
clear that the highway must also be
straight (a straight line is shorter than any curve connecting two
given points!). Furthermore, the point P cannot lie either to the
the point B. If we denote
left of the point A or to the right of
the distance AP by x it will mean that
Let the freight charges on the railway (per ton-kilometre) be k,
then the freight charges on the highway will be km. The total
freight charge
to A amounts to
for transporting loads from

straight

on which

railway

town A

from

is

situated.

the

/
/

>

0^*^/.

N = kx + km Va + (l x)\
2

Hence, we have to
f

X)

find

the least value of the function

= x + m y a + (xi)\ o<*</.
*

Take the derivative

(x

/)

DP


It

Solving Problems in Geometry and Physics

3.8.

vanishes only at one point:

= l-

j/~m 2
If

165

this point

the interval

in

lies

[0,

/],

i.e.

if

or
1

tri

it
yields the least freight charge (which is easy to check).
the indicated inequality is not observed, then f (x) increases on
0.
[0, /] and therefore the least freight charge is obtained at x

then
If

In constructing an a-c transformer it is important to insert


the coil a cross-shaped iron core of greatest possible surface
area. Fig. 45 shows the cross-section
of the core with appropriate dimensions. Find the most suitable x and
y if the radius of the coil is equal
3.8.7.

into

to a.

the source of current is an


then the effect P (watts)
obtained by cutting a resistance R
(ohms) in the circuit is expressed by
the formula
3.8.8.

If

electric cell,

E2 R

(/?+/?/)*'

Fig. 45

E is electromotive force in volts and R the internal resistance in ohms.


Find the greatest effect which can be obtained at given E and R {

where

3.8.9. A
What must

use the least

tin

be

of

volume V has the form

given

of a cylinder.

ratio of its height h to diameter

the

amount

of

material for

its

2R

so as to

manufacture?

3.8.10. In a given cone inscribe a cylinder having the greatest


surface so that the planes and centres of the base circles
of the cylinder and cone coincide.
lateral

3.8.11.

Through

Given

point

this point

draw

(1,

2)

in

a straight

with the positive semi-axes,

the

orthographic coordinates.
that it forms, together

line so

triangle of the least area.

2px
on the axis of the parabola y 2
3.8.12. Given a point
at a distance a from its vertex. Find the abscissa of the point
on the curve nearest to the given point.

Ch. III. Differential Calculus: Investigation of FuncVs

166

The expenses sustained

in one hour's sailing of a ship


empirical formula of the form
bv\ where a and b are constants for a given ship, and v is the
a
ship's speed in knots (one knot is equal to 1.85 km/hr). In this
formula the constant part of the expenses a refers to depreciation
and crew's upkeep, and the second term (bv's ) to the fuel cost.
At what speed will the ship cover any required distance at the
lowest cost?

3.8.13.

are

expressed

roubles

in

by

an

from

three boards of equal width. At


boards be placed to ensure the largest
cross-sectional area of the trough?
3.8.14.

trough

built

is

what slope should the

lateral

3.8.15. A tank with a vertical wall of height h is installed on


horizontal plane. Determine the position of an orifice, at which
the range of a liquid jet will be the greatest if the velocity of

flow (according to Torricelli's


the depth of the orifice.

3.8.16.

Two

of

point of

tance

of

is

equal to

Y 2gx,

where x

aircraft

at

aircraft

be the least and what

Convexity and Concavity of a Curve. Points of

3.9.

is

are flying in a straight line and in the


an angle of 120 to each other and with an equal
v km/hr. At a certain moment one aircraft reaches the
intersection of their routes, while ihe second is at a disa km from it. When will the distance between the

same plane
speed

law)

is

that distance?

Inflection
If f"(x)<0 (> 0) on an interval
on this interval is convex (concave),
any of its tangent lines.

(a,

b) y

i.e.

it

then the curve y


is

f(x)
situated below (above)

If /"(a: )
or does not exist but f (x ) does exist and the second
derivative f" (x) changes sign when passing through the point x Qy
then the point (x 0l f (x )) is the point of inflection of the curve

3.9.1. Find the intervals in which the graphs of the following


functions are concave or convex and locate the points of inflection:
(a)

= x' + x* 18a: + 24a: 12;


2

(b) y---3x*

8x + 6x*+l2;
3

=x+x
= 4l/>-l) + 20K(x-l)
y = ^r (*>)
l

(d)
(e)

(0

'>;

(/

]J

(x^ly,

(g)

(h)

Convexity and Concavity. Points of Inflection

3.9.

= *sin(ln*) (x >
= 2-|**-l|.

Solution,

(a)

12* 2

Hence, y"

0);

Find the derivatives:


y'

whence y"

167

at

>

= 4x + 3x 36x + 24,
+ 6* 36 = 12(V+-|- 3)
3

x1 =
2, x 2 = 3/2.
on the intervals (00,

2) and

00);

(3/2,

<

on the interval ( 2, 3/2). The sign of the second derivative determines the convexity or concavity of the curve in a given interval*
This enables us to compile the following table:

<

2 ~2

Sign of y"

Conclusion

Concavity

<

<

1
+

Convexity

Concavity

Since the second derivative changes its sign when passing through
the points (
the points x 1
and * 2
124) and
3/2,
2,

= 2

are points of inflection.

(d)

Find the derivatives:


/

10

f= 9-i77//

+T x "<

The second derivative is non-zero everywhere and loses its meaning


we have y" < and the curve is conat the point x = 0. At x <
vex, at x >
and the curve is concave.
we have y" >
At the point x=0 the first derivative y'=l, the second deriva-

x=0.

tive changes sign when passing through the point


the point (0, 0) is a point of inflection.
(g) Find the derivatives:
y'

= sin (In x) + cos (In *),

y"

= y [cos (In x) sin (In*)] = -~- sin

The second derivative vanishes


Xk

^i/A + kn
9

&

at

(-4-

In

Therefore

the points

= 0, 1,

zh 2,

...

The function sin(jt/4


In*), and together with it y\ changes sign
when passing through each point x k Consequently, the points x h
.

Ch. II L Differential Calculus: Investigation

168

In the intervals

are the abscissas of the points of inflection.


2&Jl + JT/4^

^>2fcjl-3Jl/4

the curve

is

concave, and in the intervals


^>2fcjl

it

is

(h)

Fund's

o)

2&Jl+ 5JT/4^

+ JT/4

convex.
The given function can be written in the following way:

(x

2+

x>
x<

y=

(a:

1),

1),

1,
1.

Therefore
r

x=

At the point

5x\ *>

_
~\
\

x<l.

5x\

there

no derivative. Further,

is

20a: 3

1,

x>

x<l;

20x\
y"

at

tervals:

the point x

( oo,

Compile

0),

= 0.

(0,

Hence, we have to investigate

three

in-

oo).

(1,

1),

table of signs of y"\

<

<

Convexity

Conclusion

(0,

1)

is

<

>

Sign of y"

The point

1,

Concavity Convexity

point of inflection, the point

(1,

2)

being

a corner point.

3.9.2. What conditions must the coefficients a, b c satisfy for


bx 3
cx 2 -{-dx
e to have points of inflection?
ax*
the curve y
Solution. Find the second derivative:

y"=
The curve has points

\2ax*

has different real roots,

inflection

of
e>ax

i.e.

+ Sbx + 2c.

and only

if

the equation

+ 3bx + c =

when
3b

if

the discriminant 9b 2

8ac >

0.

24ac > 0,

or

3.9.

3.9.3.

Convexity and Concavity. Points of Inflection

At what values

169

a will the curve

of

= x* + ax* + ^-x +
2

be concave along the entire number scale?


Solution. Find y"\

y"

+ 6ax + 3.

\2x*

number

The curve

will be concave along the entire


values of x, i.e. when

for all

Ax 2

For this
be

it

fulfilled;

is

+ 2ax + >0

for

all

if

y"

^0

16

scale

x.

necessary and sufficient that the inequality 4a 2

whence

|a|<2.
3.9.4.

Show

curve

that the

--=

has three points of


{

inflec-

tion lying in a straight line.


Solution. Find the derivatives:

x 2x+
*

'

'

l)

2x

6x 2

:i

-\

'

6x

(x*-f-l) 3

"

The second derivative becomes

which

three points,

zero at

the

are

roots of the equation

[i

+ 3x 3x = 0,
2

whence
x

[/

3~,

x2

- 2 + [/T,

<
Sign

of

00

<

-2-

<

J/T

-2-

y"

-2-l

< v<

<

F"3

^3
x

<

<
1

<

(- 2 - kT.

are points of

inflection.

^)
It

is

easy

<

oo

+
Concavity

Convexity

Concavity

Convexity

_L_

Conclusion

Hence,

j/"

<-2r

y"\

Let us compile the table of signs of

x,

(-2 + ^3.
to

^p-).d.

ascertain that

all

of

them

Ch. Ill

170

lie

in

Differential Calculus:

Indeed, the coordinates of these points satisfy

a straight line.

_2 }/T
-2+/" 3 -1

(1- }/T)/4

the relation

Fund's

Investigation of

(\+V 3)/4+l

3.9.5. Investigate the curves represented by the following equations for convexity (concavity) and locate the points of inflection:
(a)

(b)

= x- ^(x-3)
x
( jx/2 < x < jt/2).
y=e
2

s[n

3.9.6.
lie

Show

that the points of inflection of the curve

on the curve y 2
3.10.

+ # = 4x
2

(4

= xsmx

Asymptotes

A straight line is called an asymptote to the curve y f(x) if


the distance from the variable point
of the curve to the straight
recedes to infinity along some
line approaches zero as the point
branch of the curve.
We will distinguish three kinds of asymptotes: vertical, horizon-

and inclined.
Vertical asymptotes. If at least one of the limits of the function
or on the left) is equal to infif (x) (at the point a on the right
a \s a vertical asymptote.
nity, then the straight line x
A, then the straight line
Horizontal asymptotes. If lim f(x)
tal

y=^A

is
a horizontal
the left one as x
Inclined asymptotes.

->

00

asymptote (the right one

as

oo).

then

exist,

-*

the

cc

and

the limits

If

lim
X

x> +

straight

kv

lim
*- +

line

[f (x)

k x]=b
1

oo

= k x+b
1

is

an

inclined

(right)

asymptote.
If

the limits

Urn
X

-+

CO

=k

and

lim
X

-*

[f (x)

k x]=b
2

00

k 2 x-\-b 2 is an inclined (left) asymphorizontal asymptote may be considered as a particular


0.
case of an inclined asymptote at k
exist,

tote.

then the straight line y

3.10.1. Find the asymptotes of the following curves:


(a)

#=

-^3

(d)

#=

7 + 4,v

(b)

2
;

y=

(e)

^r+3x;

y-=xe^;

(f)

(c)

= -^;

</=--f

In

= Vl + x* + 2x\
y = 2Vtf + T.

(i)

The curve has

Solution, (a)

lim

-+

sin-L;

asymptote x

= 3,

,x
'

T-

oo

3'

point x = 3 is a point of discontinuity of the second


Find the horizontal asymptote:

(the

lim
*

And

lim
a:

->

->

00 X

asymptote

vertical

lim

x-1+0

-+

=3

and

a horizon-

x=

since

1,

^A

-r

\*~~

+ 3x = +

00.

Find the inclined asymptotes:

k=
b

=
X

lim
GO

lim

(ykx)

a;

Thus,

=3;

-*

3*

3x = 3

the straight line

y=3x+3

3a:

lim
co

->

asymptote

inclined
(e)

==

lim

an

is

Fig. 46).
curve has a vertical asymptote

The

lim
-* +

*/

Hg. 46

(see

lim
X

->

xe l/x

x=

since

-^-=+00

lim

oo

Problem 3.2.2.).
Find the inclined asymptotes:

(see

&

=
X

(a^ 1 /*

lim
*

-*

CO

lim 4"=
- CO *

x) =

lim
Xv

-+.

lim
X

->

V3

~
CO

^=

g=l;

l x
e l

kind).

= 5.

00;
(-^13^==
x ~~

lim
a:

lim
x-+

lim
X

= 5.

The curve has a

(b)

y
GO

the curve has a vertical asymptote x

so,

one y

tal

-*

since

5x

lim
x

171

vertical

0"

x^3T

+x

y=}/\

(h)

(g)

Asymptotes

3. 10.

'

.
:

lim
l/x -

Ch. III. Differential Calculus: Investigation of Fund's

172

of

Thus, the straight line y


x+\ will be an inclined asymptote
the curve (see Fig. 47). Note that
liin
-V

The function

(f)

is

->

-*

0.

and continuous

defined

xe l/x

Urn
x

<

and x

>

e ^ >

at

0,

i.e.

at

3e

Since the function is continuous at


every point of the domain of definition,
vertical
asymptotes can exist only on
finite boundaries of the domain of definition.

As x >

y=

lim

we have

^\n(e ~)

lim

In (e

liin

Fig

As x

(see

Problem

line

a;

47

^+

+ z)
=

3.2.2.),

not

is

i.

e.

the

straight

vertical asymptote.

we have

lirn
K-*\/(3e)

x\n[e

lim

3x

the line x=-l/(3e) is a vertical asymptote.


let us find the inclined asymptotes:

i.e.

Now
k

lira

X-*cc X
lira

[y

kx] =

1r

lirn

X-+cc

[n{l

-3x-e

lirn

x-*<x>

Hence,
(see Fig.
(g)

the

straight

line

y 3x2

an

is

2e

inclined

3^

2e

asymptote

48).

The curve has no

vertical asymptotes, since

continuous everywhere. Let

us

limits will be different as x+


to consider two cases separately.

look for
oo and

the function is
asymptotes. The
-oo, therefore we have

inclined

173

3.10. Asymptotes

*,=

[i

*^ +
6,

>"

',

+ 2 -

CO

Yl +

lim
X-*

[|/T+72 --;e|:

lim

x* + oo

Fig.

left

Iim

fe

lim

oo

j/

-f *

48

= 3x.

Fig. 49

(as

x-+

jClT^f^

oo):

lim

iH^J

+ x + 2a; x\ =
2

since both summands


0.
positive at x

0.

+*

the curve has an asymptote y

asymptote

1+JC 2

lim
*-*

Find the

= 3;

=
Thus, as

+2

CO

(^1+a:2 + 2x 3x)

lim

+ oo):

x-^

Find the right asymptote (as

([/l+r* and

lim

a;))

0,

the denominator are

in

<

oo.
so, the curve has an asymptote y = x as x >
The curve has no vertical asymptotes, since it is continuous
x =^=0, and in the neighbourhood of the point x=^0 the function

And
(h)

at
is

bounded.
Let us find the inclined asymptotes.

lim
-00

We

have

lim
00

1-0=0.

X-+

Then
f

+ ^sin =% _
!

AT
as x
as v

+ oo,
oo.

Ch. III. Differential Calculus: Investigation of Fund's

174

Thus, the curve has two horizontal asymptotes: y


1 and y =
1
Fig. 49). The same result can be obtained proceeding from
symmetry about the origin and keeping in mind that the function
y is odd.

(see

3.10.2. Find the inclined

= j^-^

function

as

may

asymptote

and show that

oo

of the

graph

of the function

the interval (100, oo) this

in

be replaced by the linear function y

= x

with an

error not exceeding 0.01.


Solution. Find the inclined asymptote:

b=

X2

lim TT-t
lim

t4

V1+*

x+cc

And so,
Form

1;

x)

the equation of the asymptote


the difference:

-(*-!)

l+JC

1.

is

= x 1.

Hence, assuming

x>

for all

100,

we

introduce an error of not more than 0.01.

3.10.3. Find the asymptotes of the following curves:


*2

-6*+ 3

(a)

^==

(c)

y=x+(smx)/x;

(e)

(b)

= 2x arc cos

(d)

= xarctanx;
y = In (4 x

)\

3.11. General Plan for Investigating Functions and


Sketching Graphs
The analysis and graphing

by elementary methods
Using the methods
of differential calculus, we can now carry out a more profound and
comprehensive study of various properties of a function, and explain
the shape of its graph (rise, fall, convexity, concavity, etc.).
It
is convenient to investigate a function and construct its graph

were considered

in

Chapter

of

functions
1.3

and

1.5).

according to the following plan:


1.

2.

Find the domain of definition of the function.


Find out whether the function is even, odd or periodic.

3.11. Investigating Functions and Sketching Graphs

3.

Test the function

for continuity,

out

find

175

the discontinuities

and their character.


4. Find the asymptotes of the graph of the function.
5. Find the points of extremum of the function and compute the
values of the function at these points.
6. Find the points of inflection on the graph of the function,
compute the values of the function and of its derivative at these
points. Find the intervals of convexity of the graph of the function.
7. Graph the function using the results of this investigation. If
is necessary to specify certain regions of the curve, calculate the
it
coordinates of several additional points (in particular, the x- and
^/-intercepts).

This is a very tentative plan, and various alternatives are posFor instance, we recommend the student to begin sketching
the graph as soon as he finds the asymptotes (if any), but in any
case before the points of inflection are found. It should be remembered that in sketching the graph of a function the principal reference points are the points of the curve corresponding to the extremal
values of the function, points of inflection, asymptotes.
sible.

3.11.1.
(a)

Investigate and graph the following functions:

=x

3x* + 3x*

(b)

5;

y--=\/~x Vx+l;

y- -^
1

c)

y=-zi*
-4

(e)

= x+ln

(g)

= x*e

(x 2

1);

'x
;

d)

(f)

= -j sin 2* + cos*;

(h)

= arc sin 1=^-

Solution, (a) The function is defined and continuous throughout


the number scale, therefore the curve has no vertical asymptote.
x)
The function is even, since /(
f(x). Consequently, its graph
about the (/-axis, and therefore it is sufficient to
is symmetrical
investigate the function only on the interval [0, oo).
There are no inclined asymptotes, since as x-^oo the quantity
y turns out to be an infinitely large quantity of the sixth order

with respect to

x.

Investigate the
0'

= 6*

derivative:

first

I2x + = e>x(x* 2x +
s

=x{x 2 l) 2

e>x

1)

the critical points are:


*i

Since in the interval


increases.

1,

[0, oo)

*2

= 0>

the derivative

y'^0

the function

Ch.

176

III.

Differential Calculus:

Fund's

Investigation of

Investigate the second derivative:

"

= 3(k

The positive roots

36* + 6 = 6 (5x 6x +
2

of the
*,

1 ).

second derivative:

1/K5,

*2

=1.

and pictorialness let us compile the following


the points of interest are arranged in an ascending

For convenience

where

table,

all

order:

(-

(f?0

7?)
+

y'

25

y"

(I.

<*>)

^5

-f

(5

-t-

23

On the right one more additional value of the function is computed to improve the graph after the point of inflection.
Using the results of the investigation and the above table and
taking into consideration the symmetry principle, we construct the
graph of the function (see Fig. 50). As is seen from the graph, the
function has roots
where
1.6.
(b) The function is defined and continuous over the entire number

a&

and is negative everywhere, since \/ x < \/ x + 1.


The graph has neither vertical, nor inclined asymptotes, since the
order of magnitude of y is less than unity as x* oo. Determine
scale

the horizontal asymptote:


lim

lim

(j/x

Hence, the straight


graph.

line

l)

y=^0

is

the horizontal asymptote

of

the

3.1 J. Investigating Functions and Sketching Graphs

The

177

derivative

first

3^/'(FH7

becomes zero

at

the point

x2

y
*i

and

infinity
,

xb

the

at

points

= 0.

-1

Fig. 50

Fig. 51

The second derivative


y

_ J_/

___2 \

3 V

3 J

_J

does not vanish and

Compile

-l

y'

00

y"

00

A)

1_

y#

3
is

infinite at the

same points x

1,

x3

= 0.

table:

(--{)

K-)
+

(0,

oo)

00

16
1

9yj

00

0.26

With the aid of this table, and of the asymptote y


the graph of the function (see Fig. 51).

construct

Ch. III. Differential Calculus: Investigation

178

of

Fund's

The function

is
defined and continuous over the entire axis
The function is odd, its graph is
points
symmetrical about the origin, therefore it is sufficient to investigate
the function on the interval [0, oo).
The straight line x 2 \s a vertical asymptote:

(c)

except

at

x=2.

the

lim
*

-+ 2

2x s
x2

2* 3

lim
x-2 +

-\-

OO

Determine the inclined asymptote:


2x 2

lim
v_w

_1_

m X

lim (y
X

-*

-*

2x) =

+CC

Sx

y2x =

>
<

j
I

The

first

derivative

y
in

the interval

CD

8x

lim
+ CD

inclined asymptote #

The curve has an

[0,

4) 4x
4)

6x 2 (x 2

= 2,

lim

(x 2

= 2x,

and

> 2,
<2
2x (x 12)
~ (x
x
x

at

at

4)

oo) vanishes at the points

= 0, *=2]/~33.46
at the point x = 2.
*

and becomes infinite


The second derivative

~
x=

becomes zero
Compile a

at

the point

y"

(at

(A:

-h 12)

4)

and

infinite at

= 2.

table:

(0,

y'

16a:

2)

oo

+o

00

(2,

2/" 3

r
)

2 |/"3

(2V

3",

3
2

oo)

3.11. Investigating Functions and Sketching Graphs

179

Using the results of the investigation, sketch the graph of the


function (see Fig. 52).
(e) The function is defined and continuous at all values of x for
oo,
which x 2
or |x|>l, i.e. on two intervals: (
1)

1>0

and

+ oo).

(1,

-A

4*

-1\

Fig. 53

Fig. 52

We

seek the vertical asymptotes:

lim
-

lim

lim

y=

[x

+ In (x

[x+

lim

= oo;
1)] = oo.
2

In (x

1)]

Thus, the curve has two vertical asymptotes:

x=

and x =

Find inclined asymptotes:


u

i-

lim

=
Jt

y =

lim [y
00

i-

lim

x] =

In

(a:

=
1

lim In (x
oo

In

,.

urn

(a:

1)

]-

l)=+oo.

X-+

-*

Hence, the curve has neither inclined, nor horizontal asymptotes.


Since the derivative

^ 1+
i

2*

^r

and is finite at all points of the domain of definition of the


function, only the zeros of the derivative

exists

x,=
can be

critical

- \V%

points. At the point

=-l + \f2
x = +

x2

the function

is

180

Ch.

III.

Differential Calculus:

Fund's

Investigation of

= l~l

not defined; hence, there is one critical point x


ging to the interval (
00, 1). In the interval
derivative y'
and the function increase.
The second derivative

(1,

2 belon-

00) both the

>

hence,

curve

the

convex

is

and

everywhere,

at

the

point

= ^2^ 2.41 the function has a maximum


(2 + 2^2) 0.84.
y _ J/2) J/2 +

To

In

where there are no charactethe following additional points:

plot the graph in the interval (l,oo),

ristic

points,

x=2\

*/

we choose

= 2 + ln3^3.10

and

x=

1.2;

1.2

+ ln0.44

^0.38.

The graph of the function is shown in Fig. 53.


function is defined and continuous throughout the num(f) The
ber scale and has a period 2n. Therefore in investigating we may
confine ourselves to the interval [0, 2n\. The graph of the function
has no asymptote by virtue of continuity and periodicity.
Find the

first

derivative:
y'

On

the interval

[0,

*i

= cos 2x sin x.

2k]

has three roots:

it

= ~Q~
5ji

ji

"g"

^2

3ji

*8 == ~2~

Evaluate the second derivative:


y" ==

On
x,

the interval [0, 2n]

= y,

x2

2 sin 2x cos x.
it

has four roots:

= k + arc sin (1/4),

x3

= ^,

x4

= 2n arcsin(l/4).

Let us draw up a table of the results of investigation of all


points of the first and second derivatives (the table also
cludes the end-points of the interval [0, 2ji]).
tical

Since in the interval ^0,

-^ the roots

of

the

first

criin-

and second

alternate, the signs of the second derivative in the inits critical


points are indicated only for the last
intervals.

derivatives
tervals

three

between

The

results of the investigation enable us to construct the graph


the function (see Fig. 54).
(g) The function is defined, positive and continuous on each of
the intervals (
is a disconti00, 0) and (0, 00). The point x

of

3.11. Investigating Functions and Sketching Graphs

ji

Jl

5ji

6~

~6~

181

3ji
AJ

(*

t)

(-)

x4

2ji

(* 4 .2ji)

y'

y"

]T3

3~

nuity. Since (see Problem 3.2.2.)


lim y

lim

x=0

the straight line

jt

e /*
[

limy
x-+

^-

Iirn

= oo

vertical asymptote.

is

lim x e Ux
2

*^

= )

But

= 0.

y=x e Ux
x+.oo.

There are no inclined asymptotes, eince the function


has the second order of smallness with respect to x as

\t/2

n
jc

2%

i*
-1
Fig. 54

Let

us find the extrema


evaluate the derivative:
y'

whence we
Since

for

find

= 2xe

of

[/x

the

function,

e = 2e
{

'

the only critical point

'*

{x

for

which purpose we

1/2),

*=y.

x=^=0

y" (x)

= 2*/* - e*'* + 1

= 1 e u* (2x* - 2x +

1)

>

0,

Ch.Iff.

182

on each
the
a

the

of

function

is

Differ eniial Calculus:

Investigation of

Fund's

intervals of the domain of definition the graph of


1
concave, and at the point x
/ 2 the function has

minimum

p*
*

1.87.

From

the information obtained we can sketch the graph as in Fig. 55.


oo, 0) and (V 2
oo) the
specify the graph in the intervals (
following additional points are used:

To

x=

= e~ ^ 0.37;
y = e^2J2.
l

1,

The function

(h)

x=l

defined and continuous


since at any x

is

throughout the number scale,


1-f-x

at

*>0.

continuous, the graph has no vertical asymphas a horizontal asymptote:

As the function
but

it

is

limy
x

The

first

1.

Since the function is even, we may confine


ourselves to the investigation of the function

Fig. 55

totes,

<

-*

arc sin

D=Hf.

(-

oo

derivative

2x (\+x 2x(\x
2

(1-**)*

(1+x

(l+x 2

4*

2 |*|

X (1+x 2
~

2
)

2 2
)

negative for x>0, therefore the function decreases


The derivative is non-existent
point x=0. By virtue of
at the
the symmetry of the graph about
the y-axis there will be a maxiis

mum

at

the

point

#(0)

= ^-.
=

No-

the
at
the point x
derivative is equal to
1,
1.
and the left one to
The second derivative is positice

that

right

tive:

x)

2(l+x 2 )2x
(l-fx 2 ) 4

8x
(i

+*

2 )3

>

for a

Hence, in the interval (0, oo) the graph of the function is concave.
Also note that the curve intersects with the x-axls at the points

1.

3.12. Algebr. and Transcendent. Equations

Taking into consideration the results of the investigation, construct the graph of the function (see Fig. 56).
3.11.2.
(a)

(c)

Investigate and graph the following functions:

y=\+x y; (b) y = j~^\


y = ~ + ^- (d) y = -^r
2

(e)
(f)

=y^-^=A-

y
y = x*\n(x+2);

\x

arc

(g)

tan atx^O,

10

at

= 0.

3./2. Approximate Solution of Algebraic


and Transcendental Equations
Approximate determination

of isolated real roots of the equation


usually carried out in two stages:
roots, i.e. determining the intervals [a, P] which
1. Separating
contain one and only one root of the equation.
i.e.
computing them with the required
2. Specifying the roots,
degree of accuracy.
The process of separation of roots begins with determining the
a lt a 2
signs of the function f (x) at a number of points x
whose choice takes into account the peculiarities of the function / (x).
then, by virtue of the
turns out that / (a k ) f (a k+1 )
If it
0,
property of a continuous function, there is a root of the equation
6 in the interval (a ky a k+1 ).
f(x)
Real roots of an equation can also be determined graphically as
^-intercepts of the graph of the function y
f(x). If the equation
has no roots close to each other, then its roots are easily separated
by this method. In practice, it is often advantageous to replace
a given equation by an equivalent one

f(x)

is

<

*i (*)

= * (*)

where the functions


(x) and i|; 2 (x) are simpler than the function
f(x). Sketch the graph of the functions y = ty 1 {x) and y = 2 (x)
and find the desired roots as the abscissas of the points of intery\)

section of these graphs.

The Methods
the interval

= and f
proximation x 1

f(x)

of

1. Method of Chords.
If
contains the only real root of the equation
is continuous on the interval, then the first apfound by the formula

b]

[a,

(x)
is

Approximating a Root.

184

Ch. III. Differential Calculus: Investigation of Fund's

To obtain the second approximation x 2 a similar formula is applied to that of the intervals [a,
or [x 19 &], at the end-points
of which the function f (x) attains values having opposite signs. The
process is continued until the required accuracy is obtained, which
judged of by the length of the last obtained segment.
Method of Tangents (Newton's method). If f(a)f(b)<0, and
f (x) and F ( x ) are non-zero and retain definite signs for
then, proceeding from the initial approximation x (x [a, b]) for
which f (x ) r(*o)>0, we obtain all successive approximations of
is

2.

the root g by the formulas:

10 rw

"'

/'w

X"- Xn -*

To estimate the absolute error in the nth


apply the general formula

r(*-i)'

approximation we can

where

m =
1

min

\f(x)\.

Under the above conditions the method

of chords and the method


tangents approximate the sought-for root from different sides.
Therefore, it is usual practice to take advantage of their combination,
i. e.
to apply both methods simultaneously. In this case one can obtain
the most precise approximation of a root more rapidly and the calculations can be checked. Generally speaking, the calculation of the
approximations x l9 x 2
x n should be continued until the decimal
digits to be retained in the answer cease to change (in accordance
with the predetermined degree of accuracy!). For intermediate transformations we have to take one or two spare digits.
3. Iteration Method. The equation f(x)
is first reduced to the
of

form x = cp (x) where q/ (x)


q < 1 (q = const) for a^x^b. Starting from any initial value x [a b] successive approximations of
the root I are computed by the formulas x 1 = q) (x ), x 2 = q> (Xj),
xn =
^y( xn-i)' The absolute error in the nth approximation can be
estimated by the following formulas:
|

\t x n\<Tz^\xn -iXnl
if

the approximations x n _ 1 and x n

lie

on the same side

of the root,

and
I

Xn

if

the approximations x n _ 1 and x n

X n-1

lie

Xn

on different sides

of the root.

3.12. Algebr. and Transcendent. Equations

185

Locate the roots of the equation

3.12.1.

Solution. Compile a table of signs of

at

some chosen points

00

~3

f (x)

+
+

4-

+- oo

From this table we draw the conclusion that the equation has
three real roots lying in the intervals (3, 1), (0, 1) and (1, 3).
Determine the number

3.12.2.

roots of the equation

of real

f(x)==x + e* =
Solution. Since

= +e x >

/' (x)

0; /

0.

oo) = oo;

(+

oo)

= + oo,

the given equation has only one real root.

An approximate

3.12.3.

= x*xl=0

is

value of the root

x=l.22. Estimate

of the

the

equation f{x)

absolute error

in

this

root.

We

Solution.
1.23

have /(*)

= 2.2153 1.22 = 0.0047.


1

Since

at

jc=

/ (x)

the root

4r*

in the

lies

increases

given interval

= 2.2888 .23 - 0.0588,


1

interval (1.22, 1.23). The derivative f (x)


therefore its least value in the

monotonically,

is

m = 4xl.22
1

wherefrom we get an estimate


x
I

= 4x 1.816
of

-6.264,

the error

< LZiZU =^^l 0.00075 < 0.001.


1

3.12.4. Solve graphically the equation

x log X
Solution.

=
1

0.

Let us rewrite the equation in the form

'.

Here

\f 1 (#)

= log

a:,

these functions, and

i|)

W=j-

There are tables

for the values of

simplifies the construction of their graphs.

this

graphs y

Constructing the

Fund's

Differential Calculus: Investigation of

Ch. Ill

186

= \ogx

and y

= -j

(see Fig. 57),

the approximate value

of

we

find

the only

2.5.

root 1

3.12.5. Find the real root of the

equation
/ (x)

= x 2x + 3x5 =
s

with an accuracy up to 10~ 4


(a)
by applying the method
:

of

chords,
(b)

by

applying

the

method

of

tangents.
Solution. Let us first make sure
Pig 57
that the given equation has only
one real root. This follows from the fact that the derivative
/'

(*)=3* a

4* + 3>0.

Then, from /=(1) =


that the given
it follows
3<0, (2) 1
polynomial has a single positive root, which lies in the interval (1, 2).
(a) Using the method of chords, we obtain the first approximation:

= >

^=1-=^.
4
1

1=1.75.

Since
/(1.75)

and

/ (2)

= > 0,
1

then 1.75

= 0.5156 <0,

<<

2.

The second approximation:


x2

1.75

0.25

.75

+ 0.0850 =

.8350.

< <

<

2.
0.05059
Since / (1.835)
0, then 1.835
g
The sequence of the approximations converges very slowly. Let
us try to narrow down the interval, taking into account that the
1.835 is considerably
value of the function f (x) at the point x 2
less in absolute value than f (2). We have

f(1.9)

Hence, 1.835

<<

1.9.

= 0.339 >0.

Transcendent. Equations

3.12. Algebr. and

Applying the method


will get a

chords

of

187

the interval (1.835,

to

1.9),

we

new approximation:

*3= l-835-

.3-;

059

0065 = L8434

Further calculations by the method of chords yield

x4

xb

1.8437,

1.8438,

1.8438 with
and since /(1.8437)<0, and / (1.8438) > 0, then
-4
the required accuracy of 10
(b) For the method of tangents we choose x = 2 as the initial
and /" (x) = 6x 4 >
in the inapproximation, since /(2)=1
2
4x
3 also retains its
terval (1, 2). The first derivative f'(x) = 3x
.

>0

sign in the interval (1, 2), therefore the


plicable.

The

method

of tangents is ap-

approximation:

first

^==2 1/7=

1.857.

The second approximation:


0.0779
(1.857)
QC7
7
1.857
^= 1.857-^-^=
-^27g= 1.8439.
,

The

QC

Q/1Qft

third approximation:

^= 1.8439-^^= 1.8438,
already gives the required accuracy. Here the sequence of the approximations converges much more rapidly than in the method of
chords, and in the third approximation we could obtain an accuracy
up to 10" 6
.

3.12.6. Find the least positive root of the equation tan x


an accuracy up to 0.0001 applying Newton's method.

= x with

x
logx=0 by
3.12.7. Find the real root of the equation 2
combining the method of chords with the method of tangents.
Solution. Rewrite the left member of the equation in the following way:
f(x)

= (2-x) + (- log*),

it
is seen that the function
f (x) is a sum of two monotonically decreasing functions, and therefore it decreases itself. Consequently, the given equation has a single root .
Direct verification shows that this root lies in the interval (1, 2).

whence

This interval can be narrowed

still

1.6<<

further:
1.8,

Ch. 111. Differential Calculus: Investigation of Fund's

188

since
^

(1.6)

= 0. 1959 >

0;

= 0.0553 <

.8)

( 1

0.

Then

W = _l_l

lege;

/*(*)

= -! log*

and

/'(*)<

0;

Applying

over the whole interval


rW>0interval
both the method
this

to

method
first

of tangents with
approximations:

'

the

V(i78

6)

(1

l /(i

x[=l.6pffi =

point

initial

6)

6H

0.

+ 0.1540=

1.6

,v

[1.6;

8].

chords and the


=1.6 we obtain the
of

1559=

.7559;

1.7540.

Applying the same methods

to the

interval

1.7559],

[1.7540,

we

get the second approximations:

(1.7540-1.7559) / (1.7559)
/ (1.7540)/ (1.7559)

jc,= 1.7559-

7540x'-l
l./DW
X
2

/(1

= 0.00001,

x'2
Since x 2
up to 0.00001.

3.12.8. Using the

f(x)==x 3

ox + =
1

7540)

1.75558,

1.75557.

(1.7540)

the root

computed with an accuracy

is

combined method

find all

roots of the equation

accurate to three decimal places.

Applying the iteration method

find the real roots of the


accurate to three decimal places.
sin a:.
Solution. Represent the given equation in the form x
0.25
Using the graphical method, we find that the equation has one
real root
which is approximately equal

3.12.9.

equation x

sin x= 0.25

to

1.2 (see Fig. 58).

Since

>
1.3 = 0.9636 <

sin 1.1

sin

=0.8912

1.1

0.25,

1.3-0.25,

the root lies in the interval (1.1, 13).


Let us rewrite the equation in the

form
x

cp

(x)

= sin x + 0.25.

cos x in the interval (1.1, 1.3) does


Since the derivative q/ (x)
1 in absolute value, the iteration method
0.46
not exceed cos 1.1
is applicable. Let us write successive approximations

<

xn

<

= sin x n _, + 0.25

(n=l,

2,

...),

3.12. Algebr. and Transcendent

taking x

1.2 for the

jc,

x3
x4
jc

x6

Since

Equations

189

approximation:

initial

= sin 1.2 +0.25 = 0.932 +0.25=1.182;


= sin 1.182 +0.25 = 0.925 +0.25= 1.175;
= sin 1.175 +0.25 = 0.923 +0.25= 1.173;
= sin 1.173 +0.25 = 0.9219 + 0.25 = 1.1719;
= sin 1.1719 + 0.25 = 0.9215 + 0.25= 1.1715;
= sin 1.1715 + 0.25 = 0.9211+0.25= 1.1711.

^=0.46 and

hence,

yzrj<

we have

1.171

within the

required accuracy.
3.12.10. Applying the iteration method, find the greatest positive
root of the

equation
x*

+ x=

1000

accurate to four decimal places.


Solution.

root

We

Rough estimation

gives us the approximate value of the

10.

can rewrite the given equation

x=

1000

the lorm

in

a:

3
,

or in the form
1000

l_

X2

or in the form

x=jj/l000 x and
The most advantageous
one, since taking [9,

the

of

(p

we

find that

indicated

for the

10]

so on.

main

Y 1000

methods is the preceding


and putting

interval
x,

the derivative

<P'(*)=
3

does not exceed

3/

(1000

-x) 2

1/300 in absolute value:

|q>'

(X)

<C
3

990*

~ 300
6W = 4-

Ch. III. Differential Calculus: Investigation of Fund's

190

Compute

successive approximations of x n with one spare digit by

the formula

x n+1

=y\000-x n

x2
x3

(n

= 0,

...),

2,

1,

*o= 10,
1000 10 = 9.96655,
1000 9.96655 = 9.96666,

=V
=V
= J/ 1000 9.96666 - 9.96667.

9.9667 with an accuracy of 10~ 4


put
Note. Here, the relatively rapid convergence of the process of iteration is due to the smallness of the quantity q. In general, the
smaller the q, the faster the process of iteration converges.

We may

3.12.11. Applying the


the equation

of chords,

find the positive root of

= x + l.U + 0.9* 1.4 =


3

f(x)

with an accuracy

method

of 0.0005.

3.12.12. Using the method of chords, find approximate values of


the real roots of the following equations with an accuracy up to 0.01:

l)2_2sin* = 0;

(a)

(b) e

2(1 =
x)

0.

3.12.13. Applying Newton's method, find with an accuracy


the positive roots of the following equations:

to 0.01
(a)

*3

+ 50* 60 = 0;

+ x 32 = 0.

(b) x*

method

3.12.14. Using the combined


of the

up

find the values of the root

equation
x3

on the interval

[1,

-0

with an accuracy up to 0.005.

2]

3.12.15. Applying the iteration method, find all roots of the equa5 accurate to four decimal places.
5 In x

tion \x

3.13. Additional
3.13.1.

Problems

Does the function

f{x)=
satisfy the conditions of the

if

\\/x

if

<
*>1
x

Lagrange theorem on the interval

[0,

2]?

$x
ax
3.13.2. Prove that for the function y
y the number
Lagrange formula, used on an arbitrary interval [a, b] is the

in the

arithmetic

mean

of the

numbers a and

b:

= (a-f fc)/2.

3.13. Additional Problems

3.13.3. Prove that

the equation

if

a x

191

+a x

n~

+a n _ x =

has a positive root x ot then the equation

na x n

"

+ (nl)a x n ' +
2

has a positive root less than x

+a n _ =
x

4x 1=0

3.13.4. Prove that the equation x 4


real

has two different

roots.

3.13.5. Prove

more than two

function

the

that
real

/ (x)

roots for n even and

= x n + px-\- q

cannot have
for n odd.

more than three

3.13.6. Prove that all roots of the derivative of the given


nomial f(x)
(x+ l)(x \)(x 2){x 3) are real.

poly-

3.13.7. Find a mistake in the following reasoning.

The function

f{x)

x 2 sin (l/x)

for

\
is

differentiate for any

x 2 sin

x.

cos 4-

As x tends

By Lagrange's theorem

x = x( 2| sin

whence

x =^=0,

forx

= 2 sin \

cos \I

x sin

< < x).

(0

to zero E will also tend to zero. Passing to the

we obtain limcos

(1/g)

= 0,

whereas

it

is

known

limit,

that Tim cos (l/x)

l -*

is

x-+

non-existent.
3.13.8. Find a mistake in the following deduction of Cauchy's
formula. Let the functions f (x) and y(x) satisfy all the conditions
of the Cauchy theorem on the interval [a, b]. Then each of them
will satisfy the conditions of Lagrange's theorem as well. Consequently, for each function we can write the Lagrange formula:

f(b)-f{a)
cp(ft)

Dividing the

first

(a)

= f{b-a),
= (I) (ba),

expression by the second,

f(b)-f(a)
(p(ty-cp(a)

f'($)(b-a)
y'(t)(b-a)

3.13.9. Prove the following inequalities:


(a)

<

In

J < r-

-r-

a<t<b,
a<l<b.

cp'

if

< b < a,

we

obtain:

/'(E)
'

(p'(g)

Ch. III. Differential Calculus: Investigation of Fund's

192

(b)

pyP~

(xyX^xP yP^pxP- {xy)


1

3.13.10. Prove that

all

<y<x

if

and

p>

1.

roots of the Chebyshev-Laguerre polynomial

are positive.

3.13.11. Prove that if the function f (x) satisfies the following


conditions:
(1) it is defined and has a continuous derivative of the (n l)th
~
order f (n 1} (x) on the interval [x 01 x n ]\
(n)
(x) in the interval
(2) it has a derivative of the nth order f
(*0

'0>

(3)

(*o)

= / (*l) =.-=/ {*n)

then inside the interval [x


that

i,l)

(l)

3.13.12.

= 0.
The

<X <...< *),

(X
]

there

at

is

one

least

point I such

limit of the ratio of the functions

lim

is

xn

e-'2x (cos x -f 2 sin x)


-p-:

e-*(cosx + sinx)

non-existent, since the

hm

_ ^

expression

^
1

^~ x,.1- L 2 tan*
r -Manx

,.

8"
is

-j-

tan x

discontinuous

nn n/2 (rc 0, 1, ...), but at the same time


the points x n
limit of the ratio of the derivatives does exist:

\e~ 2x (cos x~\-2 sin x)\'

lim

x^oo

nr-=
+ sinx)]'
-

(cos^

5e~

. =
-2e-*sinx
2x s'mx

l;

lim

5
2 x

lim e -*
^ x

at

the

= n0.

Explain this seeming contradiction.

number

remainder

of

the Taylor

+ h) = f(a) + hf'(a) + ^ f" {a + Qh)


/"' (x)
continuous at x a and
h *

/"' (a)=^=0.

3.13.13. Prove that the

formula of the

first

in the

order

fia
tends to 1/3 as

if

is

3.13.14. Prove that the

number

3.13.15. Prove that for

0<x^n/2

decreases.

an irrational number.

the function
obtain the inequality 2x/n
and give its geometric meaning.

From

< x < jt/2

e is

this

f (x)

<

sin

3.13.16. Show that the function / (x) = # + cos x


whence deduce that the equation x + cos x = a has no
for a < 1 and has one positive root for a > 1.
3.13.17. Show that the equation
root found in the interval (0, 1).

xe x

=2

has

---(s'mx)/x

<x

for

increases;

positive roots

only one positive

3.13.18. Prove that the

{x )

function

= {i x + x2sln T
{

is

193

Additional Problems

3.13.

for

x=0

for

x=

>

not monotonic in any interval containing the origin. Sketch the

graph f(x).
3.13.19. Prove the theorem if: (1) f (x) and
interval [a, b] and differentiate inside

and

(3)

f'(x)>y'(x)

Show

3.13.20.

nor

minima

+ px + q

2
3.13.21. In the trinomial x

mum

equals

minimum

3.13.23.

function

the

x--x 09 where n

continuous

at

=x

Given

has neither maxima,

is

/ (x)

= (x

a natural

that

q)

(x)

(b)

= \x\
= E(x)

sin 7)l*l

w = j( 2

for
for

for

extremum

number; the function

(p

at

(x) is

continuous function

/(#)

n
)

has

at

*^=>

at

x=

0.

minimum

at the point x
0, but
0.
or on the right of x

is

not

left

3.13.24. Find the greatest and the least values


functions on the indicated intervals:

y
y

= y(a)\

(a<x<b).

and (p(# )^=0.

monotonic either on the

(a)

continuous
f(a)

choose the coefficients p and q


3 and that the miniat x

Show

(2)

5.

3.13.22. Test
the point

= ^+

/ (x)

bc=^0.

so that the trinomial has a

it;

then f(x)>y(x)

that the function

ad

at

(a<x<b),

are

(x)

(p

in the

2<x<

of the following

1.

3.13.25. Do the following functions have


values on the indicated intervals?

the

greatest

and the

least

(a)

f(x)

(b) / (x)

= cosx
= arc sin x

for
for

ji/2

^ <
a;

<x<
1

ji,

3.13.26. Prove that between two maxima (minima) of a continuous


function there is a minimum (maximum) of this function.
3.13.27. Prove that the

function
a:

sin 2 (l/x) for

\0

for

=
f W

x=0,
x

Ch. III. Differential Calculus: Investigation of Fund's

194

has a

minimum

at

the point #

3.13.28. Prove that

if

at

right-side derivative, then


left-side derivative,

3.13.29.

Show

then

it

it

the point of a minimum there exists a


if there
is non-negative, and
exists a
is non-positive.

that the function


I/* 2

\3* 2
has a

minimum

at

not change sign

minimum).

(not a strict

(x>0),

(x<0)

the point #=^0, though its first derivative does


passing through this point.

when

3.13.30. Let x be the abscissa of the point of


flection on the
curve y = f(x). Will the point x be a point of extremum for the
function y
f (x)?
i

3.13.31. Sketch the graph of the function y


if
of the point
1

x=

bourhood

f(-i>=2,
3.13.32. For
probabilities"

r< i)

what choice

i.

of the

= f(x)

in the

neigh-

/"(-i)=o, r'(*) >o.


parameter h does the "curve

= -4r -*'**
V
inflection x = zko?

of

(ft>0)

Tt

have points

of

3.13.33. Show that any twice continuously different iable function


has at least one abscissa of the point of inflection on the graph of
the function between two points of extremum.

x*
8x*+ 18a: 2 8 as an example,
3.13.34. Taking the function y
ascertain that there may be no points of extremum between the
abscissas of the points of inflection on the graph of a function.

Prove that any polynomial with positive coefficients,


an even function, is concave everywhere and has only one

3.13.35.

which

is

point of

at

minimum.

3.13.36. Prove that any polynomial of an odd


least one point of inflection.

3.13.37. Proceeding
the straight line y

directly

= 2x+l

2x 4 -f-;c 3 +l

is

degree

n^3

has

from the definition, ascertain that


an asymptote of the curve y =

Chapter
INDEFINITE INTEGRALS.
METHODS OF INTEGRATION

BASIC

Direct Integration and the

4.1.

Method of Expansion

Direct integration consists in using the following table of integrals:


(1)

(2)

(3)

U du

+C

]^- = \n\u\ + C-

dw-=^a* + C;

aa

= sin u + C;

(4)

cos a da

(5)

cosh u du

(6)

f;

= e + C;
sin^dw = cosw + C;
J
= cosh w + C;
^ sinh udu
e

= sinh u-\-C\

du

du

tt

dl

f
t
2
J sin

= cot w + C;
1

(8)

f-^==l a rctan- + C= iarccot-^ + C, {a > 0);


du
= arc sin + C= arccos 4-C. (a > 0);
[ r

(9)

(7)

(10)

u1

a 2

=ln(tt

=
-

J/

i^tf)

+ C;

+ C.

In all these formulas the variable u is either an independent


variable or a differentiable function of some variable. If

\f(u)du = F(u)

+C

then

J/
a

(ax

+ b)dx = jF (ax + b) + C.

The method of expansion consists in expanding the integrand into


linear combination of simpler functions and using the linearity

Ch. IV. Indefinite Integrals

196

property of the integral:

2=

4.1.1.

(x)dx=^

fi

a i lf
(x)dx
i
J

Find the integral

*2

+5*V

\{=

|fl/|>0).
/

dx.

Solution.
j

x2+

^i~

(jc / 2

+ 5*"', x - v.) dx =

i>dx

+ b^x i>dx^x-

=j

}x

i*dx==

= fi_ + C + -3- x + C - 2*'/. + c =


3

/2

= 2^(^ + |-l) + C.
Note. There is no need to introduce an arbitrary constant after
calculating each integral (as is done in the above example). By combining all arbitrary constants we get a single arbitrary constant,
denoted by letter C, which is added to the final answer.
a

P6*3 + * 2 2*+l

4.1.3.

/=

f
2
,
2
J sin * cos *

Solution. Transform the integrand in the following way:


sin 2 *-)- cos 2 *

sin 2 * cos 2

sin 2 * cos 2 *

a;

sin 2

cos 2 *

*'

Hence,

4.1.4. I

= ^tan

f-^r-

2
J cos *

f tan 2

=J

Solution.

we

(x 2

2,

= sec

^|
xdx= [
cos *
J

t/

4.1.5.

xdx.

Solution. Since tan 2 x


/

= tan x cot x + C.
+ J[ -tsin x

+ 5)

1,

then
I

dx

= tanx + C.
jc

dx.

Expanding

the

integrand

by

the

binomial

formula,

find

/=

j(x 6 +

4.1.6. /

15jc

+75a; 2

= J (3x + 5)

17

+ 125)dx = y- + -^ +

dx.

^+

125x

+ C.

4.1.

Direct Integration and the

Method

of

Expansion

197

Solution. Here it is not expedient to raise the binomial to the


3x-\-5 is a linear function.
17th power, since u
Proceeding from the tabular integral

we

get

(3*+5)

4.1.7.

/=

4.1.8.

r
rc.

is

cos(jtx
J

Solution. Proceeding from the tabular integral

cos w du
J

(4)

= sin w + C,

we obtain

= J-sin (nx +

4.1.9.

cos

4a;

cos

7a:

+ C.

1)

d*.

Solution. When calculating such integrals


the trigonometric product formulas. Here

cos 4* cos 7*

(cos

3a:

+ cos

it

is

advisable to use

1a;)

and therefore

= IP cos 3a: dx + y
1

P
\

cos

lU^^-g-sinSAr + ^sin 11a:+

/Vote. When solving such problems


following trigonometric identities:

it

is

expedient

C.

to use the

n) x + sin (m-\-n) x];


n) x cos (m + n) x]\
sinmA:sin nx =^ [cos(m
cosmx cosnx =
Y [cos (m n)x + cos (m + n) x].
sin

mx cos nx = -^

[sin

(m

4.1.10. /

cos a; cos 2x cos

5a: dA;.

Solution.
(cos x cos

2a:)

We

have

cos 5x

= y (cos + cos 3a:) cos 5x =


= [cos 4a: + cos 6a:] +
a:

-j-

(cos 2#

+ cos 8#)

198

Ch. IV. Indefinite Integrals

Thus,

cos,

2 x dx

+ J cos d# ^ cos6xdx+ ^cos8xdx^ =


= sin 2x + sin 4x + ~ sin 6a: + ^ sin 8a:
4a;

-f-

-g-

4.1.11. /

$sin

3A:dA:.

Solution. Since sin 2

=Y
l

Since

Solution.

/=-

2!?!?

cosh 2 w

= ^2 + ^ +5

= cosh
10)]

dx =

4.1.16. /

=j

Solution.

4.1.17.

= arc tan (* + 2 + C
)

d*

V4 9x

'

r r

dx

==

7=f^J|==r7=iL
VS x-t-Ax
j/9-(x+2)
=

^?

+ C.

=.

4-1.19. /

dx

/= 1-7=^=^ = 4- 1-7====- = 4- arc sin

/=

5o/^to.

lx + l s inh(16x+10) + C.

dx

then

= ]C x + 4x+5 = ^C

Solution.

then

(8x+5)dx.

[1+ cosh (16*+

4.1.13. /

(1

11
cos6A:)dA; =-ja:
^sin6^: + C

= ^cosh

4.1.12. /

3A;

-f-

arcsin^
+
^ C.
3

_^_.

|4

Solution.
1

{
J 4

dx

4x
2

C
J

ch
<ta

t+2) 2

(a;

=TT7^1n

4^2

j/l + AT+2
/*2 (* + 2)

+ C.

C.

4.2.

Integration by Substitution

199

4.1.21. Evaluate the following integrals:

P3 2 cot 2

4.1.22.

'

(c)

d>

<

+ 3x

JiM?^^

at^.h-

cos 2*

(5)

x*

4.2.

Integrate:

f
^

cos**

cos*

dx;

(d)

(sin

5*

^
*

sin*

sin 5a) dx.

Integration by Substitution

The method
stituting

cp(/)

function.

On

of substitution (or
for a; where q) (t)

we

substituting

change of variable) consists in subis


a continuously differentiate

have:

lf(x)dx=lf[<p(t))<p'(t)dt,
and

we

integration

after

stitution

-1

return to the old variable by inverse sub-

(x).

q)

The indicated formula


(01

5 f [<P

is

(0 dt

<P'

also used in the reverse direction:

W dx

\f

where x

>

= y(t).

= J xVx^dx.

4.2.1. /

Solution.

Make

the substitution

Whence
x=

a: 5=t\

+ 5,

we

Substituting into the integral

get

+ 5)*.2* dt=2 j(/ + 5*


4

/=J(/

Now

dx=2tdt.

2
)

df

=2- +

^+

C.

return to the initial variable x:

'

4.2.2. /

= j- +d*e*

2(*-5) 6/

10(*-5)

3/

'

'

Solution.

Let us
e

make

=
t

\,

the substitution

= ln

1),

l+e*=t. Whence

dx=dt/(t

\).

200

Ch. IV. Indefinite Integrals

Substituting into the integral,

we

get
dt

Jn(/-I)'

\+e x

<

But
1

t(t

therefore

Coming back

l_

\)~t\

^ -^

= ln\t-l\-\n\t\ + C.

we obtain

to the variable x,

I^ln^ + C^x-lnil+e^ + C.
Note. This integral can be calculated in a simpler
tiplying both the numerator and denominator by e~ x

way by mul:

j^dx = - 7^- dx = -\n(e-*+l) + C =


l

= -ln^-tl = x-ln(^+l) + C.
4.2.3.

/=

4.2.4.

/2+3

dx.

V&x--5)3
(x2

/=r
4

+ 3x

- x)dx

+l)arctan^-t-J

Solution. Transform the integrand


(\

+ l/x)
Make

the substitution x

\/x*)dx

+l]arctan(.*:+l/;tr

+ -j=t;

differentiating,

(l-)dx = dt.
Whence
d

/-f

Make one more

(/

+l)arctarW

substitution: arc tan


dt
t

+\'

'

= u.

du

and
/

= j^=ln|| + C.

Then

we

get

Returning

to

first

= In

4.2.5. /

Integration by Substitution

4.2.

and then to

arc tan

we have

x,

+ C = In

t\

201

arc tan (*

+ 7) + c#
|

V a '~ x2 rlr

x*

Make

Solution.

the substitution:

x= T

dx =

-^
dt

Hence,

Now make
=

1=2.

one more substitution: \faH 2

Then 2a 2 /

dt

2zdz and

Returning to

^ 2

(3

and then to

/ ~~ r

2
2
J a sin

x+b 2 cos 2

we obtain

x,

"

a:

Solution.

a 2 sin 2

A;

+&

cos 2 a:

dx
b2

Make

the substitution

&

Returning to

a:,

Solution.

4.2.8. /

/T + 3

Make

t;

a?2

dt

tan

+
.

a:

cos 2 a:

dx
-%-

"

6 cos 2

Then

a:

we obtain
/

4.2.7. /

^-tanx

= ~ arc tan

tan

+ C.

sin xcosjcd*.

the substitution

l+3sinA:

3cosjc<2a;

= <#.

^7d^^pv. a=i.4<v.+c= (l+3 ; n ^


<

j"

cos X

Then

/,

+c.

202

Ch. /V. Indefinite Integrals

^-yr

4.2.9.

f
b
J (arc cos x) y

Solution.

Make

x'

the substitution:

dx

arccosx^/;

1^
/

4.2.11.

d/-4-^"
4

- X*

= dt.

Then

+
~ c = 4 arc cos rx + c.
^

^^L=dx.

.10.

Cr

J/ 5

sin 2x
= j_5!^L<fr.
+ sin x^
2

Make

Solution.

the substitution:

sin 2

1 -f-

x=t;

2 sin

x cos x dx

= sm 2x dx ~^dt.

Then

/=Jy-

= ln/ + C = ln(l + sin

*)

-|-

C.

Substitute

Solution.

and

+ #lnA; =

/,

(1

+ \nx)dx = dt

get
/

4.2.13.

=j = \n\t\ + C = \n\3 + x\nx\ + C.

Evaluate the following integrals:

In

x) x

4.2.14. Find the following integrals:


(a)

^yi-xdx;

In

(b)

x dx

f-i^

cos x V^sin

(c)

(d)

.xrck;

4.3.

In

Ax

y*

Integration by Parts

The formula
^

udv = uv

ydw

is known as the formula for


integration
are differentiate functions of x.

by

parts,

where w and v

Integration by Parts

4.3.

203

To use this formula the integrand should be reduced to the product of two factors: one function and the differential of another
function. If the integrand is the product of a logarithmic or an
inverse trigonometric function and a polynomial, then u is usually
taken to be either the logarithmic or the inverse trigonometric function. But if the integrand is the product of a trigonometric or an
exponential function and an algebraic one, then u usually denotes
the algebraic function.
4.3.1.

arc tan x dx.

Solution. Let us put here

= arc tan

dv

a;,

= dx>

whence
dx

^ = r+^
*

V==X)

= jarc tan x dx = x arc tan x ^ iq~^2 = x arc


4.3.2.
4.3.3.

/=
/

Solution.

an *

~^ x ^

arc sin xdx.

xcosxdx.

Let us put
u

= x\

dv

= cos

a; <*;,

whence

du

1=
We

will

^ A;cosA:dA:

= dx\

= Arsin

a:

= s'mx

sinA:dA:== Arsin

show now what would


and dv.

from an

result

x-fcosx + C.
unsuitable choice

of the multipliers u

^xcosxdx

In the integral

//

let

us put

^ cos x]

dv

= x dx,

whence
du

sin

a:

dx;

=y

2
a;

In this case
/

As

is

~ -1

cos x

+yJX

sin

a: <2a;.

obvious, the integral has become more complicated.

4.3.4.

/=

3
a:

lnA:dA:.

204

Ch. IV. Indefinite Integrals

Solution. Let us put

= In x\

dv

=x

dx,

whence
i

du
/

jc

In

4.3.5.

dx
=
X

= .,\
1

;
'

-j^x^dx = ^-x*\nx -^x + C.


4" J ^ ^ T
=
2* + 5)<T*dx.
x

nA:

(a;

Solution. Let us put

=x

2x +

dv

5;

= e~ x dx

whence

= (2x 2)d;c; u = e~ x
2x+5)e- x dx = e- x {x 2x + 5) + 2
\

=^

We

again integrate the

dv

u\

whence
du

=2

Finally
7

(jc \)e~ x
J

we

by

integral

last

x =

71

(x\)e~ x dx.

(x 2

= dx;

Put

parts.

e~ x dx>

= e~ x

dx= 2e- x (x l) + 2

e-*dx=

2*g-* + C.

get

= e-*(*

+ 5) 2xe-* + C = e~
2a;

x (x 2

+ 5) + C.

a result of calculation of integrals of the form ^ P (x) e ax dx


we obtain a function of the form Q(x)e ax where Q(x) is a polynomial of the same degree as the polynomial P(x).
This circumstance allows us to calculate the integrals of the indicated type using the method of indefinite coefficients, the essence
of which is explained by fhe following example.

Note.

As

Applying the method

4.3.6.

1=
Solution.

(3a:

17)

e 2x

of

indefinite coefficients, evaluate

(3x 3 \7)e 2x dx.

dx=

(Ax 3 + Bx2

Differentiating the right and the


(3a;

7) e

2x

Cancelling e 2x

left

+ Dx + E) e 2x + C.

sides,

we obtain

= 2 (Ax + Bx + Dx + E) e2x + e2x (3 Ax + 2Bx + D).


3

we have

3x 17 = 2 Ax + (2B + 3 A) x + (2D + 2B) x+(2E + D).


3

205

4.3. Integration by Parts

Equating the

the equal powers of x in the

coefficients at

we

right sides of this identity,

left

and

get

= 2B + 3A;

= 2A\
0-2D + 2B;
3

17=2 + D.

Solving the system, we obtain

~-

'

Hence,

(3x 3

7)

a
?

~ + -J x
^

*dx - (-| a; 3

a;

2
) e *

+ C.

Integrate:

4.3.7.

=J

(*

+ \)cosxdx.

Let us put

Solution.

a;

= cos x dx,

do

whence
da
/

where

lx

= (^3 +1) sin


=

= 3jc

a:

a:

cos

= ^ (x*+

1)

a;

+ 1) sin x 3/

lf

get

cos x

2,

we obtain

+ l)sinA;+3A: cosA: 6a: sin x 6 cosa: + C =


= 6a: + sin x + (3a: 6) cosa:+C.
2

cosA:(iA:=(A: 3

The method

indefinite coefficients

of

(a:

may

also

be

applied

sin

2x

form

P (x) sin ax dx,


J

1)

/=

+ 2/

have:

to integrals of the

4.3.8.

= xsmx + cos x + C.

(a;

Note.

dx= (x

at^a:.

I2

we

we

Integrating by parts again,

Finally,

sin x

= sinA:.

x sin x dx.

/x

/2

a;

Integrating by parts again,

where

(iA;;

+ 3a: + 5) cos 2a:

P (x) cos ax dx.

g!a:.

Solution. Let us put


(a:
jj

+ 3a:+ 5) cos
= (A

2a:g!a:

+ A x+ A
x

2)

cos

2a:

+ (B

a:

+B x+B
x

2)

+ C.

206

Ch. IV. Indefinite Integrals

Differentiate both sides of the identity:

+ 3x + 5) cos 2x = 2 (A x + A x + A sin 2x +
(2A
x+A cos 2a; + 2 (B x + B^x+B^) cos 2x+ (2B x+B
+
= [2B x + (2B, + 2A x+(A,+ 2B cos 2x +
+ [ 2A x + (2B 2A,) x -f (B 2A

(x2

2)

l)

)s'm2x=

2 )}

2 )]

sin 2x.

Equating the coefficients at equal powers of x in the multipliers


cos 2x and sin 2x> we get a system of equations:

2B

2(B 1 + A

1;

2A = 0;

= 3;

A,) =

2 (B

A
B

0;

1
x

+ 2B = 5;
2

2A =
2

0.

Solving the system, we find


A

/i

_A.
d
u, /Jo

J_2

/I

'

__J_.

'

_
A2

/i

'

_
A4

R
2

'

Thus,
J

(*

+ 3x + 5)cos2xdx= ( y + t) cos2jc+ (y * + jx + ~) sin


2

4.3.9.

/=

(3a:

+ 6a: + 5) arc tan

a:

x+C.

dA:.

Solution. Let us put

= arc tan

do

a:;

= (3a: + 6a: + 5) dx
2

whence
^w

"

= + 3a: + 5
2

a:

a:.

Hence,
/

= (* + 3a: + 5x) arc tan x ^"^ 3 5* dx.


j
ff
2

Single out the integral part under the last


the numerator by the denominator:

by

integral

=J + 3x + 2j^-3.J I ^ 1 =f + 3*+21n(x

dividing

+l)-3arctanA'+C.

Substituting the value of / lf we finally get


I

= (x* + 3x + 5a: + 3) arc tan x


2

/2

3x 2

4.3.10. Find the integral


/

Solution.

^e bx cosAxdx.

Let us put
e bx

= ^;

cos

4.v

=. do,

In (x

+ + C.
1)

integration by Parts

4.3.

207

whence
5e bx dx

= du\

sin

4a:.

Hence,

4 e
1

5*

sin \x

I 1 z=

sin

Thus,

= -j e bx sin 4a:
1

/
l.

Axdx^

e 5 *sin4A:dA:.

we obtain

Integrating by parts again,


r*

-^-e *cos4.*;

5/1
j
(

e 5 * cos

\x

+ ^-

P
\

e *cos4A:dA:.

e bx cos

4xdx)

e.

= -L e bx ^sin 4x + -|-cos 4a: ^

Whence
/

4.3.11.

=$

= ^e bx

^sin

4a:

cos

-|-

4a:

+ C.

cos(lnx)dx.

Solution. Let us put


/

= cos(lnA:);

dv

= dx

whence

= sin (In x) ^

a:.

Hence,
/

cos (In

A:)rfx

= xcos(lnA:) +

^ sin (In

a:)

dx.

Integrate by parts once again


//

-=sin (In

dv=dx,

x)\

whence
da

dx

cos (In

a:)

v=x.

Hence,
/i

=Js

n (lnA:)dx

= xsin(ln

a:)

a;)

cos (ln^)dA:.
J

Thus
/

cos (In

a:)

dv

a:

cos (In

+ x sin (In x) /.

Hence
/

=y

[cos (In

a:) -f-

sin (In

a:)]

+ C.

208

Ch. IV. Indefinite Integrals

4.3.12. /

= jxln(l

+-j)dx.

Let us transform the integrand

Solution.

In

+ 1 ) = In il = in (x + - In x.
1)

Hence
/

(x+

In

Let us integrate

and

/x

\)dx
/2

\^x\nxdx =
Ii

by

= ln(x+

Put

parts.
1);

dv

/2

= xdx,

whence

Hence
/1

= Jxln(^+l)dx = -i(x- l)ln(x+l)-l - \)dx


j
X
(x+l)-i*
In X +l)-(x-\)dx =
(

Tln

+ x + C.

Analogously,
I2

Finally
/

xln

xdx=Y^ nx

-j"

*a

we have

= J*ln(l

4.3.13.

=^

+^)^ = l(x

-l)ln(x+l)-^lnv+|+C.

= j ^^R[ln(^+l)-2 1n,]^

Solution. First apply the substitution

Then
<

2dx
= -V

or

dx

j,
<ft.

Hence,

The obtained

integral

easily evaluated by parts.

is

= \nt;

dv

= \/"tdt.

Then
du

= Y>

=-~t\ft

Let us put

4.3.

209

Integration by Parts

Whence

-1 Jj/Tln*<tf=
Returning to

x,

-g-

\jtVT\nt

we obtain

'
(^+i)

9x 3

4.3.14. /

4.3.15.

= J \n{VT^x+VT+lc)dx.

Solution.

sinx

In

[23

In

(1+33)]

+C

tanxdx.

Let us put

= In

+ /l+x)

dv

= dx,

whence

~~

'

j/t^*+ i^tt*

'

]/T=i2~~

'

^r=^

a;.

Hence,
/

= x in {VT=Tx + VTTi) - \

x^^^dx =

= ^1^/1 x +

+ x) yjc + y arc sin x-fC.

In calculating a number of integrals we had to use the


integration by parts several times in succession. The
result could be obtained more rapidly and in a more concise form
by using the so-called generalized formula for integration by parts
(or the formula for multiple integration by parts):

Note.

method

^u

of

(x) v (x)

dx

= u (x) v (x) u' (x) v (x) + u" (x) v (x) ...


+ ( l)"- ^"- (x) v n (x) ( l)"" J u (x) vn (x) dx,
1

{n)

where
v1

(x)=^v (x) dx;

v 2 (x)

=^v

Here, of course, we assume that


ring in this formula exist.

(x)
all

dx;

v n (x)

= J vn _

(x) dx.

derivatives and integrals appea-

210

Ch. IV. Indefinite Integrals

The use

of

formula

generalized

the

for

integration

by parts

is

especially advantageous when calculating the integral ^ P n (x) cp (x) dx,


where P n (x) is a polynomial of degree n and the factor y(x) is
times. For example,
1
such that it can be integrated successively
y

n+

dx =

P n (x)

P n (xf - P'n (x)

~+

+ (-!) PJ {x)t + C =

4.3.16. Applying the generalized formula for integration by parts,


find the following integrals:

2x + 3xl)cos2xdx,
(2x + 3x 8x +
V2x + 6 dx.
2

^(x 3

(a)

(b)
J

1 )

Solution.

^( x

(a )

2x + 3xl) cos 2x dx = (x
2

{3x 4x + 3)
/o

9
2

o\ f
^

cos

2a:

/c
j+(>x
,

2x + 3x 2^
2

1 )

sin 2x\
J

4)l^

= 5!?f (2^3 _ 4x + 3*) +


Sx + l)V~2x + 6dx =

c6cosj^ + Cn =
2a:

(b)

l(2x

+ 3x

= (2x + 3x
3

8x+

{2x
\)

4.3.18.

In (x

(2*

arc

4 3 19

+ Vl+x*)dx.

Yx{\nxfdx.
s ' n ****

J
x cos # dx
4.3.20.

sin 3

a:

3* cos x dx.

4.3.21.
J

6* 2

+ 6)3/2 --(e>x + 6jc 8) (2


2

+ 6) (7 0x

Evaluate the following integrals:


4.3.17.

8* + 3) + C

*y +

45x - 396* + 897) + C.


2

Formulas

211

4.3.28. Applying the formula for multiple


calculate the following integrals:

integration by parts,

2x + 5)e
2

4.3.22.

^(x 3

4.3.23.

^(l+x2

4.3.24.

^(x 2
(a:

4.3.26.

jc

3x dx.

cosxdx.

+ 2xl)sm3xdx.

2x + 3)\nxdx.

4.3.25.

2
)

Reduction

4.4.

arc tan

jc

dx.

arc cos

jc

dx.

4.3.27.
J

(a)

+ x 2) sin

^(3x 2

(3*

\)dx\

x * ~* x+

(b)

dx.

Reduction Formulas

4.4.

Reduction formulas make it possible to reduce an integral depending on the index n


0, called the order of the integral, to an
integral of the same type with a smaller index.

>

Integrating by parts,

4.4.1.

derive reduction formulas for calcu-

lating the following integrals:


(a)

(c)

=
I

/=

(**

(a

+ a*)

Solution, (a)

We

(b)

~m

dx;

dx.

integrate by parts.

(*

Let us put

dv

+ a 2\n
a

= dx,

whence
,

du

2n x dx
= r(x^
-^a n +
1

ir

,
9

= x.

Hence,
/

(x 1

a'

whence
n+

~~ lna h
l

'

{x 1

-f-

a 1 )'

2n

'

a1

212

IV. Indefinite Integrals

Ch.

The obtained formula reduces the calculation

of the integral I n+1


and, consequently, allows us
to calculate completely an integral with a natural index, since

to the calculation

the integral

of

/,

f 2
J x

+ a = a arc tan a + C.
2

'

n=l, we

For instance, putting

In

obtain

+ a + 2a? ai C
*

J (**

putting m
/
3

~~

+a

= 2,

we

~~

dx

C
2
J (*

~~ 2a 2

2 2

+a

2 3

'

a:

+a

~~ 2a 2 x*

an

+C

get
3

a:

4a 2

2a 2

(x 2

"

+a

2 2

~~

7 2 ~~~

4a 2

4a 2" (x 2

+a

+ 8a*

2 2
)

'

3
^
alX 130 "7 + C
+ 8^
i

x^fa~2

r*
"

Let us apply the method of integration by parts, putting

(b)

w=sin'-i1 x;

sin *
= cos
a
*

ay

OT

a:,'

whence
da
du

= (n 1) sin" -2 x cos
v

a:

= (m

d^:;

1X
1)

(m
v

m -1 #
cos
.

1)

Hence,
sin" -1 *

~~

(m

cos"2

-1

~ m

-2 x dx
si
C sin"

sin"- 1 *

(m 1)
(c)

- #

~ccos 7

-1

cos* 1

"" 2 a-*
'

m ^=1

/-

Integrate by parts, putting

= (a

whence

= 2nx (a

2
a;

dv = dx,

)";

2
a:

)"

-1

dx;

= x.

Hence
j

n=zX

(a*

x )"- dx =
a + a
= x(a x + 2n

x + 2n $ x
n

(tf

(a:

a:

2
)

(a

(a 2
2

a;

a;

)"

2 )"" 1

In

=x(a

2
)

+ 2na

In _ 1

Hence,
j

7/1

2na 2
x2 ) n
2/z+l
~2/i+l

2
__ x(a

-i'

2n/ + 2na I
2

Wherefrom, reducing the similar terms, we obtain

(\+2n)

dx

n_v

213

Reduction Formulas

4.4.

For instance, noting that


.

I-

we can

find

1/2

_
=

P
\

ax
dx
r -n

>

successively

= JV^"^ ^ = |(a -x

= J(a

~x + Cn
.

" arC sin

~A; 2 ) 3 / 2

^=

-J-(a

2 )i/2

+ ^/_ 1/2 ^
= -i[/5^i + -^arcsin| + C,

3/ 2
)

+ -|-a

/ 1/2

and so on.

Applying integration by parts, derive the following reduc-

4.4.2.

tion formulas:
(a)

(b)

/=

(\nx) n dx

= x(\nx) n nl n _
*>"

= \ x (In x) dx =

xn e x dx

= x? ex nl n -

^
\

(a^-1);

(c)

(d)

/= J^'sin"**^

= -noSin" ^(asinx ncos^H


Derive

4.4.3.

= (-^sin"

and use

it

reduction

formula for the integration

the
for calculating
to

integral

a:

4.4.4.
(a)

the

In

Derive the reduction formulas for the

= J tan n xdx;
J

V x*-\-a

(b) /

^-

cot" xdx\

/ 3a

of

/_,.

=f-^r-.
sin* x
J

following

integrals:

h a p

BASIC CLASSES

OF INTEGRABLE FUNCTIONS

5.1.

If

Integration of Rational Functions

the denominator

(x)

(x)

of the proper rational fraction pry-(

can

be represented in the following way:

= {xa) k {x b)

{x)

1
.

(x

+ ax+$)

where the binomials and trinomials are


the trinomials have no real roots, then
Ax

P(x)

A2

xa

(x)

x2

x b

+ yx +

different and,

\i)

s
.

furthermore,

Ak
~
~ (xa)*
,

'

'

'

(xb)*

+ ax + p
*a

(x

(xa)*

(*

+ V^+M-

"

+ cu;+p)
(*

M*

b) 1

(jca

+ Y*+M0

'

+ ouc + P)'
(*

+ Y*+H)

where
i4

i4 lf

B 19 B

.
.

M N
lf

19

7W 2

2y

/? lf

L lf

/? 2 ,

L2

They

are determined by
reducing both sides of the above identity to integral form and
then equating the coefficients at equal powers of x, which gives
a system of linear equations with respect to the coefficients. (This

some

are

real constants to be determined.

is called the method of comparison of coefficients.) A system


equations for the coefficients can also be obtained by substituting suitably chosen numerical values of x into both sides of the

method
of

identity.

(This

successful

method

is

combination

experience, often allows

us

method of particular values.)


the indicated methods, prompted by
to simplify the process of finding the

called the
of

coefficients.
If

first

the rational fraction

be singled out.

is

improper, the integral part should

5.1. Integration

of

215

Rational Functions

5.1.1.
,

\5x 2*

f*

4x oi
81
+ 4) (*-l)
<ix

Solution. The integrand is a proper rational fraction. Since all


of the denominator are real and simple, the integral will
appear in the form of the sum of three simple fractions of the form
roots

Ax 81
3) (x+4) (x
15a:

(x

where A, B,

X 3

1)

a+4

x 1

are the coefficients to be determined.


then rejecting

common denominator and

fractions to a

Reducing the
it,

we obtain

the identity

- 4x 81 = A(x + 4) (x + B {x3) (x +
+ D(x-3)(x + 4).

15x 2

l)

Comparing the
the

coefficients

we

identity,

get

1)

powers of x in both sides of


equations for determining the

equal

at

system

(*)

of

coefficients

A+B+D-

3A 4B +

15;

Solving the system


Hence,

of

equations

= 31n|x 3 + 5 In
|

we

find

us use

2D = 8

A = 3, B = 5, D- 7.

same example

the

+ 4 + 7 In |* + C =
= In

AWe. Let

4 A + 3B

D = 4;

(x3)

(x + 4)

fi

(x

to demonstrate the

I)

7
1

-f C.

applica-

tion of the method of particular values.


The identity (*) is true for any value

of x. Therefore, setting
arbitrary particular values, we obtain three equations for
determining the three undetermined coefficients. It is most convenient to choose the roots of the denominator as the values of x
3 in the identity (*),
since they nullify some factors. Putting x

three

we
we

get

A =3;

get

D = 7.

5-1.2.

5.1.3. I

=j

=$

putting x

(2

= 4,

we obtain B^=5; and putting x-=l,

+ x)(S-ir

Xi

-<Z7X

dx

of the numerator is higher than that


the fraction is improper, we have to single
out the integral part. Dividing the numerator by the denominator,

Solution. Since

of the

the

denominator,

power

i.e.

216

Ch. V. Basic Classes of Integrable Functions

we obtain
x*

3x 3* 2
x x 2x
2

x(a: 2

+2

x 2)"

Hence,
rx*

3x*3x2,

p,

Expand the remaining proper

(x

+ 2)d*

fraction into simple ones:

*(* 2)(x+l)~~

x+2

1W

2 + a:+1*

Hence

+ 2 = A(x2) (x + l) + Bx (x+ + Dx (x2).


(the
Substituting in turn the values ^ = 0, x = 2, x =
x

l)

of the

denominator) into both sides

/4= _l;

And

of the equality,

fi= |;

roots

we obtain

D= |.

so

= ^ + * + ln|*| -|ln|x 2| yln|x+l| + C.


5,4

=) ^-2 fl+x

dx

Solution. Here the integrand is a proper rational fraction, whose


denominator roots are real but some of them are multiple:

x3

2x

+ x=x(xl)

2
.

Hence, the expansion into partial fractions has the form

3*-f3 _
2x'>+x~~~

2x*
x*

whence we get the


2x

A
x

~"

B
(x l) 2

+ x

'

identity:

3* + 3 == A (x

l)

+ Bx+Dx(xl) =
^(A + D)x + (2 A D + B)x + A.
2

(*)

coefficients at equal powers of x we get a system


equations for determining the coefficients A 6, D:

Equating the
of

+ D = 2;
A=3; B = 2; D =
A

Whence

2A D+B=3\

A=3.

1.

Thus,

/=3j? + 2j 7

^-J^ = 31n|,|-^T-ln|,-H +
T

C.

5.1. Integration

The

Note.

way

roots),

At

coefficients can be

the identity

in

if

of

Rational Functions

determined

in

^=0;

x2

we put

(*)

somewhat simpler

217

(the

denominator

and x 3 equal to any arbitrary value.

x=0

we obtain

we

get

3=A\

#=1

at

5-4 + 2B + 2D;

we

will

= 3 + 4 + 2D;

have 2-=B;

whence

at

D = 1.

=2

5.1.6.

x+

+ = +

2
Solution. Since x 3
1
(x
(the second factor is
1) (a:
1)
not expanded into real multipliers of the first power), the expansion of the given fraction will have the form

x
X3

Bx-{-D

X+\

X2

# +

'

Hence,
x = A(x

x + + (Bx + D) (x+l) =
l)

= (A + B)x + (A + B + D)x + (A + D).


2

Equating the coefficients

A=

at

-T>

equal powers of
fi

x,

we

get

= T' D = T-

Thus,

To

calculate the integral

let

us take the perfect square out of the denominator:


3

and make the substitution x

\ =

t.

Then

tdt

3 V

d/

lln(/*+4) + K3 arctan^L + C.
Returning to x, we obtain
/1

= jln(x

x+l) + K3" arctan^i + C.

Ch. V. Basic Classes of Integrable Functions

218

Thus,

= y ln|* +

-g-

ln(x 2

*+l) + X^-arctan -y~


2

The denominator has two

Solution.

complex

roots,

+C.

pairs of different conjugate

therefore

_ Ax + B + Dx + E

(jc*+1)(jc 2

+ 4)~~

*a

+4

'

hence

l=(Ax + B) {x

+ 4) + (Dx+E) (x +
2

1).

Here it is convenient to apply the method of particular values for


determining the coefficients, since the complex roots of the denominator (# = i and x = 2i) are sufficiently simple.
Putting x = i, we obtain

3fl+3>U

=^
E=

whence ,4=0, B

D = 0,

whence

dx

(x

1 )

= 2i

we obtain

+ 4)~~

i_

3 J x2

y arc tan x

l,

3E 6Di =

-g-

+l

3 J x2

dx

+4

_
""

+ C.

arc tan

dx

~ i (* + x + 2) (** + 4* + 5)'
x + 4x +llx +12x-f-8
(* + 2x + 3) (x+l)
J
2

'

Expand

Solution, Here we already have multiple complex roots.


the fraction into partial fractions:
^4

4a: 3_|_ 11

(x 2

^2

+ 2x + 3)

Find the

(x

2^

*~

B = U

(jt

+1U +12a; + 8-dx+ 2x + 3) (x+\)

4a:

(a;

+ 2x + 3)

Dx + E
2

'

x2

f_

+ 2* + 3 + * +

coefficients:

-4=1;

Ax+B

+8_
1 )

Hence,
^4

1,

Thus,

2
2
J (x +1)(a:

5.1.8.

Putting

D = 0; = 0; f=L

'

5.2.

Calculate /,

^
= (x+

,2

+ 2x + 3

x2

Since
x-\-

dx.
3)2

l)

+ 2,

make

us

let

substitution

the

Then we obtain

i=\

The

(/2

+2

dt

dt

~2 I V

+ 2)

(/a

+ 2)

~~ 2/ 2'

integral
2

is

219

Integration of Certain Irrational Expressions

2
2
J(/ +2)

Problem

calculated by the reduction formula (see


.

dt

4.4.1):

Thus
/

i-

Returning to
/
l

We

i~

finally

(*2

L_
2(/>+2)

2(/

+2)

^r
i

f2

we obtain

x,

2(x 2

o rn for,

2}A2

+ 2a;+3)

2(a: 2

!_arctan^tl-J C

+ 2a;+3)

V2

^2

obtain

+ 2*+3) (a:+1)
2
= ln|x + 11 0/
o*,"^
2
+ + 3)
2

(a:

2a:

ox

YU^2

arc

tan^y + C.
V^2

Find the following integrals:


.

^"^^ 22a; 8 ^>


5x 3 -j-9x 2

5#1 10

r;;
-4a
dx

5.1.11.

(jc+1)(a:+2) 2

5.1.12.

4a:+4)

(a:

(jc

+ 3)'

4a:

+5)

dx

5.1.13.

(1+*)(1+* > )(1+* 8 )'

x +3
U4 IcT+W+T)^'
3

'

'

5.2.

Integration of Certain Irrational Expressions

Certain types of integrals of algebraic irrational expressions can


be reduced to integrals of rational functions by an appropriate change
of the variable. Such transformation of an integral is called its rationalization.

220

Ch. V. Basic Classes

If

I.

the integrand

Integrable Functions

rational function of fractional powers of

is

of

p-

2l\

an independent variable x, i.e. the function R\x x*,


x^j
then the integral can be rationalized by the substitution x=t m
where m is the least common multiple of the numbers q lf q 2
., q k
II. If the integrand
is
a rational function of x and fractional
9

powers of a linear fractional function


tionalization
ax

b
~t
cx-\-d
,

5.2.1.

m
,
'

the

of

where

integral

is

of the

effected

form

by

then
the

ra-

substitution

has the same sense as above.

l=\ x+ Y*+~x dx.

Solution. The least common multiple of the


therefore we make the substitution:

numbers 3 and 6

is 6,

= t*, dx^Wdt,

whence

= 6p
Returning to

x,

+ 6 J^^7 = y/ + 6arctan/ + C.
4

d/

we obtain
2

5.2.2.

5.2.3.

f*+K*

dx.

(2x

3)

+1

Solution. The integrand


Q
we put
3
t

refore

+6 arc tan 1/ x+C.

2x =

dx = 3t b

dt;

is

rational function of

/2x

3, the-

whence

(2x

T=
3) _L

3
\

(2x 3)

=/ 2

Hence,

1+/ 2

3-y~ 3-J- + 3-^ 3^ + 3 arc tan + C.


/

Returning to
/

=3

11
I

Integration of Certain Irrational Expressions

5.2.

x,

(2x 3)

we

-6

221

get
l

-i-

(2x 3)

y (2x 3)

-J-

(2x 3) 6"+ arc tan (2x 3) T + C

5.2.4.

/= f-

/"

The integrand

Solution.

2qr^ therefore

function of x and the

rational

is

ex-

us introduce the substitution

let

+x

+x

'

whence

Hence
j

2(l+/ 3

~~

16/

Returning to x

we

2
)

(1

/>12/ 2

'

" r
ai ___3_
~~
2j^""4/ 2 t U#

3 2
)

/=

get

'=4K(W+ C

5.2.6.

"

(*__=_.

Solution. Since

/(x-lH* + 2) = (x-l)(x + 2) ]/ 2,
5

jZTj
us introduce the substitution:

x-\

%-l

whence

_ tj
a:
/

+2

a:

-12/ a
(/4

#-f-z

,*

1)2

3/4

therefore

let

Ch. V. Basic Classes of Integrable Functions

222

Hence,
l

~ _ Jp

Returning to

(/4-l)(/4

3.3^(^ 4

1)

\2t*dt

-D

"~

3 J

dt___i_ r
/2-3/ + Lt

we obtain

x>

dx
5.2.7.

V 1 *

(1- x)

d*

5.2.8

t+1)
7^=

5.2.9.

j (,-2)

(x

4
l)

/}xx dx.

5.3. Euler's Substitutions


Integrals of the form

R(x

[/ ax 2

-\-

bx

+ c) dx are

calculated with

the aid of one of the three Euler substitutions:


(1)

V ax* + bx + c = txVa
2

(2)

V~ax

(3)

V~ax 2

+ bx + c=tx ]/"c
+ bx-\-c = (x a)t
ax

i.e.

if

5.3.1.

is

if

+ bx + c = a(x a) (x

a=l > 0, therefore we make


Vx + 2x-\-2 = x.
2

Squaring both sides

we

2x

whence

^+

2*

'

+ 2 = + <-^=%T7F

P 2(l+<)(< 2

_ + 2/ + 2
2(l +

Substituting into the integral,


.

and reducing the similar terms,

+ 2tx = 2,

*~~2(1 +/)

the substitution

of this equality

get

f$),

root of the trinomial ax -\-bx-\-c.

Solution. Here

if

real

/=f

> 0;
c> 0;
a

if

we obtain

+ 2/ + 2)

+ 4i + 4)2(l+0

(<

+ 2<+2)rf<

(l+0(<+2) 3

'

Now
tial

us

let

223

Euler's Substitutions

5.3.

expand the obtained proper rational fraction into

par-

fractions:
2

(t

B
D
+ 2/ + 2 _ A
+ 2) ~~ + + + 2 + + 2)
2

Applying the method

B = 0, D =

(/

1 )

(t

'

undetermined coefficients we

of

find:

i4=l,

^>

2.

Hence,
r2

+ 2/

Returning to

we

x,

In (x

/=

5.3.2.

Ai

-f-

d/

d/

Q p

iii

get

2
+ C.
+ + j/> + 2x + 2) H x
r
+ 2+ y x* + 2x + 2
*
_

c+ V * 2 x-\-

Solution. Since
substitution

c=l>0,

here

Vx

we can apply

x+

= tx

the second

Euler

1,

whence

(2/-l)x =

(/

-l)x

^=-2^^^;
Substituting into

/,

= ^rr;

+ Kx'-x+^y^y.

we obtain an

integral

rational

of a

r 2; + 2/
J /(/ !)(/ +

fraction:

I)

-2/ 2

+ 2/ 2

i4

t(t\)(t+\) 2

By

the

method

of

(t

'

\)

undetermined coefficients we

B = -y;

A = 2;

Z>

+
find

= - |.

= -3;

Hence
,

Cdt_

- Z9

1_

dt

o C

J/-1

dl

^J(/+l) 2

Ji/

+r

= 21n|/|[ln|/-l +
I

where

)/"x a
/

5.3.3.

x+l +

/=f (1+x)

d*

^l + x

a;

In

l
\

+C

Ch. V. Basic Clastes of Integrable Functions

224

5.3.4.

x dx
f-

(]ffx\0 x*) 3

'

<

<

and c
therefore neither the first,
Solution. In this case a
nor the second Euler substitution is applicable. But the quadratic
10
x 2 has real roots a
trinomial 7x
therefore we
5,
2, P
use the third Euler substitution:

y'7x\0x = V(x 2) (5 *) = (x2)t.


2

Whence

5
_ +
X
+
5

2)

2/ 2

v2
2

ax _

'

%
\

6/ dt

(1

2 2

Hence
,

where

+ 2)*--(-f + a
.^--fl|

/.

1/"7jc

^
10

jc

+C

Calculate the following integrals with the aid of one of the Euler
substitutions:
dx
5.3.5.

f
J

5.3.6.

J Y

j2

x Vx + 2jc + 4
dx
1

AT

dx
5.3.7.

5.3.8.

j
f

]/"(2x

2 3
)

^
^+.GI?"
yT+x
2

O^ter Methods of Integrating Irrational Expressions

The Euler substitutions often

lead to rather

cumbersome

calcu-

should be applied only when it is difficult


to find another method for calculating a given integral. For calculating many integrals of the form
lations,

therefore

they

R(x,

Vax

+ bx + c) dx,

simpler methods are used.


Integrals of the form
I.

I=f
J

Vax 2 + bx+c

dx

5.4. Methods of Integrating Irrational Expressions

are reduced

by the substitution x

Vat'+K

*J

M N K

where

are

ly

l9

The

+ -^ =

new

*J

225

to the form

Vat* + K

coefficients.

reduced to the integral of a power function,


while the second, being a tabular one, is reduced to a logarithm
(for a
0).
0) or to an arc sine (for a
0,
first

integral

is

K>

<

>

Integrals of the form

II.

P-

Vax 2 + bx + c

P m (*) is a
reduction formula:

where

=v

:?Tl+ ^+c

KaA: 2

polynomial

P m _ 1 (x) is
constant number.

where

dx,

of degree

m, are calculated by the

(x)^Hw/([-7

polynomial

degree

of

z
V ax

1,

=-,

bx + c
+r

and

is

(i)

some

The coefficients of the polynomial P m (x) and the constant


number K are determined by the method of undetermined coeffici^.

ents.

Integrals of the form

III.

dx
1

(xai)"

Vax 1 -\-bx-\~c

are reduced to the preceding type by the substitution

x-a = T
1

For trigonometric and hyperbolic substitutions see 5.7.

IV.

5.4.1.

=f

Solution.

4a:

Make

+ 4*

the substitution
/

Hence,

1
4

CiLh5)dt_

2#+l = /,

dx

whence

-^dt.

+ 14ln|/ + ^
^-4
./-tj

-4| + C.

Returning to x, we get
I

=ly^ + 4x3 + ^\n\2x+l +

r 4xi

+ 4x3\ + C.

Ch. V. Basic Classes of Integrable

226

5X

/=

5.4.2.

H*.

^x + 2x+5
2

/== f

5.4.3.

+-

F uncttons

f-x-zjL^dx.

+2x+2
Here P m (x) = x 3 x 1. Hence,
P m - (x) = Ax + Bx + D.
2

Solution.

We

seek the integral in the form


/

A x> + Bx + D) Vx 2 + 2x + 2 + K

Differentiating this equality,

/'

x2

f
j

-= dx

y * 2 + 2* +

we obtain

+ +2
2x

= (2 Ax + B) Vr x* + 2x + 2 + (Ax + Bx + D)
2

x' ~\~

2x ~

4
2

+ Vx'

Reduce to
x*

common denominator and


2

Equating the coefficients at equal powers


system of equations:

of x,

+ D) (* -f 1) + K.
we

get the following

2A + A=1,
B + 4A + B + A=0;
2B + 4A + D + B = l\
2B + D + K =
Solving the system,

+ 2x+2

equate the numerators

x = (2Ax + B) (x + 2*+ 2) + (/l* +


1

1.

we obtain

Thus,
dx
*

+ 2x + 2

where
1

Vx 2 + 2x+2

5.4.4.

/= JjAx

J
2

1)2+1

-r

/-r

4x + 3dA:.

Solution. Transform the integral to the form


4
/=[ Jf- *^ dx = (Ax + B)V4x -4x + 3 + K
2

^
A

Methods

5.4.

of Integrating Irrational Expressions

Applying the method

undetermined

of

4f

V2

]/4x

9' 3 -3' 2
f

5.4.5.

j/4j^

1 -|-

4x+3) + C.

dx.

(*+4)dx
(f

r
1

J (a:

Solution.

In (2x

/=

get

rf

K3x 2 2x+l

5.4.6.
5.4.7.

4x+3 + y

+2

we

coefficients,

V(2x-\\> + 2

J
2

227

1)

(a:+2) 2

Represent the given integral as follows:

Expand the

+ jc+1

fraction

?
\X

1 )

x+4
(x

l)(*

x+4

(x-{-4)dx

(x-l)(x + 2) 2 j7>

T\X

-\-

+ 2)

1)

dx

(a:+2) 2

yx + x+{
2

into partial fractions

9 2
Z)
.

x\

(at

(x

+ 2)

+2

Find the coefficients

Hence,
1

9(x-\)

3(x+2) 2
dx

(x

y + x+
xt

1)

dx

9 (a:+2)J

dx

f
J

(x+2) 2 |A^-f-x-fl

"

5_

The

first

integral

is

leave the solution to the reader.

5.4.8.

3
f*

5.4.9.

f
J

- 6**+H*- 6 ^

W+4x+3

3^ + 5^-7,+ 9 Wt

|^2a; 2

5a:

+ 2) /x + x-H
2

calculated by the substitution

second and the third by the substitution x

We

dx

f
J

+7

+2=

-j-'

==

the

Ch. V. Basic Classes of Integrable Functions

228

xdx

5.4.11

I (x 2 3x + 2) Vx 2 4* + 3
dx

5.4.12.

(^+l)

l+3x +
x Vf-

bers,

integral

xm

;c

of a Binomial Differential

5.5. Integration

The

"

/^ + 37+2
l)dx

5.4.13.

(a

+ bx n y dx,

where m,

n,

p are rational num-

expressed through elementary functions only in the follow-

is

ing three cases:

Case I. p is an integer. Then, if p > 0, the integrand is expanded


by the formula of the Newton binomial; but if p < 0, then we
put x=t k where k is the common denominator of the fractions m
and n.
y

Case

II.

an

is

We

integer.

put

+ bx n =

a
,

where a

is

the denominator of the fraction p.

Case
is

^-^- + p

III.

We

put a

+ bx n = tx n

where a

the denominator of the fraction p.


5.5.1. /

j/x(2+j/"x) 2 dx.

we have Case

I.

Solution.

an integer.

is

*3"

(jc

r
J

\2+x-V
y dx.

-(
3

2 \

= J*

5.5.3.

Here

/?

= 2,

i.e.

G+*

C-/T+V*
*
\

m = j2

integer; hence,

= j\

-1

dx.

dx.

+ xT ) T

J_

= -^
1

d.*:.

m+l = (-4+0 =
1,
{

3"

integer.

an

+ 4xT +4) dx = J (x T + 4x T +4xT ) dx =


- 24 ii
3

5.5.2.

Solution.

Here

i.e.

an

5.5. Integration of a Binomial Differential

We

have Case

make

Let us

II.

l+x* =f

the substitution

229

dx = 2tdt.

Hence,

=6

5.5.4. /

5.5.5.

=
[

2
t

dt

J*~(2 +

JK

+x

x b {\

= 2t + C = 2(1 +xT ) T + C.
3

T ) Tdx.
)~ dx.

5.5.6.

= jV

11

(\

+ x*)~dx.

+ = also
= -j a fraction,
=
y
---3
an integer, i.e. we have
+p =
a fraction, but
y=
=
Hence
xH
Case III. We put
+ x*
tdt
dx
x

2(f
Solution. Here p

is

is

(t

l)

I)

Substituting these expressions into the integral,

=
Returning to

5.5.7.

x,

we

get

r~
jj-

dx.

dx.

5.5.8.

5.5.9.

j-

J*

(!+**)

dx.

we obtain

_^ ^_i)M/=-^+|-4+c.
(

Ch. V

230

Basic Classes of Inlegrable Functions

\Vx Y^ + V x

5.5.11.

dx

5.6. Integration of Trigonometric and Hyperbolic


Functions

1.

Integrals of the form


I

and n are rational


the binomial differential

where
of

= ^ s'm m xcos n xdx,


numbers,
n-

/=$/*(l /*)

are

reduced to the

integral

dt

= smx

are, therefore, integrated in elementary


following three cases:

and

functions only in the

odd {j-^- an integer^,

(1)

(2)

(3)

m+n

is

odd

is

is

(^rp

an integer^,

even ^^y-^

+ ^p

an integer^.

an odd number, the substitution sin#=/ is applied.


an odd number, the substitution cosa;=/ is applied.
If
the sum m + n is an even number, use the substitution
tan#=/ (or cot^=/).
is

If

If

is

In particular, this kind of substitution


of the

is

convenient

for

integrals

form

^tan n xdx (or ^cot n xdx^>


is a positive integer. But the last substitution is inconveboth m and n are positive numbers. If m and n are nonnegative even numbers, then it appears more convenient to use the
method of reducing the power with the aid of trigonometric trans-

where n
nient

if

formations:

cos 2
or sin x cos x

5.6.1.

1=

x=y(l + cos 2x)

=y sin 2x.
s

f , iilli- dx.
J j/cos**

sin 2

A;

= y (1

cos2jc)

5.6. Integration of

Here m = 3 is an
which gives

Solution.
sin

xdx =

Trigonom. and Hyperbolic Functus

dt,

=3

j/cos a:
(

odd

y cos

We

number.

jc

+C

put

231

cosa:

/,

5.6.2. /

-^d*.
=fJ sin
x

5.6.3. /

sin 4

x cos 6

a: c(a:.

Solution. Here both


and n are positive even numbers.
use the method of reducing the power:
/

=~

(2 sin

The second

cos

a:

of the

a:)

cos 2

xdx =

obtained

sin 4 2jc(1

integrals

is

+ cos 2x) dx =

Let us

lx

+/

calculated by the substitu-

tion:

2x =t, cos

sin

/,

= gg Jsin

We

2a:

cos 2x dx

again apply to the

first

2a:

t*

dx = -^dt,

dt

==^ + C ==^sm* 2x + Q.

integral the

method

of

reducing the

power:
Ii

= 52 sin4 2x dx = y J cos 4a:) dx=


(

J_ x y sin 4#j
+26 J
128
:

And

so,

256

5.6.4. /

^
256

A*-i sin4 *+^ sin8 *+ c

0111
sin

^" + ^
^ 2048 sin
+
" 320 sin
4a:

8a:

2a:

+ 0.

2
r sin x
g-dA:.
\
6
J cos *

Solution. Here both


negative. Therefore,

tan a:=

Hence,

8a:)c(a:

finally,

/__L *^

is

+ cos

and n are even numbers, but one

we put
/;

^ = +

cos 2 a;

2
;

T~ti==dt.
cos 2 a;

of

them

Ch. V. Basic Classes of Integrable Functions

232

5.6.5.

^dx.

/=

2
[
J sin x

Solution. Here we can put cotx=t, but


by expansion:

sin 2

2
J Vsin x

= cot*

2a;

simpler to integrate

is

it

-f

j" (1

cos2x)dx =
3x\
~
r + -j\
+ C.
sin 2x

cotx +
.

5.6.6. /

5.6.7. /

cos 4

we

jj/sin 11 x cos

a:

their

(~ y

y)

and

~y = 4

sum

is

negative

are

an even number,

there-

put
dx

tan x=t;

11
4
J cos * j/tan *

j//

= dt.

11

(1

+ 4 tan

8 tan

5.6.8.

a:'

Solution. Here both exponents

fore

d*

-J

numbers and

Find the integrals

of

tanx and cot

a:

x)

C.

^/tan^Sc

x.

Solution.
\

tanxdx=

J
\

cot x dx

\
<J

sin

/=$ ian xdx.


Solution. We put tanx=tf,
/5
-'
=

=I

In

cos x

-)-

C;

AT

dx =

In

sin

at

+ C.

5.6.9.

dx =

J COS

TT7-.

K'

-jr

x = arctan?; dx

+ '-TTT.)*

tan 6 a:

We

get

tan 4 x

+ y tan

x+

In

cos x

+ C.

5.6. Integration of Trigonom. and Hyperbolic Fund's

/=

5.6.10. (a)

5.6.11. /

^cotxdx;

tan 3 xdx.

= C2^fdjc.
sin *
J

Solution. Here sin

an odd power. Let us put

raised to

is

a;

cosa:

We

(b) /

233

sin xdx =

dt.

obtain an integral of a rational function.


/

-dt.

Here,

it

methods

is

simpler

integrate by parts

to

than to use the general

of integration of rational functions (cf.

Problem 4.4.1

(b)).

Let us put

Then
du =

2(1
1

3t 2 dt\

2
)

Hence,
/

t*

2(1

dt

2
)

t*

"2(1

cos x

5.6.12.
II.

/=

dt:

_i
l

1 t*

2 J

3,3.
+ ln

2(1 P)

\t

+ cos *
+ T ln
COS X
1

-cosx

+c.

dx.
f
J COS*

Integrals of the form

i?(sin.x:,

cos x) dx where

/? is

a rational

function of sin x and cos x are transformed into integrals of a rational


function by the substitution:

tan l^-jj

This

is

=t

( ji

<x<

In this case

so-called universal substitution.


sin x

2t

IT/*'

= 2arc tanf;

cos x1

dx

n).

t
/

'

2dt

i+t 2

'

234

Ch. V. Basic Classes of Integrable Functions

Sometimes instead

make

tageous to

of the substitution

the substitution cot-|-

tany = /

is

it

<x<

(0

more advan-

2n).

Universal substitution often leads to very cumbersome calculations.


Indicated below are the cases when the aim can be achieved with
the aid of simpler substitutions:
(a) if the equality

sin

R(s\nx,
is

then

satisfied,

cosx
(b)

form

cosx)

x=

x,

effect

is

cosx)

x,

by substituting tanx

gained

t.

latter case

sin cos x) = R (sin

then a better

fulfilled,

The

cos x)== R(s\nx

the equality

if

or cot

cos x)

to the former equality,

R(
is

more advantageous to apply the substitution


and s'mx = t to the latter;

is

it

= R(s'mx

cosjc)

a;,

or

example,

encountered, for

is

in

integrals

of

the

(tanx)dx.

5.6.13.

/=

dx

/0
J sin x (2+ cos
.

Let us put

Solution.

r
sin x)
x2sj^

tan~ = t;

then we have

2dt

Expand

into simple fractions


l

t(t 3)(t\)

Find the

t\

coefficients

=b

B =ih

D =-

Hence
J

Jt_

3 J

d
t

+
,

_5_

3 J

dt

t-3

dt

= -iln|/| + |ln|/ 3 1 ln| + C =


/

T ln

tan
|

T + |-ln|tan| 3| In

tan-

X
2

+ C.

Integration of Trigonom. and Hyperbolic Funcfs

5.6.

235

dx

5.6.14. /:

5+ sin x + 3 cos x
dx

5.6.15. /:
sin

(2

a;

1)

cos 2

Solution.

expression

the

in

If

sin

cos 2 x

(2

we

substitute

1)

sin x for sinx, then the fraction will change its sign. Hence, we
cosa;; dt
sinxdx. This
take advantage of the substitution /

gives
dl

2/2_

'

1}

Since

2/ (1 2/
)(1 2<
2

(2

(1 2 )(\ 2t 2 )'

(1

<

2/ 2

then
In

1-/1^2

+ VT COS
1-^2
i

JC

COS X

+ COS X
1+^2
+ ln

cos #
I

tan

Y^2

In

2
II

""T

= -=ln
5.6.16. /

+c=

dt

dt

1 2/ 2

J sin #:+ cos #

dx.

Solution. Since the integrand does not change sign


their signs, we take advantage of
tution

and cos x do change

= tan*;

dt

= COSdx X
2

when sin*
the substi-

'

"

Hence,
7r

C tan 2 X cos 4 x
J

Expand

C sin 2 # cos #

dx

(tanx+l)

cos 2 *

dt

2
2
(/o_iw/2_
J (*+l)(/ +l)

into partial fractions

A
t+\

/*

(/+l)(/a

21)

Bt
t

+D

Et

+F

+\ M/ 2 +l) 2

Find the coefficients

B=

D--

12

'

F=-
2

Hence,

=11 '+1

lt 2 +\

dt+

<
2

Ch. V. Basic Classes of fntegrable Functions

236

In

-JL-4
^ +

^2

^+
1

sin

In

si

a;

2 tan ^ + 3
/= f sin
t
+2 cos

5.6.17.

C=

a:

4- cos

-|-

cos

a:

Solution. Dividing the numerator and denominator by cos 2


dx
substituting tanx=/;
dt. we obtain
2

cos

^_

2tan*+3

r
'

2
J sin

x+ 2 cos
=

jc

and

AT

tan x

(2

+ 3)

tan 2

a:

+ C.

x(smx + cos

cos 2 x

x+2

i=
=
j|J|^ = ln(^ + 2) + -^arctan T + C

= In (tan * + 2) + -2= arc tan tan *


2

/= [j^-dx.
J + sin x

5.6.18.

Solution. This integral, of course, can be evaluated with the aid

tany = /,

of the universal substitution

desired result by resorting


integrand:
sin#(l

sin x

1-J-sin*

(1 -j-

sin

a:)

sin x)
sin x)

sin

sin

it

easier to get the

is

following transformation

the

to

but

of the

x)

cos 2 x

(1

sin'2

sin x

COS 2 X

sin x

COS 2 X

COS 2 X

-tan 2 x.

Whence
/

5.6.19.

S1 " *
C
9
2
J COS X

/=

dx

J[ sec

x dx

+ Jf dx =
COS

tan a:

AT

+ # + C.
1

f
4
2
J cos x sin

dx.

a:

the substitution tanx^/ can be applied, but it


simpler to transform the integrand. Replacing, in the numerator,
unity by the trigonometric identity raised to the second power, we
Solution. Here

is

get
r

Sm

4
2
2
4
C sin #-f- 2 sin x cos x-f cos x

2
cos 2 x) 2
C (sin
4
2
sin'
cos
x
x
J

dx-\-2
[
4
2
J cos x
J cos *
1

~~

h C

f
2
J sin *

= y tan

3
a:

cl.s 4

J
-

ftan 2 x

* sin 2 x

~~

^ + 2 tan cot x =

cos 2

a:

a:

+ 2 tanx cot x-\-C.

Integrating Irrational Functions by Substitutions

5.7.

237

III. Integration of hyperbolic functions. Functions rationally depending on hyperbolic functions are integrated in the same way as

trigonometric functions.
Keep in mind the following basic formulas:

cosh

cosh 2 x

sinh x =
2

tanh ~2=ty then sinh x

= y (cosh 2x

(2

= 2 Artanh/ = ln({-j) (-1< <


*

/=

5.6.20.

,^2

= ^_
1

cosh x

1 );

= y sinh 2x.

sinh x cosh x

2t

= _
]

sinh 2 x

= y (cosh 2x + 1);

If

t2

1);

dx

cosh 2 xck.
J

Solution.
I

5.6.21.

= j y (cosh2*+ 1)

/=

cosh 3 ^d^.
J

Solution. Since cosh x

co$hxdx

= dt. We
/

d*=|-sinh2* + y.x: + C.

= j cosh

is

an odd power, we put sinhjc =

raised to

/;

obtain
2

* cosh

xdx

- j(l +/

= sinh x + y sinh

2
)

a:

d/

= +y + C =
*

+ C.

5.6.22. Find the integrals:


(a)
v
'

f sinh
J

# cosh 2 x dx;

(b)
v
/

x+2^

C-r-r
J sinh

cosh

a;

5.7. Integration of Certain Irrational Functions with the


Aid of Trigonometric or Hyperbolic Substitutions

Integration of functions rationally depending on x and \^ax 2


bx
c
can be reduced to finding integrals of one of the following forms:

I.

II.

III.

$/?

(ty P H* + q*)dt-

\R(t, ]fpH*
J'/?(f,

q*) dt;

dt

238

Ch. V. Basic Classes of fntegrable

where

= x + ^\

ax 2

+ bx + c = p q
2 2
t

Fund ions
out a perfect

(singling

square).
Integrals

of the forms I to III can be reduced to integrals of


expressions rational with respect to sine or cosine (ordinary or hyperbolic) by means of the following substitutions:
I.

= tan 2

t-=

or

sinh2.

P
II.

= sec2

or

= cosh 2.

p
III.

= sin2

or

= tanhz.

P
dx

I=[-r

5.7.1.

Solution.

+ 2x + x = 4 + (x+ 1)
2

/_
We

2
.

integral

of

x+l=t. Then

put

us

- j" (4+/

1 7(1+2^+^)3

have obtained an

Let

2)3

the form

I.

Let us introduce

the substitution:
*

We

= 2tan2; d*=-^-; /4+7~ = 2/1 + tan


2

cos z

get

cos zdz

=
t

tan z

+
y5 + 2* +
x

5.7.2.

/=f
J (*+l)

Solution. x

Let us put

dx

-C.
jc

V*a+2*+2

+ 2x + 2 = (x+l) + L
2

#+1=/;

then
dt

Again we have an
tf

= sinhz.

of the

integral

form

I.

Make

the substitution

Then
dt

= cosh z dz\ Vt +\ = Kl+sinh


2

= cosh z.

5.7

Integrating Irrational Functions by Substitutions

239

Hence,
,

cosh z dz
r

dz

= C0th Z + C^ =
,

U92
J sinh z

J sinh- z cosh z

^+

VT+Zi/-

V l+sinh2 2

2jc

+2

= ^x Vx^-ldx.
2

5.7.3.

5.7.4.

/-j^d*.

5.7.5.

= $K(*

l)

d*.

Perform the substitution:

Solution.

= cosh?;

dx

= sinh/d^.

Hence
/

$j/"(cQsh*/

= -i j cosh

l)

2/ df

= y j(cosh+

sinh

dt=\

j cosh

l)df

-J-

sinh 4

2/ df

sinh 2/

-J-

J
^

d/

-J-

= sinh At \ sinh 2t + 1- + C.
1

Let us return to x:

= Arcosh x = In (x + Vx*
sinh 2t = 2 sinh cosh = 2x V'x l;
sinh 4/ = 2 sinh 2* cosh 2t = 4* Vx \ (2x
t

);

Hence

5.7.6.

/=f
J

Solution.

*
V x)Vx-x*
_

(1+

We make
x

the substitution:

= sin

2
/;

dx

= 2sin*cos/df

1).

^>

Ch. V. Basic Classes of Integrable Functions

240

and
j

get
r
f

2 sin

(1

+ sinO

cos

Vsin 2

=2

dt
^

sin

2 dt

+ sin

[tzli
dt =2 tan
2
cos

^- + C

COS

'

7 i =+ c5.7.7.
r

~>

5 ,.s.

= [VZ 2x

Solution.

1)

+c.

dx.

2*+5)

5.5. Integration
5.8.1.

CLi

,=j
(x 2

2(l

of Other Transcendental Functions

= Ji2djc.
We

integrate by parts, putting

= In x\

dv

= ^;
x
2

dx

a:

a:

5.8.2.

_
-

In

C dx

T + J"^-

a:

T + ^
1

/=fi^.

Solution. Let us put:

e*dx

Apply the reduction formula

'

In

(i

(see

= dt. We

get:

+<*)*
Problem

4.4.1):

= YiJ^T) + T arctan + C = 2(1$^ + T arctan + c

5.8.4.

= $e-*ln(e*+l)dx:.

5.8. Integration of Other Transcendental Functions

Solution.

We

integrate by parts:

= \n(ex +

1);

du= +f *e xY dx;
t

= - e -ln(l+^) +
= e - *

JT

^=-

In (1

dv

= e~ x dx;

e~ x

\
*

'

-Mn(l+^) +

-^i^^ =

+ex + x ln(l + e*) + C.


)

~aarc tan *

5.8.5.

dx.

= l-

(1

g g g

+*

2
)

Pa: arc tan

x dx

jn~+T2

Solution.

241

Integrating by parts,

= arc tan x;

= T+x
|/

arc tan x

we
dv

get

In (x

xdx

+**

'

+ |/* + l) + C.
a

Ch. V. Basic Classes of Integrable Functions

242

Methods of Integration
of Basic Forms of Integrals)

5.9.

(List
No.

Method

Integral

^[(pMlfW dx

Substitution

W M dx

(p

(x)

Integration by parts

<p'

integration

of

W 9' W d*=

method

This

integrals

(x)

is

the

of

(x)

(p

form

(x) /' (x)

dx.

example,

to

applied, for

polynomial, and

is

/ (x)

(x) f (x)
f (x)

is

where

dx>

one of the

following functions:

sina#;

e^'y cos ax;

In x\

arc tan x\ arc sin x, etc.

and also to integrals of products of an exponential function by cosine or sine.

(n)
<f

W d*

Reduced

to
integration of the product
(x)q*(x) by the formula for multiple integration by parts
{n)

x)

(p

(w)

f'(x)
...

(x)

q>

dx

"- 2
>

= f (x)
(x)

- 1)

q><

+ /" (x)

(p

pn

(x)

>

w+

fi)(x)q>(x)dx
J

Applying the formula for multiple integraby parts (see above), we get

dx,

tion

where p n

(x)

is

polyno-

mial of degree n.

e**p n

(x)

dx

=
~ ~

a2

+
5

"- 3 (*)...

(-_l)i.-i/(i.-D X )q,
(

+ (-l)
4

(x)

Mx + N
+ px+q

2
J x

4q <

X>

Substitution

a3
'<

'

P"

j?

+c

Methods of Integration (List

5.9.

No.

Method

Integral

dx

a.

of Basic Integrals)

of

Reduction formula

integration

used

is

x
n

fPW
ttH

P(x)
where 7r^-J

is

proper rational

sum

frac-

tion

Integrand

= (x x
-x )" ...

(x)

1)

~(2n 2)(x 2

P(x)

Q(x)

(xx x

+ l)"-

(x 2

+ px +
+ ...

(x-x 2

(*-* 2

X
'

where

[cx+d)
R is a

function of

10

Xl

(*-* 2 )'

<7

(x*

+ px + q)*

{x *

+ px + q) k

its

dx

>

)l~T~

'

Reduced to the integral


by the substitution

of a rational

frac-

tion

rational

arguments.

Mx+W
+ ^+c

f j
I
2
J |/a^

of a

At

2
)

tion

~~n

the form

in

A2

(x x t

tion of its arguments.

Reduced to the integral of a rational fracby the substitution x t k where k is a


rational func- common denominator of the fractions

1")^,

is

2/1

(x-

a:

where

2n
1

fractions

*a+p* +

expressed

is

of partial

243

cx-\~d

By

dx
is

the substitution x-\--^

reduced to a

Mx+N
f

sum

of

two

*-mS

the

integral

integrals:

m
t

+
2

-f/n

The first integral is reduced to the integral


of a power function and the second one is a
tabular integral.

Qh. V. Basic Classes of Integrable Functions

244

No.

Method

Integral

(x,

\^ax 2 -\-bx-\-c) dx

where

function of x and
l^ax'1

-|-

bx

integration

Reduced to an integral of rational fraction


by the Euler substitutions:

rat ion a

is

ot

Vax* + bx + c = t x V~a

{a

+ bx + c=tx

(c

Vax

+c

V ax

where x

V^~c

-\~bx+c=t (xx x
the root

is

of

(Aac

>
>

0),

0),

<

0).

the trinomial ax 2 -\-

+bx + c.

The indicated integral can also be evaluated by the trigonometric substitutions:

- 4ac
sin

cos

2a

+2a=<

4ac

b2

(a

2a

<

0,

\ac

<

0)

- 4ac

b2

sec

2a

+ Ta

Aac

cosec

2a

(a

>

Aac

4ac--b

0,

<

0)

tan

2a

4ac--b 2
cot

(a

2a

>

0,

4ac- -6 2

Pn

12

}/"ax 2

where P

+ +c

(*)

dx

mial of degree n.

0)

Write the equality


t

toc
is

>

polyno-

P
j/~a# 2

(x)

dx

+ +c
6#

dx
|/~ax 2 -j-kx;-j-c

where Q n -i(x) is a polynomial of degree


1. Differentiating both parts of this equan
2
lity and multiplying by Vax -\-bx-\-c we

get the identity

Pn

(x)

i (*)

+ bx + +
W(2(w + 6) + A

(<u 2

+ |q b .

c)

which gives a system of n-\-\ linear equations for determining the coefficients of the
polynomial Q n - 1 (x) and factor k.

Methods

5.9.

No.

of Integration (List of Basic Integrals)

Method

Integral

And

245

of integration

the integral

dx

V ax' + bx + c
2

taken by the method considered

is

(M
dx

13
(x

14

Xi) m

xm

where m,

(a

Vax 2 + bx + c

+ bx n )Pdx,
p are rational

n,

numbers (an
binomial

integral of a

= 0;

in

No. 10

N=l).

This integral is reduced to the above-considered integral by the substitution

This integral is expressed through elementary functions only if one of the following
conditions is fulfilled:

if

(1)

differential).

an

is

.m+

nteger,

if

an integer,

(3)

if

is

(2)

m+\

-p

is

an integer.

1st case
(a)

if

positive

is

integer,

remove the

brackets (a
bx n )P according to the Newton
binomial and calculate the integrals of powers;

negative

integer, then the


the common
is
denominator of the fractions
and n leads
to the integral of a rational fraction;
(b)

if

is

substitution

k,

where k

2nd case
*

an integer, then

is

if

n
tion a-\-bx n

the

substitu-

applied, where
denominator of the fraction p;
t

is

is

the

3rd case
if

~~~ JrP

ls

an integer, then the substi-

is

tution a-\-bx n
x n t k is applied, where k
the denominator of the fraction p.
15

(sin

cos x) dx

Universal substitution tan -?r=t.


If

R( s'mx

cosx)=

then the substitution cosa:


If

(sin

R(slnx
/

cosx)= R

then the substitution smx=^t


If
cos x)
R( s\nx,
R
then the substitution tanx
/

is

cos

x),

applied.

(sin

cos

a:),

applied.
cos*),
(sin x
is applied.

is

246

Ch. V. Basic Classes of Iniegrable Functions

No.

Method

Integral

16

dx

(sinh x, cosh x)

The substitution tanh-^this case

sinhx

17

sin

bxdx

^ sin ax cos

bxdx

^ sin

ax

-;

sin

-i-

d* =1

In

__ /2

m
^ sin x cos" x dx,
inte-

b)

a; cos

fr)

x-j-cos (a-\-b) x]

b)

x-j-sin

[cos (a

=
[sin (a

If
is an odd positive
the substitution cosa:
/.
If n is an odd positive
substitution sin#
/.

If

m-j-n

(a+b)

x)

number, then apply

gers.

(a-\-b) x]

ax cos bx

=-^-

and n are

cosh*^^;

[cos (a

-^-

cos ax cos 6*

used.

is

sin ax sin bx

where

Transform the product of trigonometric


functions into a sum or difference, using one
of the following formulas:

^ cos ax cos bx dx

18

Integration

of

number, apply the

an even negative number, apply

is

the substitution tan#


/.
If
and n are even non-negative numbers,
use the formulas

m
.

sin 2

Reduce

^ sin^ x cos^

19

to

differential
(0

and

bers.

20

<
#

<

cos 2x
;

the

cos 2 x

integral

1+cos2x

of

the

binomial

by the substitution sin# = /

ji/2),

rational

sinP xcos?

num-

xdx=^ tP(\ t*)**-

dt

J
(see

No.

14).

Transform

into

an

integral

function by the substitution e ax

of a

rational

Chapter

6
THE DEFINITE INTEGRAL

6.1. Statement of the Problem.


The Lower and Upper Integral Sums

Let a function f (x) be defined in the closed interval


following is called the integral sum:

[a, b].

The

i-

where a

=x <

hXi

< x n _ < xn = b,
= 0,
li[x h x i+l

<x <
2

= x i+1

(J

1,

/2

1).

n- i

The sum S

2=

M,.

is

Aa:,-

called the upper (integral) sum,

and

/
A2

sn

=
i

niikXi
^
=

is

called

the

lower

M =

where

sum,

(integral)

= supf(x)
The

[m^mlfix)] iovx [x h x i+1 ].


definite integral of the function f {x)

on the interval

[a,

fr]

is

the limit of the integral sums

If

val

/ (*)

= lim

f (/) Aa;,-

this limit exists, the function


[a, b].

6.1.1.

Any continuous

function

when max

Aa:,|

is
is

> 0.

called integrable

on the

inter-

integrable.

For the integral


JX

find the

sin

xdx

upper and lower integral sums corresponding to the division

of the closed

interval [0, n]

into 3

and 6 equal subintervals.

Ch. VI. The Definite Integral

248

Divide the closed interval

Solution.

[0, n]

into 3 equal parts by

the points:

_
n

%o

_
~3~

n
^

2ji

-^2

xs

*^

The function sin* increases monotonically on the


and

therefore

= sin^- = -?y-.
*

[iT' T"j

The

m =s

= sin-^-=l. On

we

interval

this

for

value

least

have

an d the

the interval

Jtj

interval

0,

yj

= sin0 = 0, M =
on the interval

function

the

of

ra o

value

greatest

is

M =
x

the function sin* decreases

monotonically and therefore


2ji
V3
n
AA
= -^y= sin -y
M
m = sin n = 0,
.

Since

Ax k

all

are equal to -y

=Z

s3

Ax,

+ o) = ^p- ~0.907,

= iL (o +

k=0

When

subdividing the closed interval

ill
by the
==

5ji

points

x6

jt,

we

JX

rv
y

*t=-g-,

find

ji

1
'

m = sm T

5ji
m = sin-^"~
6
.

ji

x3

into 6 equal intervals


2jl

Jl

= -y

^^T*

>

= sin-^- =

== o,

== sin

1,

7W 3

= sin-y =

1,

7W 4

= sin -y =

'

M = sin -y =

Jl

m = sin T "~

= ^->

7W

m = sm TT ~~

m = sin

k]

by analogy:

m = 0,
.

x2

[0,
Jl

2ji

M = sin =
5

/3
2
1
'

Xb

Sums

6.1. The Lower and Upper Integral

249

For this division we obtain

s,=}K + i+... +m )=^-(l+K3)~ 1.43,


S= (Af + M +
+ M = (3 + /3 2.48.
5

5)

As would be expected, the inequalities


ji

s3

^ ^

^ sin

s6

x dx

^S ^S
6

hold true (the exact value of the integral


6.1.2.

8>0

At what

equal to

is

2).

does the relation


n-i

sin

xdx

2 sm %k & x < 0.001


k

max hXi

follow from the inequality

<

8.

<

<

S, then for the required inequality to


Solution. Since s n
/
hold true it is sufficient that the upper and the lower integral sums
differ by less than 0.001:

0<S-s<

0.001.

But
n-i

n-i

S n -s n

=
i

2
=

(Mim,) A*,

<

where
and m are the greatest and the
f
tion sinx on the interval [x h x i+1 ] (i = 0,

(M,
2
=
least

for simplicity that

the point ^-

is

/n f ),

values of the func-

1).

Assuming

of the

points of

1,

chosen as one

division and taking advantage of monotonicity of the function s'mx

on the intervals

0, -^-j

and

nj

we obtain

X(M m = 2 (sin-| sin0) =


f

f)

Consequently, the required


i.e.

<

not

is

satisfied

if

26

<

0.001,

0.0005.

Show that the Dirichlet function [see Problem


integrable in the interval [0, 1].

6.1.3.
is

inequality

2.

1.14.4 (b)]

In dividing the closed interval [0, 1] into a fixed numwe must take into consideration, in particular, two
possible cases: (1) all points
are rational; (2) all points
are
Solution.

ber of parts

250

Ch. VI. The Definite Integral

irrational. In the first case the integral sum is equal to unity, in


the second to zero. Hence, no matter how we reduce the maximum
length of subintervals, we always get integral sums equal to unity
and integral sums equal to zero. Therefore, the limit of integral
sums is non-existent, which means that the Dirichlet function is
not integrable on the interval [0, 1].

6.1.4. Find the distance covered by a body in a free


the time interval from t
b sec.
a sec to t

fall

within

body moves

in a free fall with constant acceleration g


Consequently, the velocity at the instant t
is equal to the velocity increment within the time interval from
to /, i.e. v(t)
kv. For a short time period A^ the velocity increment is approximately equal to the acceleration at the instant t
multiplied by A/. But in our case acceleration is constant, therefore Au = g"A/, and hence, v(t)
/.
t
gt since At
Let us subdivide the time interval from t=a to t = b into n
equal parts; then the duration A/ of each subinterval will be equal

Solution.

and

initial velocity

= 0.

to

A/^^1n^. We

each subinterval
assume that during
to

body moves uniformly with


beginning of this interval,

velocity

v n -i

time the

= g[<* + (n

The

by the body during the

find the distance covered

Vi

of

velocity at the

its

i.e.

subinterval:

equal to

=ga,

Whence we

covered by the body

distance

entire

ith
is

approximately equal to

^^s n = ^(v + v +
ba
-7T8 na+
1

+v n _ =
1)

\-2

1
n.

\-

= (ba)g
z

With n

1)

'

[a

anin
+
2~~"J
,

1)1

increasing the distance covered can be evaluated

more accu-

Sums

6.1. The Lower and Upper Integral

The exact

rately.

s=

Urn s n

value of

found as the limit

is

[a+y (b a)

lim^g(ba)

= g(b-a)

(l

251

sn

is

oo:

fl+ l(ft_fl)]

= !( & 2__ a2)

an integral sum
n-

the distance

an

is

=o

integral:
b

=^

vdt

= ^gtdt = j- (b a
2

6.1.5.

n>

-^-)J

Since s

as

2
)

Proceeding from the definition, compute the integral


i

Solution.

By

definition,

xdx =

\\m
t

2
=

S/Ax;

max

as

where

= * <*i<--. <*=!.
A#; = a^^+j
1.

Subdivide the closed interval

points

x.

= -L(i = 0,

The length
as n

>

of

1,

2,

Ax,

0,

UK
x^-.

1]

[0,

into

n equal parts by the

...n).

each subinterval

is

equal to

Ax

=^

and

-i-

oo.

Let us take the right-hand end-points of the subintervals as the


points

lt

Form an

g.

= *. +1 ==^(/ = 0,

integral

',=s,="f '1.1=1
i

As az oo

1,

1).

sum:

(1+2+

... +(l)

=o
the limit of this

sum

is

lhn

...

2n

equal to

~2-

= iiii>.

Ch. VI. The Definite Integral

252

Hence,
i

x dx =

Using

2.

we

example,

this

show that

will

for

any other choice

of points l f the limit of the integral sum will be the same.


Take, for instance, the mid-points of the subintervals as
f

+T

'

= -}T

==0

(t

o
c
+
= S-sr T=2^L 1+3+5
i

sum

integral

^'2t'+l

n ~V-

1
*

Form an

|~

---

ixl
i/o
+(2n_1)

==

2n 2

4^

= ^l

Hence
lim / n
n

-+ cc

= i-*
compute the

6.1.6. Proceeding from the definition,

integral:

\x m dx

(m=^ 1, 0<a<b).

Solution.

In this example the following points can be conveniently

chosen as points of division:

= a\

Xi=a(-\
a

x^ai^A
a J

JL

_L

,...,*

They form

a geometric progression with the

The length

of the

ith subinterval

Sx aq
Therefore the

= aq n "

(q

Now
I,-:

,.

let

i+1

common

equal to

is

aqt^aq

(ql).

length of the subintervals equals


n-l
J_

l)

=a
lim^

a<7''

+l

and

tends

to

max Ax
zero

with

l.

us choose the right-hand

= x f+1 =

ratio

maximum

increasing n, since

as

=a

(i=0,

1,

2,

end-points of the subintervals


....

1).

6.1. The Lower and Upper Integral Sums

Form an
n-

253

integral sum:

n-

= 2 8^*,= 2flY + ^(?-l) =


=
i=0
0 +
=
= i+l( 9 _l) 9 *[l+ 9 *+l+. ..
(m
l)n
+
n
q
= a^{q-\)q- qm+l _ ^(b^-a^)q1!/B

In

l>]

fl

,o

Let us calculate the limit of the


t.

e.

q+

as

integral

sum

as

max Ax

0,

1:

Hm /H^ +1 -a" +1

)H^

Thus,

d* =

am+1

bm+1
7h~T\

)-

from the definition, compute the integral:

6.1.7. Proceeding

iti

Solution. Subdivide the interval [1, 2] into n parts so that the


n) form the geometric propoints of division x t (i'=0, 1, 2,
gression:

* =1; x 1 =q; x 2

whence q= j/2.
The length of the

2
\

x3

=q

ith subinterval

Ax,
n~

=q

+1

<7'

is

..;

xn

= q n = 2,

equal to

f'^'fa-l),

- oo, i. e. as g
1.
(q 1) >0 as
q
t
us choose the right-hand end-points of the sub intervals
x i+ ^=q i+1
as the points \ h i.e., li

max Ax

and so

Now

let

Form an

i=0

fe

sum:

integral

i=0 9

'

lim
n

/=

lim
n

-* cc

-*

"( 2 "-0 =ln2>

<x>

since

~ In 2
J

as

noo.

2"

Ch. VI. The Definite Integral

254

And

SO,
*

dx

= ln2.

Evaluate the integral

6.1.8.

/=$

J/25

dx

proceeding from

its

geometric meaning.

The curve y=V2hx 2 is the upper half of the circle


x* + y = 25. The portion of the curve corresponding to the variation
to 5 lies in the first quadrant. Hence, we conclude
of x from
that the curvilinear trapezoid bounded by the lines x = 0\ x = 5;
Solution.
2

= V25x

and y

.,

area

its

is

j.

equal to

is

quarter of the circle x 2 -\-y 2

= 25\

and

25ji

Hence,
I

Evaluate

6.1.9.

= ^V25-x

the

integral,

dx

= ^.

proceeding

from

geometric

its

meaning:
/

=$

(4x

l)dx.

6.1.10. Prove that


X

^-arcsin-J
Solution.

The

integral

4
2

J1

(0<x<a).

a:

\
1

la.

,27

dx

expresses the area S 0AMx of the portion of a circle of radius a lying in


the first quadrant (see Fig. 59).
This area equals the sum of the
and the
areas of the triangle

OMx

sector

OAM.
>OMx

Fig. 59

6.1. The Lower and Upper Integral Sums

The area

where sin/

255

of the sector

Hence,
a2

^OAM ~Y arC Sin "J


c

'

and consequently,
I

= ~ya
2

6.1.11. Proceeding

# + 4r arc sin
2

from the geometric meaning

the integral,

of

show that
2ji

(a)

s\n 3

xdx

= 0;

(b)

dx^2

e~*

\e~*

dx.

Solution, (a) The graph of the function y


s\n 3 x is shown in
60. Let us show that the area situated above the x-axis is
equal to that lying below this axis. Indeed, let n^x^.2n, then
sin 3 ^.
x 1 where
sin 3 (n
and sin 3 x
xj
Therefore, the second half of the
Fig.

O^x^rc

x=n +

graph is obtained from the first one


by shifting it to the right by n
and using the symmetry about the
x-axis. Hence,
2jx

^ s'm

xdx= 0.
Fig. 60

6.1.12.

Given the function f(x)

=x

on the

find the lower (s n ) and the upper (S n ) integral


interval by subdividing it into n equal parts.

6.1.13.
integral,

Proceeding from
prove that:

the

geometric

meaning

2jx

(a)

s\n2xdx =

0\

(b)

(2x+\)dx = 6;

0;

(c)

\cos 3 xdx =
3

(d)

interval

sums

jK9=]?dx =

9jt

[2,

for the

of

3],

given

the definite

Ch. VI. The Definite Integral

256

6.1.14. Passing to the limit from the integral sums, compute the
integral

by subdividing the interval


(a)

into equal parts;

(b)

by

choose
(1)

(2)
(3)

[1, 4]:

forming a geometric progression.

points

In

both

cases

as:

left-hand end-points of the subintervals;


right-hand end-points of the subintervals;
mid-points of the subintervals [x h x i+1 ].

6.2. Evaluating Definite Integrals by the


Newton-Leibniz Formula
The following

known

is

Newton-Leibniz formula:

as the

b
b

\f{x)dx=F(x)

= F(b)-F(a),

where F

(x)

is

one

the antideri vatives of the function f(x),

of

F'(x)

= f(x)

i.e.

(a

6.2.1. Evaluate the integral

Solution.

Since

the

function

derivatives of the function

F (x)

(x)

= arc tan x

_^V2

formula we get

f
J

6.2.2.

dx

l+* 2

~~

Compute

the integrals:
ji

is

one

of the anti-

using the Newton-Leibniz


6.2.3.

6.2.

Newton- Leibniz Formula

257

Given the function


x2

<x<

for

=\

x)

for

<x<2.

Evaluate
Solution.
2

(x)dx.

J f
6

12
By

\f(x)dx=^f

12

the additivity property of the integral

(x)

dx+ ^f(x)dx=^ x dx+ \ V~xdx =


2

3
~2

_
~

X*_

2^

6.2.4.

Evaluate the integral


2

= Sl

lx\dx.

Solution. Since
,

we

for

for

0<*<
1

1,

<*<2,

obtain, taking advantage of the additivity property of the integral,

1 x\dx= 5 (1

(x

l)d;c

(1

-*) 2

(*-

l)

=1+1=1
2^2

l#

6.2.5.

Evaluate the integral

where a

< b.

Solution.

If

0^a<6,

then

/ (*)

= 1~- ==

therefore
a

= b a.
= a 6.

If

a<b<0,

Finally,

if

then /(*)

< <

ft,

and

f(x)dx

= b ( a) =

then divide the integral

^ /

Ch. VI. The Definite Integral

258

into

two

integrals:
b

\f{x)dx

= \f{x)dx+\f (x) dx=b {a).


a

The above

three cases

may

be represented by a single formula:

l\-dx

= \b\-\a\.

Note. When evaluating integrals with the aid of the Newton-Leibniz formula attention should be paid to the conditions of its legitimate use. This formula may be applied to compute the definite
integral of a function continuous on the interval [a, b] only when
is
fulfilled in the whole interval [a, b]
the equality F' (x)
f (x)
[F (x) is an antiderivative of the function f(x)]. In particular, the
antiderivative must be a function continuous on the whole interval
[a, b], A discontinuous function used as an antiderivative will lead

wrong

to the

6.2.6.

result.

Find a mistake

in the following evaluation:

V3
dx

arc
rr tan
=
tan
2
1

T-i

-77

( arc

where

Vs

2x

tan

\x

y^r) =

V 3) arc tan 0] = ^

[arctan(

ct

yq^(*=?M).

Solution, The result is a priori wrong: the integral of a function


positive everywhere turns out to be negative. The mistake is due
2x

to

the

that the function

fact

-y arc tan
{

of

the

lim

The

-rr

arc tan

2x

has a

discontinuity

x=l:

kind at the point

first

_ x2

n
= -t\

arc tan
1

urn

correct value of the integral

2x

under consideration

TC

is

equal to

V3
dx

arc tan x

V3

= arc tan ]/"3 arc tan =

-?-

Here the Newton-Leibniz formula

F (x) = arc
F'(x)

tan x

= f(x)

is

is

is

applicable,

continuous on the interval

fulfilled

on the whole

0,

interval.

since

the function

and the equality

6.2. Newton- Leibniz Formula

6.2.7.

Find a mistake

in the following

259

evaluation of the integral:

jt

dx
J

+ 2 sin H

2
2
J cos' x-j- 3 sin x

dx

71

-f

+3 tan

(The integral of a function

arc tan (V

j/"3~

positive

everywhere

= 0.

3tanx)

turns out to be

zero!)

Solution.

The Newton-Leibniz formula


7

since the antiderivative Z (x)

tinuity at the point x

F(x)=

lim

(K 3

arc tan

applicable

tan#) has a discon-

arc tan

([/"

3 tan x)

F=-arc tan(+oo) =
F

lim

(x)

arc tan (K 3 tan

x)

V
The

can be obtained

correct result

dx

__ C

+ 3 sin

2
J cos #

+3

arc tan

/3

also

be

found

arc tan (K3 tan

integration [0,

ji]

a:).

into

oo)='

2V~3

the following way:

7rr=-

tan

arc

1/3

can

sin 2 *

=
It

dx

cot 2 #

in

lim

here,

Indeed,

-S-.

lim

=-pLr

not

is

with

the

For

this

aid

(V 3 cot x)

the

of

1^3

function

F (x)

purpose divide the interval of

two subintervals,

0,

yj and

j~y

jij

and

take into consideration the above-indicated limit values of the function

(x)

as x

y=F0.

nuous function on each

Then the antiderivative becomes

of the subintervals,

a conti-

and the Newton-Leibniz

Ch. VI. The Definite Integral

2-0

formula becomes applicable:

_JT_

arc tan

3 tan

([/

arc tan (|/~3 tan

a:)

a;)

^"3

Compute

6.2.8.

the integral

dx.

Solution,

+ cos

2a:

2 cos 2 x

cos

a:

cos x,

O^Zx^-j

cos X, y < x <

Jl.

Therefore

-j-

cos 2x

dx =

cosxdx +

= sin x 7 + (sin*)
Note.

If

we

ignore

the

cosx)c(a:

^ =(l_0) + (0 ( 1)) = 2.

fact

that cos*

is

and put

f
V
we

get the

wrong

4-co s 2x
2

cos

a;,

result:

cos x dx

= sin x^ = 0.

negative in

ji

6.2.

Newton-Leibniz Formula

261

6.2.9. Evaluate the integral


IOOji

We

Solution.

Kl cos 2xdx.

have

Vl cos2a:= [/~2
Since

sin

a:

has a period

IOOji

sin

jc

|.

then

ji,

IOOji

jj

\cos2xdx=\r 2

sin

a:

rfA:

o
ji

100

J/T

sinxdA;=200|/2

J
o

6.2.10. Evaluate the integrals:

<>'

= }' 7ITW

(b)/

(c)

/=

sin a

ydx;

Ix

'

-3

-2

(d) /

-ji

J ^qry^;
o
_2_
JT

sin

i_

n
1

(g)

Htt^'
3

xdx

JI

T
(j)

7=5

Kcos* cos 3 Jed*;

jt

~
"3

(h)

i+* 8

Ch. VI. The Definite Integral

262

6.3. Estimating an Integral.


The Definite Integral as a Function of

1.

f(x)^q>(x)

If

a^x^b,

for

then
b

J f

Limits

Its

(x)dx<^

q>(x)dx.

In particular,
b

<

\f(x)dx

\\f{x)\dx.
a

a
b

m (ba) ^lf{x)dx<^M (b a)

2.

the greatest value of the funcis the least value, and


tion f(x) on the interval [a, b] (estimation of an integral).
3. If the function f (x) is continuous on the interval [a b], then

where

\f(x)dx=f(&)(ba),

a<l<b

(mean-value theorem).
4.

<p(x),

If

in

the functions f(x) and y(x) are continuous on [a, b]


addition, retains its sign on this interval, then

and

\f{x)y{x)dx = f{l)\y(x)dx,

a<l<b

(generalized mean-value theorem).


a

5.

/ (/) dt = / (*);

-gj-j

/(*)#

= /(*) at

each point x of

continuity of the function f(x).


6.3.1.

Estimate the following integrals:


JT

~3~

(a)

= j K3 +

Solution,
tonically

dx;

(b)

+2

(a)

Since the function

on the

interval

[1,

3],

/ (x)

then

= )/3 + x increases monom = 2, M = /30, ba = 2.


2

6.3.

Estimating an Integral. Integral as a Fund,

Hence, the estimation

of

263

Its Limits

the integral has the form

of
3

2-2
l.

< Jl/T+l?dA:<|/"30-2,

e.

< ^3 + *

(b)

The integrand

f(x)

dx:<2|/"30^

= j

10.95.

decreases on the interval

ji

3"

L"4

since

its

derivative
j,

Hence, the

x cos x

sin x

(x

tan x) cos x ^

value of the function:

least

m-

V3
2ji

its

greatest

value being
2 V'2

M=
Therefore
3 /*3

ji

Jt

2ji

i.

e.

6.3.2.

Estimate the absolute value of the integral


19

sin;

~>

dx.

10

Solution. Since

|sin*|<l,
[-\-x 9

<

for

x>10

10" 8

the inequality

fulfilled.

is

Therefore
19

sin

TT X

<(19 10)10" <


8

10

(the true value of the integral

^ 10

).

10"?

jt
'

Ch. VI. The Definite Integral

264

6.3.3.

Which

of

the two integrals


i

x 3 dx
is

the greater?

As

Solution.

is

Y* >

known,

x 3 for

Y xdx >

6.3.4.

1.

Therefore

x 3 dx.

Prove the inequalities:


i

(a)

0<x<

0<

/l^_ < j\

1+ *

<

[e*

dx<e.

<

Since

Solution, (a)

(b)

<

V +%

0<x<^l,

for

then
i

<

(b)

Since

*M=< [*dx=

(-

<x<

for

exists the inequality

there

< ex < e,
*

then
i

^dx

<

^e*

dx<

edx.

Hence the inequality under consideration holds

true.

6.3.5. Prove the inequality


ji

T
^e-R*tox dx< JL(i_e-)

(R>0).

= nx
si

Solution. Since the function f(x)

Problem

6.3.1

(b)],

then

decreases on

0<x<j

for

/(*)=x >^t] = c

Hence, on this interval

sin

( ji \

sin#>2 x,
sin *

<^ g

therefore
n

^0,

jx

[see
J

Estimating an Integral. Integral as a Fund, of Its Limits

6.3.

265

and
2

~2R

e~ R

6.3.6.

sin

<

*dx

Prove that

the interval

(a,

b),

for

the

dx

ji

2^
-

any functions f (x) and g(x), integrable on


Schwarz-Bunyakovsky inequality takes place:

^f(x)g(x)dx <:
Solution. Consider the function

= [f (x)-kg(x)}>,

F(x)

where k

is

any

number. Since

real

F (x)

then

0,

or
&

X2
J

/;

dxTk

(x)

g (x)dx +

/ (x)

The expression

in the

at

any X

this trinomial
i.

>

0.

is

quad-

follows from the inequality that

It

non-negative. Hence,

is

dx

side of the latter inequality

left

ratic trinomial with respect to X.

non-positive,

/* (x)

J
a

discriminant

its

e.
2

/ (x)

-Jf

g (x) dx
j

Hence

(x)

dx
J

6.3.7.

/~
<j/ ^f*(x)dx

r?

y(x)g(x)dx
which completes the

(x)

d*

<

0.

1
2

^g

(x)dx,

proof.

Estimate the integral from above


i
,

('

sin x

Solution.
sin x
I

l+x'

By

the generalized mean-value theorem

~t2
dx = sin l\
b
J l+*

we have

= sin \ arc tan x = -J-sin6(0<6<

1).

is

Ch. VI. The Definite Integral

266

Since the
sin

<

sin

1.

function

sin

on the interval
an upper estimate of the

increases

a:

Whence we

get

[0,

1]

then

integral:

sin x

-7-

It

is

theorem

possible to get
in the form

A aA
< r sin 0.64.
.

T dx

better

estimation

we apply

if

the

same

sin x

dx

+ Sj J

sin x

dx

= y^|2

(1

cos <
')

cos

0.46.

Proceeding from geometric reasoning, prove that:


f (x) increases and has a concave graph in the
interval [a, b] then
6.3.8.
(a)

if

the function
y

(ba)f(a) <^f(x)dx<(ba)(b)

if

interval

the function
[a,

b]

f (x)

increases

and has

f(a)

+ Hb)

convex graph

in the

then
b

(b-a)
Solution, (a)

f(a)+f(b)

<

f(x)dx<(ba)f(b).

limitation of generality we may assume


the graph of a function means, in particular, that the curve lies below the chord
through the points
A(a,f(a)) and
B(b, f(b)) (see Fig. 61). Therefore the
area of trapezoid aABb is greater than
that of the curvilinear trapezoid bounded above by the graph of the func-

Without

f(x)>0. Concavity

of

tion,

i.

e.

^f ix)dx< SaABb =

(b-a).L^+m,

(b-a)f(a)< \f(x)dx
a

is

obvious.
i

6.3.9.

Estimate the integral ^\fl-\-x*dx using


(a)

6.3.

Estimating an Integral. Integral as a Fund, of Its Limits

267

the mean-value theorem for a definite integral,


of the preceding problem,

(b) the result

+ x* <

(e)

the inequality

(d)

the Schwarz-Bunyakovsky inequality (see Problem 6.3.6).

Solution,

By

(a)

]f

the mean-value theorem

= J \fl+x

dx

= V'T+V,

where

0<<

1.

But
i

< VT+T* < V%

whence

(b)

The function

1< / < K2 1.414.


is concave
f(x) = VY+

on the interval

[0, 1],

since

rw=f^|>o, o<,o.
On

the basis of the preceding problem


i

we

get

< ^\f\T7*dx< -f-l&

(c)

l<I = frl+T dx< j(l

!+-!.=

\ +x*,g(x)
Bunyakovsky inequality

(d)

Put f(x)

1.207.

\/

and take advantage

\V\+#dx =^Vl+x*dx = I<y

r~\

1.1.

of the

Schwarz-

5(i+^)dx.J

l*dx

= l/T2T
6.3.10.

Find the derivative with respect

functions:

(a)

F(x)=\\ntdt

(x

>

0),

X2

Vx
(b)

F(x)=

cos

(t

)dt

(x>0).

to

of

1.095.

the following

Ch. VI. The Definite Integral

268

Solution, (a) Write the given integral in the following way:

(x)

\ In

MM- \

K2

where

*3

XZ

In tdt

X'

\nt

dt

tdt,

$ In
C

>

is an arbitrary constant.
us find the derivative F' (x) using the rule for differentiating a composite function and the theorem on the derivative of
an integral with respect to the upper limit:

Now

let

F'x

(x)

\ntdt

(x% =

\ntdt

In

cos (t*)dt-\- $cos(* 3 )d*

2x =

cos (t*)dt

=-

= (9x* 4x)lnx.

-J

F'(x)

a;

Vic

In

x 3 3x>

Xs

_C

<b) F(x)

X2

cos

(t

2
)

di
i

= COS-o1/

)*

cos (P)dt

Trrr

== -.cos

with

derivative

cos (t*)dt;
J

J
1

+ COS*
,

6.3.11. Find the


functions:

( x

1,1^cosa;.

to

respect

x of

the following

23

f(x) = ji!lii;

(a)

(b)

F (x)=^T+T*dt.
X

6.3.12. Find the points of

in the

domain x

Solution.

extremum

of the function

> 0.

Find the derivative


sin x

The

critical

points are:

x=nn

2,

...),

where sinjc =

0.

(x)

^^-dt

Estimating an Integral. Integral as a Fund, of

6.3.

Find the second derivative


cv,

M=
,

F" (nn)
7
v

Limits

269

at these points:

x cos x

Its

sin X
;

=
cos (nn) =
nn
nn
v

n
1

(
v

'

)
7

0.

derivative is non-zero at the points x


nn
these points are points of extremum of the function,
namely: maxima if n is odd, and minima if n is even.

2,

1,

second

the

Since

(n

.),

Find the derivative

6.3.13.

of y,

with respect to x

of the func-

tion represented parametrically:

x=

i/ z\n zdz\ y =

\nzdz.

yt
As

Solution.

is

known,

xt

Find x and
t

x't

y\\

= [^Y~z\n

zdz)

(t

y
t

= t\nt.3t = 9t
2

\nt'

yt

\VT

2Yt

'YT

whence
y'

= - 36/

31n

-\Vt

J/7

(f

> 0).

\nt

6.3.14. Find the limits:

^ sin I^a:
(a)

lim 2

-5

;
'

(b)

lim
x-+

+ a>

(arc tan x) 2 dx

Vx2 +

"
(c)

lim
+

oo

x2

Solution, (a) At x^-=0 the integral

sin

equals zero;

it

is

easy to check the fulfilment of the remaining conditions that ensure

Ch. VI. The Definite Integral

270

the legitimacy of

using the L'Hospital rule. Therefore

^ sin V".x dx

V^xdx

sin

= lim

lim

We

have an

L'Hospital

the

of

2
'

type. Use

the

2 ^

lim

= lim 2x 3xsin x

form

indeterminate

^e* dx

X*-

rule:

3x 2

x-+0

(c)

(*

dx-?*

lim
*-*

CD

<?*

00

e*

J
X->

6.3.15. Find the derivative

(a)

^e~ t2

dt+

e*

CO

lim

2-^ = 0.

of the following implicit functions:

tdt=0\

(b)

sin 2

lim

^e

dt+

sin/d*

= 0;

J
o

(c)

|/"3

2sin

ede+ ^cos/d/=0.

Solution, (a) Differentiate the


respect to x, putting y=y(x):

side

of

the

equation

-x 2

dy
sin"

J
_o

.0

Hence, solving

left

the

-' 2

^ + sin

* 2 2x
-

= 0.

equation with respect to j-

^^_2A;/? + ^
dx

sin 2

2
A:

we

get

with

Estimating an Integral. Integral as a Fund, of

6.3.

the

Differentiate

(c)

putting y

Its Limits

271

side of the equation with respect to x,

left

= y(x):
'

2 sin 2 z dz

^ cos

dy
/

dt

dx

= 0.

-0
X

Whence

/3 2 s
Find:

6.3.16.
inflection

2
i

(a)

V3 2 sin

^
+ cos y ^ = 0; dx

points of extremum
the function

the

on the graph

of

X
/

l)(f

cos y

and

the

points

of

2) df;

curvature of the line defined by the parametric equations:

(b)

= aVn

cos
(

%- dt,

o
.

y = ay
(the

Cornu

sir
sin

dt

-77-

spiral).

Solution,

(a)

The function

is

defined and continuously differenscale. Its derivative

tiate throughout the entire number


/;

= (*-l)(*-2)
x =
x = 2 and
2

when passing through


l,
equals zero at the points 1
2
the point x 1 it changes sign from minus to plus, whereas in the
neighbourhood of the point x 2 the sign remains unchanged. Conse1, and there is no
quently, there is a minimum at the point
extremum at the point x 2 2.
The second derivative
y

^=

rx = 3x
vanishes at the points x 1

lOx + 8

-~y* x 2

=2

and changes sign when pas-

sing through these points. Hence, these points are the abscissas of
the points of inflection,
(b)

We

have
x't

=ay

n cos

Ttf^

y\

= a yr n sin

Tit

2
,

Ch. VI. The Definite Integral

272

hence,

^ cos 3 -g-

*/

*7

whence the curvature

K_

\y"\

__

Vnt
a

U
6.3.17. Prove that the
oo) by the integral

(y')

'

2
l

function

L(*),

defined

in

the

interval

(0,

an inverse of the function

is

Solution.

ex

Let us take the derivative

=7

(*>0).

Since the derivative is positive, the


and, hence, has an inverse function

x=L~
The derivative

of this

increases

(y).

equal to

is

__

Ty~L r{x)~ X

whence

y^--L(x)

inverse function
dx

function

'

follows (see Problem 3.1.10) that

it

x = Cey.

To

find C,

substitute x

Since

l.

= 0,

L(1)

i.e.

y\ Xssl

then
1

which proves our

=C^-C,

assertion:

x=L'
6.3.18.

Given the graph

of

(y)=ey.

the function

y=f(x)

(Fig.

62),

find

the shape of the graph of the antiderivative I='^f(t)dt.


o

tive;

On

the
interval [0,
a], the given
consequently, the antiderivative increases.

Solution.

function is posithe interval

On

6.3.

0,

the derivative of the given function

yj

curve

Estimating an Integral. Integral as a Fund, of Its Limits

= / (x)

of the given function


is

On

concave.

is

convex, the point

is

the interval

being

point

of

derivative

the

curve

negative; consequently, the

*^y

hence, the

positive;

is

273

The

inflection.

= / (x)
inter-

is a
val [a, 2a] is considered in a similar way. The point x 1
point of minimum, since the derivative I'(x)=f(x) changes its
sign from minus to plus; the point x 2
a is a point of maximum,
since the sign of the derivative changes from plus to minus.

3a

Fig. 62

2a.

Fig. 63

The antiderivative

a periodic function with period 2a,


is
I (x)
the areas lying above and below the x-axis are mutually
cancelled over intervals of length 2a. Taking all this into account,
we can sketch the graph of the antiderivative (see Fig. 63).

since

6.3.19.
a

the polynomial P (x) of the least degree that has


equal to 6 at x=l, and a minimum equal to 2 at

Find

maximum

= 3.

Solution. The polynomial is an everywhere-differentiable function.


Therefore, the points of extremum can only be roots of the deriva-

Furthermore, the derivative of a polynomial

tive.

P\x) = a (x

Since at the point

x=

(x

a
1

polynomial.
and x 2 = 3

3) = a (x Ax + 3).
2

there must be P(l)

= 6,

we have

P(x)=l

1)

is

The polynomial of the least degree with roots


has the form a(x
l)(x
3). Hence,

P' {x)dx-\-&

= a^(x* 4x + 3)dx + 6 =
i

= a y 2* + llx )
1
(
2

The
a

coefficient

= 3.

is

determined from the condition

Hence,

P(x)

= x 6x + 9jc-|
3

2.

-!-

P(3)=2, whence

Ch. VI. The Definite Integral

274

6.3.20. Find the polynomial P (x) of the least degree whose graph
points of inflection: (1,
1) and a point with
1), (1,
at which the curve is inclined to the axis of abscissas
abscissa

has three

at

an angle of 60.

Solution. Since the required function is a polynomial, the abscissas of the points of inflection can only be among the roots of the
second derivative. The polynomial of the least degree with roots

0,

1,

has the form ax(x 2

Consequently,

1).

P"(x)=a{x 3
Since at the point x

x).

the derivative P' (0)

= tan60 = j/3,

we

have
X

P'

(x)=P"(x)dx + V3 =

Then, since P(l)

l,

we

a^-^) + VZ.

get

= a(- + l)+V3(x-l)+l.

P(x)=^P'(x)dx+\
1

The

coefficient

/>(_!)

1,

Jrom
^(T/a-i)

the last remaining condition

determined

is

whence a

H ence,

"

]/ 3 ~"

P(x)-

Taking

6.3.21.

definite integral,

(3a:

advantage
prove that

of

the

Ojc

3
)

+ x J/3.

mean-value theorem

for

the

(a)

3< \ Vq + x*dx<

10,

o
JT

(b)

]/

+ ysin 2 *d*<^

|,

2Jt
/

(C)

?n
13

dx
P
J 10 3 cos x
|

2jt
*

Using

6.3.22.

V +x
1

dx

the

< -Xp-

Schwarz-Bunyakovsky

Make

inequality, prove that

sure that the application of the

value theorem yields a rougher estimate.

mean-

275

6.4. Changing the Variable in a Definite Integral

Find the derivatives

6.3.23.

of

the following functions:


x2

(a)

= ^lntdt(x>0);

F(x)

(b)

F(x)

= y.

_2_

Find the derivative

6.3.24.

functions represented paramet-

of

rically:
In

(a)

= ^dz,
j

e z dz\

j
5

2
sin

(b)

x=

C
\

VT

arc sin zdz,

6.3.25. Find the points

(a)

F(x)

)e

(l

2
t

of

C sin\

z2

dz.

extremum

of

the following functions:

)dt;

X2

(b)

/7

W =f!!=gid/.
+
J

e<

Changing the Variable

^=

In

a Definite Integral

the following conditions:


defined in [a, p]
and has in this interval a continuous derivative q/(/);
y(t)
(2) with / varying on [a, P] the values of the function x
do not leave the limits of [a, b]\
If

(1)

a function
q)(/)

is

cp(/)

satisfies

a continuous single-valued function

(3)

q)(a)=a and y($)

= b,

formula for changing the variable (or substitution) in the


definite integral is valid for any function f (x) which is continuous
then

the

on the interval

[a

b]:
b

lf(x)dx=\f[<p(t)] y'(t)dt.
a

y(t) the inverse substitution


the substitution x
frequently used. In this case the limits of integration
a and p are determined directly from the equalities a ^(a) and
x|;(fr).
In practice, the substitution is usually performed with
p
the aid of monotonic, continuously differentiate functions. The
change in the limits of integration is conveniently expressed in
Instead

= yp(x)

of

is

Ch. VI. The Definite Integral

276

the tabular form:

6.4.1.

Compute the

Make

Solution.

^T-^^C-y.

the

\ \x*dx.

= 2sin/,
# = (/) = 2 sin

substitution

function

sa ^ s fi es a "

[""IT' "t]

integral

The

3~

^e

on

(p

the

that

interval

conditions of the theorem on changing

the variable in a definite integral, since

is

it

continuously differen-

monotonic and

tiable,

<p(-t) = -^. <p(t) = ^ 3


And

so,

= 2 cos

= 2 sin

a:

cos/>0

since

assuming

je

V ^ x* = 2

dt\

Tl

on the interval

Jl

cos

= 2 cos

/,

3"

~3~J

Thus,

J
s

dx = 4

cos 2

"

tdt=2

JT

Jl

"a

+ cos20d/=

(l

=2

lsin2<]y = ^ + )/3.
[/ +
iL
3

6.4.2.

Compute

Solution.

Make

p 1/^2

the integral

the substitution

= 2 sec/;
d* = 2^df;
cos
2

*l

ji

On

the interval

0,

fore the substitution

~J

is

the function 2 sec

valid.

is

monotonic, there-

277

6.4. Changing the Variable in a Definite Integral

Hence,

~4

J^sec 2

_/

16 sec 4

^ sin/

^cos 2

/^

H
3

=4i sin
si

6.4.3.

Compute the

cos tdt

=j2

s ' n3

^
o

Vz
*

32

integrals:

VI

dx

x*Vtf=^dx;

(a)

(b)

^=

6.4.4.

f
J

(a)7
v

Compute

"8
6 5

the integrals:

dx

*f

sin x-\- sin ,2 *

(b)
7
v

+ cos x

'

Solution, (a) Apply the substitution


sinje

cos x dx

=
= dt\

/;

ji

~2~

The
all

inverse function x

= arc sin t[Q <

-y

< <

for

satisfies

1)

conditions of the theorem on changing the variable. Hence,

cos
/

6 5 sin

xdx
x-\- sin 2

Make

"

:ln-

3
2

_4_
'

(b)

6- 5/-j-/

the
jc

substitution

= 2arctanf,

= tan-^X2
dx

2dt

=
l

+ t*'
ji

~2

which

is

valid due to monotonicity of the function tan 4"

on

the

Ch. VI. The Definite Integral

278

interval

+ cos.v

2+

2dt

J 2

\+t*
/

j/3

j/3

J/3

Compute

the integral

= -7=- arc tan -=


6.4.5.

df

9 f

dx
2

Solution.

Make

tan

''arc

cos 2 x-\-b 2 sin 2

U arc tan 0^ =
^3

71
'

3 1^3

b>0).

(o>0,
a:

the substitution

tan#

/,

dx
dt,

Hence,
<4

dx

cos 2 x-\-b 2 sin x

f
J cP

dt

+ bH ~ b*J
\

_
~

dt

a2

= b a arc tana = ab
-r arc tan
a

If

#,=

6=1,

then

'

ab

arc

tan
a

cides with the result of the


integral

\o

= arc tan
1

substitution

ji
,

which exactly coin-

= 6=1

jt

dx

I
6.4.6.

Compute

[*

the integrals:

dx
dx;

(b)

f-

J X l^l

(0

dx.

+ ln *

ji

dx

a 1 cos 2 x-j-fr 2 sin 2 #

'

"

into the initial

279

the Variable in a Definite Integral

Changing

6.4.

.1

Compute

6.4.7.

/=\

the integral

cos i x

sum

this integral to the

Reduce

Solution.

x sin x

dx

two

of

integrals:

ji

3T

~2~

To

the integral
x sin x
'

-j-cos 2 x

dx

apply the substitution

= n
dx = d/,
x

/,

ji

ji

Then
JI

Q sin
(ji

' 2

l+cos 2 (Jt-0

r (n

t)

/)

sin

1-j-cos 2
1

'

d/
/

smt

1-f-cos 2

/"*

sin

+ cos

df.
/

Hence
.

Since the

=/i

first

/ s

_ C x sin x
- J + cos x^

sin

71

and the third integrals


we have

of the variable of integration,

n
)

+ cos

/ rf/

+ cos

f*

differ

sin

+ cos

only in the notation

Ch. VI. The Definite Integral

280

To

integral apply the substitution

this

du

= cos
= sin
t

dt,

ji

The

Jl'

1+M a

indefinite

integral

x sin x

r*

/Vote.

du

du

t1

not

is

COS X

-j-

expressed

in

elementary functions. But the given definite integral, as we have


shown, can be computed with the aid of an artificial method.
6.4.8.

Evaluate the integral


\n(\+x)
1-f-x

Solution.

Make

dx.

the substitution

dx

= tan/,
-=

dt

cos 2

Jl

Hence,
Jl

In

(1+ tan/)
sec 2

Transform the sum

sec 2

<tf

= Jln(l +tan/)^.

+ tan/:
/2~sin

+ tan = tan
/

-f-

Substituting into the integral,


jt

~T
/

tan

/ =

-|

cos

we obtain

n_

n_

- j lln2d/ + Jlnsin(/ + -J-)df Jlncos/<//--

(I

T
= 1/1112 T + jlnsin(/ + -5-)d/ Jlncos/d/ =
4

Jt

JT

T
= -y ln2 + jlnsin^/ +

dt

In cos

dt

ln2

/2

6.4. Changing the Variable in a Definite Integral

Now

us

let

I^=I

show that

To

281

end apply the substitution

this

dt

= dz

jt

JT

T
to

the

integral

/2

In cos

dt.

Then

/2

= ^ In cos

ji

In sin

j"

^- + 2^

lnsin

dz

=I

t.

Therefore

/=T ln2
Note that

this problem,

in

indefinite integral

well as in the

as

^~~jz^r ^ x

no * expressed

ls

preceding one, the


in

elementary fun-

ctions.

6.4.9. Prove that for any given integral with finite limits a and b
one can always choose the linear substitution x = pt
q (p, q constants) so as to transform this integral into a new one with limits
and 1.

We

pt
Solution.
notice that the substitution x
q satisfies
explicitly the conditions of the theorem on changing the variable.

Since

we have

must equal zero at x = a and / must equal unity


for p and q the following system of equations

= p-0 + q,
b = p.\+q,
a

whence

= b a,
b

= a.

Hence,
l

lf(x)dx = (b a)$f [(b a) + a] dt.


t

at

=b

Ch. VI. The Definite Integral

282

6.4.10.

Compute

sum

the

two integrals

of

_2_

-5

$e<* +5

2
>

.(-*)' dx.

dx+3

each of the given integrals into an


preceding problem).
To this end apply the substitution x =
t
4 to the first inte-

Solution. Let

transform

us

and

integral with limits


gral.

Then dx =

dt

(see the

-5
l i==

e ( x+

and

dx=-le<-

^ dt
2

=-

-4

Apply the substitution x


dx =

= -^-\-~-

the

to

second

2
1

>

dt

integral.

Then

and
_2_

o
3

=3$

/2

2 \

e*\

~)

dx

\ev-^dt.

Hence

=-

+
Note that neither

of the

je

'-

1)2

dt+

integrals

= 0.

and $ ^*

e u+5)2

evaluated separately in elementary functions.


6.4.11. Prove that the integral
p sin 2kx
\
i

equals zero

if

is

Make

Solution.

an

dX

sin y

integer.

the substitution

=
dx = dt,
x

tc

t
1

/,

n
Jt

Then

at

k an integral number

we

get:

Ji

P sin 2kx

sin

a:

P sin

~"

2fe

(ji

sin(Jt

/)

/)

^_

P sin 2&/
sin/

rfx

is

6.4. Changing the Variable in a Definite Integral

283

Since the definite integral does not depend on notation of the variable of integration, we have
/

Compute

6.4.12.

= /,

whence

= 0.

the integral

dx

VT-

s\nt (the given function is


Solution. Apply the substitution x
cost dt. The new limits of integration t 1 and
not monotonic), dx
t 2 are
found from the equa-

y = sin
i

tions

We may

=y

y~3

/;

=y

put

other

but

= sin

also be chosen,

and

t2

may

values

instance,

for
2jt
'

3
Fig.

In both cases the variable


x
s\nt runs throughout the

64

^3

entire interval'
2

monotonic both

on

|_<r>

show that the

Let us
Indeed,

Fig.

(see

the

64),

function sin/ being

J
2jt

and

tJ

results of the

5ji

~6~

two integrations

will coincide.

dx

r
a:

1^1

r
J

c s

sin

di

cos

r
,1

sin

= lntan-

tan

-In

tan

In

12

On

the other hand, taking into consideration that cost

on the interval

j^-y;

-yj

we obtain

is

/V^3
negative

Ch. VI. The Definite Integral

284

2n

dx

*
i

xVlx

5jt

cos

cos

sin

dt

t)

dt

__

sin/

5jt

2jx

tan

= In

-In

tan

2-f

ln

tan^
Note.

Do

not take

=^

interval

|y

the

limits of

the interval

6.4.13. Prove

-yj

since,

with

= sin

varying oo
t

lie

beyond

'

L(x)

function

the

that

=y

the values of the function x

the

/a

^"3

^"3

on the

defined

interval

(0,

oo) by the integral

(x)

= possesses
J

the following properties:

L(x l x2 )

= L(x + L(x
l)

2 ),

L^j=L(Xl )-L(x
Solution.

By

2 ).

the additivity property

X,

Let us change the variable in the second integral


z

=x z
dt = x dz
t

1
y

Then
L(x,x 2 )

^=

= j +

Putting

here x 1 x 2

=x

3]

x2

L(x l ) + L(x 2 ).

= x
x
3

we obtain

L(x 3 )^L(Xl )

+ L^,

i.e.

L(^)=L(x )-L( Xl
3

).

It

Changing

6.4.

L\x

also easy to obtain the other corollary

is

any integral

and

and

m
)=
n

L(x)

for

n.

Indeed, for positive

L{x

285

the Variable in a Definite Integral

and n

=^mL\x n

from the relations

this follows

L(x)=nL(x n

negative exponent, from

for a

L(1)

= 0,

L(x- l )

Now, taking advantage

of

ction of the upper limit,

Note. As

= L()=L(l)-L(x) = -L(x).

known, L(x)

is

the continuity of the integral as a funget the general property L(x a )


aL(x).

we

= \nx.

Here we have obtained the prinfrom its determi-

cipal properties of the logarithm proceeding only

nation with the aid of the integral.


3

Transform the integral ^(x

6.4.14.
{x

2fdx

by

the

substitution

2f--^t.
A

Solution.
interval

function x

has
not

formal application of the substitution throughout the


would lead to the wrong result, since the inverse

3]

[0,

= y(t)

two branches: x = 2
attain values x > 2, the

rect

we have

i.e.

x 2 -=2

[/" t\

result

= 2Y~i>
the function x
+ Y The former branch canvalues x < 2. To obtain a cor-

double-valued: x

is

latter

t.

to break up the given integral in the following

way:
3

(x

2)

dx

= J (x 2)

dx

+ J (x 2)

dx,

and

to put

second.

= 2Y

Then we

in the

first

and x

integral,

=2+V

in

the

get

/2

=j(^-2)Mx=j/ -^ =i-Ji/rd/=i.
2
r
2

Hence, /

= 8

= 3,
1

-(-

which

is

a correct

result.

It

can be easily

Ch. VI. The Definite Integral

286

verified

by directly computing the

{x-2)* dx
Compute the

6.4.15.

integral:

initial

(x-2f

= i+T= 3

integrals:

Jt

4
Jt

4
,

(e )

+ cos x
+ sin2*

C sin #

=j

dx

>

a
(f)

/=jV ]/ j^-d*. a>0;


b

2a

(h)/ =

/ = ^2ax-x*dx;

(g)

j fr

6.4.16, Applying a suitable change of the

variable, find the fol-

lowing definite integrals:


dx

dx
.1

^+1+^+1)3

<b>J;

+ / x
2

'

V(a* + b*)/2
(c

).fmf^)

(d)

yV a )(&

fc

it

is

equal to -j

=
+
d#

2-

-2

a;

-7T

2
)

)/2

6.4.17. Consider the integral

that

.f

V'(3a*

_^

It

is

easy

to

Indeed,

arc tan

-2~~

Jl

[t-(-t)

conclude

On

Changing

6.4.

287

the Variable in a Definite Integral

making the substitution #--=y, we have

the other hand,

X
,

d*

dt

2
1

2
2

dx

arc tan 2t

+l

'

This result

obviously wrong, since the integrand

is

1
2

_JX

+* >
s

and,

0,

consequently, the definite integral of this function cannot be equal

^-

number

to a negative

Find the mistake.

6.4.18. Consider the integral


to

tut ion tan -^

/=

dx

Making& the

2 cos x

J 5

substi-

we have

2ji

dx

2n
P

2<ft

5-2 cos*
j

(1+<1))

5-2

0.

The result is obviously wrong, since the integrand is positive,


and, consequently, the integral of this function cannot be equal to
zero. Find the mistake.
6.4.19.

Make

change

sure that a formal

the

of

variable

_2

leads

to

wrong

the

result

in

the

integral

dx.

Find

the

-2

mistake and explain


6.4.20. Is

it

it.

possible

to

make

the substitution

x=sec/

in

the

integral /

J/V+l

dx?

o
i

6.4.21.
jc

= sin*.

for /?

Given the integral

dx.

Make

Is

it

possible to take the

numbers n and

the

substitution
as the

limits

Ch. VI. The Definite Integral

288

6.4.22. Prove the equality


a

+ f(-x)]dx

f(x)dx=l[f(x)

-a

any continuous function

for

f (x).
2ji

f(x)cosxdx by

6.4.23. Transform the definite integral

the sub-

stitution sin^

6.5. Simplification of Integrals Based on the


Properties of Symmetry of Integrands

1.

If

the function

f (x)

even on

is

a,

a],

then

= 2^f(x)dx.

f(x)dx

-a
2.

the function

If

f (x)

odd on

is

a,

then

a],

f(x)dx--=0.

-a
3.

the function

If

f (x)

periodic with period 7\ then

is

+ nT

f{x)dx^

where

/i

is

/ (*)

a+ nT

an integer.
i

6.5.1.

Compute

the integral

|*|d#.

l
Solution. Since the integrand

/(a:)

= |a:|

is

an even

function,

we

have
i

^\x\dx =

\
)

6.5.2.

Compute the

i
i

\x\dx = 2^ xdx = x

integral
7

a:

sin #

-7

Solution. Since the integrand


the integral equals zero.

is

odd, we conclude at once

that

289

6.5. Simplification of Integrals

6.5.3. Evaluate the integrals


ji

(a)

cosnxdx;

J /(*)

jr
jt

(b)

^ / (x) sin

nxdx,

-71
if:

(1) f (x)

is

an even function;

f(x)

(2)

is

an odd function.

6.5.4. Calculate the

integral

^4

+ 2* +
2

dx.
t

sin

Compute the

6.5.5.

Therefore

and lower

jt)

= cos

ji)

+ sin

jt,

(x

:/(*).

"

+ ^)

cos 4 x-f sin 4

a:

number n from the upper

limits:

37

sin

cos 4

a:

2a:
-4-

sin 4 x

sin

cos 4

a:

_2

2a:

+ sin

tan

2
J cos x

(1

xdx

+tan 4

a:)

JT

Make

the substitution
X
t

= tan

dt

a:,

ji
'

cos 2 x

JT

r
J

cos 2

tan x dx

#0

J \+t*''

4" tan 4 x)

2t dt

arc tan

JT

6.5.6. Prove the equality

Solution.

It

is

cos xf (x 2 )dx

sufficient

cos(

to

x)f

=2

cos xf (x 2 dx.
)

show that the integrand


[(

since

sin 2a

possible to subtract the

is

it

(#4- n)

a:

a periodic function with period

is

sin 2 (x-\4

2a:

cos 4 x-\- sin 4

The integrand

Solution.

/(*

integral

x) 2 \

= cosxf (x

2
).

is

even:

290

Ch. VI. The Definite Integral

Compute the

6.5.7.

2s 7
~~

C
J

integral

3* >^
+
+2
6

10a: 5

7a;

a;

+ +

2a: 2---r^-ri
x
1

-K~2

Solution.
V~2

2x7

3a;

0x b

7x*
+2

a;

2a;

+x+

dX

~~

-V~2
r

2xi\0xb 7x*+X j
*2

3a; 2

aX_t"

+2

- 4) +
^+2

(a;

-K~2

-K~2

V~2

[3(^-2x )+ 1?iT ]^ =

= +2

_ 4^3 +

arc t an

x
|/"

K~2

v-2

|o

= _ 16

"

2 l/"2

Tn calculating we expanded the given integral into the sum of


two integrals so as to obtain an odd integrand in the first integral
and an even integrand in the second.
6.5.8.

Compute

the integral
2

cos*

In
1

Solution.

The function

+X
q>(x) = lny^

f(x)

x dx.

= cosx

is

even. Let us prove that the

function
cp

is

odd:

<_*) = in {= _ In ({f )-

= -In {f - _ W

Thus, the integrand is the product of an even function by an odd


one, i.e. an odd function, therefore

cos*

ln\^dx
X = 0.
1

"
6.5.9.

Prove the validity

of the following equalities:

(a)

x s s'm d xdx -=0;

Integrals

6.5. Simplification of

(b)

e cosx dx

ji

291

= 2 ^e cosx dx\
o

"T

~T
ji

(c)

sinrnxcostt^djc

s'mxf (cosx)dx

= 0.

(m and n natural numbers);

JT
Q

(d)

-a

6.5.10. Prove the equality


h

^f(x)dx^^f(a + b
a

Solution. In the right-hand integral

x^=a + b

dx=^

/,

dt

x) dx.

Then we obtain

make

the substitution

baa

j f
a

(a

+ bx) dx=lf(t)dt^lf(t)dt = \f (x) dx.

Note. The relation established between the integrals can be explained geometrically.
The graph of the function f(x), considered on the interval [a, b]
is symmetrical to that of the
function f(a
b
x)
considered on

+
x = ^-^-.

the

same

point

which

A
is

interval, about the straight line

Indeed,

if

the

on the x-axis and has the abscissa x, then the point A\


symmetrical to it about the indicated straight line, has

lies

the abscissa x'

=a + b x.

+b x)] = f(x).

Therefore,

(a

+ b x') =

[a

+ b (a+

But symmetrical figures have equal areas which are


expressed by definite integrals. And so, the proved equality is an
equality of areas of two symmetrical curvilinear trapezoids.
6.5.11. Prove the equality

\f{x)g{t

x)dx=\g(x)f(t x)dx.

Ch. VI. The Definite Integral

292

Solution.

then

Apply the substitution

x=z in the right-hand integral;

we have
o

-lg(t-z)f(z)dz~lf(z)g(t- z) dz.
Jl

Jl

6.5.12. Prove the equality

sin

xdx=^

cos m

xdx and apply

J
o

obtained result in computing the following integrals:


JX

Jl

#dx and

cos a

On

Solution.

sin 2 **!*:,

Problem 6.5.10 we have

the basis of

71

Jl

sin"

xdx

Hence,

sin

(y

cosmxdx.
J

in particular,
71

71

sin 2

/^=^

xdx=^

add these

cos 2 xdx;

integrals:
7l_

71

2/

hence, /

(sin 2

+ cos

jc

x)

dx =

dx =

~\

= -j

6.5.13. Prove the equality


2

ji

f (sin

jc)

dx = 2

(sin x) dx.

Solution. Since

(sin x)
^ /

/ (sin x)
jj

dx

+^

(sinx) dx,

the

it

sufficient

is

6.5.

Simplification of Integrals

293

to prove that
ji

ji

f(s'mx)dx=

(sin x) dx.

_jx_

In the left

make

integral

x=n

the substitution

/,
71

dx= dt,

JT

T
n

Then
JT

f(s\nx)dx =

^ f [sin (n

t)]

dt

sin

*=J

/ ( sin

x) dx

6.5.14. Prove the equality


n

ji

^ xf
Solution. In the

left

(s\n x)

dx = ^J

integral

make

/ (sin

dx.

the substitution

x=n

a:)

t,

dx= dt,

JT
ji

Then we obtain

a:/

(sin

x)dx =

(ji
{[

[sin

(ji

/)]d/ =

Jt
JT

Jt

= 5"/ (sin/)* S'/ (smt)dt.


Whence

%\xf (s\x\x)dx= n
which

is

f(sin x)dx,

equivalent to the given equality.

Ch. VI. The Definite Integral

294

6.5.15. Using the equality

s\n[n +

j]x

= + cos^-|-cos2a;+
j

-x-

im-

-j-cosnx,

prove that

^H- "^") x

sin

ji

^
x

sin

6.5.16. Prove that


-\-b 2 sin

2.x:

q>(x)

if

=ya +a

^osx + b 1 sin

x+a cos2x+
2

then

2ji

= Jta

cp (jc)

+an cos nx + bn sin fix,

2ji

(a)

dx n.

(b)

<P

(x)

cos kx dx

= na k

2ji

(c)

(p

kxdx = nb k

(x) sin

(k=l,

2,

tt).

Integration by Parts. Reduction Formulas

and

If

t;

are functions of x

and have continuous derivatives,

then
b

(#) u'

(a:)

dx

&

= u (x) v (x)

or,

more

(x) u' (x)

dx

briefly,
b

^udv = uv^ ^vdu.


a

a
I

6.6.1.

Compute the

integral

^xe x dx.
o

Solution. Let us put

x=
du =

ex

u,

dx
v

dx\

dv\
= ex
>

x and v
quite legitimate, since the functions u
continuous and have continuous derivatives on the interval
Using the formula for integration by parts, we obtain

which

is

xe x dx

= xe x

ex

dx = e

ex

= ex

are

[0,

1].

6.6.2.

Integration by Parts. Reduction Formulas

6.6.

Compute the

integral

I=

\ e

ax sin

295

bxdx.

Solution. Let us put

= e x dx;
v = e x

= s'mbx,
du = b cos bx dx,

dv

Since the functions u

= smbx,

= eax

together with their deri-

vatives are continuous on the interval [0,


tegration by parts is applicable:

ax smbx
/=
e
a

a J

ax cos bx

dx

= a
Now

let

us integrate

the formula for in-

jt],

[ e ax cos
J

by parts the integral

= cos bx,
du = b sin bx dx,

/t

bx dx

- L.
1

Put

= eax dx,
v = e ax
a

dv

Then

ax cos bx
e
a

b
,

f
\

ax s\n

bxdx

e~

\\

b*

a J

a2

G T +l)

Hence
a?

b{eT +\)

+ b*

In particular, at a

= b=

we

b{e

get

^e x s'mxdx =

-j(e n +1).

T +\)

a2

b*_

a2

Ch. VI. The Definite Integral

296

Compute

6.6.3.

the

^\n 3 xdx.

integral
jt

Compute

6.6.4.

Solution.

First

^s'mV x

the integral

make

dx.

the substitution

Vx=t,
x = t\
dx = 2t dt

ji

ji

Whence
.n

sin J/* dx:

=2

sin

dt.

Integrate by parts the latter integral.

Put
t

= u;

du =

sin tdt
v

dt\

= dw,
= cost.

Then

sin tdt

Compute

6.6.5.

=2
the integral

171

~z

+)

/=

cos

dt

sinH

=2.

lo

^-^y==^-

dx.

6.6.6.

Compute

the integral

6.6.7.

Compute

the integral

In

x 2 s\nxdx.

=^(a

2
)

where n

dx>

is

a na-

tural

number.

The integral can be computed by expanding the integrand


according to the formula of the Newton binomial, but it
involves cumbersome calculations. It is simpler to deduce a formula
for reducing the integral / to the integral I n _ l To this end let
Solution.

(a 2

expand the

us

Integration by Parts. Reduction Formulas

6.6.

integral I n in the following

way:
a

(a 2

a;

and integrate the

2 )"" 1

(a

(a 2

by

n~x

x(a 2

a:

)"" 1

xdx

parts:

xdx = dv,

= ^-(a
x
2n
1

dx = a 2 l n _ x

integral

latter

= x;
du = dx\

in

We

297

'

(n=0).

obtain

= *i n -i +

ix

-*r -~k I ^ ~ x2
|

Y'

dx = a2/ -

~i

7 -

Whence
/
l

This formula

is

valid

=aU

any

at

2n

I
2/1+1 '"-I*

n other than

real

and

~.

In particular, at natural n, taking into account that


a
Io

= l dx = a>
o

we

get

2n (2/1-2) (2/1-4)

*~~ U

(2/i+

1)

(2/i

1) (2/1

3)

6-4-2
.

aw + 1

5-3

(2/z)H

(2/1+1)!!'

where

=2-4-6

(2/i)!!

(2/i+l)!!

1-3-5 ... (2/1+1).

result of the preceding

Using the

6.6.8.

... (2/i),

problem obtain the

lowing formula:

where

"I-

-r

are binomial coefficients.

Solution. Consider the integral

2/1

(2/2+

'

1)!!

fol-

Ch. VI. The Definite Integral

298

Expanding the integrand by the formula of the Newton binomial


to 1, we get:
and integrating within the limits from
i

In

\(\x*) n dx =
o
i

= J(1 C} x + C
2

~T~

*4

- C*x + ...+(
7

~T

6.6.9.

Compute

2/7

^3

3^5

(-1)"

" "

2n+l

proof.

the integral

// w

(m

C nn x 2n ) dx =

^2

^1

which completes the

n
1 )

m
==5 sin^jcrfA;^^ cos xdx

a natural number).

Solution.

The substitution

=
cosjcd.x: = d/,
sin x

f,

31

"2

reduces the second integral to the integral

Hm =
considered

in

(1 sin 2 *)

cosxdx=\{\t*)

Problem 6.6.7 with a=^\ and n

the reduction formula

m
is

valid here, since

(m=^0, m^=

1)

dt

Therefore,


If

6.6.

Integration by Parts. Reduction Formulas

299

an odd number, the obtained reduction formula reduces

is

ti m to
2

cos#djc=

1,

therefore

(m-l)ll

*
If

Hm

is

ml!

an even number then the reduction formula transforms

into
2

therefore

(m 1)

6.6.10.

Compute the

ji

!!

integral
ji

= ^xsm m xdx
o

(ra

a natural

number).

Solution. Taking advantage of the results of Problems 6.5.14


we get

and

6.5.13,

oo

ji

ji

ji

m xdx =
m
/ = xsm
j sm xdx = n

which, taking into consideration the result


(ji

-2

ji

'

(m 1)!!
L
L
-7^fT -

rr
mil

xdx,

Problem 6.6.9, gives

of

lf

s\n m

is

15

even

'

odd.

6.6.11.

Compute the

integral

/n

x m (\n x) n dx\

m>

0,

is

natural number.
^= x m (\n x) n
f (x)
can be made continuous on the interval

Solution. First of all note that, though the integrand

has no meaning at #

it

Ch. VI. The Definite Integral

300

1]

[0,

any

for

m>

> 0,

and n

lim x m (\nx) n
X-+

by virtue

of

Hence,
0, n

in

m>

it

u =(\nx) n
,

\nx)

follows that the integral l n exists at


we integrate by parts, putting

it

To compute

du

Indeed,

3.2.4.

particular,

> 0.

\x

lim

^=0.

f (0)

X-+ +

+Q

Problem

by putting

= x m dx,
xm +
a =
m+

dv

n(\nx) n - 1

ax,

Hence,
i

The formula obtained reduces /

In particular, with a na-

to I n _ v

tural n, taking into account that


i

we

get
"

= (-!)" (m+l)" +
!

'

Compute the

6.6.12.

integral / Wt

xw

x)

( 1

dx,

where

and n are non-negative

integers.

Let us put

Solution.

( l

n
)

= a;

du= n(l x)

dx

n~l

dx;

= dv;
v=

Then

The obtained formula


is

a positive

times,
/

we

is

then,

valid for all n

applying

this

>

and

0,

formula

1.

If

successively

get

m+

integer,

/
1
i

l)

m + i.-i""( /0 +l)(m

==

+ 2)
'

~~

(n I). .[n (n \)]


(m+l)(m + 2)...(m + /i)

m+n

'
*

Approximating Definite Integrals

6.7.

301

But
l

xm + n + l

m+n dx:
u0 =\x
f

m+n+

'

/i

+n+

I)

m+

Hence,
. n
(

The obtained

result,

m+

J)

3-2-

2)

..(m

/i)

(m

'

non-negative integer, can be written

form

in the

_ m\n\
~ (m+n+

m
6.6.13.

Compute

"

1)!

the integrals:

(a)

l) (/i

m + 2).

with

(/i

/i

V x dx\

arc tan

\)e~ x dx\

^(x

(b)

~3~
i
.

\c)

(e)

C
j

Xdx
iiH^'

r>

(d)

+x

J*ln(l

)dx\

(f)

a:

arc tan

ln(l

a: g(a:;

+ tanx)dx;

o
ji

16

(g) ^ sin

arc tan (sin x)dx\

2a:

(h)

arc tan j/" V^xldx.

6.6.14. Prove that


i

^(arccosArJ^dAr^Ai^y

(/t

1)

(arc cos

a;)"

~2

dA;

(n>

1).

6.6.15. Prove that if


lowing formula is valid

f" (x) is

continuous on

[a, &],

then the

fol-

*r

(x)

= [bf (b)-f(b)] - [af (a)-f (a)].

6.7.
1.

Approximating Definite Integrals

Trapezoidal

formula.

parts by points x k
apply the formula
b

/ (jc)dx

Divide the interval

[I / (*

-^
b

where h =

= a-\~kh,
)

+/

[a,

= 0,

b]
1,

into n equal

and

n,

+ /(*_,) + y f (*)]

Ch. VI. The Definite Integral

302

The

error

in this formula

estimated as follows:

is

|R|<^,

where

M=
a

sup

\f

(assuming that the second derivative is bounded).


2. Simpson's formula.
Divide the interval [a,
parts by points

xk

= a + kh

^f(x)dx^ -^.{f(x

<)

Assuming that
is

where

in )

(*, - 1)]

{x) exists

and

+2[f(x t
is

into

b]

2n equal

an d apply the formula

+ f(x + 4[f(x + f(x

+f
iv

^ = ~^

(x)\

)+...

+ f (* +...+f (x
4)

bounded, the error

in this

2n _,)]}.

formula

estimated in the following way:

Approximate the

6.7.1.

formula

at

dx

integral

I=\y^r

using the trapezoidal

10.

Solution. Let us tabulate the values of the integrand, the ordinates y {


10) being calculated within four de0, 1,
/ (x ) (i
cimal places.

yi=

+*i

TTT

Xi

M= TT7

+Xi

0.0000

1.0000

1.0000

0.6000

1.6000

0.6250

0.1000

1.1000

0.9091

0.7000

1.7000

0.5882

0.2000

1.2000

0.8333

0.8000

1.8000

0.5556

0.3000

1.3000

0.7692

0.9000

1.9000

0.5263

0.4000

1.4000

0.7143

1.0000

2.0000

0.5000

0.5000

1.5000

0.6667

Using the trapezoidal formula, we obtain


,

dx

/=3T+7~Tol

1.0000+0.5000

n qoqq
0.9091+0.8333
+
+ Aftnn1
.

+ 0.7692 + 0.7143 + 0.6667 + 0.6250 + 0.5882 + 0.5556 +


+ 0.5263 ) = 1 6.9377 = 0.6937 0.6938.

We

Estimate the error in the result obtained.

0<jc<1,

Since

number 2

as

then |f (x)|<2.

and estimate the

have

Consequently,

f" (x)

= ^_^

we may take

the

error:

ll<72^ = 6^0<
We

303

Approximating Definite Integrals

6.7.

0017

calculated the ordinates accurate to four decimal places, and

Q
the round-off error does not exceed

Q5

Q,Q

precisely,

numbers^

exact

= 0.000045,

Q5

(l

+9xl) = 0.00005

the ordinates y

since

and

^more
are

*/ 10

Thus, the total error due to using the trapezoidal

formula and rounding

the ordinates does not exceed 0.0018.


integral by the Newtonobtain
off

Note that when computing the given


Leibniz formula

we

dx
l+JC

ln(l+x)

'

= In 2^0.69315.

Thus, the error in the result obtained does not exceed 0.0007,
a result accurate to three decimal places.

i.

e.

we have obtained

1.5

Evaluate

6.7.2.

by

Simpson's formula the

integral

C
\

e'

lx

^-dx

0.5

accurate to four decimal places.

To give

Solution.

we

racy,

fiv

(A;

find /

iv

(a;).

2n which ensures the required accu-

a value of

Successively differentiating f(x)

(0.0001a;*

0.004a; + 0.1
3

2a;

e-^-,

we

2.4x + 24) = ^g-

get

1 *,

the polynomial in parentheses. On the interval


function cp (a;) =e- lx increases and therefore reaches
lb
1.2. The upper estimate
eits greatest value at x =1.5: (p (1.5)
b
of the absolute value of the polynomial P (x) divided by x can be
separate
terms.
of
its
sum
moduli
The
greatest
of
obtained as the
value of each summand is attained at a:=0.5, therefore

P (x)

where
[0.5,

1.5]

\P(X)

is

the

0.0001

0.004

0.12

x2

*3

<

2.4

24

x*

xb

< 0.0002 + 0.0 6 + 0.96 + 38.4 + 768 < 808.


1

And

so,

iv

|/

be taken as

(a;)|

Mv

<

1.2x808

<

1000. Hence, the number 1000

may

Ch. VI. The Definite Integral

304

We have to compute the integral accurate to four decimal places.


To ensure such accuracy it is necessary that the sum of errors of
the method, operations and final rounding off should not exceed
0.0001. For this purpose we choose a value of 2n (which will determine the step of integration h) so that the inequality

|i?|<{
is

-0.0001

= 5- 10"

satisfied.

Solving the inequality


l

xl 000

180

(2/2)

<5xl0"

we obtain
2n
Let us take 2n

A more

= 20;

>

19.

then the step of integration h will be equal to

accurate calculation shows that at 2n

|Z?|<3.5xl0-

= 20

5
.

we

calculate y t within five decimal places, i. e. with an error


5
then the error of the final rounding off will
not exceeding 10"
6
Thus, the total error will be less
also be not greater than 10~
If

than

4.5xl0" 5

Now

< 0.0001.

compile a table

of values of the function

lues of x from 0.5 to 1.5 with the step h


are carried out within five decimal places.

xi

OAxi

y=^~

= 0.05.

e o.

IX,

f r

^e

va '

The calculations

yi

0.50

0.050

1.05127

2. 10254

0.55

0.055

1.05654

1.92098

0.60

0.060

1.06184

1.76973

0.65

0.065

1.06716

1.64178

0.70

0.070

1.07251

1.53216

0.75

0.075

1.07788

1.43717

0.80

0.080

1.08329

1.35411

0.85

0.085

1.08872

1.28085

0.90

0.090

1.09417

1.21574
1.15754

0.95

0.095

1.09966

10

1.00

0.100

1.10517

1.10517

11

1.05

0.105

1.11071

1.05782

6.7.

Approximating Definite Integrals

305

e 0.lXi

%i

0. l*j

12

1.10

0.110

1.11628

1.01480

13

1.15

0.115

1.12187

0.97554

14

1.20

0.120

1.12750

0.93958

15

1.25

0.125

1.13315

0.90652

16

1.30

0.130

13883

0.87602

17

.00

0. loo

14454

f\

18

1.40

0.140

1.15027

0.82162

19

1.45

0.145

1.15604

0.79727

20

1.50

0.150

1.16183

0.77455

For pictorialness sake we


following calculation chart:

x\

use

at

1
i

3
4
5
6
7

8
9
10
11

12
13

14
15
16
17

18

19

20

0.50
0.55
0.60
0.65
0.70
0.75
0.80
0.85
0.90
0.95

the

y\

OA
U.o47ol
*70

tabular data to compile

= and
= 20

at

an odd

at an

even

the

2.10254
1.92098
1.76973

1.64178
1.53216
1.43717
1.35411

1.28085
1.21574

1.15754
1.10517

1.00
1.05
1.10
1.15
1.20
1.25
1.30
1.35
1.40
1.45
1.50

1.05782
1.01480

0.97554

0.93958
0.90652
0.87602
0.84781

0.82162
0.79727

0.77455
2.87709

Sums

12.02328

10.62893

Using Simpson's formula, we get


I

.5

^^^^(2.87709 + 4x

12.02328

0.5

+ 2x 10.62893)==^.

72.22807-

1.2038.

Ch. VI. The Definite Integral

306

6.7.3. The river is 26


sive depths of the river

0.9

0.3

1.7

wide. The table below shows the succes-

measured across
10

3.4

2.8

2.1

12

its

14

3.3 3.0

section at steps of 2 m:

18

16

3.5 2.9

Here x denotes the distance from one bank and y


metres). Knowing that the
1.3 m/sec, determine the flowrate per second

depth

ing

(in

20

22

24

26

1.7

1.2

0.8

0.6

the correspondrate of flow is


of the water in the
y

mean

river.

By

Solution.
tion

formula

the trapezoidal

the area

of the cross-sec-

26

= J#dx2 y (0.3 + 0.6) + 0.9+

1.7

+ 2.1+2.8 + 3.4 +

+ 3.3 + 3.0 + 3.5 + 2.9 +

1.7+1.2 + 0.8

55.5 (m 2 ).

Hence,

Q = 55.5 x

1.3

72

(m 3 /sec).

impossible to estimate the error accurately in this case. Some


methods of estimation enable us to indicate approximately
2
hence, the
the order of the error. The error in 5 is about 3
3
/sec.
error in Q is about 4
It

is

indirect

Compute

6.7.4.

the following integrals:

^y?-dx

(a)

accurate

to three

decimal places, using Simpson's

j
n
4

formula;
i

(b)

^e-* dx accurate

to three decimal places,

by

the trapezoidal

formula.
6.7.5.

By Simpson's

formula, approximate the integral


1

.36

1.05

/ (*)

Ax*


if

the integrand

307

defined by the following table:

is

1.05

1.10

1.15

1.20

1.25

1.30

1.35

2.36

2.50

2.74

3.04

3.46

3.98

4.6

fix)

Additional Problems

6.8.

Additional Problems

6.8.
6.8.1.

Given the function

at

0<*<1,
l<x<2,

at

2<x<3.

lx

f(x) =

at

(2 xf

Check directly that the function


X

F(x)
is

and that its derivative


and is equal to f(x).

continuous on the interval

interior point

6.8.2.

[0, 3]
interval exists

of this

Show

f(x)

on the

6.8.3.

Can one

on the interval
6.8.4.

line

x In x

interval

assert that
b]

[a,

then

if

at

at

x=\

is

function is absolutely integrable


integrable on this interval?

tangent to the graph of the function

point

x=a

forms an angle -j with

angle

the point x

at

the

axis

of

(x)
J f"

dx

= b.

if

f" {x)

is

continuous function.

6.8.5.

Prove that

(x)

dx

= E(x) {E M ~ + E (x) [x E (x)].


{)

/ (x) at

the

and

an

abscissas

Evaluate

each

1].

[0,

it

r\

=\
^

integrable

at

the function

that

is

= \f(t)dt

Ch

308

The Definite Integral

VI.

ji

dx

f*

Given the

6.8.6.

integral
te

1+

=-

COS 2 X

Make

sure

that

the fun-

ct ions

fc

(xy

}^2cosx

= 7r=-arc cos
1

'

Vr + cos

z?
and r 29
i

m = -7=^
>^2
/

tan ^
tan^

j.

arc

1^2

are antiderivatives for the integrand. Is it possible to use both antiderivatives for computing the definite integral by the NewtonLeibniz formula? If not, which of the antiderivatives can be used?

For

6.8.7.

f (x)

magnitude y = y

filled?

at x

=x

At what value

6.8.8.

Show

(Cauchy's problem).

of

the equality

is

^e 2x dx=e 2 ^(b

a) ful-

that

Investigate

6.8.9.

an antiderivative which attains the given

find such

function

the

defined by the

definite

integral

6.8.10.

Show

that the inequalities


1

0.692

<

are valid.

6.8.1L With the aid

<)

show that

af the

inequality

inequality,

1.096

6.8.13.
are given

Assume

^x

(q^Lx^L

\<^dx<$
smx^x ^-(x^O) and theSchwarz-

6.8.12. Using the inequality

Bunyakovsky

x^s'm x^

show

<

that

V xs\nxdx<

1.111.

that integrable functions p x (x), p 2 (x) p 3 (x), p A (x)


[a, b], the function p (x) is non-negative,

on the interval

and the functions p 2

6.8.

(x),

p3

Ps

309

Additional Problems

(x),

satisfy the

{x)

p4

(x)<Pt (x)<P

inequality

(*)

Prove that

p3

(x)

(x)

dx^\p

(x)

dx^

(x)

px

(x) pi (x) dx.


I p,
a

6.8.14. Let the function f (x) be positive

on the

interval

[a,

b].

Prove that the expression


h

reaches

the

only

value

least

f (x)

if

constant on this interval.

is

6.8.15. Prove that


ji

farctanx^l

^
/

is

2 Jf sin

6.8.16. Prove that one of the antiderivatives of an even function


an odd function, and any antiderivative of an odd function is

an even function.
6.8.17.

Prove that

if

f (x)

period 7\ then the integral

is

a continuous periodic function with

a+T

/=

depend on

f(x)dx does not

a.

v(x) and their derivatives


6.8.18. Prove that if u = u(x), v
through order n are continuous on the interval [a, b], then
b

$
a

UV {n)

dx=[uV Kn -

1)

U V
,

{n

-2

>+.

+(l) n - 1 U in -

l)

v]

\a

+
b

+ ( 1)"

u {n >vdx.

Chapter

7
APPLICATIONS
OF THE DEFINITE INTEGRAL

7.1.

Computing the Limits of Sums with

the Aid of Definite Integrals


to compute the limit of a sum when the
increases unlimitedly. In some cases such limits can be found with the aid of the definite integral if it is possible to transform the given sum into an integral sum.
It

is

number

often

necessary

summands

of

2
1

For instance, considering


division of the interval [0,
for

the

points

into

1]

each continuous function f(x)

7.1.1.

n equal parts

as points of

length

of

A*=

we have

Compute

n
jt

UI1J
im
n

-*

ji

sin

2ji

h sin

(n
v

+ sin
,


n
1)}

Jil
.

Co

Solution. The numbers in brackets


s\nx at the points
function f{x)

values of the

represent the

2ji

ji

subdividing the interval


Therefore,
will

be

interval

[0,

ter

we add

if

the

ji]

[0,

{n

intorc equal parts of

summand sin^ =
sum for the function

the

integral

1)ji
length

to our

\x = ^.

sum, the

f(x) = s\nx

ji].

definition, the limit of such an integral sum as n > oo


sin x from
to n:
definite integral of the function / (x)

By

ji
n f
sin
lim n
n
\

lat-

on the
is

the

\1

sin

2ji

4-

sin

1
'

(nl)n.
n

hsin
1

nn\
n

7\

=j
o

sin

xdx =

cos

a:

2.


7.1.2.

Computing

7.1.

Compute the

lim ( r

+
'

V~4n 2 -2 2

sum

Solution. Transform the


1

311

limit

V~4n 2 -l

Sums

the Limits of

in parentheses in the following

way:

.+-=1
r

\ 4n i -

'W- V -(4)'

+ ..-+

The

sum

obtained

on the interval

Y==^

The

the

is

limit of this

of this function

lim ( r

sum

to

.-

for

the

subdivided

1]

[0,

as

from

sum

integral

oo

is

equal

to

into

the

function

f (x)

n equal parts.
definite integral

1:

=1
7.1.3.

lim JL
AZ

-* CD

= arc sin

-77

+/^+/HT5+/5T5+---

+ /-+3fcr,]

expression in the following way:

given

The obtained sum

il

Compute

Solution. Transform the

011

H l+

^e

the

is

interval

^^
[0,

=
J

sum

integral
3];

therefore,

V T^

the

function

/ (x)

by definition,

+ /--+s5j=d) -

dx = ^(l

for

x)~

Y dx = 2VT+x = 42 = 2.

312

Ch. VII. Applications of the Definite Integral

Using the definite

7.1.4.

(C)

lim

compute the following

integral,

limits:

^V*+V}:.+V'n.

^(l + cosg + cosg + ...+coseLzi)

(d)K ra

7.1.5.

Compute

4= limi/n!

the limit

n-.cc

Solution. Let us take logarithms


In

= lim In

The expression

J^JiL ^

in brackets

Consequently,

li

In

is

In #dA;

In

n + ln n +

7.2.

+
1

In

for the

integral

and lim

n->-cc

sum

the integral

= (A;ln x x)

= g"

1
.

Finding Average Values of a Function

The average value

of

/ (x)

over the interval

[a,

b]

is

the

number

li=~f(x)dx.
%

root
j

The square

[/ (x)]
b __ a j

of the average value of the

square ol the function is called the root mean square (rms)


function f (x) over [a, b].
7.2.1.

Find the average value

the interval

Solution.

[0,

jli

the function

of

1].

In this

case

j/
c

xax =

o~~

of

f(x)=i/ x

the

over

7.2. Finding Average Values

Find the average values

7.2.2.

= sin

(x)

(b)

/(*)=?qrr

over

j^b =
^
a
2

[0.

2 ]-

function

f (x)

of all

consists

= 2y = 2 V~x

finding the average value of

in

vertical chords of the

a^x^2a.

over the interval

The problem

Solution.

the

of the functions:

Determine the average length

hyperbola

313

Function

x over [0, 2n]\

(a)

7.2.3.

of a

over the interval

[a,

2a]:

2a
1

C b
a

2b
'

K^^-f

a2

ln(*

Vx^a^ =b[2\^3-\n(2 + V3)].

7.2 .4. Find the average ordinate of the sinusoid


the interval [0, ji].

= sinx

over

Solution:

ji

u
^

=
jt

[ sin

xdx

jt

- 0.637.

- cos x

ji

ji

Rewrite the obtained result

\i-

Using the geometric

= 2

in the following

meaning

ji

C
\

way:

sin

of the

x dx.

definite

integral,

that the area of the rectangle


with the altitude
&

ii
^

we can say
and the base

ji

equals the area of a figure bounded by a half-wave of the sinusoid y=smx,


and by the #-axis.
ji

O^ix^n,

Find

7.2.5.
circle

x2

7.2.6.

+y =
2

average

the

length of all positive ordinates of the

1.

Show

that the average value of the function f (x)> contithe interval [a, b], is the limit of the arithmetic mean
values of this function taken over equal intervals of the

nuous on
of

the

argument

x.

Solution. Subdivide the interval

points x ( ^a-\-i

- (/=0,

1,

2,

[a,

b]

into n equal parts by the

n).

Applications of the Definite Integral

Ch. VII.

314

Form
n points

mean

the arithmetic
of division x
/(*o)

x ly

of the values of the

xn _

..,

+ /(*i)+-..

Vf/r)

This

mean may be

function f(x) at

represented in the following form:


n~

Vn= ^f(X
l

i)

Ax i>

where

AX:-

The

sum

latter

the

is

sum

integral

for the function f(x),

the-

refore
n

V>n = ~a Hm
l

lim

(*d Ax ~b~^a

\ f

x )dx

\l,

which completes the solution.


7.2.7.
to

10

the average value of pressure (p m ) varying from 2


the pressure p and the volume v are related as follows:

Find

atm

if

_3_

pv

160.

Solution. As p varies from 2 to 10 atm, v traverses the interval

[4j/4, 4 j/T00];

hence
j/7oo

pm =

Jf

4(j/l00-j/4)

160u"

'

do

3
4

320

/4
3

4(^/100-^4)

J/ 4

3-

4.32 atm.

^20(^/10+^/2)

7.2.8. In hydraulics there is Bazin's formula expressing the velocity v of water flowing in a wide rectangular channel as a function
of the depth h at which the point under consideration is situated

below the open surface,


v

= v -20V HL (A)

2
,

the velocity on the open surface,


is the depth of
is
the channel, L its slope.
Find the average velocity v m of flow in the cross-section of the

where v

channel.

We

Solution.

7.2.

315

Finding Average Values of a Function

have

Determine the average value of the electromotive force


over one period, i.e. over the time from =0 to t
T if
electromotive force is computed by the formula
7.2.9.

Em

the value

sin-^-

duration of the period in seconds, the amplitude


value) of the electromotive force corresponding to

T is the
maximum

where
(the

=0.257. The

2nt

fraction

called the phase.

is

Solution.

e *=t)

s,n

*=ra

- cos "f-Jo

=0

Thus, the

average value of the electromotive force over one pe-

riod equals zero.


7.2.10. Each of the two vertical poles OA and CD is equipped
with an electric lamp of luminous intensity i fixed at a height h.
The distance between the poles is d. Find the average illumination
of the straight line OC connecting the bases of the poles.

7.2.11. Find the average value of the square of the electromotive


force

(E 2 ) m

the

over

from

interval

to

=T

~2

see Prob-

lem 7.2.9).
Solution. Since

E =E

sin ~y~

we have
{

j ^o J

2nt
sin'

dt=rEl

C
\

- Q0S
Y^-dt^
T

t-

4jt

7.2.12.
[0,

oo),

If

then

a
its

on an

function f (x) is defined


average value will be
b

\i= lim

~
\

f(x)dx

-T- Sin

4nt
-TfT-

infinite

~ _ o
~ 2

'

o"

interval

316

Ch. VII.

Applications of the Definite Integral

this limit exists. Find the average power consumption of an


if
alternating-current circuit if the current intensity / and voltage u
are expressed by the following formulas, respectively:

=/
u=u
/

cos

(cot

cos

(cot

+ a);
+a+

cp),

where cp is the constant phase shift of the voltage as compared


with the current intensity (the parameters co and a will not enter
into the expression for the average power).

Solution.

The average power consumption


T

wm =

lim

C/ cos((o/-f a) u cos

(cot

+a

-\-

w) dt

Taking into consideration that

= y [cos (a + P) + cos (a P)]

cos a cos P

we

will get

wm =

lim

Iim

|^o.

Hence,

it

+ 2a +

[cos (2a>/

sin(2cor

+ 2a + cp)-sin
why

clear

is

+ cos

cp)

so

2a

cp]

dt

+ +
cp)

cos

much importance

is

^|

attached to the

quantity coscp in electrical engineering.


7.2.13. Find the average value
indicated intervals:
(a)

(b)
(c)

f(x)=2x 2 +
/(*)=- 7

over

over
f

'

2x + 3
f(x) =

7.2.14.

A body

[0,

23

3*

res a velocity v l

that

fa lling

of

the function

f (x)

over the

1];

over

= \^2gs^

jli

[0, 2].

to the ground from a state


on covering a vertical path

the average velocity v m over this path

is

of rest acqui-

s=s v Show

equal to -y-

7.2.15. The cross-section of the trough has the form of a parabolic segment with a base a and depth h. Find the average depth
of the trough.

7.2.16. Find the average value

over time interval from

to

l m of

(see

alternating current intensity

Problem

7.2.12).

317

7.3. Computing Areas in Rectangular Coordinates

Prove that

7.2.17.
ellipse p:

8 COS

eccentricity,

p=\

where

a,

are the semi-axes and e

is

(p

equal to

is

average value of the focal radius of an

the

b.

the segment AB of length a a point P is taken at a


distance x from the end-point A. Show that the average value of
the areas of the rectangles constructed on the segments AP and

On

7.2.18.

a2

PB

equal to

is

-g-

7.2.19. Find the average value of the function


f(1 X ')=
'

over the interval


to

~,

is

the

yj. Check

^0,

value

cos2 *
sin 2 #

+ 4 cos

directly that this average, equal

of the function

f (x)

for a certain

x=

lying

within the indicated interval.

Computing Areas

7.3.

Rectangular Coordinates

figure is bounded by the straight lines x=a, x =


and the curves y=y 1 {x) y=y 2 {x), provided y 1 {x)^
(a^x^b), then its area is computed by the formula

plane

If

in

= b(a<b)

^y

2 (

x)

s=
In certain cases

the

x = b) can degenerate

left

into

[ftW- yi
boundary
a

point

(x)] dx.

x=a
of

(or the right boundary


intersection of the curves

(a)

Fig. 65

=y

and y
y 2 (x). Then a and b are found as the abscissas
points of intersection of the indicated curves (Fig. 65, a, b).
7.3.1. Compute the area of the figure bounded by the straight
2x , y
2xx2 (Fig. 66).
2 and the curves y
lines x
0, x

(x)

of the

318

Ch. VII. Applications of the Definite Integral

= 2x x

Solution. Since the maximum of the function r/


tained at the point x=l and is equal to 1, and
x
on the interval [0, 2], we have

is

at-

the function

y=2 ^l

s-ii 2 "-< 2 *-*>i-i:-("-?)i:-^-f


o

7.3.2.

x=

Compute

the area of the

2y\ x=l 3y 2

(Fig.

figure

bounded by the parabolas

67).

Fig. 66

Fig. 67

Solution. Solving the system of equations

find

y2

x = 2y 2

x =l3y\

the ordinates of the points of intersection of the curves y x


Since 1
then we have
2y 2 for
3y 2

S=

= 1,

[(l-m-(-W)]dy=2(y-^)\l = l

Find the area of the


between the
parabola x 2 =\y and the witch
7.3.3.

figure contained

of

Agnesi y

= -p-^

(see

Fig.

^y

U = J-

68).

Solution. Find the abscissas of the points A and C of


intersection of the curves. For
this purpose eliminate y

from

<Zf

-z
Fig. 68

Computing Areas

7.3.

in

Rectangular Coordinates

319

the system of equations

y=
x2

whence
a:

The
#2

+4

As

of

equation are

this

the

points

from

seen

is

+ 4x 32=0.

or x*
4

roots

real

= 2.

the

figure,

x2

A.

xt =

and

on the interval

+4

[
2, 2]. (It is also possible to ascertain this by directly computing
the values of these functions at any point inside the interval, for

instance, at

jc

= 0.)

Consequently,
2

dx

+4
7.3.4.

= x +l
2

= ^4 arc tan-|-

Find the area of the


and the straight line x

\2J

-V

bounded by the parabola

figure

+ y = 3.

7.3.5. Compute the area of the figure which lies in the first qua2
drant inside the circle x2
3a2 and is bounded by the paraboy
las

x2

= 2ay

and y 2

+ =
= 2ax(a > 0)

(Fig.

69).

Solution. Find the abscissa of the poA of intersection of the parabola


2
2
2
circle
x2
y -3a
y =2ax and the
Eliminating y from the system of equations
int

we obtain x2

= 2ax,
+ 2ax 3a = 0,
y

whence we

Fig. 69

get the only positive root: x A


a. Analogously, we find the abscissa of the point

+ =

2
x2
3a2 and the parabola
y
Thus, the sought-for area is equal to

circle

x2

of

= 2ay\

intersection of the
r

xD

= a\

aVl
Sr=

[y*(x)yi(x)]dx
J

where y x

(x)

=^

= \V2,ax
r

V 3a

for

0<x<a,

for

a<x^aV2.

2^.

320

Ch. VII.

By the

Applications of the Definite Integral

additivity property of the integral

aV2

V
2

2a

it
3

6a
s-

^~2

3a

a2

+
1

\^

x + arc sin -f7t


6a Jo
a |^3
2

^~2

6^2

3a 2

arcsm |A
1

arcsin f=
drLMn
}A
3

Here we make use


arc sin

of

arcsin

a\

-tt

the trigonometric formula:

arc sin p = arc sin (a K p Vla


2

(aP

> 0)

transforming

for

arcsin

= arcsin ^

|/^y arcsin

= arcsin y
7.3.6.

Compute

V3 V3

the area of the figure lying in the first quadrant


and bounded by the curves y 2
4x
2
x2
5.
\y and x2
y

Compute the

7.3.7.
figure

x+l,
(Fig.

-7

70).

Solution.

y=f(x)-.

x+
cos

near trapezoid

5=

is

interval

if

1 <x<0,

a:

if

0<x<-|

y
1,

The area

of the curvili-

equal to

f(x)dx= ^ {x+ l)dx + ^cosxdx =

7.3.8.

The function

continuous on the

+ sinx

Find the area of the segment of the curve y l


2 is the chord determining the segment.

the line x

the

area of

bounded by the lines y =


y = cosx and the A:-axis

Tt/2

Fig. 70

is

+ =

= x ~x
6

if

Computing Areas

7.3.

in

Rectangular Coordinates

321

2
x 2 (x- -1) it follows that
equality
y
In other words, the
or
either x
2
2
consists
x3
domain of definition of the implicit function y
and the interval [1, oo). In computing the area
of the point x
the isolated point (0, 0) does not play any role, therefore, the

From the
* (;e 1)>0, therefore
Solution.

interval of integration

Passing over

to

that the segment

is

is

2]

[1,

(see

Fig. 71).

representation

below by the curve y

= xVxl.

Hence,

S_ J
Make

= .xVx 1, we see
= xVx and

y
bounded abov e by the curve y

explicit

[x\fjT^l

(x\fl^l)]dx = 2lx)/'x\dx.

the substitution
X

x l-/
dx=2t

2
,

dt,
1

Then
1

S = 4j(/'+l)/<tf
7.3.9.

= 4[ + f);-M5.

Determine the area


by two branches
= x 3 and the straight

bounded

(y x) 2

of the figure
of

the

line

curve

x=l.

Fig. 71
Solution. Note first of all that y, as an
function of x, is defined only for
x^O; the left side of the equation is always non-negative.

implicit

Now

the equations of two branches of the curve y = x xVx,


we
x + xVx^x xV x, and
Since x^O, we have
y =x+x V
find

x.

therefore

S=^(x + xV~xx +xV~x)dx = 2 *V~x dx7.3.10.

Compute the

y*=x(x-\) 2

area

enclosed

by

the

loop

of

the

curve

Solution. The domain of definition of the implicit function y is


oo. Since the equation of the curve contathe interval
ins y to the second power, the curve is symmetrical about the

0^jc< +

Ch

322

VII.

Applications of the Definite Integral

The positive branch y

x-axis.

X)

_V

\x-

(x)

given by the equation

is

- y-{x _
<j

The common points of the symmetrical branches y


= y (x) must lie on the x-axis. But y (x) = *
and at x 2 = 1.
at x l =
l

Consequently,

and y

the

loop

is

>

l)f

formed by the curves y

= Y x(lx), 0<*<1

(see

Fig.

72),

and y 2

(x)
\

{x)

=
_

only

= \f x(\ x)

the area enclosed

being

S=2jV*(l x)dx = 2 ^x x
2

Find

7.3.11.
y*

the

area

enclosed

by

the

)dx = ^.
loop

of

curve

the

= {x-\)(x-2)K
2
y =x(x-1)

Fig.

Find

7.3.12.

y= x

Fig.73

72

the area of the figure


the line tangent to

2x + 3,

bounded by
it

at

the parabola
the point /W(2, 5)

and the y-axls.


Solution. The equation of the tangent at the point M(2 5)
5
6(x 2) or y = 7 >x. Since the branches
has the form y
of the parabola are directed downward, the parabola lies below the
6a:> x2 2x 3 on the interval [0,2] (Fig. 73).
tangent, i. e.

+ =
7

Hence,
2

S= j*[7 6x ( x 2x +
l

3)]

dx = J(x2

4x + 4)dx = -|

7.3.

Computing Areas

in

323

Rectangular Coordinates

y=x

2
2x+2,
7.3.13. Find the area bounded by the parabola
the line tangent to it at the point 7W(3, 5) and the axis of ordinates.

We

7.3.14.

take on the ellipse

+ -1

(a>b)

M(x,

y) lying in the first quadrant.


that the sector of the ellipse bounded
axis and the focal radius drawn to the point
a point

Show

by

So =

ab
-77-

arc cos

With the aid of this result


area of the entire ellipse.

xa

its

semi-major

has an area

deduce a formula

computing the

for

y
1

Solution.

Sqmao

M(x,y)

Fig. 75

Fig.

74

We

have (Fig.

^aom#+ S MABM
a

74):

S^ 0MB y

Smabm = ydx = V~=T* dt


X

= ~[t

|/^=7~2 + 2 arc sin

= [-xVtf^Z+cP (y- arc sin


Since

arc sin a = arc cosa


>MABM

2a

-x

,
1

we obtain

Va x + a
2

arc cos

~
j

Hence

Sqmao

$ao mb + Smabm + ~y arc cos

324

Ch. VII. Applications of the Definite Integral

At x = 0, the sector becomes a quarter

4 ^ellipse
9

A
--arccos0

"

and consequently, S ellipse = nab. At


^circle

of the ellipse,

ab
ab
n
=
y=T
a = b we get the

ab

i.e.

area of a circle

bounded

7.3.15. Find the area

and the straight

line

by

the parabolas y

= 4x #=g2

= 2.

Solution. In this case it is advisable to integrate with respect


to y and take advantage of the symmetry of the figure (see Fig. 75).
Therefore, solving the equations of the parabolas for x, we have:

x=
By symmetry

of

the

figure

about

S 0AB0

equal to the doubled area

the (/-axis the area sought

S = 2S 0AB0 = 2

is

V~y-\ V~y ) dy= 5 J Vy dy = ^p-

j (3

From an arbitrary point M(x, y) of the curve y x m


perpendiculars
and ML(x>0) are dropped onto the
coordinate axes. What part of the area of the rectangle ON
does
the area
(Fig. 76) constitute?
7.3.16.

MN

> 0)

(m

ML

ONMO

Fig. 77

7.3.17. Prove that the areas S


5X
x-axis and half-waves of the curve y
,

geometric progression with the

S2 53
,

bounded by the
form a

= e~ x s\n$x, x^0
common ratio q = e P.

Solution. The curve of Fig. 77 intersects


at the points where sinpx=0, whence

the

Ox

Xn

nil

u,
/-v

l,

q
z,

...

positive

semi-axis

Computing Areas

7.3.

in

Rectangular Coordinates

325

The function y = e" ax

sin fix is positive in the intervals (x 2ki x 2k+1 )


and negative in (x 2k+l x2k+2 ) i.e. the sign of the function in the
1)". Therefore
interval (xn xn+1 ) coincides with that of the number (
y

>

(n+

Sn =

(n+

1) jt

\dx={\) n

l)jt

e- x s\nfixdx.

nn
3

But the indefinite integral


e~ ax sin

fix

dx

is

=a

equal to
a

+fl

a ^

+ P cos P*) + ^-

Consequently,
(n+

[^^(asinpx + pcosPx)
:

tP

1) ji

13

[g-a(+l)/pp (_l)+i_gaJV3p (1)"]

* 2

+P

Hence
-a(/z +

I )

jt/3

which completes the proof.


7.3.18. Find the areas enclosed between the circle x 2

+4(/ 11=0 and


Solution.

Rewriting

(x-l)
y

the parabola

+ (*/+2)

the
2

we

=_t6,

= -{x-\f-2 V 3 + 2.

the centre of the


the point C(l,
2)
and the radius of the circle equals
The axis of the parabola coin4.
cides with the straight linex=l
and its vertex lies at the point
fl(l,

2,

at

lies

-2J/T)

(Fig. 78).

The area S ABDFA of the smaller


figure is found by the formula
u

SaBDFA

(*/par

i/circle)

- 2x+

equations of the curves,

Consequently,

circle

+y

y= x + 2x+l2\/ ~3.
r

dXf
Fig. 78

have:

326

Ch. VII. Applications of the Definite Integral

where xA and x D are determined from the system


( + 2) = 16,_
=
-(x-\)
-2V3 +4,
y+
xD = 3.

1,

equations

(*-l) +

whence x A

of

Hence,
3

Sabdfa

t(-^

+ 2x + - 2,/T + 2 + VT6-(*-l)*)]
1

>

~V

16 (at 1)* +
= [-j + x* + (3-2V3) x + x
+ y arc sin ~1J = 8 |/T + 2 J/"T2 + 16 arc sin 1 =
*

32

.
,

The area

of the second figure is easy to determine.


Note. The computation of the integral can be simplified by using
the shift x
1
2 and taking advantage of the evenness of the
integrand.

r/

7.3.19. Compute the area


6
1
x2 and the #-axis.

7.3.20.

Compute

bounded by the curves y =

x=^y

7.3.23.
4 (y 2 ~~x 2 )

7.3.24.

by the curve y 2 =(\

Compute the

area

enclosed

by

area

of

the

and the straight

Compute the

loop

of

the

2
)

curve

area

of

the
line

the

bounded by the curve


x-\-y=l.
figure

figure

enclosed by the curve

2
).

Compute the

+ x y = 0.
2

the

== 0.

Compute

= x (l x
7.3.26.

x3

the hyper-

enclosed

+]/' y

7.3.25.

cut off by

area

= 8.

Compute the

Yx
2

=\

+x

4)

+l.

7.3.21. Compute the area of the portions


2
2
from the ellipse x 2
bola x 1
3y
4y
7.3.22.

the area enclosed between the parabolas

x = y2

(x

area

enclosed

by

the

loop

of the

curve

7.3.27. Compute the area bounded by the axis of ordinates and


2
the curve x
y).
y (l

Compute the

area bounded by the curve y


x*
2x 3
the axis of abscissas and two ordinates corresponding to
the points of minimum of the function y(x).
7.3.28.

+ # + 3,
2

7.4. Areas with Parametrically Represented Boundaries

7.4.

327

Computing Areas with Parametrically Represented

Boundaries
If

the boundary of a figure

the

of

figure

= -^y(t)x'(t)dt;

represented by parametric equations

= x(t),

Art

then the area


mulas:

is

y=y(t)

is

by one

evaluated

S=

S=x(t)y'(t)dt;
a

of the

three for

| J (xy' -yx') dt,


a

where a and

P are the values of the parameter / corresponding


respectively to the beginning and the end of the traversal of the
contour in the positive direction (the figure remains on the left).

Compute the

7.4.1.

area enclosed by the ellipse

x = acos/,
Solution. Here
xy'

convenient

is

it

(0^t^2n).

y=bs'mt

~yx'=a cos txb cos /

compute

to

first

+ b sin

= ab.

x a sin t

Hence
2,1

=yj

2ji

x y'

yx')dt = -j^

abdt

= nab.

o
2

Find the area enclosed by the astroid

7.4.2.

_2_

T
(^)

+("f)

=1-

Solution. Let us write the equation of the astroid in parametric


asin 3 /,
Here it is also convea cos 3 /, */
form: a;
nient to evaluate first

xy'

yx'

=a

(cos 3

0^/^2k.

3 sin 2

cos

+ sin

3 cos 2

sin

=
= 3a
t)

cos 2 /sin 2

/.

Hence,
2ji

2jt

S = -i

(xy'yx')dt=ja*

7.4.3.

cycloid x

Find

the

area

= a(t sin/),

Solution.

sin 2 2tdt

= \a

n.

of

the

region

= a(l cos/)

bounded by an arc
and the #-axis.

of

the

Here the contour consists of an arc of the cycloid


a segment of the #-axis (0^x^2na). Let us

(0</<2ji) and

apply the formula

S = ^yx'dt.

Ch. VII. Applications of the Definite Integral

328

Since on the segment of the x-axis we have r/ = 0, it only remains to compute the integral (taking into account the direction
of a boundary traversal):
2ji

(I cos*)a(l cost)dt=a

(1 cos t) 2 dt =

2n

=a

2cos/

+ 4"0+ cos2 ')]

dt

= 3na

2
.

7.4.4.
jc

Compute the

= asin/,

area

of

the

region

enclosed

by the curve

= bs\n2t.

Solution. When constructing the curve one should bear in mind


that it is symmetrical about the axes of coordinates. Indeed, if we
t
for t the variable x remains unchanged, while y
substitute ji
only changes its sign; consequently, the curve is symmetrical about
for / the variable y remains
t
the ;t-axis. When substituting n
unchanged, and x only changes its sign, which means that the
curve is symmetrical about the y-axis.

x=asint

Fig. 79

Furthermore, since

common

period

2jt,

the

it

is

functions
sufficient

bsm2t have a
a sin/; y
x
to confine ourselves to the fol-

lowing interval of variation of the parameter: 0^.t^l2n.


From the equations of the curve it readily follows that the variables x and y simultaneously retain non-negative values only

when

the parameter

varies

on the

interval

u
'

0</<y we

-2

therefore at

obtain the portion of the curve situated in the

first

quadrant. The curve is shown in Fig. 79.


As is seen from the figure, it is sufficient to evaluate the area
enclosed by one loop of the curve corresponding to the variation

7.4. Areas with Parametrically Represented Boundaries


of the

parameter

5=2

oo
J

from

to

n and then to double the

ji

ji

yx' dt

7.4.5.

=2

329
result

ji

^ b sin 2t

x a cos / dt

= \ab ^ cos

smt

dt

Find the area

of the

by the

enclosed

region

loop of the

curve

0=4(6-0-

%=4(6-0;

Solution. Locate the points of self-intersection of the curve. Both


functions x(t) and y(t) are defined throughout the entire number
oo
scale
t <oo.
At the point of self-intersection the values of the abscissa (and
ordinate) coincide at different values of the parameter. Since

<

x=

y(/

3)

2
,

to take

tion y

(t)

of the

parameter

the abscissas coincide at

= 3

on one and the same value

^=
= 0,

point

M(x,

and
t

at

the

t2

point

we have x(t x )

>

(0,

changes from

(the figure

remains on the

of the

3t x

= -^left).

= x(t^) = 0,

and

0) is the only point of selfto 6, the points of the curve


/
varies from
to 3, the

the first quadrant. As


y) describes the lower part

in

indicated interval x(t) and y(t)

must

= 3.

increase,

tion x(t) begins to decrease, while y (t)


gure 80 shows the traversal of the curve
ing

same values

Fig. 81

i.e.

When

intersection.

are found

the

Fig. 80

Thus, at
= y(t 2 )

at

For the func-

iid^(3^X) = -^^(3 + X)

the equality

/,

be fulfilled for X=^=0, whence X

y(t l )

1 'k.

loop,

since

in the

and then the func-

keeps increasing. Ficorresponding to increas-

still

330

Ch. VII. Applications of the Definite Integral

In computing the area


nient to use the formula

enclosed by the

loop sought

(6-0^Ul
Find the area

by the

enclosed

is

conve-

27
'

24

7.4.6.

it

loop of the curve: x

2
\

= tJCompute

7.4.7.

a cos

(1

+cos

the

/);

by

enclosed
cos /).

area

= as\n

(1

the

cardioid:

Solution. Since a;(/) and */(/) are periodic functions, it is suffito consider the interval [
ji, ji]. The curve is symmetrical
t for / the value of the
about the x-axis, since on substituting
variable x remains unchanged, while y only changes its sign, and
cient

i/>0
As
1

varies from
/
changes from

to

and

1,

the

decreases from x

to

u= _ 1

= 0. We

ji.

the function u

jt

can show

= cost

= au (1 + u) =a
=
to x\

= 2a

The curve

shown

is

traversal as

its

in
t

(^u -f

y)

ordinate y increases on the

the

that

decreases from

and then increases

and decreases on the interval

interval

tion of

to

abscissa

first

a:|

to

as
/

^y</^Ji^.

Fig. 81, the arrow indicating the direcincreases.

Consequently,
ji

S=

ji

(xy'

yx') dt = a

+ cos

( 1

t)

dt

= j na\

-JT

7.4.8.

Compute

cos/, y

7.4.9.

Compute

(a)

x=t \,

(b)

x = 2t

(c)

x=t

7.4.10.

2
;

area

region

enclosed

by

the

curve

t;

= 2t P\
2

y=L (3 - n
= b sin

Compute
x

the

of

1.

the areas enclosed by the loops of the curves:

Compute

x = a cos/; y
7.4.11.

the

= bs\n

the area
cos 2 /.

of

the

by the curve

region enclosed

the area

= cos

/;

enclosed by

y~

^-sin

/;

the evolute
c

=a*b

of
2
.

the ellipse

7.5.

The Area

Polar Coordinates

in

331

7.5. The Area of a Curvilinear Sector in Polar


Coordinates

In polar coordinates the area of a sector bounded by the curve


and the rays q) x
and (p 2
P is expressed by the integral

= p(cp)

=a

S = -j jp 2
7.5.1.

Find the area

of the region situated

bounded by the
a and x = a.
y=x

and

(<P)d<P-

parabola

= 4ax

in the

and

the

quadrant

first

straight

lines

Solution. Let us introduce a polar system of coordinates by


placing the pole at the focus F of the parabola and directing the
polar
axis
the positive
direction
in
along the #-axis. Then the equation of

parabola

the

be p^

will

coscp

whe-

p is the parameter of the parabola.


In this case p
2a, and the focus F has
the coordinates (a, 0). Hence, the equation of the parabola will acquire the
re

form

=i

zzz^z
cos cp

lines

will

straight
cp

(Fig. 82).

and

,
'

those

become

cp

the

of

and

Therefore,
Jl

4a 2

SpARP
FABF

"

(f COS(p) 2

dcp

= 2d

dcp

4sin^"

JT

Changing the variable:


Z

cot tt

= z,

2 sin 2 (cp/2)

dz,

ji/4 cot (k/8)

k/2

we obtain
cot (Jt/8)

S FABF = a*
or,

(i

+z .) dz=aBfl .(cot-J + lcoti-l-l)

taking into account that cot

>FABF

~=

4*

"t^fj"/
sin (jx/4)

= 2fl"(l+y^2

= +K 2
1

Ch. VII. Applications of the Definite Integral

332

Compute

7.5.2.

the area of the region enclosed by

= l+cos(p;
= acosq).

(a) the cardioid p

(b) the

curve p

7.5.3. Find the area of the regions bounded by the curve


a and situated outside
2acos3(p and the arcs of the circle p
p
the circle.

function

Solution. Since the

= 2acos3q)

r = -y
2ji

has a period

the radius vector describes three equal loops of the curve as q) van and jx. Permissible values for q) are those at
between

ries

which cos3(p^0, whence

_ +

^ <(p<

Consequently, one of the loops

and

= 0i lf 2

{k

described as

is

and the other two loops as


5ji

-g-

ively (Fig.

belonging
the

cp
a

is

it

7ji

-g-

and

3ji

and

respect-

83). Cutting out the parts,


to the circle p=a, we get

whose area

figure

arly,

between

varies

q)

varies between

and between

...)

equal

is

the

to

sought. Cletriple area

Smlnm*
Let us find the polar coordinates of
the points of intersection
and N.
For this purpose solve the equation

2a cos

3cp

= a,

(k=0)

~q"

is

specified

As

is

&MLNM

by the polar angle

q) x

=y

y only

and

Fig. 83

e cos 3(p

i .

the

roots

Between

and

are found. Thus, the point


,

and the point

M by

q) 2

=y

seen from the figure,

~ SoMLNO

^OMNO

~
JT/9

JT/9

=|

4a 2 cos 2 3(pdq)

7.5.4. Compute the


p=3|/"2 acoscp and p

Solution.

The

first

a2

dcp

a2

^+ iLij

-Jt/9

-!t/9

through the pole p

area

circle

= 0,

of

the

figure

bounded

by

the

circle

= 3asincp.
lies

in the right

touching the vertical

half-plane and
line.

The second

passes
circle

The Area

7.5.

in Polar Coordinates

333

in the upper half-plane and passes through the pole


touching the horizontal line. Consequently, the pole is a point
of intersection of the circles. The other point of intersection of the

situated

is

= 0,

circles

is

found from the equation 3|/2 acoscp

B (arc tan 1^2

a|/6). As

is

= 3a sin

q),

whence

seen from Fig. 84, the sought-for area

V3'

Fig. 85

Fig. 84

is

and

sum

equal to the

the areas of the circular

cf

segments_CMBO

OCBO

arc

BAO

the

first

The
adjoining each other along the ray (p=arctan^2
by the end-point of the polar radius p of
is described
.

arctan}/2

circle for

and the arc


second

of the

polar radius p

end-point of the

^arctan^2

<(p^y,

OCB

circle for

by the

O^cp^

Therefore

cos 2

J
arc tan

cp

dq>

=4 a2 (t arctan ^ 2

>

arc tan

S 0CB0 -=Y a2

sin 2 q)dq)

= -|-a

^arc tan|/2

^p)

Hence,

Soaho + Socbo

= 2.25a

(ji arc tan

]/YVT).

7.5.5. Find the area of the figure cut out by the circle p
from the cardioid p = l-f-cos(p (Fig. 85).

Solution. Let
ves.

To

us

whence

find the points of

first

this end solve the

<Pi

=y

>

^2

=n

cp,
q),

sin

cp

intersection of these cur-

system

= |/~3 sin
p == + cos
p

= ]f3

O^cp^rc,

Ch. VII. Applications of the Definite Integral

334

The sought-for area is the sum of two areas: one is a circular


segment, the other a segment of the cardioid; the segments adjoin
each other along the ray

<P

=y

The

BAO

arc

described by the

is

end-point of the polar radius p of the cardioid as the polar angle

changes from

to

ji,

and the arc

OGB

by

the end-point of

cp

the

O^cp^y.

polar radius p of the circle for

Therefore

= -I

3 sin 2

cp

dcp

+-I
J

(1

+ cos

cp)

dcp

= 4(<P-^)|f + |(cp + 2sincp + f

sin 2<f

= T (lt
7.5.6.

Find

the

= a(l coscp)

area

and the

of

the

circle p

bounded

figure

by

]/T).

the

cardioid

= a.

7.5.7. Find the area of the region enclosed by the loop


y*
3axy.
folium of Dtscartes x 9

Solution. Let

us

the

of

pass over tj polar coordinates using the usual


psincp. Then
.
formulas x
p cosq), #
the equation of the curve is:

3
3
p (cos

cp

+ sin

cp)

= 3ap

sin

cp

cos

cp,

or
3a sin
cos 3

cp

cos

(p

+ sin

cp

cp

3a sin

and

and secondly, p

The

at

cp

2cp)

that,

3ji

oo as
latter

cp

means

and
that

folium of Descartes has an asymptote, whose equation y =


a can be found in the usual way in rectangular coordinates.
x
Consequently, the loop of the folium of Descartes is described

the

as

equation

cp

cp

this

at

firstly,

Fig. 86

from

follows

It

2cp

+ cos(p) (2 sin

(sincp

cp

(see

changes
Fig, 86).

from

to

and

is

situated

in

the

first

quadrant

Area

$ 7.5. The

Thus, the sought-for area

C 9a 2 cos 2

Taking advantage

the

of
les

loop

^from

cp

cp

(\

= ~,

to

cp

sin 2

+ sin

335

cp

cp) 2

symmetry about the

curve's

the

about the ray

i.e.

(cos 3

of

Polar Coordinates

equal to

is

>OAO

y = x,

in

we can compute the


an(J then double

P=-j)

bisector

area of
it.

half

This enab-

us to apply the substitution

tan

cp

= 2,

dcp

cos 2

(p

which gives
4

^oao
Still

cos 2

~* y "

(cos 3

cp

sin 2

cp+sin 3

cp

2a(p

_ qya

~""

cp)

'

tfz

Jd + z

3 2
)

new substitution
z

+ = v,
3z dz = da,
2

leads to the

integral
dv

Compute

7.5.8.

the

area

of

3
2

a2

the region enclosed by one loop

of

the curves:
(a)

p=acos2cp;

(b)

p=<2sin2(p.

Compute the

7.5.9.

area enclosed by the portion of the


lying inside the circle p=acoscp.

=a (1 cos

7.5.10. Compute the area


=osin(pcos 2 (p, a > 0.

=a cos

7.5.11.

cp)

Compute

3
(|

(a

> 0).

the

area

of

the

region

enclosed by

of

the

region

enclosed

cardioid

the curve

by the curve

336

Ch. VII. Applications of the Definite Integral

Compute the area

7.5.12.

= yL=^

of the portion

bounded by the

of the figure

flying inside the circle

Bernoulli's lemniscate

=aVcos 2cp.
7.5.13. Passing over to polar coordinates, compute
2 3
the region enclosed by the curve (x 2
Aa 2 x 2 y 2
y )

the area of

7.5.14. Passing over to polar coordinates, evaluate the


2
y*
the region enclosed by the curve x*
a 2 (x 2
y ).

7.6.

Computing

The volume

the

of a solid

area of

Volume of a Solid
is

expressed by the integral


b

V = \S{x)dx
a

where S(x)

is the area of the section of the solid by a plane


perpendicular to the x-axis at the point with abscissa x\ a and b are
the left and right boundaries of variation of x. The function S(x)
is
supposed to be known and continuously changing as x varies
between a and b.
The volume V x of a solid generated by revolution about the
x-axis of the curvilinear trapezoid bounded by the curve y
f(x)
(f(x)^0) the x-axis and the straight lines x a and x b (a<b)
is expressed by the integral

Vx

ji

\y

dx.

The volume Vx of a solid obtained by revolving about the x-axis


bounded by the curves y = y 1 (x) and y = y 2 (x) [0
y x (x)

<

the figure

^^W]

an d the straight

lines

x=a

=b

is

<

expressed by the

integral

V x = n\{y\y\)dx.
a
is
represented parametrically or in polar coordinates,
the appropriate change of the variable should be made in the above
formulas.

If

the curve

7.6.1.

Find the volume

of the ellipsoid

7.6. Computing the Volume of a Solid

Solution.

an

The section

the

of

337

by the plane # = const

ellipsoid

is

ellipse (Fig. 87)

+
y

with semi-axes b

cy

Hence

the

area

of

the

section (see Problem 7.4.1)

r i^

S(x)=TLbJ/

Therefore the volume

V = ^ nbc

^ = nbc[\

xc

of the ellipsoid

and we have Inhere

is

^ dx = nbc x x

3 a2

In the particular case a

=b=c

-a

7.6.2.

x2

Compute

+y +z =
2

7.6.3.

the

at right

common

Solution.
(Fig.

88).

abc.

the ellipsoid turns into a sphere,

the volume of the solid spherical segment of two


2 and x
3 from the sphere
the planes x

16.

The axes

intersect

4
o

Fig. 88

by

out

cut

= 4 na*

Fig. 87

bases

(a:

of

two

angles.

identical cylinders with bases of radius a


of the solid constituting

Find the volume

portion of the two cylinders.

Take the axes of the cylinders to be the y- and e-axis


The solid OABCD constitutes one-eighth of the sought-

for solid.

Let

us

cut

this solid

a distance x from

0.

In

by

the

plane perpendicular to the #-axis at


we get a square EFKL with

section

Ch. VII. Applications

333

side

EF =

Va
2

2
,

therefore

of

5 (x)

the Definite Integral

=a x
2

and V

=8

(a

dx

**)

of
all chords (parallel to one and the same direction)
radius R symmetrical parabolic segments of the same
altitude h are constructed. The planes of the segments are perpendicular to the plane of the circle.
Find the volume of the solid thus obtained (Fig. 89).

7.6.4.

On

circle

of

Fig. 89

Fig.

Solution. First compute the area of the parabolic segment with


base a and altitude h. If we arrange the axes of coordinates as
indicated in Fig. 90, then the equation of the parabola will be
h.
y = ax 2
Determine the parameter a. Substituting the coordinates

point

0^,

b(Jj,

equation

we

get

the parabola

of

a_

a= ^;

= aj-M, whence
y=-a^x' + h> and
1

is

the

of

the

hence the

desired

area

o_

S = 2 ydx

= 2^^x +h}dx=jah.
2

Now find the volume of the solid. If the axes of coordinates are
arranged as indicated in Fig. 89, then in the section of the solid
by a plane perpendicular to the x-axis at the point with abscissa x
we obtain

a parabolic

= 2/ R x
2

2
.

segment

area

S^~ah

where

= 2y =

Hence,
R

S(x)

of

= j\^W^ handV=[S(x)dx^jh^W^ dx = ^
2

-R

nhR\

The plane

7.6.5.
the fixed

Computing

7.6.

diameter

of

the

moving

Volume

339

of a Solid

triangle remains perpendicular lo


the base of the triangle
vertex lies on a straight line pa-

of a circle of radius a:

of the circle, and its


the fixed diameter at a distance h from the plane of the
circle.
Find the volume of the solid generated by the movement
of this triangle from one end of the diameter to the other.
7.6.6. Compute the volume of the solid generated by revolving
about the x-axis the area bounded by the axes of coordinates and
_L
_L
_L
2
the parabola x 2 -\-y 2
is

chord

rallel

to

=a

'

Solution. Let us find the points of intersection of the curve and


the axes of coordinates: at x
x
a. Thus, we
a, at y
y
have the interval of integration [0, a],

From

we

the equation of the parabola

get

y=

\a

- -Y
x
2

there-

fore
a

=n

y dx

=n

_1_

\a

_1

dx

=n

^a2

4a

Aa

_3_
2

_1_

+ 6ax-

+x jdx= ^ na*.
2

7.6.7. The figure bounded by an arc of the sinusoid y


s\nx
the axis of ordinates and the straight line
\
revolves about
the y-ax\s (Fig. 91).
y

y^

>
1/2
Fig. 92

Fie. 91

Compute
Solution.
interval [0,

the volume

The
1].

of the solid

inverse function x

of revolution thus generated.

= arcsinj/

is

Therefore

V=

2
j\^ x dy

= n^ (arc sin y)

dy.

considered on the

340

Ch. VII. Applications of the Definite Integral

smy = t.

Apply the substitution arc

Hence
y

= sin
dy = cost dt,
t

*/

n/2

And

V = jt

so,

cost dt. Integrating by parts, we get

y=n

n2 &)
4

Compute the volume

of the solid generated by revolving


the figure bounded by the parabola */
2
0.25x 2
and the straight line 5x
14
0.
8^/

7.6.8.

about the

.xr-axis

+ =

ABCA

The solid is obtained by revolving the area


92) about the #-axis. To find the abscissas of the points
and B solve the system of equations:
Solution.

(Fig.

= -x + 2,
2

5x8y+H== 0.
Whence x

i4

x B =2. In our case y x

=y;

= (5/8)jc + 7/4.

(x)

=^ +2
2

and y 2

(x)

Hence,
,

lra(T*+ 7 )

-(T + 2 )']^'

891

JC,

280

Jl.

1/2

7.6.9.

about the
y*.
8x

Compute
(/-axis

Solution

It

is

the volume of the solid generated by revolving


the figure bounded by the parabolas y
x 2 and

obvious that x 2

from the origin

(y)

= \^y^x

(y)

= ^-

on the

in-

the coordinates to the point of intersection of the parabolas (Fig. 93). Let us find the ordinates of the
points of intersection of the parabolas by excluding x from the sy-

terval

stem

of

of

equations
y
{ y

We

obtain y 1

= 0,

#2 =

4.

= x\

= 8x.

Hence,

V=n

J [y

fj) dy=* 24

jt.

Compute the volume of the solid torus. The torus is a


generated by revolving a circle of radius a about an axis
lying in its plane at a distance b from the centre (b^a). (A tire,
for example, has the form of the torus.)
7.6.10.

solid

7.6.

Computing

the

Volume

of a Solid

341

7.6.11. Compute the volume of the solid obtained by revolving


about the x-axis the figure bounded by two branches of the curve
(yx) 2 = x 3 and the straight line x=l.

Find the

7.6.12.

volume

of

the

solid

= 2a the figure bounded


line x = a (Fig. 94).

about the line y

and the straight

generated by revolving
by the parabola y 2 = 4ax

a
-2a

C\ B

-x

-x

0'

Fig. 93

Fig. 94

we transfer the origin of coordinates into the point


retaining the direction of the axes, then in the new
system of coordinates the equation
of the parabola will be
Solution.

2a)

O'(0,

If

(y'

2a)

= 4ax.

Hence y 2 = 2a + \Z~4ax (for the curve OAB), and y 1 = 2a


]^4ax (for
the curve OCD). The sought-for vo-

lume

is

equal to

V = n(yl yl)dx = n[(2a +


o

+2^) ~ (2a 2 V^f] dx = f na\


2

Fig. 95

7.6.13. Find the volume of the solid generated by revolving about


the x-axis the figure enclosed by the astroid: x
a cos 3 1\ y
as\n 3 t.
Solution. The sought-for volume V is equal to double the volume

obtained by revolving the figure


'

OAB

= 2n ^ y

(Fig. 95). Therefore,

dx.

Ch. VII. Applications of the Definite Integral

342

Change the variable


x = a cos 3 /,
dx =
3a cos 2

= a sin

t
/

sin

dt,
jx/2

Hence,

V = 2n^

a 2 sin 6

/ (

3a cos

ts'mt) dt

it

IT

= 6jxa

J
-

Using

the

integrals,
t,

from

formula

we
c

sin 7 /d/

Problem 6.6.9

sin 9

dt

computing the above

for

get
o

/ 6

32

2 \

7.6.14. Compute the volume of the solid generated by revolving


sin/), y
cos/) about the
a(t
arc of the cycloid x
a(l

one

Ar-axis

7.6.15.

Compute the volume

of the solid

obtained by revolving

= a (1 +cos

about the polar axis the cardioid p

cp)

shown

in Fig. 81.

Solution. The sought-for volume represents the difference between


and OKLO
the volumes generated by revolving the figures
about the x-axis (which is the polar axis at the same time).
As in the preceding problem, let us pass over to the parametric
representation of the curve with the polar angle cp as the parameter:

MNKLO

p cos = a cos
= p sin =a sin
cp

(p
cp

(p

( 1

(1

+ cos
+ coscp).

cp),

equals 2a (the value


obvious that the abscissa of the point
cp=0), the abscissa of the point K being the minimum of
a(\ +cos cp)cosq).
the function x
It

is

of

x at

Let us find this

minimum:
dx

^ = a sin
At

^=0

we obtain

<p,

=0;

xM

= 2a

(p 2

cp

(1

+2 coscp) =0,

at

(p 2

ji.

= |n,

xK

Computing

7.6.

Volume

the

Hence, the sought-for volume

343

a Solid

of

equal to

is

2a

V z=n ^ y\dx

y\ dx.

tc

Changing the variable x = acosq)(l

+ coscp),
X

( 1

get

*
1

= a + cos
sin
=
dx
a sin (1 +2 cos
2

r/

we

a/4

q),

(p)

cp) dcp,

cp

2jx/3

>

a/4

2n/3

2a

ji

Thus,
o

V=

ji

a2

(1

+cosqp) 2 sin 2

qp

a sin

cp

+ 2 coscp)] dcp

(1

jt

a2

(l

+coscp) 2 sin 2

(p

a sin

(p

(1

+ 2 cos

(p)] d(p

==

= jia

sin 3

q)

(1

+coscp) 2

+ 2cosq))d(p =

(l

o
i

= jia

+w) 2

(l

+2u)du

= ^ na

= cosqp).

(w

7.6.16.
(a)

nes

the

Compute the volume


hyperboloid
1

of

of

the solid bounded by:

+ ^r ^ =

one sheet

an d

the

pla-

and z=l;

2=4

(b) the parabolic cylinder


a\
and the plane x

2
*/

the

planes

of

coordinates

(c)

the elliptic

paraboloid

= ^ +^

and the plane

=k

(k

>

0).

7.6.17. A wedge is cut off from a right circular cylinder of radius


a by a plane passing through the diameter of the cylinder base and
inclined at an angle a to the base. Find the volume of the wedge.
7.6.18.
the figure
(a)

(b)

Compute the volume

of

the

bounded by the following

xy = 4,
y = 2x

x=

\,

x'\ y =

= 4,

solid

generated

lines:

about the x-axis;


*/
about the #-axis;

by revolving

344

Ch. VII. Applications of the Definite Integral

3
y = x y = 0, x=2 about the (/-axis;
=
s\nx (one wave), y =
(d) y
about the #-axis;
2
2
(e) x
about the (/-axis;
# = 4,
a) 2 = ax x = 0, y = 2a about the x-axis.
(0 (y

(c)

y=2
y

7.6.19. Find the volume of the solid


ax ~7*
about the x-axis.
curve yu 2
1

obtained by

revolving the

7.6.20. Compute the volume of the solid generated by revolving


the #-axis the figure bounded by the lines y^s'wiX and

about
y

= 2 x.

7.6.21. Compute the volume of the solid generated by revolving


about the x-axis the curvilinear trapezoid bounded by the catenary

= ^-(^e

=c

(c>

-\-e

= acosh-^

and the straight

lines

xt

=c

x2

0).

7.6.22. Compute the volume of the solid generated by revolving


cosxand
about the #-axis the figure bounded by the cosine line (/

the

parabola

7.6.23.

about the
the

y^^*

2
-

Compute

the volume of the solid generated by revolving


2
and
#-axis the figure bounded by the circle x 2
y

+ =l

parabola

= ^x.
=
=

x 3 take two points A and B, whose


7.6.24. On the curve y
2 respectively.
and b
1
abscissas are
Find the volume of the solid generated by revolving the curvilinear trapezoid aABb about the #-axis.

a=

acost\ y
7.6.25. An arc of the evolute of the ellipse x
situated in the first quadrant revolves about the x-axis
Find the volume of the solid thus generated.
7.6.26.

Compute

the

volume

of the solid

= bs\nt

generated by revolving

the region enclosed by the loop of the curve x

= at

2
>

= a[t

^j

about the #-axis.


7.6.27. Compute the volumes of the solids generated by revolv2 2
2
a 2 (x 2
the region enclosed by the lemniscate (x 2
y )
)
about the x- and (/-axes.

ing

7.6.28. Compute the volume of the solid generated by revolving


acos 2 cp about the polar axis.
the region enclosed by the curve p

7.7. Arc Length of a Plane Curve in Rectang. Coord's

345

7.7. The Arc Length of a Plane Curve in Rectangular


Coordinates

If
a plane curve is given by the equation y
y(x) and the
derivative y' (x) is continuous, then the length of an arc of this
curve is expressed by the integral

l=lVl+y'

dx

where a and b are the abscissas

Compute

7.7.1.

(0,

and

0)

8) (Fig.

(4,

the arc of the


the points

length of

the

parabola y 2

semicubical

=x

end-points of the given arc.

of the

between

96).

The function y(x) is defined for


Since the given points lie in the first qu-

Solution.

x^O.

adrant, y

=x

_3_
2

3
= ~Vx

Hence,

and l/l+*/' 2

j/ 1+-

Consequently,
Fig. 96
3

=A (10 KTo-i).

7.7.2.

=x

Compute

the

length

by the straight

7.7.3.

Compute the

line

Solution. Since y'

arc

cut

from the curve

off

= -g.

arc length

the points with the abscissas

the

of

of

x=0,

= tan x,

y=\ncos>x between

the curve

x=^.

then |/l

+ y' = \f\ -f tan x = sec x.


2

Hence,
n
4

3n
= lntan-g
.

'

sec

J
7.7.4.

x1

=a

to

Compute
x2

=b

(b

xdx= In tan

+\

= In Jx
e

the

> a).

arc

length

of

the

curve

from
|

346

Ch. VII. Applications of the Definite Integral

Find the arc length

7.7.5.

y=l

the points with the ordinates

Solution. Here
variable; then

it

the curve

of

convenient

is

and y

x=-^y 2 y

between

= 2.

adopt y as

to

\ny

independent

the

'

*'-U-w

^ T+ 7 1

and

/(T^+i7=^+i-

Hence,
2

= J ^jqp^i dy = I (i y + 4) dy =T + T ln 2
_2_

Find the length

7.7.6.

the astroid x

of

_2_

_2_

+y =a

As is known, the astroid is symmetrical about the axes


of coordinates and the bisectors of the coordinate angles. Therefore,
sufficient to compute the arc length of the astroid between
it is
the bisector y = x and the x-axis and multiply the result by 8.
Solution.

In the
at

first

=\a

quadrant y

T
~3 ~\
3
x

and y

at

= a, y=x

Further,

'

a ^-jc T

-|) x~^~ = x~~* [aJ-x^)

'

and

Consequently,
/

a
c

=8

)
a
2

jVo/e.

the

first

If

we compute

_J_

.L

Jx =

6a.

3/
/f

the

arc

length

of

an astroid situated

quadrant, we get the integral


a

i
x

dx

whose integrand increases

infinitely as

().

in

Arc Length

7.7.

Plane Curve in Rectang. Coord's

of a

347

OABCO

7.7.7. Compute the length of the path


consisting of por2
tions of the curves y 2
2x 3 and x 2
20 (Fig. 97).
y

Solution.

It

sufficient

is

symmetry

since by

+ =

of the

compute the

to

figure

arc lengths

and

about the

x-axis

Solving the system of equations

+ y = 20,
2

x2
y

we

the point

find

Find

= 2x\

4).

(2,

fa. Here

J/2*,

V\+y'*= )/ i+ J*.

Hence,
2

Since on the circle of radius ^20 /;


*ab xs
responding to the central angle arc ta
tan 2,

*' ie

en th

of

an arc cor-

Finally

(b)

*/

= A(ioi/TO l)+4/5

Compute

= y

(a)

= ]/20arc tan 2.

we have
I

7.7.8.

XB

arc tan 2.

the arc length of the curve:

cut off by the x-axis;

= ln(2cosx)

between the adjacent points

of intersection

with

the x-axis.

2
2
between the adjacent
(c) 3y =x(x
l)
with the j-axis (half the loop length).

7.7.9.

Compute the
y

points

arc length of the curve

= Ux V~#-\ In (x + KF=T)]

between

and x

= a+l.

of

intersection

Ch. VII. Applications of the Definite Integral

348

7.7.10. Find the arc length of the path consisting of portions of


the curves x2
(y+\y and y 4.

7.8. The Arc Length of a Curve Represented


Parametrically

is given by the equations in the parametric form x = x(t),


and the derivatives x' (t), y' (t) are continuous on the interval [t lt t 2 ], then the arc length of the curve is expressed by the

If

a curve

= y(t)

integral

= [Vx

,2

+ y'

{t)

{t)dt,

t x and t 2 are the values of the parameter


t 2 ).
the end-points of the arc (t 1

where

corresponding to

<

7.8.1.

Compute

the

= a(cos/ + /sinO.

length

arc

involute of a circle x

the

of

= a(sini tcost)

from

Solution. Differentiating with respect to

= at cos

x\

whence Vx'

+ y' = at.
2

t=2n.

to

we obtain

/,

= at sin

y't

t,

t,

Hence,

2jt

at dt

2n

=2an

2
.

7.8.2.

Find the length


x

7.8.3.

one arc

= a(t sin

Compute the

Solution.

of

t),

length of

of the cycloid:

= a(l cos/).
the astroid: x = acos
y

Differentiating with respect to

= asm*t.

we obtain

t,

= 3a cos ts'mt;
y' = 3a sin
cost.
2

x't

Hence
J/"*;*

Since

the

+ ^; = \f9a

sin 2

cos 2

= 3a

sin

cos

= ^2

sin 2t

|.

|sin2/| has a period

function

2
/

= 4xy jsin2/df

=6a.

Note.

If

we

forget that

the root and put

Yx

-\-

y' 2

we have

to take the arithmetic value of

= 3asmt cos

we

shall obtain the

wrong

Arc Length

7.8.

of a

Curve

Rep res' d Parametric ally

349

since

result,

2ji

= ySin

sin/cos/d/

Compute

7.8.4.

length

the

-0.

x=V3t

loop of the curve

the

of

2ji

y = t-t*.
Solution. Let
x(t)

us

the

find

and y(t) are defined

limits
for

all

2
,

Both functions
Since the function

integration.

of

values of

x= V 3/ ^0,
2

the curve lies in the right half-plane. Since with a


change in sign of the parameter
x(t) remains unchanged, while
y(t) changes sign, the curve is symmetrical about the #-axis. Furth-

ermore,

function

the

takes

x(t)

on one and the same value not


more than twice. Hence, it follows
that the points
of the
at

curve

of

self-intersection

on the

lie

x-axis, i.e.,

(Fig. 98).
y=
The direction in which the mo-

ving point
(x, y) runs along the
oo to oo
curve as / changes from
is

Fig.

98

indicated by the arrows.

But y

at

= 0,

= 1.

Since x

(t 2 )

=x

(t 3 )

= J^3,

the point

point of self-intersection of the curve. Conse-1 and / =1.


quently, we must integrate within the limits t 2
8
Differentiating the parametric equations of the curve with respect

(^3, 0)

to

f,

we

is

the

get x't

only

= 2V~3t,

y't=\3t\ whence

Consequently,

i(l+3t*)dt
7.8.5.

Compute

between the points


7.8.6.

Compute

= 4.
t

the
of

arc

length

of

the curve #=-g-

= 2 -j
t*

intersection with the axes of coordinates.

the arc length of the ellipse

^ + ^=1.

Solution. Let us pass over to the parametric representation of the


ellipse

x=acost,

= bs'mt

Differentiating with respect to

xt

= a sin

t\

O^t ^ 2n.

we obtain
y't

= b cos

t.

350

Ch. VII. Applications

the Definite Integral

o)

whence

Vx + y' = Va
2

where

sin 2

+b

cos 2

= a V\e

cos 2

e is the eccentrici-ty of the ellipse,

= a = -V a*a b*
c

Thus

2ji
l

The

= a \V 1

zos 2 tdt

cos 2 tdt

integral

= Aa[\f\ e
is

cos 2

df.

not taken in elementary func-

tions;
/

the

called

is

it

= Y x,

we reduce

integral of the second kind. Putting

elliptic

the integral to the standard form:


ji

ji

T
\

V\

e cos

/d/ = $ Kl

sin 2

xdT=(e),

E (e) is the notation for the so-called complete elliptic integral of the second kind.
Consequently, for the arc length ot an ellipse the formula
/
4a(e) holds good.
sina and to use the tables of vaIt is usual practice to put e
lues for the function
where

E
For instance,

if

(a)

=E

(arc sin e)

10 and 6

= 6,

=E

(e).

then

r Q0
V l0^ 6 = A0.8Q = sin 53.
8 =-

Using the table

we

find

7.8.7.

= 40

of values of elliptic integrals of the

(53)

Compute

= 40xl.2776 51.1.

the arc length of the curve

</=y(' 2 -3)

between the points of intersection with the x-axis.


7.8.8.

Find the arc length


x=a

of the cardioid:
(2 cos

cos 2t),

y = a(2s\n sin 2t).


t

second kind,

7.9. Arc Length

Find the length

7.8.9.

the closed curve

of

= 4 \^2as\nt;

351

a Curve in Polar Coordinates

of

= asm2t.

7.8.10. Find the arc length of the evolute of the ellipse

= cos

Compute

7.8.11.

^-sin

/,

y = (2
/,

On

7.8.12.

and

7.9.

cycloid
the

the

=a

2
.

+ 2/ cos/,
+ 2/ sin/

)cos/

= a (/ sin

length

coordinates, then

the

arc length

= a(\ cos/)

find the
arc of the cycloid in

first

Polar Coordinates

in

by

given

is

/);

the

of

The Arc Length of a Curve


smooth curve

If

ji.

point which divides


the ratio 1 :3.

/,

the arc length of the curve

x=(t 2 2)sin/

between

of

the equation p
p((p) in polar
the curve is expressed by the

integral:
*pi

where

cp

and

7.9.1.

Find

medes p

= aq).

(cp,

changes from

first

of

the

polar

angle

rp

at

the end-

cp 2 ).

the

of

first

turn of the spiral


Therefore

turn of the spiral ot Archi-

is

formed as the polai angle

to 2n.

2ji

2ji

=\

<

length

the

The

Solution.
cp

values

are the

cp 2

points of the arc

\fa y

+a

dy = a\ Vy +
2

dcp

=a

[n

|/"4ji

-f

1 In (2n +

Find the length of the logarithmic


a certain point (p
(p ) and a moving point
7.9.2.

spiral
(p,

p=ae m v

cp).

between

Ch. VII. Applications of the Definite Integral

352

Solution. In this case (no matter which of the magnitudes, p or


is greater!)
p
,

<p

<Po

= a|/l + m

e '*dq)

=a

the length of the logarithmic spiral


increment of the polar radius of the arc.

Find

7.9.3.

the

length

arc

cardioid

the

of

Po

proportional

is

i.e.

V +m

IP

2
.

Ap|,
to the

= a(l+coscp)

(a>0, 0<q)<2ji).

p^=

Solution. Here

asincp,

VV^+V^- K2a (l+cos


= 2a cos W^J
2

(p)

- K4a

_ 2a

>

Hence, by virtue

cos 2 (cp/2)

C os (cp/2),

jx

< < 2k.


cp

symmetry

of

2jt

= 2a

cos
|

d(p

= 4a ^ cos

dcp

Find the length of the lemniscate


the right-hand vertex corresponding to cp
7.9.4.

polar angle
Solution.

(p

If

<~

= 2a 2 cos2cp between
and any point with a

0<(p<~,
p

= 8a.

then cos2q)>0. Therefore

= a j/*2 cos 2cp;

pjp

V2

sin 2(p

j^cos
s in
t

2cp \

cos2(p 7

2cp

/"2

]^ C os2(p

Hence,

V 2sin
1

The
It

can

latter

be

integral

is

reduced to a

did of special tables.

cp

called the elliptic integral of the first kind.


form convenient for computing with the

Arc Length

7.9.

7.9.5.

Find the arc length

7.9.6.

Compute

p = asec ^q)
y^
Solution.

K*P

p^

between

is

on the interval

^(p

and

= asec^q) y^

+ p; = sec((p-y)

cp

=y

tan

=a

sec 2 ^ q)

j*

2
2
]/ l+tan ((p-f )=asec ((p-f

function sec(q)

in the

-^-j

omitted,

is

this function

[o.f

V3

4
^

dq)

length

the

of

a.

closed

p = asin y.
4

Solution. Since

the function p
r

even, the given curve

is

= asin

a period 4jx,

symmetrical about

the polar axis. Since the function sin 4

Fig. 99

has

during half the period from

increases from
its

line

p=asln

Find the

7.9.7.

curve

of

of the straight

positive.)

is

(p

segment

the

of

353

= asin y.

curve p

of the

length

the

(The sign of the modulus


since

a Curve in Polar Coordinates

of

and

to a,

symmetry

(Fig.

to

will describe half

2n the polar radius


the

curve

by virtue

- a sin

99).

Further, p^--=asin 3 (q)/4)cos(q)/4) and

Kp + P^ = Va? sin
2

if

(cp/4)

+a

sin 6 (cp/4) cos 2 (cp/4)

(cp/4),

0<(p<2jT.
Hence,
'271

= 2a

JT/2

sin

(cp/4) d(p

= 8a

7.9.8.

and p

3
j sin /d/

= ya

((p

= 4f).

Find the length

of the

curve

(p

= y(p+l/p)

between

= 4.

Solution.

d/

The

differential of the arc dl

= Kp + p; dq) = Kp
a

d(p

+ dp =

is

equal to
2

|/

(-^) ^ 1*.

p=2

354

Ch. VII. Applications of the Definite Integral

From

the equation of the curve

we

find

^=y

^2")

Hence,

4( -^) + ldP=j
1

V|(p -2+^ +
2

Find the length

7.9.9.
3

=t

^=y

and

7.9.11.

Compute the

7.10.

t0

p(p=l between

spiral

the length of the closed curve p


arc

= 2a(sinq) + cos(p).

curve

length of the

1+ c 0S(p

rom

^^y-

Area of Surface of Revolution

The area
the arc

hyperbolic

Compute

the

7.9.10.

<Pi

of

4)dp

= 3 + ^.
^

;-.(/Kf*=i(f+'"'
<Pi

of the surface generated

of

the

curve

by revolving about

= y(x) (a^x^b)

is

the x-axis
expressed by the

integral
b

P = 2n\y

VT+V dx.
2

It

is

more convenient

where

to write this integral in the form

P = 2n

^
L

ydl

the differential of the arc length.


coordinates,
is represented parametrically or in polar
then it is sufficient to change the variable in the above formula,
expressing appropriately the differential of the arc length (see 7.8
If

and

dl

is

a curve

7.9).

7.10.1. Find

the
2

_2_

area

of

the

surface

formed

by revolving the

_2_

3
a 3 about the #-axis.
astroid x 3
y
Solution. Differentiating the equation of the astroid

fx

+4*/

whence

y'=- y-

y'

= 0,

we

get

7.10. Area of Surface of Revolution

Vl+y'* =

Then,

Since
3

X"

the

355

astroid

is

sym-

\x\

metrical about the (/-axis, in computing the area of the surface we


may first assume
and then double the result. In other words,
the desired area P is equal to

x^0

P = 2x2n
Make

yV\ +

y' 2

dx = 4n

a3

the substitution

a3

x =t\
3

4" x

a3

dx = 2tdt,
a

y_

Then

P=\2na

l/3

fl

12

t*dt=^n<&

J
o

7.10.2. Find the area of the surface generated


the x-axis a closed contour
formed
2
A;
and x y 2 (Fig. 100).
by the curves #
Solution. It is easy to check that the 1
given parabolas intersect at the points
The sought-for area
1).
(0, 0) and B (1,
P P 1 P 2 where the area P x is formed
by revolving the arc OCB and P 2 by revolving the arc OAB.
Compute the area P v From the equation

by revolving about

OABCO

x = y2

we

get

= V*

and

#'

= ^y=f.
Fig.

Hence,
i

P = 2nV~x }/~
1

4
+ dx = 2n [ ^ *+

^=

4(4* +
Now compute

100

the area

P 2 We
.

P 2 = 2n

have

x2

y~x

Vl + Ax

!)"

y'
,

dx.

= 2x

|(5K5-1).
and

356

Ch. VII. Applications of the Definite Integral

The substitution # = ysinh, dx = ~ cosh


Arsinh

P^JI

dt gives

sinh 2 /cosh 2 /d/

= ^^sinh4^

Arsinh

= 9-TL -3> ln
Thus,

p=p 1+ p =

(5

1^5-0^

+ ^).

+ _qgL _^ Jtln(2+|/-5 ^
)

Compute

7.10.3.

by revolving:

the area of the surface generated


2

the portion of the curve y

(a)

= 3
(b)

line

about the

cut off by the

straight

line

(/-axis;

the portion of the curve


about the x-axis.

x=2

X
= -^,

= 4 + x,

cut

by

off

the straight

Find the surface area

7.10.4.

of the ellipsoid

(a>b).

the ellipse -^--j--p-=l about the #-axis


Solution. Solving
for
0, we get

the

equation of

formed by revolving

the

ellipse

with respect to y

y^
b

Va* x 2

'

Hence

-a

where the quantity


ellipse.

When

+a

~b

e= j/ a

=~

is

the

eccentricity

of

the

the eccentricity e tends to zero and


t

arc sin e

lim
e - o

I,

since the ellipse turns into a circle, in the limit


area of the sphere:

P = 4jia

a
.

we

get the surface

7 JO. Area

of

357

Surface of Revolution

7.10.5. Compute the area of the surface


the ellipse 4x2Jry 2
4 about the (/-axis.

by

obtained

revolving

7.10.6.

An

arc of the catenary

= ~{e a +e" a ) = a cosh

whose end-points have abscissas

and

x, respectively, revolves

about

the x-axis.

Show

P and

that the surface area

the volume

generated are related by the formula


Solution. Since y'

hence,

= sinh

we have V\

of the solid

thus

2V

P=

+ y' = cosh
2

Therefore

7.10.7. Find the area of the surface obtained by revolving a loop

= y(3a y)

curve 9ax 2

about the (/-axis.


described by a moving point as y changes
to 3a. Differentiate with respect to y both sides of the
from
equation of the curve:

of the

The loop

Solution.

whence

is

= (3a y)
a
^ ~y a~y

8axx'

xx'

2y(3ay) = 3(3a y)(a y)


Using the formula

for

computing the

area of the surface of a solid of revolution about the (/-axis,

P = 2n j xV~l+x' 2 dy=2n j

Vx + (xx')
2

dy =

3a

3a

y )dy = 3na
2

7.10.8.

Compute

the curve 8y 2
7.10.9.

=x

we have

2
.

area of the surface generated by revolving


x* about the x-axis.

the

Compute the

area

of

generated

surface

about the x-axis an arc of the curve x

2
\

y=-^(t

by revolving

3)

the points of intersection of the curve and the x-axis.

between

Ch. VII. Applications of the Definite Integral

358

Solution. Putting y

= 0,

and

= K3,

and, hence,
the curve intersects
with the x-axis at two points: (0, 0) and (3, 0). When the parameter t changes sign, the sign of the function (x)t remains unchanged, and the function y (t) changes
y,
its sign, which means that the curve
1
is
symmetrical
about the #-axis
m

^=

and

jc

= 3.

2>3

find

Whence

(Fig.

find the area of the surface

it

to confine

ourselves to
the lower portion of the curve OnB
that corresponds to the variation
the parameter between
of
and
sufficient

is

Fig.

+ 1^3.

101

Differentiating with respect

we find
y = t*-l

to
-.2t;

and the

2t3

that

101).

To

X
-1

follows

it

ty

linear element
dl

= Vx? + y? dt

(l

)dt.

Hence,

P = 2n\\y(t)\Vx' 2 + y'
t

dt

VT

vT

= 2n$

(t*3)(l
j
i"
+ )dt = ^-n
\

L "

I 1

(t*

2t* 3t)dt=3n.

7.10.10. Compute the surface area of the torus generated by revolving the circle x 2
(y b) 2 r 2 (0 r b) about the *-axis.
Solution. Let us represent the equation of the circle in parametric
form: x
rzost\ y = b
rs'mt.

Hence
x't

The

< <

desired area

= r sin

t\

y't=r cos

t.

is

2JT

P-2jiJ

(b

+ r sin

t)

)/*(

r sin

tf

+ (r cos tf dt =
= 2nr\

7.10.11.

Compute

he

are a

of

the

(b

+ r sin

surface formed

t)

dt

= 4n

br.

by revolving

the lemniscate p
a}/cos2q) about the polar axis.
Solution. Real values for p are obtained for cos2(p>0,

X^^^T^^
3

-j-ft^

ty^-j n

ri ght-hand

branch

of

the

(the left-hand branch of the

lemniscate),

lemniscate).

i.

e.

or

for
for

7.10. Area of Surface

The
d/

linear

element

359

Revolution

of

of the lemniscate

is

equal to

^l

= Kp + p^dcp= if a cos2y+( a
Y dip =,"|Acos2(p/
cos
|^cos2(p
2

'

2(p

psincp=asin(p}/cos2q).
Besides, #
The sought-for surface area P is equal to double the area of the
surface generated

by revolving the right-hand branch. Therefore


4

P = 2 x 2n

= 4o
f y dl

^^^P^ =

2a (2

- j/2

).

cos 2cp

7.10.12. Compute the area of the surface formed by revolving


2
2
2
about the straight line x
y = a the quarter of the circle x
y =a
between A (a, 0) and B (0, a).
(x, y),
from the moving point
Solution. Find the distance
2
2
a:
to
straight
x
a
the
line
lying on the circle x 2
y
y

MN =
since

the

for

MN

+ =

+ =

\x+ ya 2 x 2 a\

:+ Va? x* c

V2

V2

points

the

of

circle

that

lie

in

the

first

quadrant

x-\-y^a. Further,
dl

= yi + y'2 dx= if + (
i

Ydx=-^=

Hence,

P=

'

2n

x+ Va* x

2 -

a dx

V^ x

V~2

V2na [- V'c
7.10.13.
the surface

Compute
formed

x2

+ x aarcsin-

= (*-*)

the

area of
revolving
lemniscate p

by

=
one branch of the
r cos2q> about the straight line
=a\
_
it

From the triangle OMN


we find the distance MN

Solution.
(Fig. 102)
of

an

arbitrary

axis of revolution

point
q)

=x

MN = p sin

of

the

Fig.

right-hand

102

branch

(p^

=a|/cos2cpsin

qp^

from

the

Ch. VII. Applications of the Definite Integral

360

then
j1

dl

a dw
r
V cos 2cp

Jl/4

P = 2n j aKcos2q5

Therefore

sin

q>)

y=?= = 2na

2
.

-Jt/4

Compute

7.10.14.

area

the

of

formed by revolving

surface

the

about the x-ax\s the arc of the curve #

= xy

between x

=2

and

= 2.
Compute the

7.10.15.

one half-wave

of the

area of the surface generated by revolving


s\nx about the x-axis.

curve y

Compute

the area of the surface generated by revolving


4ay between the points
the arc of the parabola x 2
3a.
of intersection of the curve and the straight line y

7.10.16.

about the

(/-axis

7.10.17. Find the area cf the surface formed by revolving about


e'sin/; y^^cost between
the ;t-axis the arc of the curve x

*=0

and

*=y
Compute

7.10.18.

the area

of the

surface obtained by revolving

the points of

its

the arc of

the curve x

= -^-\
t

about the #-axis

=4

^
t

between

intersection with the axes of coordinates.

7.10.19. Compute the area of the surface generated by revolving


2asinq) about the polar axis.
the curve p

Compute

7.10.20.

the

area

of

the surface

formed

by revolving

about the x-axis the cardioid

x
y

= a (2 cos cos 2t),


= a(2s'mt sin 2t).

7.11.

Geometrical Applications of the Definite Integral

7.11.1. Given: the cycloid (Fig.

x = a{t sin

t)\

103)

y^a(l cos/);

0< ^ 2n.
/

Compute:
(a) the areas of the surfaces formed by revolving the arc OBA
about the x- and (/-axes;
(b) the volumes of the solids generated by revolving the figure OB AO
about the */-axis and the axis BC\
(c) the area
of the surface generated by revolving the arc BA
about the axis BC\

volume

(d) the

ODBEABO

the solid generated by revolving the figure


touching the figure at the
tangent line

of

DE

about the

vertex B\
(e) the area of the surface formed by
revolving the arc of the cycloid [see

item

361

Integral

7.11. Geometrical Applic's of the Definite

jj

(d)J.

Solution, (a) When


the jc-axis the arc
face of area

revolving about
generates a sur-

OBA

c
103

Fig.

231

P x = 2n

dl

= 2n j

( 1

cos

t)

y dt =

2a sin

8a

When

revolving about

'ji

sin 3

OB A

(/-axis the arc

the

64jta 3

y dt

generates a sur-

face of area
TT

Py = 2n

xdl

+ 4to

= 4na

(t

sin

t)

sin

y dt +

sin0sinyd/ = 4jia

sin

(/

siny dt

16jt 2

2
.

Jl

(b)

When

revolving about

the y-axis the

figure

OBAO

generates

volume

a solid of

oo

2a

Vy

= n^

2a

(x\

x\)dy = n

where x = x 1

2a

jc|

rf^

Jt

x2

dj/,

x2 (y) is
(y) is the equation of the curve BA, and x
the equation of the curve OB.
Making the substitution y a(l
cos/), take into consideration
that for the first integral t varies between 2k and ji, and for the
and n. Consequently,
second integral between

TT

Vy = n

Jt

a2

(t

sin

t)

asm tdt n

a2

sin

t)

sin tdt

(t

asm tdt =

2jt

= na

(/

sin

t)

2n
-

= jia

J
_2t

For computing
the figure

OBAO

sin/d/

t{\ cos2t)dt+

2jt

the

volume

3
$ sin /d/

=6ji 3 a 3

2ji

of

about the axis

the

BC

it

solid obtained
is

convenient

by revolving
first

to trans-

Ch. VII. Applications of the Definite Integral

362

the origin into the point C, which


tions in the new system of coordinates

fer

= a(t n sin

x'

y'

t)\

yields the following equa-

= a(l cos/).

Taking into account only the arc BA, we get


2a

V=n

jt,

x'

dy' = na

^ (t

sin
ji

t)

\ (z

+ sin z)

sin

sin z dz

dt.

2ji

Putting

n =

we obtain

z>

JT

V = na

\(z

+ sin z)

na

zdz

sin

=^
(c)

Making

we

the above-indicated shift of the origin,

= 2a sin y

dl

dt

(9jx*

16).

get

= 2a sin y dt.

Therefore
2a

P= ^2nxdl = 4na

2 j*

sin/) sin y dt =

(/

jt

2n
ji

= 4na

(z+

sin z) cos

J*

y dz = 4 (^2n

(d) Transferring the origin into


direction of the r/-axis, we get
x'

Putting

n =

= a(t

changing from

sin

/),

B and

point

=a (1 +cos

changing the

/).

we have

z,

x'

ji

the

= a (z + sin 2),

ji

to

ji

for

y'

=a

the arc

( 1

cos

OB A.

2),

Hence

ji

V=n

a3

{l cos z) 2 (\

+ cos z) dz = ri a
l

3
.

-ji
ji

ji

(e)

P = 2n

J
-jt

ydl

= 4na

j (1 cosz)cosyd2

=y

jta

2
.

-ji

the volume of the solid bounded by the surfaces


and x2 + y 2 = 2x.
Solution. The first surface is a parabolic cylinder with generata:) in the
rices parallel to the r/-axis and the directrix z 2 = 8(2
plane xOz, and the second is a circular cylinder with generatrices
7.11.2. Find

Z2

= 8(2 x)

7.11. Geometrical Applies


parallel to the z-axis

and the

directrix

+ y = 2x
2

x2

363

Integral

of the Definite

in the plane xOy.


2

The volume V

computed by the formula V

is

=^

S(x)dx. S(x)

re-

presents the area of a triangle whose base


tude to 2z\

S(x)

= 2yx2z = 4 V2xx

is

equal to 2y and

alti-

1/8(2*).

Hence,
2

V=

Vx

x) 8

(2

2
(

x)

dx = 4

\/"8

$ (2

*) Vx

= 4/8 (|2 J/*

dx

=
256

--?-!/**)

:
'

15

7.11.3. Prove that if the figure S is bounded by a simple convex contour and is situated between the ordinates y x and y 2 (Fig. 104),
then the volume of the solid ge.y
nerated
by revolving this figure
about the #-axis can be expressed
by the formula
t

b/

y*

V = 2n

&
^

yh dy

yt

where

h=x
x

=x

left

{y)x

(y) 9

being the equation of the


1 (y)
portion of the contour and

Si

x = x2

(y) that of the right portion.


Solution. Let the generating figure S be bounded by
a simple

Fig.

104

convex contour and contained between the ordinates y t and y 2


Subdivide the interval [y ly y 2 ] into parts and pass through the
.

points of division straight lines parallel to the axis of revolution,


thus cutting the figure 5 into horizontal strips. Single out one
strip and replace it by the rectangle ABCD, whose lower base is
equal to the chord
h specified by the ordinate y its altitude
AB being equal to Ay. The solid generated by revolving the rectangle
about the #-axis is a hollow cylinder whose volume may
be approximately taken for the element of volume

AD =

ABCD

AV

n (y + Ay) h ny h = 2nyAyh + nh (Ay)


2

2
.

Rejecting the infinitesimal of the second order with respect to Ay,


we get the principal part or the differential of volume

dV
Knowing

= 2n yh dy.

the differential of the

volume, we get the volume proper

Ch. VII. Applications of the Definite Integral

364

through integration:

= 2n

Thus, we obtain one more

yhdy.

formula

computing the volume

for

of

the solid of revolution.

2x 2 -\-3,
7.11.4. The planar region bounded by the parabola y
revolves about the
and
the *-axis and the verticals
(/-axis. Compute the volume of the solid of revolution thus generated.
Solution. Divide the area of the figure into elementary strips by
straight lines parallel to the (/-axis. The volume AV of the elemen-

x=0

x=\

tary cylinder generated by revolving one strip

AV = n (x+ Ax)
where Ax

is

is

nx y = 2n xy Ax + ny (Ax)
2

2
y

the width of the strip.

Neglecting the infinitesimal of the second order with respect to Ax,

we

volume

get the differential of the desired

dV = 2n xydx.
Hence
i

V=

2n xydx

= 2n

+ 3) dx = 4n.

x (2x2

7.11.5.

Compute the area

portion of

of the

x2

-f-

=t

ax

the cylinder surface


situated inside the

sphere

x2

+ y + z =a
2

2
.

Solution. The generatrices of the


cylinder are parallel to the 2-axis,

the

serving

^
a

circle

shows

as
a

jY+y

^
directrix

quarter

=^

105
the sought(Fig.

for surface).

Subdivide the
circle

shown

in

portion of the
Fig. 105 into

small arcs AL The generatrices


passing through the points of division cut the cylinder surface

y
Fig.

of

105

into

higher order are neglected,

strips.

the area of the strip

If

infinitesimals

ABCD

of

is

equal to

point

D, then

CD- AL
If

and cp
and

= acoscp

the

are

CD

]/

a2

polar

coordinates

p = asinq),
2

and

of

A/

the

= a.A(p,

whence we

7.11. Geometrical Applies


find the

element

of the Definite

365

Integral

of area:

dP

=a

sin cpdcp.

Hence,

P=4

a 2 sin

cp

dcp ==

4a 2

7.11.6. Find the area of the surface cut off from a right circular
cylinder by a plane passing through the diameter of the base and
inclined at an angle of 45 to
the base.
Solution. Let the cylinder axis
be the e-axis, and the given diaC/
meter the x-axis. Then the equation of the cylindrical surface
q
2
2
will be x
y =a\ and that of
the plane forming an angle of 45

with the coordinate


will be y = z.

The area
row

strip

the

of

ABCD

xOy

plane

y^^^?^

infinitely nar-

Fig.

(see Fig. 106) will

106

be dP
zdl (accurate to infinitesimals of a higher order), where dl is the
tary arc of the circumference of the base.
Introducing polar coordinates, we get
z

Hence dP

=a

= y = a sin

s\nq)dq)

dl

(p;

the

length of

elemen-

= a dqp.

and

jt

P = a2
7.11.7.

The axes

of

sect at right angles.

ting the part

sin

qp

=a

dy

2
[

cos

cp]*

= 2a

2
.

two circular cylinders with equal bases

inter-

Compute the

common

surface area of the solid constituto both cylinders.

Compute the volume of the solid generated by revolving


the y-axis the figure bounded by the parabola x 2
1, the
axis of abscissas and the straight lines
and x=l.
7.11.8.

= y

about

x~0

7.11.9. Find

the area

Ax 2 + 2Bxy+Cy 2

l(8

of the

= AC

>

ellipse
0;

given

C> 0).

by

Solution. Solving the equation with respect to y,

_ Bx VCdx
where the values

of

1
.

we

_ Bx+ VC bx*

x must satisfy the inequality

C 6jk 2 >0.

the
get

equation

Ch. VII. Applications of the Definite Integral

366

we obtain

Solving this inequality,

Consequently, the sought-for area

/?

the limits of integration:

is

equal to

Vt

C_

7.11.10. Find the areas of the figures bounded by the curves represented parametrically:
(a)

x=2t t

= 2t

3
;

7.11.11. Find the areas of the figures bounded by the curves given
in polar coordinates:
(a)

= asin3(p

(b) p

(c)

cos

(a

three-leaved rose);

cp

= 3sinq)

and

= K3cosq).

IP riirvp
7.11.12. Find the arc length of the
curve

by the straight line x =


7.11.13. Find the length

(0,

7.11.14.

Al,
2

0);

Compute

= 4^ (2 x)

cut off

1.

of the arc

OA

of

the curve

where

/y 2

.In-i
a In

the arc length of the curve

tained inside the parabola

= 2 (x l)

con-

= -^.

7.11.15. Prove that the length of the ellipse

x
is

= V^2sin/;

= cost
= s\nx.
parabola y = ^ x2

equal to the wavelength of the sinusoid


7.11.16. Prove

that

the arc of the

correspon-

O^x^a

has the same length as the arc of


ding to the interval
Pcp corresponding to the interval
the spiral p

O^p^a.

7.11.17. Find the ratio of the

curve
of

y=

which

is

-o-

* Wx

area

enclosed

by the loop

of the

to the area of a circle the circumference

equal to the length

of

the contour of this curve.

7.12. Computing Physical Quantities

7.11.18. Find the volume of the segment cut

paraboloid

cal

7.11.19.

boloid

an

ellipse

of the solid

and the planes


r

7.11.20. Find the


is

volume

the

^t + ^t 4-=

off

from the

ellipti-

by the plane x = a.

^+^ =x

Compute

367

volume

bounded by the hyper-

=c

and

cone whose base

of the right elliptical

with semi-axes a and

its

fr,

= />c.

altitude being equal to h.

7.11.21. Find the volume of the solid generated by revolving


x+l;
about the x-axis the figure bounded by the straight lines y
2x+l and x 2.
y

Find

7.11.22.

volume

the

generated

of the solid

about the #-axis the figure bounded by the

2ay

the straight line

7.11.23. Find the

curve

= acos

q)

bx =

and the axis

volume

of the solid

by revolving

hyperbola
of

1,

abscissas.

generated by revolving the

about the polar axis.

7.11.24. Find the areas of the surfaces generated by revolving the


following curves:
(a)

= tan x ^0<jc<^-^

about the #-axis;

= x \f -^-(O^xO)
about
x + y 2rx =
y

(b)

(c)

about

the #-axis;

and

the x-axis between

h.

7.12. Computing Pressure, Work and Other Physical


Quantities by the Definite Integrals

I.

To compute

law,

which

an area

at

states
a

the

force

the

that

depth

specific

weight of the liquid.

II.

a variable force
this force

If

force

of

immersion h

of

X = f(x)

then the work of


by the integral

pressure
pressure

liquid

of

is

P = yhS

we

use

Pascal's

of a liquid
y

where y

P
is

on
the

acts in the direction of the *-axis,


interval [x lt x 2 ] is expressed

over an
x2

A=
III.

city v

The kinetic energy


is

defined as

^f(x) dx.

of a material

point of mass

and velo-

368

Ch. VII. Applications of the Definite Integral

IV.

repulse each

charges

Electric

where e x and e 2 are the values


between them.

other

with a

force

the charges, and r

of

is

the distance

Note. When solving practical problems we assume that all the


data are expressed in one and the same system of units and omit
the dimensions of the corresponding quantities.

Compute

the force of pressure experienced by a vertical


and altitude h submerged base downwards in
water so that its vertex touches the surface of the water.
Solution. Introduce a system of coordinates as indicated in Fig. 107
and consider a horizontal strip of thickness dx located at an arbitrary depth x.
Assuming this strip to be a rectangle, find the differential of area
dx. From the similarity of the triangles
and ABC
dS
7.12.1.

triangle with base b

= MN

we have

BMN

= 4-,
-^
b
h

whence

MN = ^-h

and dS = ^dx>
h

Fig.

Fig.

107

108

The force of pressure experienced by this strip is equal to dP xdS


accurate to infinitesimals of higher order (taking into consideration
that the specific weight of water is unity). Consequently, the entire
force of water pressure experienced by the triangle is equal to
h

xdS = -^

P =

x*dx = -jbh*.

J
n

7.12.2. Find the force of pressure experienced by a semicircle of


radius R submerged vertically in a liquid so that its diameter is
flush with the liquid surface (the specific weight of the liquid is y).

dam

has the form of a trapezoid whose upper


lower one 50 m, and the altitude 20 m.
Find the force of water pressure experienced by the dam (Fig. 108).
Solution. The differential (dS) of area of the hatched figure is
MNdx. Taking into consideration the
approximately equal to dS
7.12.3.

base

is

70

vertical

long,

the

7.12. Computing Physical Quantities

OML and OAE, we


MN = 20 x + 50 = 70 x.

similarity of the triangles

whence

ML = 20

xdx=(70 x)dx

and the

equal to

~=

find

x
;

2Q

MNx

Thus, dS =
water pressure

differential of the force of

is

dP = xdS = x (70 x) dx.

Integrating with respect to x from

P=

(70x
\

to 20,

x )dx=

work

7.12.4. Calculate the


of a

369

11

performed

we

get

333-

in

pumping

the water

out

semispherical boiler of radius R.

A rectangular vessel is filled with equal volumes of water


water is twice as heavy as oil. Show that the force of pressure of the mixture on the wall will
I
reduce by one fifth if the water is
replaced by oil.
Solution. Let h be the depth of
h_
the vessel and / the length of the
2
wall. Let us introduce a system of
7.12.5.

and

oil;

coordinates as shown in Fig. 109.


Since the oil is situated above the
water and occupies the upper half of
the vessel, the force of the oil pressure experienced by the upper half
of the wall is equal to

a
Fig.

1(

jl
2

The pressure
oil

column
h
,

at a

depth

A
2

of height

>y

and that

is

of

made up
the

of the pressure of the

water

column

of

height

and therefore

dP.=

h
2

Xy+U-

Consequently, the force


the wall is

of

of

ldx=[ x

~)idx.

pressure of the mixture on the lower half

370

Ch. VII. Applications

The entire pressure

of the Definite Integral

mixture on the wall

of the

PP 4P
r-^
- 4+ fi
+i6 - m

lh 2

i6

If the vessel were filled only with


the same wall would be

oil,

is

equal to

'

the force of pressure

on

IK1

=4.f xl dx = -r

Hence,

= i-P.
P P = l:lh*
16
5
7.12.6. The electric charge E concentrated at the origin of coordinates repulses the charge e from the point (a, 0) to the point (ft, 0).
Find the work A of the repulsive force F.
Solution. The differential of the work of the force over displaceeE

ment dx

is

=F dx = -^-dx.

dA

Hence

As

b*

oo

work A tends

the

eE
to

7.12.7. Calculate the work performed in launching a rocket of


P from the ground vertically upwards to a height h.
Solution. Let us denote the force of attraction of the rocket by
the Earth by F, the mass of the rocket by
Ri and the mass of

weight

by

the Earth

mE

According to Newton's law

=k
X'

where x
Earth.

is

the distance

between

km Rm E =K>

Putting

S et

being the radius of the Earth. At

the weight of the rocket P,

and F

(x)

i.e.

and the centre

the rocket

we

F x
(

x=R

F(R)

= -^>

the force

= P = -^2y

Thus, the differential of the work

dA = F(x) dx

is

= ^-dx.

of the

+
F (R)

whence

will be

K = PR*

7.12. Computing Physical Quantities

371

we obtain

Integrating,

R+h

R+fi

PRh

The

limit lim
h

(h)

lim
h

> <n

PR

is

equal to the work performed

l~

by the rocket engine to achieve complete escape of the rocket from


the Earth's gravity field (the Earth's motion is neglected).
7.12.8. Calculate the work that has to be done to stop an iron
sphere of radius R rotating about its diameter with an angular velocity

(0.

The amount of required work is equal to the kinetic


the sphere. To calculate this energy divide the sphere
into concentric hollow cylinders of thickness dx; the velocity of the
points of such a cylinder of radius x is cox.
Solution.

energy

of

volume of such a cylinder is dV = 4nx]^R 2


x2 dx
the element of mass dM = ydV, where y is the density of iron, and
2
the differential of kinetic energy dK = 2nya) 2 x 3
x2 dx.

The element

of

VR

Hence,
R

K = 2nya

3
jx

VR x
2

AnyR*

dx =

co /?

Mco 2 /? 2

7.12.9. Calculate the kinetic energy of a disk of mass


and radius R rotating with an angular velocity co about an axis passing,
through its centre perpendicular to its plane.

7.12.10. Find the amount of heat released by an alternating


nusoidal current
I

2ji

si-

/ sin

T in a conductor with resistance R.


Solution. For direct current the amount of heat released during
a unit time is determined by the Joule-Lenz law
during a cycle

= 0.24 I

alternating current the


0.24 a (t)Rdt, whence

For

dQ

R.

differential

of

amount

of

heat

is

Q=0.24/? [l*dt.
In

this case

Q -0.24 RIl j

sin 2

{^t ^dt^
2jc
v

-0.12 RI

T_
'

2ji

0.12/? 77*.

Ch. VII. Applications of the Definite Integral

372

Find

7.12.11.
vertical

the pressure of a liquid of specific weight d on a

with axes 2a and 2b whose centre

ellipse

is

submerged

in

h(h^b).

the liquid to a level

7.12.12. Find the pressure of a liquid of specific weight d on the


wall of a circular cylinder of base radius r and altitude h if the
cylinder is full of liquid.
7.12.13.
gravity in

the work performed to overcome the force of


the water out of a conical vessel with the vertex
the radius of the cone base is R and its altitude is H.

Calculate

pumping

downwards;
7.12.14.
if

Compute

the

work required

one kilogram

force of

is

to stretch a spring

required to stretch

it

by

by 6 cm,
cm.

7 J 3. Computing Static Moments and Moments of


Inertia. Determining Coordinates of the
Centre of Gravity
In all problems of this paragraph we will assume that the mass
distributed uniformly in a body (linear, two- and three-dimensional)
and that its density is equal to unity.
about
1. For
a plane curve L the static moments
x and
y
the x- and (/-axis are expressed by the formulas
is

M x = J y dl, M y = J x dl.
L

The moment

of

inertia

about the origin

/o=S(^ +

of coordinates

)d/.

L
If

the curve

given by the explicit equation y

is

= y(x) (a^x^b)

then dl has to be replaced by Vl+y' 2 dx in the above formulas.


x(t),
the curve L is given by the parametric equations x
If

y = y(t)(ti<:

U)> then dl should be replaced by

Vx' 2 + y' 2 dt

these formulas.
2. For the plane figure bounded by the curves y
y1
b
a, x
y 1 (x)^y 2 (x) and the straight lines x
static moments are expressed by the formulas

M *=\ \
The centre

dinates:

xc

=y

(x),

the

(yl

of gravity of a plane curve has the following coor-

Mv

=~

{x),

(a^x^b)

yl)dx; M y = x(y y )dx.

3.

in

yc

= j-x

where

is

the

length of the curve L.

7.13.

where S

=^Y>

373

Inertia

of

of gravity of a plane figure has the coordinates: x c

The centre
yc

Computing Static Moments and Moments

the area of the

is

7.13.1. Find the static

moment

a*

= -^-,

figure.

of the

upper portion of the ellipse

b*

about the #-axis.


Solution. For the ellipse

ydi
since y 2

=b

^-x

ydl
where

dx

and yy
r

]/

b2

= ~i x

^x + ^x
2

we have

dx

=^Va e x
2

Integrating from

we

to a,

the eccentricity of the ellipse, e

is

dx

find

Mx =

= Vy + (yy'ydx;

Vet 1

*
2

dx =

[ V e

^r

x 2 dx

-a
Z==:

~a{K
In
since

the
8 =

case
~

of a circle,
arc sin e
11.
and urn
e

7.13.2.

e.

i.

Find the moment

ar s n 8
^

~T

at a

= b,

we

=^ ^+
shall

have

arc sln p )

M x = 2a

2
,

of inertia of a rectangle

altitude h about its base.


Solution. Let us consider
out from the rectangle and

with base b and

width dy cut
base and situated at
a distance y from it. The mass of the strip is equal to its area
dS bdy, the distances from all its points to the base being equal
by 2 dy and
to y accurate to dy. Therefore, dl x

an elementary
parallel

to

strip of

the

Ix

by 2 dy

= -^-.

7.13.3. Find the moment of inertia of an arc of the circle x*


lying in the first quadrant about the #-axis.

+y =R
2

7.13.4. Calculate the moment of inertia about the #-axis of the


bounded by the parabola y 2 =--Aax and the straight line x- a.
2
Solution.
have dI x
x' dS, where dS is the area of a vertical

figure

We

strip situated

at

distance x from the y-ax\s (Fig.

dS = 2\y\dx = 2\/4axdx.

110):

Ch. VII

374

Applications of the Definite Integral

Hence,
a

Ix

=J

4x2

Vaxdx = 4V~a ^x T dx = -f a *b

In designing wooden girder bridges we often have to deal


logs flattened on two opposite sides. Figure 111 shows the
cross-section of such a log. Determine the
moment of inertia of this cross-section
about the horizontal centre line.

7.13.5.

with

- -z-^x

*~x

Fig.

110

Arrange the system

Solution.

Fig.

Ill

of coordinates

as

is

shown

in

the

accompanying drawing. Then


dI x

=y

dS = MN dy = 2xdy = 2

dS, where

VR y
2

dy.

Whence
h
lx

= l\

VW^

dy

= \\y

VW^

dy.

-h

Substituting

we

= Rs\nt;

lx

=R cos

dy

^=0;

dt;

= arc sin (h/R),

<;et

=4

Jy

arc sin
2

VR y
2

arc sin

= 4R*

dy

=4

R 2 sin

(h/R)

arc sin

sin 2

(h/R)

cos 2 /d/

=^

cos tR cos

dt

(h/R)

(1 cos4/)df

= ^ arc sin A + A (2h R VR h


2

When

about one of

we obtain
its

diameters:

the

moment
'

of

inertia

of

the

2
.

circle

7.13. Computing Static Moments and Moments

375

of Inertia

7.13.6. Find the moment of inertia about the *-axis of the figure
bounded by two parabolas with dimensions indicated in Fig. 112.
Solution. Arrange the system of coordinates as shown in Fig. 112
and write the equations of the parabolas.

The equation

parabola

of the left

is:

b2 (

y ^Ya[ X +2

the equation

right parabola, # 2

= ^(y

For the hatched


inertia

of

dI x

strip

the

of

moment

the

is

= y*dS = y*\MN\dy,

where

\MN = x
\

y =
f-

Hence,
6/2

/,=

b/ 2

y*(a-%tf)dy = 2

ab*

j>

i,2

-b/2

7.13.7. Find the static moments about the x- and (/-axis of the
arc of the parabola y 2
2x between x
2 (y>0).
and x

Find the

7.13.8.

moments about

static

the axes of coordinates

segment -^--|--|-=1 whose end-points

of the line

lie

on the coordi-

nate axes.
7.13.9.

Find

the curve y

the

= cosx

7.13.10. Find
bounded by the

between x

the

=x

moments

of

the #-axis of the arc of

and x,=-2

moment about

static

lines

7.13.11. Find the

moment about

static

the x-axis of the figure

r
\/

x.

inertia

the triangle bounded by the lines x

about the

x-

and

= 0, y = andy + -|-=

(/-axis of
1

(a>

0,

b>0).
7.13.12. Find the
base
if
equal to h.

its
is

AD

moment

of inertia of the trapezoid

AD = a, BC=b

and the altitude

of

ABCD

about

the trapezoid

7.13.13. Find the centre of gravity of the semicircle x 2 -\-y 2 --=a 2


situated above the x-ax\s.
Solution. Since the arc of the semicircle is symmetrical about the
(/-axis, the centre of gravity of the arc lies on the */-axis, i. e. x c
0.

Ch. VI I. Applications

376

To

find the ordinate y c

7.13.1:

M x 2a

2
;

of the Definite

Integral

take advantage of the result of Problem

therefore y c

Thus,

xc

= 0,

yc

= -.

7.13.14. Find the coordinates of the centre of gravity of the ca-

=y +
= cosh x
Solution. We have

between

tenary y

dl=\/
whence we

*/'

+ sinh

dx= V\

/I (0,

xrfx

and B(a, cosh a).

1)

= cosh xdx

find

^ d/
l

xdx = sinh a.

cosh

Then
a

M y =^xdl=^ x cosh x dx = x sinh x\ J sinh


a

= a sinh a cosh a +

Hence,
a sinh a

(cosh a = a-

cosh a-

1)

sinh a

sinh a

1.

= a tanh--

Analogously,
a

M x = ^ydl=

cosh

x dx =
y

( 1

+ cosh 2*) djc =


I

sinh 2x

sinh 2a

=t[ x +-tsinh 2a

TH

cosh a

4
2 sinh a

sinh a

7.13.15. Find the centre of gravity of the first arc of the cycloid:
2n).
cos /) (0
t
sin /), y
a(1
(t
Solution. The first arc of the cycloid is symmetrical about the
na, therefore the centre of gravity of the arc of
straight line x
na. Since the length
the cycloid lies on this straight line and x c
Sa we have
of the first arc of the cycloid l

=a

< <

2ji

2ji

ye

=-j ydl=--2a J
2

(1 cos t)smdt=-j

sin 3

~2

dt

=Ya

'

7.13.16. Determine the coordinates of the centre of gravity of the


2

portion of the arc of the astroid x


quadrant.

'

+y =a*
3

situated in the

first

7.13. Computing Static Moments and Moments

of

377

Inertia

7.13.17. Find the Cartesian coordinates of the centre of gravity of


and y
n.
cosq)) between q)
the arc of the cardioid p=a(l
Solution. Let us represent the equation of the cardioid in parametric form:

x
y

= p cos = a + cos
= p sin = a (1 +cos
( 1

q)

q)

(p)

cos
sin

cp)

(p;
(p.

As the parameter qp varies between and ji the running point describes


the upper portion of the curve. Since the length of the entire cardioid equals 8a and
dl

=V

+ (y'^f
9/

d(p

^ d(p

2a cos

Problem

(see

we have

7.9.3),

JT

xc

= \ \y ^ ^Va

+ cos

asin W

2a

cos 4

<P)

^ a cos-|- dcp

y sin

-|-

dcp

= 4- a cos y

4
5

Analogously,
JT

yc=z

Ta \
L

xdl

=^cosq)(l + cos(p) 2acos^d(p =


n

ji

=aj

y=

Putting

we

cos

cp

cos

get (see

yd(p = a

^2cos 6

-|-

cos

dep.

Problem 6.6.9)

_JT_

T
yc

And

so,

= 2a

xc

(2

cos b

cos* t)dt =4ftg^ 2ay =

-^-a.

= y =^.
c

interesting to note that the centre of gravity of the aboveconsidered half of the arc of the cardioid lies on the bisector of
the first coordinate angle, though the arc itself is not symmetrical
It

is

about this bisector.


7.13.18. Find the centre of gravity of the figure bounded by the
2
2
36 and the circle x 2
9 and situated in the
9y
y

ellipse 4a: 2
first

quadrant (Fig. 113).

+ =

Ch. VII. Applications of the Definite Integral

378

Solution. Let us
3

moments

calculate the static

first
3

dx
3

=y

x V~9

dx

= 3\

b
3

The area
radius 3

is

x2

dx =5.

quarter of

of

equal to

a circle of

and the area

of a

quarter of an ellipse with semi-axes a

and

b=2

equals

x
Fig.

113

of

the

9jx

figure
3jt

=3

3ji
,

therefore

under

the

area

consideration

is

3ji

Thus,

My

Mx _

20

~S"~

3n'

7.13.19. Find the centre of gravity of the figure bounded by the


_L

parabola

of

_L

_L

+y =a

and the axes

of coordinates.

7.13.20. Find the Cartesian coordinates of the centre of gravity


acos 3 q) (a
the figure enclosed by the curve p
0).
Solution. Since
in all cases, the given curve is traced

p^O

when

q)

y to y

changes from

ction coscp

it

is

the area

S = 2xy j

By

virtue of evenness of the fun-

symmetrical about the polar axis and passes through

the origin of coordinates at

Compute

>

dy

= y.

the figure obtained:

of

=a

(p

iX3x5
6
2
Jcos (pd(p= a 2x4x6 X

ji

'32

7.13. Computing Static Moments and Moments

Now arrange the axes of coordinates as


the parametric equations of the curve are
x p cos

(p

y = p sin

(p

= a cos
= a sin

shown

379

of Inertia

in Fig.

Then

114.

cp;

cp

cos 3 (p.

The centre of gravity of the figure lies on the x-axis, i.e. y c


by virtue of symmetry about the #-axis. Finally, determine x c

r*

ji

jt

xy ax

= ~- J

JL_

-y

cos 10

(5/32)jia 2

figure

by

bounded

sect at

The

(p

dtp

= ~-

(cos 10

coordinates
the

(Fig.

straight

12

line

21

ji

~~40

fl

"

115

centre of
2
y=x

d(p

(p)

1x3x5x7x9x11 \
2x4x6x8x10x12;

the

of

cos

gravity of

and

the

the

sinusoid

115).

straight line y

the points (0, 0)

bounded by these

cp

Fig.

the

= sinx (*>0)
Solution.

sin 2

114

7.13.21. Find

*/

(p

/ 1X3X5X7X9
^2x4x6x8x10

8fl 3
:

Fig.

=
\

and

lines

is

5=

^sin

= 2 x
,

jc

and the sine


1^

dx-

The

line y

area

of

= s\nx

inter-

the

figure

380

Ch. VII. Applications of the Definite Integral

Hence,

yj( sin2 *-^*


4

d*
)

jt

^x(^sinx

J_

^-x^dx

ji

sin 2x

Xr

4
3jx2-

"6(4

ji)

12

3ji"

Jt

dx

(4-

3(4

ji)

12

ji

7.13.22. Prove the following theorems (Guldin's theorems).


Theorem 1. The area of a surface obtained by revolving an arc
of a plane curve about some axis lying in the plane of the curve and
not intersecting it is equal to the product of the length of the curve
by the circumference of the circle described by the centre of gravity
of the arc of the curve.
Theorem 2. The volume of a solid obtained by revolving a plane
figure about some axis lying in the plane of the figure and not
intersecting it is equal to the product of the area of this -figure by
the circumference of the circle described by the centre of gravity of
the figure.

Proof. (1)

Compare

the

revolution of the curve

formula

L about

for the area of the surface of


the x-axis (see 7.10)

P=2n\ ydl
}

with that

for the ordinate of

the centre of gravity of this curve

MX
f
Ai
y c = -f=T)ydi.
I

Hence we conclude that

P = 2n lyc =
where

l-2ny ct

the length of the revolving arc, and 2ny c is the length


radius y c9 i.e. the length of the circle described by
the centre of gravity when revolving about the x-axis.
/

is

of a circle of

7.13. Computing Static Moments and Moments

(2)

Compare the formula

by revolving

for

the

volume

of

381

Inertia

of

solid

generated

plane figure about the x-axis (see 7.6)

-yl)

dx

with that for the ordinate of the centre of gravity of this figure

yc

b
x

= -j

Hence we conclude that

V=

ji

2S y c

=S

2 ny c

where S

is the area of the revolving figure, and 2ny c is the length


the circumference described by the centre of gravity when revolving about the x-axis.

of

7.13.23. Using the first Guldin theorem, find the centre of gravity of a semicircle of radius a.
Solution. Arrange the coordinate axes as shown in Fig. 116. By
If the semivirtue of symmetry
0. Now it remains to find y c

^=

circle revolves

about the *-axis, then

the surface P of the solid of revolution is equal to Ana 1


and the
na. Therefore, accorarc length l
ding to the first Guldin theorem,
,

Ana 2

= na

2ny c

yc

2.

-a

7.13.24. Using the second GulFig- 116


din theorem, find the coordinates
of the centre of
gravity of the
figure bounded
by the x-axis and one arc of the cycloid: r
=a (t sin/); y a ( 1 cos t).
Solution. By virtue of the symmetry of the figure about the
straight line x~na its centre of gravity lies on this straight line;
hence, x c
na.
The volume V obtained by revolving this figure about the x-axis
is
equal to 5n 2 a 3 (see Problem 7.6.14), the area S of the figure
being equal to 3na 2 (see Problem 7.4.3). Using the second Guldin
theorem, we get
_5a
5ji 2 a 3
V
~~ ~6
y ~~ 2nS
2jt-3jia 2

'

7.13.25. An equilateral triangle with side a revolves about an


axis parallel to the base and situated at a distance b
a from the base.
Find the volume of the solid of revolution.

>

382

Ch. VII. Applications of the Definite Integral

Solution. There are two possible ways of arranging the triangle


with respect to the axis of revolution which are shown in Fig. 117,

a and

b.

The altitude

of

the

equilateral

aV 3

triangle

is

the area

h--

a 2 |/"3

The centre
intersection of the

gravity 0'

of

at a distance

cas.

Fig.

By

of b-

Q ^"3

point of

from the

in the second.

117

the second Guldin theorem

2na 2

V3

aY3_

b+

and

first

at the
a }A3

medians and

axis of revolution in the

situated

is

2na*V~3

a]/"3

a*bV3
2

a*bV3
ji

"*\
4
6
2
V
J
7.13.26. Find the centre of gravity of the arc of a circle of radius
subtending a central angle 2a.
7.13.27. Find

the

centre

the arc of the cosine line y

the straight

line #

of

gravity

= cosx

the

of

between

x=

figure

bounded by

y and x = ^

and

= y.

Find the coordinates of the centre of gravity of the


x4
by line y 2 = ax 3
7.13.29. Find the Cartesian coordinates of the centre of gravity

7.13.28.

figure enclosed

of

the arc

7.13.30.
its

sides.

of

the

logarithmic spiral p

= aev

from

(p,

= y to

(p 2

ji.

A regular hexagon with side a revolves about one of


Find the volume of the solid of revolution thus generated.

7.13.31. Using Guldin's


a semicircle of radius R.

theorem, find

the

centre

of

gravity of

7.14. Additional Problems

383

Additional Problems

7.14.

7.14.1. Find the area of the portion of the figure bounded by


the curves y m
xn and y n
x m (m and n positive integers) situated
in the first quadrant. Consider the area of the entire figure depending on whether the numbers
and n are even or odd.

7.14.2.

(a)

Prove

that

the area
of the
curvilinear trapezoid
straight lines x
a,
b and parabola
x

=
=
bounded by the x-axis,
= Ax* Bx 2 + Cx + D can be computed using Chebyshev's formula

'

(a-\-b

a\

fa + b\

(b)

Prove that an analogous area

y = f(x)

for a

fa + b

parabola of the

a\~]

fifth

order

= Ax + Bx" + Cx + Dx + Ex + F
3

can be computed using the Gauss formula

S=

^[ ,(-/F?) + 8f(^) +
5

7.14.3. Show that the area of a figure bounded by any


dius vectors of the logarithmic spiral p
ae m and its arc
portional to the difference of the squares of these radii.

'+

two
is

ra-

pro-

7.14.4. Prove that if two solids contained between parallel planes P and Q possess the property that on being cut by any plane
R parallel to these planes equivalent figures are obtained in
their section, then the volumes of these solids are equal (Cavalien's principle).

(O^x^h)

7.14.5. Prove that if the function 5 (x)


expressing
the area of the section of a solid by a plane perpendicular to the
x-axis is a polynomial of a degree not higher than three, then the

volume

of

this

solid

is

equal

toV = ^ S (0) + 4S

^ +S

(h)

deduce formulas for computing the volume of


Using
a sphere, spherical segments of two and one bases, cone, frustrum
of a cone, ellipsoid, and paraboloid of revolution.
this formula,

7.14.6. Prove that the volume of a solid generated by revolving


(/-axis the figure
O^y^y(x), where y(x) is
a single-valued continuous function, is equal to

about the

a^x^b

V=

2ji

xy

(x) dx.

Ch. VII. Applications of the Definite Integral

384

7.14.7. Prove that the volume of the solid formed by revolving,


about the polar axis, a figure O^a^cp^fJ^rc, O^p^p(q)),
is equal to

2ji

V ==

r
l

((p)

sin q)d(p.

the arc length of the curve given by the pa-

7.14.8. Prove that

rametric equations
jc

y
is

equal to

= nOcos/ + nOsin*,
= f"(t)sint + f'(t)cost

[f (t)

+ f (t)]l\.

7.14.9. Find the

length of the curve represented parametri-

arc

cally

cos
^ dz,
= C
z

between the origin and

the

nearest

C sin z
\

dz

point from the

vertical tan-

gent line.
7.14.10. Deduce the formula for the arc length in polar coordinates proceeding from the definition without passing over from
Cartesian coordinates to polar ones.

Prove that the arc length l(x) of the catenary y


measured from the point (0, 1) is expressed by the forand find parametric equations of this line,
(#) =sinh a;

7.14.11.

= cosh x
mula

using the arc length as the parameter.


7.14.12. A flexible thread is suspended at the points A and B
located at one and the same height. The distance between the
points is AB =2fr, the deflection of the thread is /. Assuming the suspended thread to be a parabola, show that the length of the thread

at a sufficiently small

y.

7.14.13. Find the ratio of the

curve y

ference

is

x Vx

to the

area enclosed by the

area

of the circle,

loop of the

whose circum-

equal in length to the contour of the curve.

7.14.14. Compute the length of the arc formed by the intersection


parabolic cylinder

of the

and the

7.14.

cone

elliptic

between the origin and the point


7.14.15. Prove that

Ax
is

385

Additional Problems

M (x,

z).

the area of the ellipse

+ 2Bxy + Cy* + 2Dx + 2Ey + F =

(AC

B > 0)
2

equal to

where A

(ACB-

A B D
B C E
D E F

-=

7.14.16. Find: (a) the area S of the figure bounded by the hyy'1
perbola x 1
U the positive part of the x-axis and the radius
vector connecting the origin of coordinates and the point M(x, y)
lying on this hyperbola.
(b) The area of the circular sector Q bounded by the x-axls and
the radius drawn from the centre to the point
(x, y) lying on
y* 1. Prove that the coordinates of the points
the circle x 2
and N are expressed respectively through the areas S and Q by
the formulas

a;^

=cosh 2S,

7.14.17.

yM

= s'mh2S,

xN

= cos2Q, y N = s')n2Q.

Using Guldin's theorem, prove that the centre of grais one third of the altitude distant from its base.

vity of a triangle

7.14.18. Let be the abscissa of the centre of gravity of a curbounded by the continuous curve y f(x), the
x-axis and the straight lines x--^-a and x b. Prove the validity
of the following equality:

vilinear trapezoid

(ax
\

+ b)

f (x)

dx

- (a% + b)

\ f

(x)dx

(Vereshchagin's rule).
7.14.19. Let a curvilinear sector be bounded by two radius vecand a continuous curve p^/((p). Prove that the coordinates
of the centre of gravity of this sector are expressed by the following formulas:
tors

<p 2

<p 2

cos

(p

dcp

sin

cp

cp

3"
N-'

*P

dcp

Ch. VI /. Applications

386

of the Definite

Integral

7.14.20. Prove that the Cartesian coordinates of the centre of


gravity of an arc of the curve p
/((p) are expressed by the following formulas:

<p 2

pcoscp

|/"p 2

-|-

p"2

sin
^ p

rfqp

2i
;

=JBi-

cp

^~p 2

+ p'

cfcp

Chapter

8
IMPROPER INTEGRALS

8.1.

Improper Integrals with Infinite Limits

Let the function

be defined for

f (x)

all

x^a

and integrable on

any interval

Then

A],

[a,

integral of the function


+

noted by the symbol

f (x)

lim
in

(x)

\ f

dx

the interval

the improper

called

is

[a,

+00] and

is

de-

00

We

f(x)dx.

similarly define the

integrals

+00

^ f

(x)dx and

GO

(x)dx.

00

Thus,
+

GO

/ (x)

lim
A-> +

dx;

^f(x)dx\

lim

A-*-a> A

-oo

00

(x)
^ f
a

^f(x)dx=
+

f{x)dx=

-oo

lim

\f(x)dx+

A -~ A

lim
fl-*

[f(x)dx.

+ co

and are finite, the appropriate integotherwise, they are called divergent.
Comparison test. Let f (x) and g(x) be defined for all
and
If
(x)
(x) for
integrable on each interval [a, A],
If

the above limits

exist

rals are called convergent;

x^a

A^a.

0^/

^g

00
f

all

x^a,

then from

convergence of

the

integral

g(x)dx

it

fol-

a
GO

CO

lows that the

integral

^f(x)dx

is

also

convergent, and ^((xjdx^.

388

<

Ch. VIII. Improper Integrals

g (x) dx; from divergence

of the

integral

^f(x)dx

it

follows that

a
OO

the integral

^g(x)dx

also divergent.

is

Special comparison

test.

If

>

infinitesimal of order X

as

as

oo

the function

compared with

-~

f(x)^0

is

an

then the integral

+ X

f(x)dx converges

for

>

and diverges

for

X^l.

Absolute

and conditional convergence.

Let

the

function f(x) be

00

defined for all

x^a.

If

the integral

/ (x)

dx converges, then the

a
00

integral

[f(x)dx

also converges and

is

called absolutely convergent.

In this case
00

CO

\f{x)dx <^\\f(x)\dx.
a

the

If

integral

f(x)dx converges, and

\f(x)\dx diverges, then

the integral

^f(x)dx

is

called conditionally convergent.

The change of the variable in an improper integral is based on


the following theorem.
Theorem. Let the function f (x) be defined and continuous for
x^a. If the function x y(t) defined on the interval
oo and oo), is monoto(a and P may also be improper numbers
has
a continuous derivative q/(/)^0 and limq) (t) = a,
nic,

a</<|3

t->

lim
t

>

cp

(t)

= +

a+O

then

o
oo

J
(

[f {x)dx= \f(y(t))v'
a

(t)dt.

Integration by parts involves no difficulties.


Evaluate the following improper integrals with infinite
limits or prove their divergence taking advantage of their defini8.1.1.

tion.

w]lW-x>
e2

W L+t +
-co

(c)J*sin*d*.

Solution, (a)

Improper Integrals with Infinite Limits

8.1.

By

389

definition,

jxln 3 *
e2

In 3

Jx

aI x

e2

2 In 2 *

=
(b)

By

lim

definition,

oo

+ 2x + 5

may

(instead of the point x


any other finite point of the #-axis
be taken as an intermediate limit of integration).

Compute each of the


above equality:

limits

standing

the right

in

side of the

= y arctan T + T
,

Jl

1*

o
,

'i

4- arc tan

ji

=T

arctan -2'

Hence,

+ 2* + 5

X
(c)

By

"

definition,

qo

^xs\nxdx=
o

Putting w

cfo

= sin xdx

lim

lim
+

[xs'mxdx.
00

and integrating by
A

^xsinxdx^

lim

14

a;cosa;|^

parts,

we

get

+ J cosArdA; j==
o

lim
j4 -+

But

the

last

limit

QO

a:

sin at^a; diverges.

does

not

exist.

A cos A + sin

^4).

CO

Consequently,

the

integral

Ch. VIII. Improper Integrals

390

8.1.2. Evaluate the following improper


limits on the basis of their definition:

with

integrals

infinite

dx

(O

x+x*

V" (4x

+l) 3

'

QO

dx

dx
(!

(e)

+ *)'
By

Solution, (a)
OD

x dx

x2

6x+10

(f)

definition
A

xdx

j.

>

(x 2

_l_

lim
/4

3)~

GO

1/2

-\I2
\

lim
->

y4

8.1.3.

GO

<4

-3

=
1

1.

Prove that the integrals of the form

e" px dx and

converge for any constant p


8.1.4. Test

>

g^dx

and diverge

for

/?

< 0.

the integral
00

dx

f
J

l+2x 2 + 3x 4

for convergence.

The integrand

Solution.

+ 2x + 3x
2

is

and

positive

j as

a;

oo.

is

an infinitesimal

Since 4

>

1,

of order

A,

=4

as

compared with

the integral converges according to the

special comparison test.


8.1.5. Test the integral

dx

x+sin 2

for convergence.

Solution.
tive for x

The integrand f(x)-

As jc* oo
as

x+sin 2 x

is

continuous

and posi-

1.

the

compared with

integral diverges.

function
;

f(x)

is

an infinitesimal

of order

according to the special comparison

X=l

test

the

8.1.

Improper Integrals with Infinite Limits

391

8.1.6. Test the following integrals for convergence:


co

cc

+ cosa:

3+arc

,n

sin

f arc tan x
V
.

j/

8.1.7. Test

the integral

(*+

1)

dx

for convergence.

Solution.

Determine

The integrand

continuous

is

and positive

order of smallness X with respect to

its

for

as

x^\.

oo;

since
1

x+ Vx+\
*2

the

order

+2

^+l

+ 1^ * +

1+2

l/_L.

According to the special comparison

of smallness
00

test

the integral

Hy

*+

diverges.

8.1.8. Test

the integral
GO

dx

r
J

Vx (x-\)(x-2)

for convergence.

Solution. Since the function

f(x)=

is

=~ x

_r_--

an infinitesimal

of order

= y3

with respect to

as

+oo,

according to the special comparison test the integral converges.


8.1.9. Test the integral

for convergence.

Ch. VIII. Improper Integrals

392

Solution.

Determine

The integrand
its

and positive

continuous

is

order of smallness with

= -TT

respect

to

x^2.

for

x + oo:

as

v-35

\/2

Since the second multiplier has the limit

have 1

= -^- <

1.

8.1.10. Test

as x

<*),

we

Consequently, the given integral diverges.

the integral

cosx

dx

for convergence.

Solution.

The integrand
'

is

cos- = 2sin 2

f(x)=

'

x^l.

positive and continuous for

Since 2 sin 2

y ~2 = -p-,
1

the given integral converges (by the special comparison

test).

8.1.11. Test the integral


i

"

n>0

dx.

In

for convergence.

Solution. Transform the integrand:

Since the function

f(x)<

is

an

infinitesimal

In other words,

lim

/(*)

as

x>,

According to

the special comparison test the given integral diverges.


8.1.12. Test

the integral
1

for convergence.

4 sin 2x

dx

then

8.1.

The function

Solution.

f(x)

^ sin 2x

numerator. Test the integral

in sign of the

4 sin 2x|

'[/

convergence.

Since

dx

4 sin

for

2a:

<^
5

COnV/erges, the integral


j

f
J

c h an 2es its sign together

TT^"
y x

x 3 -\-

with the change

393

Improper Integrals with Infinite Limiis

and

^-^^-dx
x*+y x

the

integral

converges as well (ac-

cording to the comparison test). Thus, the given integral converges


absolutely.
8.1.13. Prove that the Dirichlet

converges conditionally.
Solution. Let us represent

integral

given integral as the

the

sum

of

two

integrals:

C slnx
= \
dx=
.

The

first

is

dx+ \

C sinx
\

proper integral (since

of

integration by

lim

dx

J
parts to the second integral,
\

method

P sin*

x-> o

Applying the

we have

oo

smx

dx

= m

dx'-

\\

C cos x dx

IL

lim
A

-> co

But the improper integral

-^^dx

Jl

T
converges

71

2
co

and the

integral

P cos*

x*

converges.

absolutely,

since

Ch. VIII. Improper Integrals

394

the

Therefore,

^-^~dx

integral

converges.

Reasoning

way

in a similar

easy to prove that the integral

is

it

CO

c sx

dx

also

Now

converges.

prove

us

let

that

the

integral

n
2

J_5!2_LL

dx diverges. Indeed,

J
sin x

sin 2 x

cos 2x

2x

but the integral

PI cos2x

fdx

/I

pcos2x

CO
Jl

31

00

=4

4 - 1 In -1 j

lim In

It

ji

C cos 2x

<te

diverges, since

lim In

f*

A =oo, and

the integral

cos 2x

converges.

JX

T
8.1.14. Prove that the following integrals converge
00

Ot

^ s\n (x

(a)

2
)

dx;

00

cos

Solution, (a) Putting

(a; )

dx;

(b)

= K*,

we

jVin (x 2 )dx

= -^

us represent

2x cos

4
(jc )

dx.

find
00

00

Let

-y^dt.
^
o

the integral on the right side as the

sum

of

two

= 0.

Let

integrals:

V'
The

first

summand

is

proper integral, since

lim -^i/- + o V t

8.1. Improper Integrals with Infinite Limits

apply

us

summand

second

the

to

method

the

395

integration by

of

parts, putting

u= jV U

sin

sin

cos

00

_L

= dv,

dt

CQS

dt

converges

integral

last

Jt/2

Jt/2

The

cos

dt.

jt/2

absolutely,

since

COS

^"^yr

f3/2

an ^

QO

the integral

We

converges.

^"^72

can prove analogously that the

Jl

T
QO

integral

cos(x 2 )dx

is

convergent. The integrals considered are cal-

FresneVs integrals. They are used

led

non

(b)

By

explaining the phenome-

in

diffraction,

light

of

x2

substitution

the

this

integral

reduced

is

to

the

QO

integral

2
Jcos(/ )d/. The

integral converges as

latter

has

just

been

proved.
Note. Fresnel's integrals show that an improper integral can conoo. The last
verge even when the integrand does not vanish as x
convergent integral considered in item (b) shows that an improper
integral can converge even if the integrand is not bounded. Indeed,

at

= l/nn

=h \/ tin,

(n

i.e.

=
is

it

1,

2,

integrand

the

...)

attains

values

the

unbounded.

8.1.15. Evaluate the improper integral


QO

?
J

Solution.

x=0

at

Make

dx

(l+* 2 )"'

n natural number.

the substitution x

/=0, x

+ 00

= tan

>
jt

as

quently, by the theorem on changing

t,

where

0^<y.

Then

and

x't^^^Q.

Conse-

variable in an improper

integral
jt

ji

~2~
2

oo

I(TWI xsec2M/= I
On changing
was computed

the
in

variable

we obtain

Problem 6.6.9.

cos2,,

"

2/d/

the proper

integral

which

Ch. VIII. Improper Integrals

396

Therefore,

dx
aX

+*

J<

=
>"

n/2

>

>

3)

1.3.5. ..(2/i

2.4.6. ..(2.-2)

^
*>

ji

'T-

L
,

Compute

8.1.16.

the integral

Apply the substitution

Solution.

*=l/f;

dx

If

= j-j-p^d*.

another integral

= (l/t

*2

l+* 4

//y

= 00,

)dt\

00

_? (^
J

added

is

the

to

)^

_?

l// 4

Make

substitution

the

H-O, 00

\/t

dz

(I

lim
2

)/"

-*

left

sides then

we

l/t

)dt

= dz.

Then,

as

Hence,

a
dz

lb

-00

= 0;

dt.

+ 00, z ++ oo.

00
1

= t l/t,

and as

and

right

r2 -f

f2

arc

tan^L-H

V2

CD

..

!=-

2^2

dZ

,4

lim
-+

arc
/

tan

JI

^2

Jl

Jl

8.1.17. Evaluate the following improper integrals:


X

00
2

(a)

l~Y^dx\

8.1.18.

(b)

Compute

e-* x 2m+1

dx.

the integral

accurate to two decimal places.


Solution. Represent the given

integral

in

the form of a

two integrals

x5
1

7v

+ * +l
2

sum

of

8.2.

Improper Integrals

Unbounded Functions

of

397

Compute

the former with the required accuracy, using Simpson's


formula, and estimate the latter. Since for
1
we have

x^

then

0<

/2

= ^x-T*dx = ^N-*'*.
N

N=

At

we

Computation

of the

-= <

L<^ X

get the estimate

0.0031.

integral

by Simpson's formula

step

for a

gives

5^0.2155,
and

for

step

-^-

= 0.5
S

Since the difference between


gives a more accurate value

=0.2079.

values

the

is

= 0.2079

0.0076, the integral I t


of the order

with an error

^L6 - 0.0005.
Consequently, the sought-for
/

integral

with an error not exceeding 0.004,

mal

approximately equal to

or

= 0.21

with

all

true deci-

places.

8.2.
If

Improper Integrals of Unbounded Functions

the function
b

f (x)

is

defined

0<e<b a

interval

[a,

point

then, by definition,

b,

e],

for

this limit

is

a^x<b>

if

left

of

the

we put
b-v

(x)dx= lim

(x)dx.

existent and finite, then the improper integral is


is called divergent.
it
the function f (x) is unbounded to the right from

said to be convergent. Otherwise

Analogously,

integrable on any

and unbounded to the

b
\

If

is

& 0.208

the point a, then


b

f (x)

dx=

lim

f(x)dx.

398

Ch. VIII. Improper Integrals

Finally,

the function

if

is

unbounded

interior point c of the interval


b

f {x)dx = $

an

the neighbourhood of
then, by definition,

in

6],

fa,

(x)dx

(x) dx.
J f
r

be continuous on the interval [a, b] except


If there exists a function F (x) continuous on [a, b] for which F' (x)
at a finite number
f (x) except
of points, then the Newton-Leibniz formula

Let the function

at a

finite

number

f (x)

of points.

]f(x)dx=F(b) F(a)
a

holds good.

Sometimes the function F

(x)

is

called a generalized ant [derivative

on the interval [a, b}.


For the functions defined and positive on the interval
convergence tests (comparison tests) analogous to the comparison
tests for improper integrals with infinite limits are valid.
Comparison test. Let the functions f (x) and g(x) be defined on
the interval
and integrable on each interval [a, b e],
< e <b a. If ^f(x)^g(x), then from the convergence of the
for the function

f (x)

a^x<b

a^x<b

\^g(x)dx follows the convergence

integral

^f(x)dx,

of the integral

a
h

and

^ f

(x)dx^^

g(x)dx;

from

divergence

the

the

of

integral

^f(x)dx

follows the divergence of the integral

^g(x)dx.

Special comparison test. If the function f(x)^0


continuous on the interval
and is an

infinitely

large

quantity of the order X as compared with jzr^ as

x>b 0,

then

a^x<b

is

defined

and

the integral

^f(x)dx converges

for

X<

and diverges

for

X^\.

In particular, the integral


b

dx

C
(b

<

xf

1
and diverges for %
1.
converges for X
Absolute and conditional convergence. Let the function f (x) be
defined on the interval a?Cx<b and integrable on each interval

8.2.

[a,

b e];

Improper Integrals

Unbounded Functions

of

from the convergence

then

the

of

integral

399

^\f(x)\dx
a

follows the convergence of

the

^f(x)dx.

integral

a
b

^f(x)dx

In this case the integral

called absolutely convergent.

is

a
b

But

^f(x)dx converges, and the

the integral

if

integral

^\f(x)\dx
a

diverges, then the integral ^ f(x)dx

is

called conditionally convergent.

a
b

Analogous

tests

are

valid for improper integrals

also

^f(x)dx,
a

where f(x)

is

unbounded

to the right

from the point

from the definition, evaluate


improper integrals (or prove their divergence):
Proceeding

8.2.1.

a.

the

following

dx
(a)

l4^T

(b)

I cos,-

3
(c )

f4 x-x>-3

FTT^f

+ ?/* 2

P* 3

dx

The integrand

Solution, (a)

neighbourhood

[1+e,

e]

of the point

since

it

is

jt

/ (x)

l.

=
x y
..

It

is

unbounded

is

in the

\wx

integrable on any interval

continuous function.

Therefore

f^==

xr/\nx
V

\-J*

lim
S-++Q J
1+8

xW\nx
V

lim

8^+0
(b)

The

integrand

of the point *

=y

3
/lH^|
e^+o [i

1
+-ej

|l

lim

!* /ln(l+e)

!(k)

and

3_
'

is

unbounded

integrable on any

in the

interval

neighbourhood

as

400

Ch. VIII. Improper Integrals

a continuous function. Therefore


-t

J cos*

dx

lim
e

Jf

^+

cos x

lim

W4 + ^1

tan

In

e-> + o

= Hm

b->

(
\ 1

In tan

+o

Hence, the given integral diverges.


(c)

x=

The integrand is unbounded in the neighbourhood


and x = 3. Therefore, by definition,

V~4x

x'

dx

dx

3 ~~ J

V~4x

dx

x2

of the points

VT?

-a;

2 we can take any other interior point of


{instead of the point x
the interval [1, 3]). Let us now compute each summand separately:

dx

lim

==

2)

lim arc sin

=
3

==
^-^-3
d*

tr

<?

lim

[0

arc sin (e

1)]

3-t

lim
6-.

-7=^==
r \-(x2)*

lim arc sin (x


8-.

2)

'

= ^[3^ sin (l-e)

-0]=f

Hence,

(d)

The

bourhood
interval of

integrand
of

the

f (x)

point

x=l, which

integration. Therefore,
2

is
an interior
by definition,
l

dx

unbounded

is

dx
i

JC

2
|

in the neigh-

point

of the

8.2. Improper Integrals

Evaluate each

separately.

_ Hm

dx

If

< x<2,

then

1,

lim [arc sin

(1

8- +

401

dx

lim arc sin x


8 -*

0^*<

If

-e

dx

summand

Unbounded Functions

of

e) -0] =~1

then
dx

J V^l

^2_!

_+

J
1

lim

j/- x 2

+e

lnU + K*

1)

8-> +

lim [ln(2

->

+ K3) ln(l + e +

+e) 2 -

l)]

= In

'2

j/"3

).

Hence,

= | + ln(2 + |/3).
(e)

sum

Represent the given integral as a

numerator by / x

ing each term of the

of

three items, divid-

3
,

dx
"

3/5

The

first

summand

a proper

is

evaluated by the Newton-

integral

Leibniz formula:
i

17'

lo

17'

The second and third summands are unbounded


point x

= 0.

to the right of the

Therefore,
i

dx
* 4/15

lim

x 11 / 15

_ !^

8-> +

analogously,

f-^L =

ii

tn

JL = m A ^2/6 __5_
~2
1

ii

Ch. VIII. Improper Integrals

402

Hence,
i

(f)

of

Represent

integrand

the

625
"187*

= T7 + T- 2 'Y =

dx

f(x)

the

in

form

of

sum

partial fractions;
1

/(*)

TU
Then

P
j

(1 x)(l

_ _
djc

x+2

a:

dx

= - jC
1

+x+x

+ - Jf
,

2
)

+2

c
Since
.

1-8
l-e

dx

lim

lim

I-*

8->
E _

In (1

x)

the given integral diverges. There is no need to compute the second


representing a proper integral.
Note. Evaluation of the improper integrals from Problem 8.2.1
(a to f) can be considerably simplified by using a generalized antiderivative and applying the Newton-Leibniz formula. For instance,
3 3 s
In 2 * is continuous on
in Problem 8.2.1 (a) the function F (x)
-^

summand

the interval
1

[1,

<x^ie, and

e]

F'

and differentiate at each point of the


= f (x) on this interval. Therefore
dx

= \\/\tfx

_3
*

8.2.2. Proceeding from the definition, compute


improper integrals (or prove their divergence):
2/JT
('

(a)

interval

(x)

2x dx
(b)

fsinl.;

the

following

403

8.2. Improper Integrals of Unbounded Functions

8.2.3. Evaluate the following improper integrals:


3

-3

Solution, (a) Find the indefinite integral

TCT =
r

The function

J"

(9 arc sin

i-, ^9=^) + C.

= y^9 arc siny xK9 x


f(x) = y==== on the interval
2

(x)

derivative for

a generalized anti-

is

^j

since

3],

3,

it

is

continuous on this interval and F'(x)=f(x) at each point of the


interval (3, 3). Therefore, applying the Newton-Leibniz formula,

we

get

= Tr

-7=

x y 9

9 arc sin

=-r

x-

n.

-3
(b)

Transform the integrand


f

The

(x)=s

\/~*x==

indefinite integral

is

+* =

equal to

]/ |^dx^2arcsin|-l/4^7 + C.
2

j
The function f

(x)

=2 arc sin

-|-

]/~^

is

generalized antideri-

on the interval [0, 2], since it is continuous on


and F'(x)=--f(x) on the interval [0, 2).
Therefore, applying the Newton-Leibniz formula, we get

vative for
this

f (x)

interval

^/2^ dx== ^ 2arcsin ^_

/4ZI72

= + 2.
Jl

j| o

8.2.4. Test

the integral
1

dx
x

/x

-1

for convergence.

Solution. At the point x

the integrand goes to infinity. Both

i^

integrals

3*
f
*/ X
J X v

-\

and

3*

/X

J X y

diverge, since ^

-g-

>

1.

Gorisequent-

Ch. VIII. Improper Integrals

404

ly, the given integral diverges. If this were ignored, and the Newton-Leibniz formula formally applied to this integral, we would
obtain the wrong result:
1

dx

1:

And

this

because the integrand

is

8.2.5. Test

positive.

is

improper

following

the

(a)

6.

integrals

convergence:

for

j
\F==dx\
e*

Psin#-{-cos#
^

u\
b)

dx.

Solution,

The integrand

(a)

infinitely

is

cosx = J/2 sin ~ -J^p x


h=l

the integrand has the order

large

as x

as

compared with

as

+ + 0.

Since

(),

~.

According

to the special comparison test the given integral diverges.


(b) Rewrite the integrand in the following way:
r

/(*)=

sin #-|-cos x
5

l/\

This
h

to

to

function

-^r

as

integral

is

compared with y~^,

as

y\~x

as

large

infinitely

+x +x*

x+
the

since

order

its

1,

is

equal

multiplier tends

first

x >0.

Therefore, by the special comparison test, the given


converges.

8.2.6. Test the following improper integrals for convergence:

(a)

e5in
\

,_,

dx;

(b)
T

b
i

cos

(c)

J v/ x

a:

d#
sin

Solution, (a)

The

integrand

interval (0, 2) and is not


oo. Indeed, since
lim f(x)

(x)=

defined

In
n (l 4-

i/x?)

J,;
at

= 0.

is

positive in the

Let

us

show

e smx_i

_ s'mx~ x,

In (1

l/'x*)

as

x-*0,

that

8.2.

lim

Improper Integrals

Unbounded Functions

of

405

we have

v
.

*-+ o

hni
x

At the same time we have shown


that

an

/(a;) is

pared with

infinitely

in

j/16-x4

by

Consequently,

/(x)~y-L=

that

point x

of the

= 2 <

>

=2

0,

i.e.

com-

as

test,

large function

infinitely

the neighbourhood

as x

comparison

special

the

the

of

= oo.

large quantity of order k

given integral converges.


(b) Determine the order

lim .
- o ^/* 2

->

the

f (x)

with respect

to

T
^4^2 x'

this

end transform the expression


r

quantity

order

of

comparison
(c)

obvious

is

it

tt

= y<l
/ (x)

of the point
cos

+
k=-j <

as

CQS x

*2.

= 0.

smx

y sin x
.

a:

the function

jc>

as

f (x)

is

an

According

'

infinitely

large

to the special

the given integral converges.

test

the function

that

The integrand

bourhood

as

A,

1/2 x

y4+x*l/ 2 + x

j/l6-A: 4~

Hence

for f (x):

is

unbounded

Since
cos*

in

the

neigh-

i/*

/ (*) is

compared with

an

infinitely

large quantity of order

and, by the special comparison

test,

the integral converges.


8.2.7.

Investigate

the

following

improper

gence:
ex

(a)
.1

dx

,i

Tct

(b)

'

y u=ji?

j K

fe)
V

x l dx

^
x-sinx

/n
'

r in
J

gtanx_
1

integrals

for conver-

Ch. VIII. Improper Integrals

406

8.2.8. Prove that

the integral
f sin

<x<

dx

Vx

converges.
Solution. For

sin

Vx
But

the

C dx

y=

integral

converges,

therefore,

by the comparison

b
i

sin(i/*)
test,

integral

the

^x

conver g eS) an(j

a j sQ

consequently

the given integral converges absolutely.


8.2.9.

Prove the convergence

and evaluate
Solution.

=^

the integral

of

In sin

it.

Integrate by parts, putting u

lnsin xdx=x\ns\n x

lim

sin x

'

as *

at

1= =0; x

n/2

=j

at

t2

= ^.

tan x

/.

is

proper

Then dx =

we

substituting

dx.

2dt;

get:

Jl/4

JT/4

In sin xdA: = 2

On

= dv:

dx

integral

one. Consequently, the initial integral converges.


Now make the substitution x 2t in integral

C
J

^e

i^^ Q'

lim

1,

tan x

= \n(s\nx),

COS X j
dx=
x -

Since

xdx

lnsin2^=2

(In 2

+ In sin + In cos f)df =


JT/4

JT/4

= 2Hn2f +

lnsin/<

lncos/df

JT/4

= yln2 + 2

jt/4

lnsin /df

In cos /d/.

In

the

Improper Integrals

8.2.

make

integral

last

=
dt = dz\

the

JT/4

substitution

t-=n/4

at 2,=ji/2;

Unbounded Functions

of

at

= jx/4.

22

2^

JT/

Then

77/2

2^

tdt

= jt/2 z.

Hence,

JT./4

In cos

407

z^dz = 2

In cos

\n sin zdz.

jt/4

Thus,
JT/4

JT./2

In sin

xdx

= j In 2 + 2

JT./2

In sin

+2

tdt

In sin 2

J
Jt/4

Ji/2

=J

ln2

+2

lnsinfd*

= -jln2 + 2/.

Whence

^=
Compute

8.2.10.

\ns\nxdx=-

In 2.

the integral
1

dx
yr==z(n
fx

Solution.
A,

=y

The integrand

natural number).

quantity of order

an infinitely large

is

x>1 0.

with respect to -j-^ as

Therefore, the integral con-

verges.

Make
x

at

the substitution x
^

= 0, x=l

at

= sin

00
1

Jl/

rxn dx =

The

last

1^

integral

-*

is

= n/2.

in the

On

integral.

substituting

JT/

C s\n n /-cos tdt

cos/

Then dx = costdt,
we get

s\n n tdt.

evaluated in Problem 6.6.9:


n

Consequently, the given


formula.

integral

^
2

is

jx

yy,
1

~3

also

ft

even,

n odd.

computed

by the same

8.2.11. Evaluate the following improper integrals (or prove their


divergence):

f
(a)

dx

Tun;

r
(b)

dx

f
(c)

J
1

3a:

+2

y^rdx.
k

408

Ch. VIII. Improper Integrals

8.2.12.

Compute

the improper integral


i

= x m \n n xdx

In

Solution. At n

m>

(n natural,

the integral

1).

evaluated directly:

is

=^xm dx= m+
:

m+

"

For n

>

integrate I n by parts, putting

= In" x\

J
n-i
n
du=n\n
1

We

<lX

get
1

m+

"

any natural
I

m-f-

of

n:

XT Jf ^In^^dxm+

This gives a formula by means

And

= xm dx\
X m+l
u =
m

do

n ^n

/-,.
n 1

which one can reduce


x

__

)"

/ to /

for

/z!

finally,
w

Compute the

8.2.13.

(m+l) + 1

"

integral
2.0

_
~

C
J

e~ x dx

^/ 2-\-x x 2

0.3

accurate to 0.03.
Solution.

The

+ x = (2
2

integral has a singularity at the point

x)(l+x). Let us represent

it

as the

x=2,

sum

of

since

two

in-

tegrals:
2

-e

-e

Now compute the first integral to the required accuracy, and estimate
the second one. For e^O.l we have

0</ <
-

^lp==r

n5x ^'= (u53s

8.3.

Putting e

409

Geometric and Physical Applies of Improp. Integr's

= 0.1,

we

get the estimate / 2

< 0.028.

Evaluation

of the

integral
1.9

e~ x dx

V*by Simpson's formula with

S
and with

a step ft/2

step h

= 0.8

gives

= 0.519,

= 0.4,
S

= 0.513.

And so, integral /j gives the more accurate value, 0.513, with
error not exceeding 0.001. Taking into consideration that integral
is positive, we round off the obtained value to
/

an
2

^0.52

with an error not exceeding 0.03.

By putting

Note.

computation

of the

= 0.01,

we

get the estimate / 2

< 0.005,

but the

integral
1.99

e-*dx

f
4

0.3

would involve much more cumbersome calculations.


8.2.14. Investigate the following integrals for convergence:
i

dx
(a)

/i

f-=;
V sin*

dx

(b)

ex

Jf

< c)

Ccos 2 xdx

Jo^gr:

F=;

f tanxdx

(d)

(e)

j
_1_

sin#dx

8.3.

Geometric and Physical Applications of Improper

Integrals
8.3.1.

Find the area

of the figure

bounded by the curve

y-l

(the witch of Agnesi)

Solution.

and

its

asymptote.

The function y =
}

tire

number

scale,

and lim y
X-+

tote of the given curve

to find the area

+x*

_^

is

continuous throughout the en-

= 0. Consequently, the x-axis

is

the asymp-

CO

which

is

shown

of the figure that

in Fig.

118.

It

is

required

extends without bound along the

Ch. VIII. Improper Integrals

410

In other words,

x-axis.

is

it

required to evaluate the improper integ-

CO

ral

dx

S= JC

-t-t
r
-\-

y-ax\s

By

X"

virtue of the

^=

GO

-4^- = 2[ T

S=f

arctanx

lim

generated by

Find the surface area


arc of the curve y

8.3.2.

Solution.
face

Jl.

revolving about the

= oo

The area

of

the sur-

equal to the improper in-

is

at

Making the substitution e~ x = /,


= e~* dx, we get x^=0 at

118

Fig.

31

= 9Z'-tt^ =

tegral

-7

about the

the

y\

of the figure

we have
CO

x-axis

symmetry

oo

= 0;

hence

S = 2ji [\fT^dt = 2n-[tVT^ + \n(t + VT+T*)]\ =

= ji[J/'l+ln(l+j/~2)].
8.3.3.

Compute the

area enclosed by the loop of the folium of Des-

cartes

x 3 -f if

3axy =

shown

Solution. The folium of Descartes is


present the curve in polar coordinates:

x^pcoscp;

Then p 3 cos 3
we get

cp -f-

sin 3

cp

3a p

cos

cp

3a cos

cp

^ ~~ cos 3 (p+

0.

in Fig. 86.

= psincp.
= 0, whence,
sin
cp

sin

cancelling p 2 ^

cp

sin 3

cp

Since the loop of the curve corresponds to the variation of

and

the sought-for area

equal to

is

jt

r*

9a
^

n
r

Let us re-

2
2
cp
sin'cpcos'cp
sin
cp cos

cp

between

8.3. Geometric and Physical Applic's of Improp. Integr's

To evaluate
tan

cp

t\

= dt\

CO gf

9a 2 f

proper

the obtained

dt

cp

at

9a 2

,.

= 0,
i

make

integral
cp

=y

at

3a 2

df

411

the substitution

Thus we

oo.

,.

get

<4

8.3.4.

soid y

Find the volume

= 2a

^s

a k u *

The

generated by revolving the

of the solid

as Y

"

cis-

m ptote x=2a.

shown

in Fig. 119. Transfer the origin of


coordinates to the point 0' (2a, 0) without changing the direction of
the axes. In the new system of coordinates
x
2a, Y
y the equation of the cissoid
has the following form:

Solution.

X=

cissoid

is

(X

Y2
The volume

-X

the

of

+ 2a)*

= Ji$ X

dF

about
asymptote, is

of revolution

solid

the axis X = 0, i.e. about


expressed by the integral

the

= 2jcJX

dY.

Let us pass over to the variable X. For this


purpose we find dY= Y' dX. Differentiating the
equation of the cissoid in the new coordinates
as an identity with respect to X, we get

2VT'
whence

for

=
Y

3(X + 2a) 2 X (X + 2a)3

2 (X~^~2a) 2

X2

>

Fig.

(X

119

a)

we have
(X

+ 2a)*(X-a)

(X

+ 2a) (X-a)
'

X2Y

X }f-(X + 2a)/X
2

Hence,

= 2n C(X 2a) (X a) :dX.

V- (X + 2a)/X

Make

the substitution (X

/ = oo.

+ 2a)/X =

2
;

X = 2a

at

= 0, X=0at

Then:

2a

1+/ 2

dX:

2at*

\at
:

+
X-a =
(1

2 2

X + 2a- +

dt\

3o

fl/

2
(

412

Ch. VIII. Improper Integrals

whence

v
V ~ 9

+ at
t(\ + t*)(\ +

f
)

2at 2 (3a

_
~

4at dt

t'*)(\

+ t*)*

= 48

(trw dt +

16

ttf

dt -

Putting t
tanz, dt
Hence,

= sec

zdz, we get

at

= n/2.

JT/2

V=

A8na's

= 0, ^=oo

at

JT/2

sin 2 z cos 4

zdz
JT,/

= 48jia

16jta 3 ^ cos 2 z sin 4 z dz


JT/2

48na

cos 4 zdz

cos 6 zdz

JT/2

JT/2

16jta 3 ^ sin 4

zdz

16jia 3

sin 6 zdz.

JT/2

Using

known

the

formulas

the

for

integrals

s\n n

xdx,

JT/2

^cos^xdx

(see

Problem

V = 64 jca 3
8.3.5.

Prove that

t/=-7=L=,
V

the

finite

is

the

64 ra 3

1x3x5 -

area of the region


abscissas, the

finite

and equals -y

is

^e

the

bounded by the curve


ordinates and the

axis of

area of the region

axis of abscissas and

and equals

the curve

axis of

Prove that

y^=t

get

asymptote x
8.3.6.

we

6.6.9),

# = ~2>

6,

and the area

the

of the

bounded by the curve

straight

lines

x=:l

is

region contained between

the axis of abscissas and the straight lines

x=:l

infinite.

8.3.7.

Find the volumes

of

the

generated by revolving the lines y


(a)

about the x-axis,

(b)

about the y-axis.

solids

= e~ x

enclosed by the surfaces

= 0,

(0

<

-(-

oo):

8.3. Geometric and Physical Applic's of Improp. Integr's

and

= 2a _ x

area contained between the cissoid y 2

Compute the

8.3.8.

413

asymptote.

its

Compute

8.3.9.

and the axes


8.3.10.

the area bounded by the curve y

= e~ 2x

(at

>

0)

of coordinates.

Find the volume

generated by revolving, about

of the solid

= 2^ ^j

the x-axis, the infinite branch of the curve y

for

x>

1.

and attract a
be located at the origin
8.3.11. Let a mass
and
found on the x-axis at a distance x from
material point

having a mass

of

1,

with

F=

force

-r

(according to Newton's

moves
law). Find the work performed by the force F as the point
along the x-axls from x=r to infinity.
Solution. The work will be negative, since the direction of the
force is opposite to the direction of motion, hence
N

oo

A=\
J

^xT dx=
2

lim

n-

=
^rdx
2

oo

J
t

from infinity to
During the reverse displacement of the point
r the force of Newtonian attraction will perform posithe point x

tive

work ^-

This

quantity

under consideration at the point


potential energy accumulated at
8.3.12.
"ballistic"

the

called

is

x=r
a

potential of the force

and serves as the measure

of

point.

In studying a decaying current resulting from a discharge


instruments are sometimes used whose readings are pro00

portional to the "integral

current

intensity"

g=

dt or the "inte-

00

gral square of current

intensity"

S=^I

dt

and not

to the

taneous value of the current intensity / or to its square


is time measured from the beginning of the discharge; /

instan-

2
.

Here

alternating-current intensity depending on time. Theoretically, the process


continues indefinitely, though, practically, the current intensity becomes imperceptible already after a finite time interval. To simplify
the formulas we usually assume the time interval to be infinite in
all calculations involved.
Compute g and S for the following processes:
(a)

cient,

=/

e~ kt (a simple aperiodic process); k


is greater than zero.

which

is

is

a constant coeffi-

414

Ch. VIII. Improper Integrals

=I

e~ kt sin
are constant.
Solution.
(b)

(a)

g=

e~ k1

l
\

(simple oscillating process); coefficients k and a>

cot

dt=

lim

V"*' dt =

lin

oo

J /;*-*<= -'JA

oo

(b)

g=

\ I

()

e~ kt s\n

cot

dt

lim
A -

= -^f7T2
+
o)

/2

^ _ n

oo

5-

00

sin

(o/d/= lim
A -

[(cocosorf
LV

=A
46

lim
- 00

co

+6

=
7

'

li e

-* kt

~ co

* 2<ot

dt

/l

dt

cot

+ ftsinwOe-"]^-^^;
or-f-&

co

sin

V"*'

^4

f
J

2
(A cos 2cot
;

2 *

J o

(o& sin 2a)t)

9 -2kt

4k (k*

8.3.13. Let an infinitely extended (in both directions)

co )

beam lying

by a concentrated force P. If the


x-axis is brought to coincidence with the initial position of the axis
of the beam (before the latter is bent) and the y-axis is drawn
through the point
(at which the force is applied) and directed
downwards, then, on bending, the beam axis will have the folloon an

elastic foundation be bent

wing equation
y

= ^ e~ a

1x1

(cos

ax +

sin

a x
\

|),

where a and k are certain constants. Compute the potential energy


of elastic deformation by the formula
oc

W = Ee J (y"f dx
Solution.

Find

(E, e const).

y"\

y"

Pol
ax [(cos ax
= --e~
+ sin ax) 2 ( sin ax+cosax) +
-j-(

sin ax cos ax)] = -e~ ax (sin ax cos ax).

415

8.4. Additional Problems

Hence,

P 2 a 6 Ee

k2

e~ 2ax

(1

2 sin ax cos

ocx)

dx =

J
o

P 2 a Q Ee

2a
8.3.14. What work has to be performed
from the Earth's surface to infinity?

Determine the work which


e 2 =l
from infinity

8.3.15.

charge

electric

charge

8.4.

e1

to

move

has to be
to

P 2 a*Ee

2a
4a + 4a 2

unit

body

4fc

of

mass

done to bring an
distance from a

Additional Problems

8.4.1.

Prove that the integral

dx
xP In* x

converges

for

>

and

<

1.

8.4.2. Prove that the integral


00

converges absolutely
tionally for
8.4.3.

l<(/?+l)/g<0

for

0<(p+

l)/</

<

and converges condi-

1.

Prove that the Euler integral

of

the

first

tion)
i

B(p,

q)=\ xP-

(l

x)"-

dx

converges for p

>

and

> 0.

8.4.4. Prove that

lim
7
if

-> a>

sinax-sinP^dx^O,

|a|=HP|.
8.4.5. Prove that
oo

= ^e-

xi

-x* n +

dx=Y

(n natural).

kind (beta func-

Ch. VI II. Improper Integrals

416

Prove that

8.4.6.

the integral

if

converges

J-^

any posi-

for

and

tive a

if

f(x) tends to

x> 0,

as

then the integral

00

/(ax)-/(N

<fa

(a>0( p>0)

converges and equals

/4ln(P/a).

Prove that

8.4.7.

00

cc

g"

^x

__ f cos

ax

cosfk
x

dx
X
~ \nn

^
'

"cT

JT/2

At what values

8.4.8.

of

cos X

does the integral

dx

con-

verge?
ji

Prove that the integral

8.4.9.

P
J

diverges

if

dx

j^^k

converges

<

and

if

converges

1,

k^\.
00

Prove

8.4.10.

<s<

and

4,

8.4.11.

sin

that

the

integral

*(*^~ CQS

*)

converges absolutely

if

<j

if

< s < 4.

Suppose the integral


+

00

lf(x)dx

(1)

converges and the function y(x)


Does the integral

is

bounded.

lf(x)<p(x)dx

(2)

necessarily converge?
What can be said about the convergence of integral (2),
ral

(1)

converges absolutely?

8.4.12. Prove the validity of the relation


/ (x)

where f(x)

= 2/ (ji/4 + x/2)
^ In cos

y dy.

2/ (n/4

jc/2)

In 2

if

integ-

Compute with

8.4.

417

Additional Problems

the aid of the relation obtained


JT

8.4.13.

(-^)

= j

In cos

Deduce the reduction formula

ydy.

for the

integral

JT

In

and evaluate

=^\ncosx-cos2nxdx

this

integral.

(n

natural)

ANSWERS AND HINTS

Chapter

p
=
2

(b)

1.1.5.

Hint. Prove by the rule of contraries, putting 2

common

are positive integers without


1.1.8.

Hint.

1.1.9.

(b)

You may

take k

where/? and q

multipliers.

2s

1.1.11.

of

x^?4,
x

(a)

which

for

<

*<0;

b\

16|

>

The equality

////i/.

2^x<:3.

(b)

0;

values of x for which x 2


2
1.1.13. (a) x < -g- or
\a

x^sl.

or

4<x<2.

(c)

valid for those values

is

//m/. The equality holds true for those

5x-\-6^0.
a:

>

(b)

8;

<

<

or

when a and

good

holds

are

<

//m/. The inequality

5.

opposite

in

sign or

when

\a\<\b\.
' 2 - 3

'

+
+ a&+a
=
10
5x2*.
+
/(*)

1.2.4.

1.2.11.

[aV + ta +

&'-

/t

<+>"->
3
)

_L

1-2 .6.

V'l+l;

4"

'
;

^10-5.

,,,,

= 5; / (0) = 4; / (0.5) = 4; / (-0.5) = -I / (3) = 8.


+d
= a Xn a a
follows that n + = a Xn+i = a Xn
1.2.15. tfm/. From
+
+ =
1.2.18.
(x) = 527.
1.2.17. / (x) = x 5x+6.
1.2.16. x=2; 3.
(*) = 23;
S = &^1--^x.
1.2.20. P = 26 + 2
or x> 2.
1.2.19. x^
^
1

.2. 14. / (2)

it

i/

1.2.21. (b)

2,
(g)

...).

(-^,

Since

and

< j+2kn(k =

(c)

3);

(2,

Hint.
2)

a:;

(3,

oo);

( oo, 1)

sin#^l,
(h)

1, 2,

[1,

...).

4);

and

the
(i)

(2,

oo);

function

(-2,

0)

<p

is

and

(d)

= ^ + 2kn(k =

defined only
(0,

1);

(j)

0,

1,

when sinx=l;

-iL + 2tot<*<

Answers and Hints

1.2.22.

points

1.2.24.

(1,

(d)

The function

(d)

defined

is

x= 2.
(~oo,

(a)

and

0)

1.2.26.

(b)

oo);

1.2.25.

oo).

(0,

(3
(b)

2ji<a:<(2+1)

(a)

ji

(b) Hint. Consider the

1.3.3.

Ch.

to

over the entire number scale, except the

2jt,

and

ji)

5<x<6.
(* = 0, 1, 2,
*2

difference

(b)

increases

It

.-ffcri

for

<

\+x\

^ + kn

<

3];

y,

(b)

[1,

(c)

4);

(3,

...);

\+x\
1.3.4.

419

(k

1,

2,

...)

and decreases on the other intervals.


JI

< x^-

The function decreases on the interval

1.3.7.

from 4-oo to 2 and

ji

<

increases on the interval

The function

ji

rorn 2 to

+-

neither even, nor odd, (d) even.


1.3.10. (a) Even; (b) odd; (c) odd; (d) neither even, nor odd; (e) even.
1.3.9.

(c)

1.3.12.

\A |==5,

(a)

is

co

14
= 4,

= 5, =

cp

= 0, ^=y;

(b)

= 4,

(0

= 3,

q>

-5-,

= arc tan y r = //m/. 3 sin y + 4cos y =


= 5sin ^y+cp^, where cos q^^, sincp = y. 1.3.13. (b) T = 2n\ (c) T=
The function reaches the greatest
1.3.16. The greatest value /(1) = 2.
T=

2ji

(c) |/1

(o

a:

cp

a:

4ji.

1.

////if.

value at the point where the quadratic trinomial 2x 2

4#+3

reaches the least

value.

Even;

(b) even;

odd;

1.3.17.

(a)

1.3.18.

(a)

1.3.19.

Hint, (a) Assume the contrary. Then

T = n;

r = 6ji.

(b)

(c)

#+r+sin
(

whence cos

x-\

T \ =

(x-\-T)

which

(d)

even.

= x-\-s'm x,
impossible

is

for

any constant 7\

the

since

2sin "^-

side

left

cos \^x+T = cos


= 2nk (k = \

not

is

constant;

whence either

x,

suppose

(b)

the

7= = =
>Y

2, ...), which is impossible, since the left-hand


t
of these equalities are functions of a continuous argument x.
t

1.4.6.

x=

(a)

log 2 ^
1

g2

1.6.3.

(c)

==
1

(a)

1+arcslnj/

(b)x

lQ gj/

(0

<

<

= 3sin*/;

2 or 2

<

<

(c)

= y lo 8

(y

Then

contrary.

Vx+ T4- V~lc=2nk, or

>

members

(d)*

0);

00).

.og|

0,

(b)

T>

- y,

2;2.25;2g; 2{g
1.6.9.

Hint.

The

inequality

|^^~2 <

is

satisfied

lor

> N

Answers and Hints

420

= Ey= 0.01

At 8

lj.

= 0.1

the

inequality

is

beginning with /z=10, at

fulfilled

beginning with /z=100, at 8


0.001 beginning with /z= 1000.
Hint. Verify that the sequence {x 2n -\} tends to
1.6.10.
as n
oo,
oo.
as n
and the sequence {x 2n \ tends to
(c) it has; (d) it does not have.
1.6.12. (a) It has;
(b) it does not have;

Hint. For a

1.6.19.

= (\-\-a n

inequality a

/ =-

<a)|*J<-|;

Hint.

1.6.14.

(a n

V/ a

>

put y/~a= + a n
>
> na n prove that a n
1

n
)

1.7.1.

(b)

A;

1.7.4.

(b)

1;

xn

of

>

and, with the aid of the

0)

of the

(c)

(e)

0;

1.

a =

inequality

1;

1.7.2. (b)

(e)

1.7.5.

(g)

(h)

1.

Hint.

111

J_

2'

(a)

finitesimal,
1.8.6.

(b)

and cos n 3

(b)

2x3

form x n
(c)

1;

1.

'

/i(/i+l)~

'

= ^
1

-f

1'

--i.

(d)

0,

11

summand

each

Represent

Hint. The quantity

is

an

in-

bounded quantity;

is

Hint. The
wand

X3x...X/?^2

put

> na n

bring xn to the

will

an

(f)

1;

the form of the difference


1

([

-\-

Hint. Multiply and divide by imperfect of a sum, square

(f) 0.

1X2

<

an infinitesimal. For a

is

and make use

0)

divide by n

in

which

(a n

and then

\<.

<x n

-)-

\x n

(b)

sequence

is

(f)

(e) 0;

bounded

due

n\=^\x2x

to the fact that

therefore

^H4)'+-.+(^n=3-()-'<,
(b) 0.

1.8.7.
1.8.9.

<

//t/i/.

<

For

all

/z,

fulfilled;

beginning

with

therefore

J=-

<

xn

certain
,"/

= n

value,

<

{!/ n,
*

<

i.

o
the inequalities
-f-

and lim iV

/i

iy

= lim

1.8.10.

= J^G/
The
lim y n
fl

are

2!

Hint. Take advantage of the fact that

->

Hint.

>

The sequence {y n \

decreases,

since y n i

=a

'l

= a 2flX2 =

0-

boundedness of the sequence from below


by & and from the relation y n + i= Vy n

follows from a

>

1.

Denote

b=\.

oo

1.8.11.

//m/. Ascertain

ness from the

that

the sequence increases. Establish the bounded-

inequalities

n(n\)

*<'+('-)+(T-T)+-+(.-r-7)--f

A n skits and

/lints to Ch.

Hint. Transform x n into the form xn

1.8.12.

421

2n

yn +
2

and take advan1 -j-

tage of the inequalities

2n

2i

2n

<

<

Y& + + n

1.

////i/. See Problem 1.8.7 (a).


//m/. Establish the boundedness of
with the sum of some geometric progression.
1.9.2. (b) Hint. Choose the sequences

1.8.13.

and

and

ascertain

sequences

the

that

xn

by comparing xn

sequence

the

1.8.14.

</i=l,

...)

2,

appropriate values of the function

of

have

different limits:

lim 2
1.9.3.

arc tan x <

=0.

lim 2

the inequality

of

arc tan

tan (
V 2

x'
n

+ oo,

Xn

Hint. Take advantage

(e)

71

(f)

(x

>

0).

Hint. Transform the difference

=
1

sin x

-\

(d)

(e)

~; (f)~.
o

jx

-g-

|sina|^|a|.

into a product and apply the inequality


1.10.1.

sin
.

sin x

Hint. Multiply

the

numerator

and de-

\Z

nominator by imperfect trinomial square (jj/ 10

x-f-2);

(g)

~~

(h)

log fl 6.

'

Hint,

lim
log a
at-v3L
V x

= log a

+ 6 3J

lim

(*-3)(^* + 6 + 3)

= log a 6;(i)

x3

L*-3

<j)^.
1.10.2.

(e)

On removing

////i/.

the irrationality to the denominator divide

the numerator and denominator by x.


1.10.3.

(b) 32.

-5-.

(c)

as

Put

Hint.

(g)

'

x=z 15

(f)

Hint.

oo.

x =z;

Put

3. //m/. Put s\nx = y.

i
1

1.10.5.

1.10.7.

1.10.11.

(b) e

(b)

(c)

(a)

*"

(d)

(f)

4;

cot a

1.10.8.

(b)

(d)

1;

_1_

(c)

'

(cl)

2
1.10.12.

(a)

(h) 2.

(g)

j_
20

(b)

-2;

(c)

y>

(d)

(f)

(e) 0;

'

n
(

e)

~ 24

1.

Answers and Hints

422

(a) e 4 ;

1.10.13.
(h)

(i)

1;

9;

-1;

(b)

(k)

1;

(j)

1.11.6.

(b)

They

(a)

100*

is

(a)

It

(e)

of

the

a sin a

tan

~~T'

a3

smallness;

order of

first

order

second

of the

an infinitesimal of an order of smallness lower than x.


smallness;
is of the fourth order of
(b) of the first order of
of the third order of smallness;
(d) of the third order of
(f) of the order of
of the first order of smallness;
smallness

(c)

(g)

(i)

L=a 0.

are of the same order;


(c) they are equivalent.
an infinitesimal of the same order as x\
(b) x 2 is an
order higher than x\
(c) 6 sin a: is an infinitesimal of the
infinitesimal of an order higher than x\
(d) sin 3 x is an

infinitesimal ot an
as x\

-fax

~* Q

is

same order

smallness;
smallness;

1;

1.11.8.

\/ tan 2

(g)

It

|.

1.11.9.

(at

rj

lim e?*i
X

a-

(e)

(f)e-J;

j/lT. Hint. Replace arc cos (1 x) by arc sin }/~2a; x 2 (b) l;(c)a.
is of the third order of smallness. Hint.

1.10.14. (a)
(b)

(e)

Hint.

$X

oiX

lim
X-*

1.11.5.

p.

(d) e 3

2 In a;

(c)

(h) of the first

order of smallness;

smallness. Hint. Multiply and divide the difference

oi

jj/cos x by imperfect trinomial square;


(j) of the first order of smallness.
The diagonal d is of the first order of smallness; the area S is of
the second order of smallness; the volume V is of the third order of smallness.

cos

1.11.10.

1.12.3.

(b) 4;

(f)

3;

1.12.7.

(a)

(b)

(f)

(g)

(h)

(a)

/( 0)

~;

1.12.8.

(i)

(a)

1.12.9.

1.

/(-0) =

/(

1.12.6. (a)

2.

(b)

Hint.

10.14.

Ml 0) = 2,

+ 0)=+co.
1.13.3.

(c)

_2;
(b)

1.13.1.

/(2

1;

(g)

1042=

/(l+0) = 2;

_ .
f

= 0;

(b)

10 3

the

discontinuity

of

( oo)=

first

kind:

^-

kind at the point #

first

arc tan

(+ oo) =

=5

=
(c)
+
/( 0)=1, / ~H 0) = 0;

1.14.6. (b)

At

(e)

(1+0.042).

/(-0) = 0,

/(

+ 0)=+oo;

/( + 0)=1.

1.

(b) The function has a discontinuity of the


The jump is equal to 27.
1.14.3. (c) The function is continuous everywhere;

/(5_0)=

/(2-0) = -oo;

kind

first

= 3.

arc tan

(d)

(f)

1.14.2.

(c)

(b) 2.

1;

the

point #

JI

= 0;

/(5

0)

there

at

the function

(e)

ji.

has
Hint.

first

kind:

jump equals

the

-]--^is

discontinuity of the

JX

the point

at

the point

a:

0,

discontinuity of the

a
JT

the point ^

(d) at

=y

an

infinite

dis-

continuity of the second kind:

1.14.7.

(a)

At the point

the discontinuity

it

x~0

is sufficient

there

is

removable discontinuity. To remove

to redefine the function, putting /

(0)=

1;

(b) at

Answers and Hints

is

there

is

the points x

is

to extend

sufficient

there

423

removable discontinuity. To remove the discontinuity


(c) at the point #
the function putting /(0)=1;
-j- oo;
kind: /( 0)
(d) at
a discontinuity of the second
0, / ( -|-0)

the point x
it

to Ch.

(2k hO-j

(^

= 0

1 2,
f

flx)=UmJs\nx)*=\

= kn (k~ 0.

the points x

(e) at

removable discontinuities, since

).

1, 2,

if

jf

sin x

<

1,

s in^

1;

discontinuities of the

...),

1st

kind,

since
-

if

sin x

if

sin

1, 2,

...

sin x

(f)

at the points

= n 0,
.

At the point

sin

x=

a:

0,

0;

removable discontinuities, since

x=n

if

>
<

a:

is an infinite discontinuity of the second


discontinuity of the first kind (the jump
being equal to 2); (c) at the point #
0, an infinite discontinuity of the second
kind, at the point x=\ a discontinuity of the first kind (the jump being equal
(d) at the point x\, an infinite discontinuity of the second kind.
to 4);

1.14.8.

kind;

(a)

(b)

point

the

at

there

2,

1.14.9.

(a)

/(0)=1;

1.15.2.

(b)

The function
The function

1.15.3. (b)

lim

(c)

lim

\imy=
Jl

/(0)=^;

(c)

(d)/(0)

= 2.

is

at the points

= --|;

is

y\\mu =\\

continuities, since

/(0)

continuous on the interval (0, +oo).


continuous everywhere. At the only possible point
we have

of discontinuity

(b)

(n

lim y
U-*cc

= 0,

lim u 2

=l\

1, 2,

x_

there

...)

=y

B= _ t

are removable

~>

Yes.
on the interval

1.16.12.

1.16.2.
1.17.1.

1,

No. For instance, the function y

1.16.13.

1.53.

1].

Hint. Multiply the obvious inequalities:

(a)

<

n'

V2(n-\) <

-4-

/(S=T^<=1;

.f.

rr*

b) Hint.

Let

^-o-X
2
D

2n

T34 X T56 X...X >2n


2

1
.

2n
'

Then A

< B

since

dis-

1.

2/712/t

<
2n

. and

2n-\-\

'
'

2n

f-

A'1

< AB^--

r
2n -f

=x

Answers and Hints

424
1.17.2.

Hint.

(a)

Extract

the

from both sides

root

101st

inequality

of the

2
and reduce both sides by
(b) Multiply the obvious inequalities:
1

99
98
2

1.17.3.

(a)

<

<

100

199

or

X
X

101

x
X

198

<

102

<
<
<
<

200

<

100 2

100 2

100 2

100 4

<

(b)

3;

>

or x

(c)

the

inequality has no solutions, since it is equivalent to the contradictory system


1.17.4. Yes.
1.17.5.
(a) No; (b) Yes.
2 > 0, a:(4a: 2
*+4) < 0.
1.17.7. Hint.
Apply the method of mathematical induction. At /i=l the
relation is obvious. Supposing that the inequality

(l+*i)(l+* 2

...

(l+*-i)^l+*iH-*a+---+*-i

holds true, multiply both its sides by \-\-x n


(i=l, 2,
conditions \-\-x n >0, Xi>x n >
1.17.8.

(a)

a:=0, 1, 2,

(e)

[-4, -2] or
1.17.9.

for

all

cp (a:)

a:

(a)
7= 0,

only

for

+oo);

[1,

(c)

No:

and

cp

(0)

x^0;

take into consideration the


1).

< x< (2/i+

1)

for

(x)

0)

(/i

f (x);

= 0,

jx

(/i

= 0,

1,2,

nowhere

is

1, 2,

...);

defined;

...).

and / (0) is not defined;


(b) No: f (x) is defined
only for x > 0; (c) No: f (x) is defined for all x, and
Yes;
(e) No: f (x) is defined only for x > 2, and cp (*)

(x)

cp

( oo,

*=(2/i+l)y

(f)

4];

[2,

(2/zjt) 2

(b)
(d)

and

(d)

2 and for x < 1.


3) (6
1.17.11. V
8xt (x
1.17.10. (a) (0, oo);
x), 3 < x < 6.
(b) [1, oo).
1.17.12. (a) x
5. ////i/. The domain of definition is specified by the inequalities x-f-2^0, x
5^0, 5 x^O, which are fulfilled only at the point
that the number x
5 satisfies the given inequality, (b) Hint.
#
5. Verify
The domain of definition is specified by the contradictory inequalities x 3 > 0;
2
x > 0.
r, X
nX _J_ n X
9
fiX
y
"
(b) a*=
(see Pro1.17.17. (a) f (x )
7l
for

>

+ T1?

blem

1.17.16).
1.17.18, An even extension defines the function
,

f(x)

=x

-{-x for

x for

An odd

0<

a:<3,

3 < x <

0.

extension defines the function

f(x)=x 2 + x

for

0<*<3,

If the function
a period 7\, and the function cp (a;)
/ (x) has
and T l
n 2 d (n lt n 2 positive integers), then the pen 1 d, T 2
nd, where n is
riod of the sum and the product of these functions will be T
the least common multiple ot the numbers n x and n 2
1.17.22. Hint. For any rational number r

1.17.21.

////if.

has a period

7\>,

>=m*)={
'

But there

is

no

least

number

in

i
for rational x,
n
; for irrational x.

the set of positive rational numbers.

Answers and Hints


1.17.23. Hint.

wc denote
we get

II

Ch.

to

425

the period of the function

= f{0) = f(-T)

f(T)

aT =

sin T-\~ cos

whence

= sin 7)-]- cos aT),

7 = 0, cos^r^l, and hence T = kn

sin

by T, then from

f (x)

aT 2nn

2n

is

rational.

k
1.17.25. The difference of two increasing functions is not necessarily a monox and g(x)
x 1 increase for
tonic function. For example, the functions f(x)
x^O, but their difference / (*) g (x) ==x x 2 is not monotonic for x^O: it

and decreases on

increases on

0,

Example:

1.17.26.

a:

-{1.17.27.

(a)

= -j\n

if

a:

if

1+0 (-1 <

-y

is

<

rational,
irrational.

is

1);

(b)

x=<

oo

<

<

lor

V~y

1,

16,

<

16

log 2 y for

<

oo.

The functions y=x 2 -\-_

1.17.28. Hint.

+2x+l

for

= X+V*
=
+

and y
mutually inverse, but the
2
x has
x, i. e. x
2x-fequation y
no real roots (see Problem 1.4.4).
1.17.30. (c) Hint If is the domain

(x^O)

(x

are

1)

the
definition of
then the function y
of

function

= f[f(x)\

(x),

is

defi-

xE

for
those
which
the points of the desired
graph are plotted is shown in Fig. 120.
2 (b
1.17.32. Hint. The quantity T

ned only

f(x)E.

for

How

=
a)
a period: from the conditions
= f (a x) and f(b + x) = f(b x) follows that
[x + 2 (b-a)] = f[b + (b + x2a)]=f(2a-x) = f[a + {a--x)] = f(x).

symmetry
/

1.17.33. (a)

diverges;

It

(b)

= n

that

xn

=n

xn

= 3,

yn

The converse

|.

= 0,

(x n y n )

= oo.

oo

(b)

incorrect.

is

No.

Take

into

Example:

Hint.

1.17.36.

may

The sequence a
1.17.38. Hint.
turned
9. If this sequence
0,1,

-+

= n-{-\;

...).

4,

(x n y n )

-* oo

lim
n

(n

lim

yn

n{n ~ 2)

1
1

converge or diverge. Examples:

either

+ (-I)].

Example: xn

1.17.34. (a) No.

a =

may

it

--

yn
xn

of

it

1.17.35.

is

(a -\-x)

account
1)"

1
.

only the following values:


out to be monotonic, then the irrational
number would be represented by a periodic decimal fraction.
1.17.39.

Hint.

If

the sequence

i.

~
On

6 +l a

attain

increases,

<

an +

bj

then

(/=

1,

2,

)>

Answers and Hints

426

whence

follows that

it

t>

+ a +...+a n

+ i(<*i

<

+ b +...+bn

a n + l (b 1

and hence
"1

&1

+ ^2+ + +
+ &2 + ... + ^ w +

0/1

+ 02+ "-+fl/l ==
+ &2+...+&*
g
^ +
+
== W + ft +
(bi + b +...+b n +
^1

&1

1.17.40.

nx

(a)

< E

(b)

2;

(nx)
v

'

^ nx

From

1.17.42. Hint.

<

fl2

From

Hint.

< ^

x
n

>

'

nX ^

the

inequalities

x.

the inequalities
n

ix 2

^=1
it

)(b l

1.17.41.

0.

(c)

0;

follows that x

it

+ + +
+ b +...+b n

^2

fe=i

/z=i

follows that

/i+l

2/i

nl

2n
i

Hint. Take advantage of the fact that

1.17.43.

lim a

lim
n

-* go

yfa

(see

cc

Problem

lim

1.6.19),

rt

"

=1, and

00

qualities a
1.17.45. Hint.
1.17.46. (a)

->

for

>

1,

/i

<

the ine-

co

1_
rt

lim

ah

<

<

take

place.

Divide the numerator and denominator by x m

a=l;

= l;

a=l; b=-^

(b)

Hint. To find the coefficient

a divide the expression by x and pass over to the limit.


1.17.47. (a)

= x
M-{
W
1

/
f

for

0<x<

for

<

>

1,

1.

(b)

+/2JI,
JX

for

for

/(*)H

= -{-nji

Take advantage
(1

(/i

1.17.48. Hint.

?=

)...(1+a: 2 ")

1.17.49. Generally speaking, one can't.

lim
X ->

and

if

result:

we

+ *) + ln(l-*) =
X2

~ X =Q. +

replace In

lim

( 1

a:)

1,

2,

...).

of the identity

x)(\+x) (1+x
ln(l

= 0,

by x and

For example,
]ijn

X
In (1

= 1 x* n

ln_0- f 2)
X2

-*

x)

by

=_

we

will

get the

wrong

Answers and Hints

1.17.50.

Hint.

then

sideration,

If

chord

the

equal

is

subtended by the arc under con-

angle

a central

is

2R

to

427

Ch. I

to

sin

Ra,

and

the

sagitta

to

a2

R(\ cosa) ~ R-.


1.17.51. 2. Hint. The difference
inscribed regular /z-gons is equal to

/.

on
2tf/z

jt

,
n

where a

sin a
n
n = 2nR..tana
~ tiRcc
a
jr

tan

sin

perimeters of a circumscribed and

the

of

.,

2
,

and the side

of

an inscribed rc-gon

2Rs\n = 2R sina~

2Ra.

1.17.52.

On

the equivalence of (1-j-a) 3

1.17.53.

No,

logd+^l!^-^

is

and 3a

as

>0.

as*-+0.

If
the function cp (x)
/ (x) -\-g (x) is continuous at
then the function g (x)
cp (x)
f (x) is also continuous at this
g(x) sign x (see Problem 1.5.11 (p)); both
point; (b) No. Example: / (x)
and their sum is identically
functions are discontinuous at the point x
0,
equal to zero, and is, hence, continuous.

1.17.54. (a) Yes. Hint.

=x

the point x

for

1.17.55. (a)

No. Example:

Xf:0, g(0)

functions
pie:
r

= g (x) =

/(#)

point

= 0,

<

m
No.

for

^
<

''

product

their

1-r ec
1.17.56.

=x

is

continuous everywhere, and

#=

continuous at

/ (x)

being discontinuous at the point #

function

is

=
=

Example:
K
i

f /

0; (b)

of these

No.

Exam-

functions

are

discontinuous at the

0;

f (x)

i-

g(x)~ sin-^-

The product

lim x sin =
x

since

both

= 0.

f(x)
v

=<
I

being
1

if

continuous everywhere.
is
.

rational,

We may

irrational.
the Dirichlet function (see Problem
'

if

a:

.,

write

is

1.14.4 (b)).
2\ (x)
1, where X (x) is
1.17.57. (a) #
is a disconti\
is a discontinuity
of the second kind,
nuity of the first kind;
(b)
is a discontinuity of the first kind: f(\
0,
0)
f (x)

( 1

H 0)

1.17.58.
x-+

(c)

(a)

cp

(x)

is

discontinuous at

= n = 0,
y=y

1,

lim

n-0

(n\

x=\

y=\,

lim

x==n

points except x
are discontinuities
all

2, ...
The function has

= 0.

= 0.
of

a period of

1;

the
(b)

first

kind:

x=

x-> n +
...) are points of discontinuity of the first kind:

2,

lim
x

-*

Vn-0

= 2n

1;

lim
y
Vn-rO

x=V'n

= y\ X ~_ y = 2n

The function is even;


(c)
(/i=1, 2, ...) are the points of
discontinuity of the first kind; at these points the function passes over from the
value
to
and returns to 1. The function is even;
1

Answers and Hints

428

(d)

y=]

if

if

-75-

|
1

sin x

sin
s

I
1

i.e.

^-\-nn < x <

i.e.

>

x= -\-nn

= ~2

a:

sin x

if

<

i.e.

^--f-jx/z,
6~

111

-^-f-ji/z,

<

J1/1

<

|-jw.

JI

are discontinuities of the

first

kind.

1.17.59. The function f[g{x)\ has discontinuities of the first kind at the
The function g[f(x)] is continuous everywhere. Hint.
points x
1; 0; +1.
The function f (u) is discontinuous at u
0, and the function g (x) changes sign
since f (x) attains only the
at the points #
0, 1. The function g[/(#)|==0,

values

0,

1.
Write the function

1.17.61. Hint.

f(x) = {

that

sure

2,

= 0:

Choose the points

and

Hint.

= f(x) x.

0,

'

< x<2.

from

The function

[0, 2].

to

on the interval

= /(*o)

c,

/(&)-/

[a,

b\

is

[/

to),

. ,

2X

<1

(*)].

(* 2 )

= /(*o) +

e.

theorem

value

[/

to)

to) +...+/ to)]

< max

Apply the intermediate value theorem

on the interval

We

x 2 so that

*2
*o)Apply the intermediate

/ (x n )]

1.17.65. Hint.

0)

2,

chosen.

to

the

function

1.17.64. Hint. Apply the intermediate value theorem to the function


the interval [x lt x n ], noting that

min

discontinuous at the point

is

given and the point

is

Apply the intermediate value theorem

= min(^ *i>

1.17.63.

g(x)

increases

e<min[/(x )-/(a),
x x and x 2 x 1 < x <
/(*i)

and put 6

2<x <

tor

interval [0, 2].

1]

/(-0)=1, /(+0) = 0.
1.17.62. Hint. Suppose e >
may consider that

for

function

the

3
to on the
and from

to the intervals

for

(*+l)2
Make

the form

in

x+

lj

[/

to),

to the function

f (x)

on

(x)].
(x)

At sufficiently large values of the independent variable the


values of the polynomial of an even degree have the same sign as the coefficient
at the superior power of x; therefore the polynomial changes sign at least twice.
1.17.67. Hint. The inverse function
1.17.66. Hint.

Yy\
Vy\
is

continuous in the intervals (00,

= 0.

for

for

<-l,

= 0,

for y

1) and

(1,

>

00)

and has one isolated point

Answers and Hints to Ch. II

Chapter

II

2.1.1.

20
-

(b)

= 3x

2.1.6. (a) y'

2.1.2.

the indicated points.

2*arctan*+l.

._ ,
+r
l^x

~2x
f

e''

a;

'

'

(f)

i/'

6a:

(b)

2.2.6.

*/'

(b)

3;

-f-

= -3(3-sinA:)
o /o

u'2e* +
3

2M
l

-j

2*

x9
2

In 2

*'

(g)

VI;

= 2e*cos*;

(h)

j,'

3^==+5;
y
2 cos

(c)

a;

(e)

l)

^'

(f)

*/'

a:

(2a:

2 sin

cos

sin x

= 3 cos

+
5^

h)

h-ri 5;
tanx^\+x*

(i)

(/'

5)cos(a: 5A:-H)-Asec

a:

sin

=-

3a;

arc

a:

x/l + ln**

cosA:;

5 In 4

3/(2e*-2* +

sec'2

="2^

3 sin x

(d)
K)

*/'

(d)

(g)s\ny^=

301nMtan3*)^;
/

2(1 a:)
~

(c) y'

2.2.2.

a:?*

(cos a: sin a;) sin


x2e x

2.2.5.

m/sec.

= 2e*+y;

*'

(0

= 25

+ 8^+ 2^./x
w ,__ 3^6(*-2

D2

v av

non-differentiable at

is

2.2.4. (a)

^ 7y+^+

(0

2
= -j
ax

9000.

2.2.3. (b)

T5r

(b) y'

2.2.1.

2.1.5.

2.

The function

2.1.7.

10a;

y'

(b)

(b)

429

a:

= 21narc tan-^-3

arc tan

9+^
tti

3
2.2.8.

'

(b)

X(Jtsinh2x

=-

sinh ,

cosec* x;
*+ cosh 2
= e sinh e ta<acosh+6>_

{^

= (tan*)

(d)

</'

= 3*x

y'

(e)

j/^y-p^sinSArcos 2 ^^

(c)

(x+\)
(d) y'

taax)

+ coshjt*.sinh2x*);
I

2.2.9.

2a;

lntanA:-[ sin^

2a:

2.2.13.

(coshy + sinh-0

= -Jj
e + b)x
(cosh fo + sinh ^) = (a +
j/cosh

=y

/' (*)

(a)

*+l;

(d) /'(x)

(b)

j/'

= (cos
cos

s,y
51

cos

y'=-

j/ sin 2

3a: (1

5a:

"* (cos x

a:)

sin
x

= 4 sinh

cos a; tan x sin

j_

jl

U+2)

/' (x)

3a:)

(*-j-3)

r2

= tanhx;
f

4a;;

24

(c)

3(x-l)

In

3a:

2 -}-

(b)

(a

fr)

2.2.14. (a)

(e) /' (a;)

*);

(i) f

(x)

(c)

/' (x)

= (a+b)e ax X

Answers and Hints

430

In 3

2.2.17.

...

(b)

(a)

^81*

2.3.1.

k n e**\

(b)

+ y sin

2.3.4. (b) e*

(a:

2"" 1

(e)

#-ln x

^5x

sin f 2x

(sin

5/ cos

-1- sin

(f)

(c) e

ax

71

j^a"-

j sin $x

^~

1)

+ 3'

(b)

x
a:

* 3 sin

(a)

+ cos

2
)

cos

a:

x 60 x

(c)

x)\

4n^sinA:].

'

+^ are sinx

A
a:

\,;
+7r
'(1

= 2 (1 x)

+ 6/: +

x)

2
)

(n 1) 4];

3rt

2.4.3.

(b)

2.4.4. (b)

y'

= 7^-+sin
:

2.4.6.

y'

=JL+ e "T(e x

(b)

tr

~W

Yt=F

2.5.1.

(d) *,;

C)

= - 2>-

(b) 6a:

/.

(c)

ifi=^=|;

J.

(c)

(d)

2.4,,.

~~

2t (2/ 2

+ 2/+l)

jA.

,.,

cos/

6 sin

/)

(e)

2a:

=-

(a
;

~ ^l^
6y+ 37 =
9 = 0;
+
2*/

(l^)

2 ''.

1^=57^7

4 sin

fe)

201

^2

^2_|_i

(d)

2/z

-ey )V-* x+y )

. JtjJ,
l

rr3

2.4.5. (b)

2 4 7

a:)

y'x = 3 sin/ sec


x

[(4/z

a:

= -cot^f;

^=_*L_

(b)

2e~*

(b)

( 1)"

lx3x5x...X l97x(399 x)

(d)

(1

4 cos

"

/,

_1_

(6

...

(c)2r-"x

[_^__^_]

(a) .00!

2.4.1. (b) Xyy

a- 2 p 2 +

x 1140 X sin x-j-8640 cos x\

cos

[3* 2

1_

+ny +

-l).

Hint. y

{c)

(x

2ioo

^2

+ /iy

IH/i-^8

n (n

(a)

2.3.9.

In3

--"- "ix'.'gr --^-]}-

2.3.8.

+ 1

arc tan

+ n ^)

(1
(2x 2

j/

(1+a: 2 ) )^1 +a: 2

cos 2 In 3 *)

+ 48a: + 551);

2* 2
2.3.6.

In 3

sin
r

+ n-y)+^sin

^3*

+ COS p*

In 3

J^arcsin3- 2 *

5x j/cos 4

^arc sin3" 2 *

tan

=r.

0.

cos 3

y
;

Answers and Hints

2.5.3.

(b)

2.5.15.

= arctan2|^2.

cp

-J;
2.5.16.

(d)

(b)

2.5.8.

y=l,

(b)

(a)

Ch. II

to

+ 2</-2 = 0;

(c)

431

+ 2 = 0; y = x.
0+^= 3 (*+|- J

11.

-J.
2.5.19. s

2.5.17. 26,450.

= at-^; v = a-gt;

s max

= s[

= r;=^^sinM(l+2ecosM).

2.5.20. y
2.6.5.

(b) log 10.21

2.6.7.

(c)

= cos* A x (d) A y = + tan


d*i/ = 4-* 21n4(2A: ln4 l)dA:
2

2.6.9. (a)

oaift (a)d.2
y

2.6.10.

in particular at

2.6.11.

= 4 s'm2xdx

d 3y

AV = 4nr

dy

= 0.05.

a:)

A*.

(b)

d2

~ 4 ~ ln *
y (In 4)

4 ln *
a:

a:

4(1+3a: 4 ),

d2

/,

dx 2*'>

^^4)2

:i

= tan

Ay

0.9942.

( 1

(c)

2.6.3.

(d) cot4510'

1.009;

(TZ^r ^ - f="p ^
4(1+3a: 4 ),

4a:

2
1

y= ~ CQS

Ar-\-4nr Ar

2/

+o jxAr

the

is

volume contained between

Ar; dV
4nr 2 Ar is the volume of a thin layer
two spheres of radii r and r
with a base area equal to the sphere's surface area 4:xr 2 and a height Ar.

= g/A/-|i- g At
the distance covered by a body within the
the distance covered by a body which would move at
time At; ds=gt At = vdt
a velocity v = gt during the entire interval of time.
2

As

2.6.12.

is

is

2.7.1. (a) It does not exist;


2.7.2. 90. Hint. Since

(b)

exists and equals zero.

it

x^0

(e*,

y
/1(0)

= -I,

2.7.3.
2.7.4.

e-* x <
%

0,

/;(0)=1.

r_(A)=-<p(fl); /;
the derivative
Hint. For x ^

n*) = -cos(l)+2xsin(l
At

a:

the derivative equals zero:


a

/'(0)=
Thus, the derivative /'
second kind at the point x

XT-^^

lim

exists

(a:)

Aa:- sin

for all x,

'

but has a discontinuity of the

= 0.

2.7.7. Hint. The formula for the sum of a geo2.7.5. a


2x
b
x\.
metric progression represents an identity with respect to x. Equating the derivatives of both sides of the identity, we get
"'" +1
1
1
2r
1
;
3*
,

+ +

+^- =

^+;>'" +

Answers and Hints

432

multiplying both sides

by x and

equality

of this

\+x-{ n+

+ 2*+...+iV-=
2.7.8. sin x + 3 sin 3x +

+
_

(2/i

(2

sin

1)

differentiating

again,

we

X + Un^+*i-\) X+>-nx>> +

\)*

(2/i-|- 1) sin (2/i

1)

(2/i

) jc

1 )

sin

get

(2/1+1)*

4 sin 2 x

//m/. To prove the identity multiply its left side by 2 sin* and apply the
To deduce the desired formula
cos (a
cos(a
formula 2 sin a sin P
p).
P)
differentiate both sides of the identity and equate the derivatives.
2
2
<*> [e x
/' (cos *)];
2.7.9. (a) sin 2x [/'(sin *)
(b)
f (e x ) /'(*)/ (e x )\;

^11?}

(C )
'

\|)

ln

(*)

<P'

(#)

cp (a:)

(x)

(p

ln
.
"

In 2

(*)

cp (*)

(d) No.
2.7.10. (a) No;
(b) No;
(c) Yes;
2.7.11. Hint. Differentiate the identity /(*)
/(*) or f( x )
f(x).
This fact is easily illustrated geometrically if we take into consideration that
the graph of the even function is symmetrical about the y-axis, and the graph
of the odd function about the origin.
2.7.12. Hint. Differentiate the identity / (*+ T)
f (x).
2.7.14. y'
2\x\.
2.7.15. The composite function
2.7.13. F'(x)
6x 2
/ [cp (x)] may be non-differentiable only at points where cp' (x) does not exist
u at which f (u) does not exist.
and where cp (#) attains such values of q>(x)
But the function y
u2
\x\ 2 has a derivative y'
Q at the point #=0,
though at this point the function u
\x\ has no derivative.

2.7.16.

(a)

y"

= 6\x\;

tage of the
that

show

(a)

of

= (n 1)

XX

2 sin

(b)

cos-!

= C

(k

= 0,

....

1,

the binomial coefficients.


u n -^
w_ 2 and use the

0,

and take advan-

/i)

method

Designate:

(b)

sin i- at

= 0.

discontinuous at x

is

Verify that

property
u'

y" (0) does not exist, since y' (x)


2.7.17. Hint,

of

un
f(x)
mathematical

induction.
2.7.18.

Hint. Apply the Leibniz formula for the

derivative of the pro-

/ith

duct of the functions u

=e

and v

=x

2
.

at

2.7.19.

yM(0)={ [1x3x...X(2/j
I

Hint.

Differentiate

y(n)

e-*

2.7.22.

2.7.23. x 12

Hn +

formula

Leibniz

the

V =-2 X

= 2)
(/i

Take advantage

2.7.21. Hint.

and

(0)

yx

(x)

for

\fn

-2

at

= 2k
= 2k+\

n
n

(k=\,

identity

the

l)]

times
>

(0)

2,

and,

(/i

...).

putting

nth

2 )<n

= ( 2xe-**Y n

(_oo <{/<l),

*3,4=Kl-l/7=^
(t=l,
4x/(l-jtf)

obtain

(0<y<l),
2,

>

derivative of the product

=j^jy

= 0,

of the definition

= (e-x

the

= V + Y\- y

2)

3,

4) for Xi

0,

1.

u~e~ x2

and

Answers and Hints

433

Ch. Ill

to

2x

Hint. Solve the biquadratic equation x*


obtained functions x ( (y).
2.7.25. (a) x x
(b)
\.
3; * 2 =1;

-{-y

and

find

the domains

of definition of the

x=

Hint. Note that the function x=-2t

2.7.26.

==

t
\

<J

vative at

= 0.

But

2/ 2

0,

>

rywhere.

2
[ 2x
through x: y
y=<
*

< ^

'

<

2.7.27. a

=c=

-!-

This function

'

'

0,

<

2
i.e.

sin ax-\-f

f' (x)

differentiate eve-

The curves

2.7.28. Hint.

is

0.

4
2
points where sinaA:=l. Since at these points
y'

we can ex P ress y ~t 2 + * V

therefore

<0

haS n deri "

/^q'

intersect

at

the

cosa#=0,

= f (x) = y'v

(x)acosax

the curves are tangent.

For

2.7.30. Hint.

y=

nn

normal

and the

the tangent

the equations of

are

reduced to the form:

= co\^-{x at)-\-2a\ y tan y (x at),

n(2k 1) (k=\, 2, ...) the tangent line (y 2a) touches


respectively. For l
at) passes through the highthe circle at the highest point, and the normal (x
2kn (k
for t
at) pasest and lowest points;
0, 1,
.) the tangent line (x
ses through both points, and the normal (y
0) touches the circle at the lowest

^+ y.y
dt

2.7.34.

point.
v

reliable result,
45.
cp

2.7.35.

The

= ^^

relative error 6

the result with the least relative

i.e.

Chapter
3.1.2.
3.1.5. l

to the

III

Yes;

(b)

The most

corresponds

error,

value

sin 2cp

= e\.

(c)

No, since the


No, since

derivative

non-existent

is

g( 3) =g(3).

3.1.7.

3.1.9.

(d)

at the point 0.
Hint. Consider

Ihe functions
f(x)

2x

= arc sin

-j-2 arc tan

for

a:

>

1,

g(x)

3.1.15. (a)

2x
= arc sin y-j
2 arc tan x

= y;

a-^;

(b)

(c)

for

<

= 10

(d)

it

is

not

appli-

# = 0.

cable, since the function has no derivative at the point


3.1.16. 1.26 < ln(t+?) < 1.37. Hint. Write the Lagrange formula for the
function f(x)~\nx on the interval [e, e-\-\] and estimate the right-hand side
in

the obtained

relation:

In (1 -\-c)

(e

<

<

e-\- 1).

Apply the Lagrange formula to the function f (x) = \n x on the


l+*]> x > 0, and estimate the right-hand side in the obtained re-

3.1.17. Hint.

interval [1,
lation

3.2.3.

ln(l+x)
(b) 0.

=y

Hint.

(1

<

Represent

<

cot x

3.2.1.

-=

(c)

JI!iLf
;

c)

2;

JL

(d)

0;

3.2.5.

(f)

(b) gi

e.

Answers and Hints

434

3.2.6.

(a)

(b)

1;

3.2.9.

1.

-i

(a)

lna-1;

(b)

(c)

(d)

2;

rr.2

(f)

0;

(n)i-;

(h)

1;

(g)

^;

(o)

In a;

(j)

e~;

(p)

-1;

(k)

-1

(r)

1;

(q)

n
2

(i)

(e)

3.3.5.

4":

(m)

e;

(1)

0.34201.

(b)
1

3.3.6.

^ 3.018350.

83

^83 =

tfm/.

V 81+2 =3 (l+gy)

*
-

Apply the

bi-

'

nomial formula and retain four terms.


3.3.7. Hints, (b) Write the Maclaurin formula for the function /(A:)
tan*
with the remainder R 4 (x);
(c) write the Maclaurin formula
for the function
j_
2
with remainders R 2 (x) and R 3 (x).
(1 +*)
/ (#)

3.4.2. (a) / (x)

+ g+o(*
3.4.3.

_^ x +
,

(jcB);

/w = x _^ + f!_fl +

(b)

).

(b)

i;

(c)

-1;

(d)

1;

(e)

1.

1+2,+, 2 -4,3_|, 4 __L, 5; (b)-^-g+g;

1-f +

(c)

720*

3.5.1.
(0,

3.4.4. (a)

+ 12

= 1 ^ _i. x

(d)

oo);

(e)

The function decreases on the

( oo,

interval

and decreases on (^-^

and

y^j

increases on

and

(3,-j-oo)

number

over the entire

the function increases

(f)

0)

^ oo,

the function increases on the intervals

scale.

3.5.2.

(b)

The function

increases on the

intervals

JT

0,

and

5t

^-

2jt

decreases on

and

(b) the funcincreases throughout the number scale;


(c) the function
tion increases on the interval (1, 0) and decreases on (0, 1);
(d) the function increases on both
decreases throughout the number scale;
(e)
oo, 0) and (0, oo) where it is defined;
the function decreases
intervals (
(f) the function
on the intervals (0, 1) and (1, e) and increases on (e, -j-oc);
oo,
and '(I, oo),
on ( 1, 1).
increases
on the intervals (
1)
decreases
3.5.11. b^{.
3.6.1. (b) The minimum is / (1)-=/ (3)
3, the
3.5.10. a<0.
7 \
3.6.2.
(b)
the
minimum
The
(d)
minima
maximum f(2) 4;
/
(a)

3.5.8.

The function

are

(\)=V~3;

the

maximum

3.6.3. (b)
3.6.7. (b)

The maximum
The minimum

3.6.8.

On

mum

(b) the

(b)

j=

'

the interval

minimum

3 6 -10- (a)
-

is

is

is

/(0)

= 2.

( 2)=

/(0)

[0,

= 0.

2jt]:

160;

the

the

the

minimum

minimum

The minimum

/( 2) = 1,

is / (0)

maximum

is

= 0,
/

= 2.

^-^-j=r 4;

the

(2)=

/(0)

1;

maximum
(c)

the

the maxi-

/ (2)

=^4e~ 2

maximum

is

Answers and Hints

25
f
5 =
y

/(0)

= 0,

the

minimum

minimum

the

= 7;

/(0)

to Ch. Ill

maximum

the

(e)

is

maximum

(d) the

3)

435

(2) ==

is /

1,

minimum

the

3,

/(2)=-j/44.
There

3.6.11. (a)

ximum

is

3.7.1.

The

(c)

greatest value

3.7.2.

is

3.7.6.

The

(a)

value

) = --(-0.25
Yl) 6
=^
value
j

/(0)

the least value

= y,

j/(0)

value

the least

/(2)

value /(

least

= 0.

the least value /(3)

In 3, the

is

value

est

3)
'

the least value

value, the least value

est

3.8.3.

H=R

/ ^-^j= 2; (e)

=
2
In 2);
there
(2)

of

is

2,

2.

to

is

of

x)

(sin

at

3.8.8.

The radius

3.8.10.

the radius of the cone base.


.

x
,

15

the cylinder
base
j

of

The equation

3.8.11.

of

is

R
r=
2
the

where

desired

straight

T+T
x=a
p

for a

3.8.12.

>

p and x

v= 1/
V

a^p.

for

3.8.13.

no great-

is

i/

2tl
is

the great-

is

(f)

(1

4Wi

3u~

/(0)=1.

is

(y

interval

the radius of
R
V =where H =the height the cylinder,
acos a, where a = 0.5 arc tan
a sin a,
reduced
finding the greatest value
the function
S = 4xy-\-4x = 4a
2a sin a)
P max = ^r
W = W 3.8.9. A = 2/?=
< a < ~
4

3.8.7. x
the sphere.
Hint. The problem

in the

= ^4

= 0; c the greatest
= ^-0.25 In 3; (d) the

the least value

if"

/(l)=l,

is

^i^y^^lh

the least value

/( 2) =

is

(d) the

0.

f(\)

greatest

= 0;

the least value /(0)

ma-

the

(c)

= 0.

the least value y(0)

= 2,

/(0)

is

f(

is

is

value

greatest

the greatest value

(b)

^L_

6,

y(4)

is

is

is

greatest value

the greatest value

(c)

no extremum;

(b) there is

minimum

the

(d)

greatest value

The

(b)

no extremum;

is

= 0;

/(0)

v
1

-ttt

//in/.

will

It

take

2b

hours

cover

one

The

knot.

T = ^~^- = ~-\-bv 2

appropriate expenses are expressed by the formula

to

TL

3.8.14.

cp

Hint. At the

board

width a the

area of

cross-sectional

the

trough is equal to a
walls to the bottom.
3.8.15.

(1

+cos

cp)

Hint. The point

sin

cp.

where

cp

is

the angle of inclination of the


V

of

fall of

the

jet is at a

2
the tank base, where
h
x is the height at
located, v is the rate of flow; therefore the length
the expression

V2gx

y ^^=2

distance of

from

g
which

the orifice should

of the jet

Vx(h-x).

is

be
determined by

Answers and Hints

436

hours the least distance will be equal to ^- km.

3.8.16. After

2v
3.9.1.

The intervals

(b)

xity

tervals of concavity are

)/~~3,

the curve

(e)

oo,

<

the intervals

(f)

the in-

>^3)

oo,

)>

'

^>

concavity are

of

J
3

(0,

convexity

conve-

of

(c)

13);

(1,

oo), of

oo),

(1,

V~3, _ "7(^)>
^

concave everywhere;

is

and

0) and (|^~3,

V"s); the points of inflection are

(0,

1^; the points of inflection are

and

concavity are

of

x x ) and

(x 2)

convexity

of

oo),

points of inflection are (x lt y x ), (x 2

3.9.5.

concave

The point

(a)

>

for x

the curve

is

concave

in

is

where x x
where

3);

(3,

=e

the curve

arc sin

V"5-A
-

is

=e

convex

and convex

in

the

for

<

3 and

l/~~5

#=arc

x2

VI

the abscissa of the point of inflection

y2)

of inflection

(b)

3;

x2)

(x lt

-V~5

sin

i
;

V^\
-

arc sin

2~

3.10.1.

>

(c)

oo

x=2;

(d)

= 0;

= 0;
y = 2x
= 3, y = x 3;

(d)

(e)y

function

3
=y
(1) ==y (

is

asymptote

maximum

1)

6 \

has

no

oo,

4) =

(c)

y=2x as
(c)y = x;

it is even. The graph is symasymptotes. The minimum is y(0)-=l

The points

/
of inflection are

1)

V"3
g

23 \
the

(b)

J;

function

the

2,

1;

+oo). The graph has a vertical


and (
1,
and an inclined asymptote y=x
3. The minimum is y (0)^0,
/
256
3296
The points of inflection are
and
6,

defined in

x=

and

JTX
y=

defined everywhere,

is

and

metrical about the (/-axis

maxima y

(b)

= 2x-^.

The function

3.11.2. (a)

x>+oo

as

(i)

3.10.3. (a) *

defined

is

in

( oo,

0)

and

+oo). The graph

(0,

J;

has a vertical asymptote x

tion

( --

is

V~*

\
)

(1,
two
is

1)

and

vertical
(

(1,

oo);

it

asymptotes

V 3) = +3

inflection is (0, 0);

= 0.

The minimum

(d) the
is

function

is

is

y (^-^-^=3. The point

defined in the intervals

of inflec-

oo,

1),

odd. The graph is symmetrical about the origin, has


1 and an inclined asymptote y x. The minimum

x
the
(e)

maximum

y{V 3)--=

the function

is

2~

point

'

defined everywhere,

it

is

even.

of

The

Answers and Hints


graph

symmetrical about the

is

minimum

(0)

asymptote

The point

is

= 2.

is

The horizontal asymptote

0.37;

the function

y_

(0)

i/

defined in

is

(0)

= 0,

0.075); (g) the function

as x

The

oo.

+oo). The

2,

maximum

the

maximum

The

inflec-

vertical

0.73)^0.12.

defined everywhere.
/ 3 \ 3
/ 3 \

is

is

Ae

is

-z-vi

defined and continuous everywhere. The horizontal asymptote


is i/(0)
0, the point (0, 0) being a corner point on the

y\. The minimum

graph:

is

(tzVjy vi-sy (*_o, (ttfiy

is

inflection are (0, 0),

of

-v~$
(h)

is

= 0.

and has a horizontal asymptote y


2 jj/ 2. The points of
y ( }^ 2)

the function

(f)

The minimum

of inflection

The points

(/-axis

maxima

the

j[/ 4,

(2, \/ A)\

are

tion

is

437

Ill

to Ch.

= -i

f
,

y +{ 0)

=
= +~.

^=2.330;

3.12.11. 0.6705.
x2
2.128.
0.202; jc 3
3.12.13. (a) 1.17; (b) 3.07.
3.12.14. 1.325.
To approximate the smaller root more precisely
- 1 to find a more accurate value of the
write the equation in the form x
e- 8x
1 .25
larger root represent it in the form
In x).
(1
3.13.1. No. Hint. Show that at the point
the derivative is non-exis3.12.6.

4.4934.

3.12.8.

3.12.12. (a) 0.27; 2.25; (b) 0.21.


3.12.15. 0.5896 and 2.2805. Hint.

x=

x~\

tent:

Ml)=-1.

/'-(1)=1;

3.13.2.

Hint. Check the equality

/ (b)

f (a)

= (b a) f

+b

2
Hint. Apply the Rolle theorem to the function / (x)
a^ n -f,..-\-a n - x (x) on the interval [0, x ].
f
A(x 5
3.13.4. Hint. Make sure that the derivative f (x)
one
1) has only
real root, #=1, and apply the Rolle theorem.
3.13.5. Hint. The derivative /' (x)
nx n ~ l
p has only one real root at an
even n and not more than two real roots at an odd n.
3.13.6. Hint. The derivative is a polynomial of the third degree and has
three roots. Take advantage of the fact that between the roots of the polynomial lies the root of its derivative.

3.13.3.

3.13.7. Hint.

From

the correct equality

limcos-r =0

determined from the mean value theorem,

it

~>

(0

<

<

x),

where

is

does not follow that lim


x

~>

cos = 0,
x

since it cannot be asserted that the variable attains all intermediate values
0. Moreover, g takes on only such a sequin the neighbourhood of zero as x

ence of values

3.13.8. Hint.

The mistake

is

=0

().

that in the Lagrange formula one and the

same

is

coefficients
powers of

which lim cos

taken for / (x) and cp (x).


3.13.9. Hint. Apply the Lagrange formula to the function \nx on the interval
a]; (b) apply the Lagrange formula to the function zP on the interval [y, x].
3.13.10. Hint. With
the
aid of the Leibniz formula ascertain that the

point
[by

for

<

of

the Chebyshev-Laguerrc polynomial alternate in sign, the odd


for
having negative coefficients. Whence deduce that L n (x) >

0.

xn
3.13.11. Hint. Using the Rolle theorem, show that inside the interval \x
1
roots of the second derithere are at least n roots of the first derivative, n
vative, and so on.
3.13.12. Hint. The L'Hospital rule is not applicable here, since the derivanumerator and denominator vanish at all points where the
tives, of both the

Answers and Hints

438
factor

sin* (which we cancelled

tives)

vanishes.

+ h) = f(a) + hf'

f (a

with

it

+ Qh) f"

f" (a

the

expansion

= }_f,
a+^ n
6

(a)

(a)

R2

by using

3.13.14. Hint. Prove

h'

(a)+-^

f"

given

the

where p and q are natural numbers, p


> p

/"' (a

>

equality

the

get

limit as h

to the

rule of contraries.

>

+6^).

problem,

the

in

and pass over

h2

deriva-

limit ol the ratio of

Write the Taylor formula with the remainder

3.13.13. Hint.

Comparing

computing the

in

> 0.

Suppose that e== "~

and, using the Taylor formula,

1,

get for n

i =1+ _L + Jr+
Multiply

both

..4

\-

sides

and

n\,

integers
n\ are positive
b
r

n\ J

1!

by

equality

this

of

+ Jr+ _^r (), <9<

...

and

noting,

+r-r(
/i

,,

~ n\ and
< 77 <
n-\-\
that

1,

\ q J

obtain a contradictory result.


3.13.15. Hint. Verify that the function
i

f(x)

sin*

x<

=}

'

n
2

'

continuous on the interval

is

x=--0
Ascertain that the derivative f
3.13.16. Hint. Show that /' (*)
\

1,

/m

/(0)

<

(x)

0.

inside the interval.

is

Ascertain that

> for a <


= l-a<[ <Qfora>1>
1,

and take advantage

of the fact that the function increases.


3.13.17. Hint. Show that the function f(x)=xe x
2 increases
site signs at the end-points of the interval (0, 1).
3.13.18. Hint. Show that the derivative

/'

is

equal to

the

points

3
a:

W=-^+ 2xsin T C0S T


points *=
(n =
9

{x

and has oppo-

0)

at

the

= 2~~

0,

i-

derivative

the

changes

1, 2,
sign

in

...),

and to

any vicinity

at

of the

origin.
cp (x)
in3.13.19. Hint. Ascertain that the auxiliary function \p (x)
/ (*)
creases.
3.13.20. Hint. Make sure that at all points of the domain of definition of the
bc^O. But if ad bc 0, i.e.
function the derivative retains its sign if ad

then the function

is

3.13.22.

A minimum

/(*

p 6,
>0 and n

constant. 3.13.21.

if

cp

(x

#=14.
is

even; a

maximum

and n is even; the point x is not an extremum if n


if
f(x )
(p (* ) <
the funcis odd. Hint. At an even n, in a certain neighbourhood of the point x
tion retains its sign and is either rigorously greater than zero or rigorously less
than zero, depending on the sign of cp (x ). At an odd n t lie function changes
sign in a certain neighbourhood of the point x
.

Answers and Hints


For x

3.13.23. Hint.

the

derivative

/' (#)

>

f (x)

hence

0,

= 2 sin + cos

xx

is

/ (0)

439
a

is

minimum. For

positive at the points

x = /

and negative at the points

IV

to Ch.

>

= -^
ZiTin

The case x

(2/i+1)ji

<

investigated ana-

is

logously. 3.13.24. (a)


and 0; (b) 1 and 2.
3.13.25. (a) The least value is non-existent, the greatest value equals 1; (b)
the function has neither the greatest, nor the least value.
3.13.30. Yes. Hint. Since /" (x) changes sign when passing through the point x
the latter is a point of extremum for the function /' (x).
3.13.31. The graph passes through the point
(
2) and has a tangent
1,
line y
is a point of inflection, the curve being concave down2
ward to the left of the point M, and upward to the right of it. Hint. The functhrough
passing
x
tion
increases
and changes sign when
1.
f" (x)
1

=
h

3.13.32.

= L=.

3.13.33. Hint. According to the Rolle theorem, between the roots of the first
there is at least one root of the second derivative. When
passing
through one of these roots the second derivative must change sign.
2
2n
3.13.35. Hint. The polynomial has the form a x' -\-aiX^"- -\-{-a n - x x 2 -{-a n
Polynomials of this form with positive coefficients have no real roots.
3.13.36. Hint. Take advantage of the fact that a polynomial of an odd degree
(and, hence, also its second derivative) has at least one real root and changes
sign at least once.
2
**
lim ( *L
3.13.37. Hint. Find
!

derivative

+ +

Chapter

IV

/=A: 3 + A: 2 + 0.51n|

4.1.2.

-2-

4.1.7.

2x |+C.
1

= (*+l) + ^-*
2

+C.

Hint. Eliminate the irrationality from the

denominator.
4.1.14.

= ~arctan~ + C.

/=-!=arctan-^J+C.
^

4.1.15.

^3

4.1.18.

= ln|* +
Vx* + iyx+_\ + C.
V\Qx-V7
= U= In
+c
2 ^70
V 10*+ V~7
x3
3-

4.1.20.

4.1.21.

(a)

+2cotA: + C;
4.1.22.

arc

tan

(d)

x
(a)

In

4.2.3.

5-l^

5
/

(b)

T (*-4)

+C;

(c)

3tanx-f

l-arc tan x-\- C.

In (x-\-

2
(c)

C;

2 "*

2
1

-f-*

+ C;

)-farc sin x-f-C;


(d)

0.2 cos 5*
37

=
4

Y~2x

(b)

sin x

xsin5a + C.

5 + C.

cos x-\-C\

440

Answers and Hints

t= 2 Vcosx+C.

4.2.8.

+ In

4.2.13. (a) 0.75

(d)

^arctan^ + C;
4.2.14.

y y sin

2 +
.1(8

4a:

arc sinA:

4.3.19.

3* (sin *

4 3 21

3a: 4 )

arc sin

(c)

x+ In

In 2

In

a:
|

-^-{-C;

+C.

+ C;
*+C;

+ C.
+ In tan
2

-J

+ C.

+ cos sin 3)

l+(ln

3)*

+ ^C
,

(x*

4.3.24.

10x + 21) sinA: +

cos 3*-)
2

A;(4A:

20) cos + C.
a:

sin3x + C.

4.3.25.

^j-xt+ix^J \nx ^+^_3*+C.

4.3.26.

i^arctanx-^+-J + C.

4.3.27.

y arc cos * -H^ j/T^F+C.

4.3.28.

(a)

_ 18^+g-13

n(6x+2) _6^tl cos(fa+2)+ ^,3 +


2

2_^ +C;

27

express

+C.

4.3.23.

2 *+!)

4.4.2.

^T^7 +C.

(x-x* + lx+!fje** + C.

+ C;

1)

4.3.22.

320

3a:

i-

(f)

j/sin 3

a:

J^l

"

+ +

+ rO |/"i+x + C.
^xf/x [(ln*)*-|ln* + | + C.
2 |Ar+Iarcsin* + 4 J/T^t+C,
xln(x+

4.3.18.

_|_i_ x

ln|ln*|

)(1-a:)

2
a:-|- sin 4

a:

tan x

In

a:

(a:

2 cos yV+C;

(e)

(b)

|-(ln*-5)^l + lnx+C;

cos

4.3.17.

+ C;

(b)

(d)

4.3.14.

__(35-40x+ 14*
/

a:

a:)

(a)

(c)

4.3.2.

4.2.10.

-i3

(b)

+C

(d) Hint.

.|_

(jc

2__ 7jc+1)

( 2x-f-l)3

-A

2a:-7)(2a:+1)
(

Apply the generalized

sin""
e

n --

'

formula

for

integration by parts and

from the relation thus obtained

In

i
1

x (a sin x

n cos x)-\
x

(n\)
3

n1
,

/->

'

n-

Answers and

4.4.3.

(n\) sin"cos x

(b)

'

(a)

cos

_L_tan"-iA:-/_ 2

/i

Vx

v"- 1

a;

a/_ 2

/i

= In|

2);

+ C.

tan

= ln

cos x

sinx|+C;
/

|+C;
a:

= In

Chapter V
5.1.2.

*-2*+| In

5.1.5. 2 ln

In |*

t^t-

V7

3
5.1.10.

^
>^7

arc tan

+ ln x

+
+ 50x + 68
4 (x + 2) (x + 3)

5x
9* a

5.1.11.

(x-

9x4-1

2
5.1.8.

I)

arc tan (x

2 +C

(x+\)
ln

(*

Jn

x+2
5.1.14.
2(jc .

arc
1

^+1

+ In

_"V?+T

tan

tanx-

+ C.

1^3

L^arc

+ C.

arc tan

V3

y x+24

16

3)H

C,

x +x

2 arc tan x
l)

/*+6

5.2.4.

2)

+ 2)

(*

(1+,).

6(1+*)^

+ 2) + C.

(x

5.1.13.

+ C.

~ arc tan +

5.1.12.

5.2.2.

+y ln|*+2| + C.

(Jt+1)

$/'

x+24\n
\

^ x\\ +

C.

?4+ln

V"3

+ C,
5.2.7.

5.2.9.

j/^1x +
f

C.

\--x V
)

5.3.3.

-2

5.3.5.

2 ln

5.2.8.

where

arc sin * + C.

arc tan

j/V-f- 2*

-[-4

*|

2(/*

+C

+ 2*- -4 x
r

= + C;

j/> + a + C;

441

2 sin 2 x

'

= -'

(nS*

nl

Ch.

to

2 sin 2 x

4.4.4.

Hints

l)

= a;-|-C;
/

(c)

Vx*+~a

+ C.

Answers and Hints

442

5.3.6.

5.3.7.

1+l ^-**+2arc

|/ j^+C.

tan

(*+/T+?)" +e
|5.3.8.
15
X
~
^
5 Yx* + 2x 1-5 ln(x+l + y X + 2x+5) + C.
3* + *--l
y 3x 2_ 2x+ + c.
o
Hiii j/x + a:+1 + |- In
+ 2 J/V-j-x+l + C.
14x + 111) Vx*-\-4x-r3- 66 In * + 2 +
+ 3 |+C.
o
.

'

2x

'

5.4.2.

5.4.5.

5.4.6.

5.4.8. -1-

5.4.9.

(jc

2a:

-f-

-f-

(32a- 2

Vw + bx+7

20* 373)

329 '
-\

128

64

/2

In

4jc

4x+5+-

+ 2 V\x*+ 10a:+14| + C.
+

5.4.10.

5 .4. 1

..-^-

4X
,
1

5.4.12.

x2

+C

y j-pnh + + X + 3^ +
2

5.4.13.

+ 3 -2arcsin-L + C.
2

+ C.

////i/.

make

First

the

substitution

t.
5

5.5.2.

3arctan

x-{-C.

5.5.4.

\2 + x*

-y \2 + x

A(l+xV -|-(I+*V + jg (+**) +C.


+ C.
^ |/ (l + ^/^) _3 ^(i
s

5.5.5.

5.5.7.

5.5.8.

5.5...

5.5.10.

5.5.11.

5.6.2.

3 In

V
,

(b)

77= + c

C+^f-^+C.

^+^^-

-11

|V

(a)

+C

3
+ / T4 + C

('

3 sin^ #
5.6.10.

a/ - +

_L
5 sin 5 x

cot

JL tan 2 a:-1

5.6.6.

f_c.

x+y cot x

In

( 1

+ tan

x)

tan*

cot 5 x

+ 4-tan**-|-C.
3

x+C\

+ C \ tan x + In
----

cos

a|

+ C.

+C.

Answers and Hints

sinx

5.6.12.

1+

sin 3 x-\--^ In

]r-

^arc

5.6.14.

tan

443

+C

^tanh-^+r

(b)

(x+

j^

arctan^

+ C.

j/'^T+C.

5.7.4.

1)

/ = arc sin ^^in-C.

5.7.7.

*~

/==

5.7.8.

5.8.2.

.o
5.8.5.

= 4 ^ x+ 2 In (2 x 2 V a:) 2 (l +
cos + sin

= arc tan x.
\-C where
a r^
*

6.2.2.

1;

(a)

2/i

(b)

125
r"T~92

4/i

(c)

(f)

6.3.1.

(a)^HL^;
(a)

0;

2,

(c)

<

<

- V 1-f-x

(b)

is

the

*=1,

at

maxima

- (substitution x = a sin 0;

1;

(a)

^2

3
^U+ln 2+]^
2

]A 3

at

1=4

(b)

0.2

(a)

(b)

(c)

i;

(h)

(i)

M = /(0)=A

^ = jjL

minimum

x=

In

e~;

6.3.14. (b)

6.3.24.

the

i-

(b)

at

(b)

112;

^=

(b)

(substitution

*= tan

(b)

yi=

1;

2(y 3-l);

(C )

the

0-

+ ?-^ji.
2

In

j|

6 3. 15. (b)

jx;

1.

Sm

2 1n2;

Hint.

6.4.J5. (a) 2

is

5.

lb

6.4.6.

4/z

arc tan

(g)

1;

(b)~.

In x;

The maximum

= 2;

(a)

2/z

6.2.10.

\-

-J
In 2;

(e)

,125

ift

= 16-4,175
1

6.3.23.

are at x
(a)

+ C.

4x51 = 19.

175

6.3.11.

= -e-smx.

6.4.3.

and

it:

= /(2)=A.

whose height

as the area of a trapezoid

J^lziO.

k)

6.3.25.

-J- arc tan -J;


(

In

VI

=16

(a)

x)

-f-l

= 4 ^~^ ^ = 44

i
io s
6.1.12.
n
"

4x1 1=3

and bases

cc

Chapter
/

-\-C.

+5

I'-'-e^

6.1.9.

]/> 2a:

minima

a:

}^?^) + ^a:(2*
o
+ +CIn (s + V^TT)--

5.7.3.

sin

sin x\

sinh4*

(J)4-

-+5!|^ + C;

5.6.22. (a)

(d )

7=

Ch. VI

to

n
24

(c)

sin

sin

(d)V3 0.5

In (2

Answers and Hints

444

(e)0.251n3

V^3);

(substitution x
6.4.16.

tution x 2

at

(g)

(b)

-5-;

cos 2

The substitution

tinuous at x

(substitution x

a:

w iH

= tan x

(d) -^(substi-

t)\

n t do, since this function

disconti-

is

will not do, since this function

is

discon-

x=

\/ x 2 dx =

]/ x 2

dx

+^

x 2 dx

\/

-2

-2
and apply the substitutions

Y~t b in

2 <

<

x=-f

and

<

V~t b in

2.

6.4.20.

It

is

impossible,

It

is

possible; see

since

sec

and the interval

of integration

is

1].

[0,

6.4.21.

Problem

6.4.12.

On

G.4.22. Hint.

stitution x

writing

dx ==

/ (x)

-a

in the first

/ (*)

-a

(arc sin

0^+

(n

arc sin

/)

dt+

'

/ (2ji

+ arc sin

/)

sum

of three integrals for the in-

tne

variable:

'

f (x)

If

(1)

is

an even function, then

31

31

f (x)

cos

nxdx 2^

71

f (x)

cos nx dx, and

(x) sin

X dx

Jl

0.

JT

-JT

(2)

the sub-

t,

If

make

-l

2jT
and substitute
)
y) (t' "IT")
x = n arc sin/, x = 2ji + arc sin respectively.

tervals:

dx,

/ (*)

o
/

Hint. Represent the given integral as the

6.5.3.

+J

6.4.23.

arc sin

d*

integral.

-^-^-j

(h)

/);

b
V~ t is double-valued. To obtain
necessary to divide the initial interval of integration into

is

it

parts:

<

a3

(i)

/).

The inverse function

6.4.19. Hint.

= 2a sin

(substitution

-j- In

(c)

x cos x = t)\

7i.

the correct result

<

sin

= 0.

6.4.18.

two

6 2 sin 2

The substitution x = -j~

6.4.17.

nuous

= acos/);

(a) -g-;

=a

(substitution

(a:)

is

an odd function, then


J

JT

/ (x)

cos nx dx ==

0,

and

/ (x)

sin

nxdx

JT

-JT
JT

/ (x)

sin

/i

dx.

6.5.4.0.

(b)

-|;

<g)f-l;

6.6.3. 6

2e.

6.6.5.

(c)

(h)i|l-2^3.

jt

}/" 2

4.

(d)

6.6.6.

1;

jt

6.6.13. (a) -5-

2.

(e)

In

2-

(f) In

Answers and Hints

6.6.14. Hint. Integrate by parts twice, putting


~
(arc cos x) n l the second time.
x.
6.6.15. Hint. Integrate by parts, putting u

time and u

6.7.4.
2/z

= 6;

(b)

445

Ch. VI

to

= (arc cos x) n

the

first

0.601. Hint. Estimate

(a)

1V (x)
/

on the interval

yj

and put

0.7462. 6.7.5. 0.96

6.8.1.

0<x<l,

for

-i

F(x)--

for

< *<2,

Continuity is checked directly. The assertion concerning the derivative re2.


quires checking only at the points *=1, x
6.8.2. Hint. Make sure that the function / (x) is continuous both inside the
and lim f(x)
interval (0, 1) and at the end-points ( lim f(x)
f{\)).
f(0)
*->l-0
X-> +
6.8.3. No. Hint. Consider the function
if x is rational,
1
|
cp (x) == 1
if x is irrational on the interval [0,
1].

6.8.4.

]/"3.

Hint.

/" (x)

dx

f* (b)

(a).

6.8.5.

Hint. Putting for definiteness x

(x)

= n^ x

>
<

and

/z+1,

take advantage of the additivity of the integral


x
(x)

dx=

(x)

dx+

(x)

dx+

...

(x) dx-\-

n-l

the

(x) dx.

6.8.6. The antiderivative F 1 (x) will lead to the correct result and F 2
wrong one, since this function is discontinuous in the interval [0, jc].

(x)

to

X
6.8.7.

ted

in

the

{x)

=y + J

form F

(x)

(0 dt. Hint.

f (t) dt

+ C.

Any

antiderivative

Putting *

=x

0t

find

(x)

can be represen-

C yo=

x
6.8.8.

l=-^\n
2

2b

2a

'

it
is odd, and
The function is defined on the interval
1,
1),
creasing; convex on the interval [1, 0] and concave on the interval [0,

6.8.9.

the point [0, 0] is a point of inflection.


6.8.10. Hint. The function
x x at

< a:^
at, =

'<H

in1];

1_

is

continuous on the interval,

and the greatest value

it

reaches

at

the

least

and

value
at

m=e

xl.

^0.692

at

446

Answers and Hints

6.8.11.

Hint. Integrate the inequality

6.8.12.

Hint.

sin x

Integrate the inequality

and write Schwarz-Bunyakovsky inequality

V x sin x dx ^

J
6.8.14.

x dx

"J/

sin # dx

-=

8"

Apply the Schwarz-Bunyakovsky inequality

//i/i*.

~|2

VI

the form

in

-I

6.8.15.

Hint.

Make

6.8.16.

Hint.

If

= -y-

the substitution arc tan

X
f (x)

is

an even

F(x)

function, then

^f(t)dt

is

an

odd

function, since

F(-x)

f(t)dt = ^f(z)dz = F(x)

(/

z).

And

if

f (x)

an odd function, then

is

F(x)^f(t)dt

is

an

even function,

since

F(x)=

= -^f{-z)dz = F(x)

^f(i)dt

all

the remaining

antiderivatives

have

Chapter
7.1.4.

7.2.2.

(a)

7.2.13.(a)
7.3.4.

~.
6

(a)

~2

'

= -z);

{x)-\~C

and, therefore, are

with respect to a equals zero:

0.

VII
(b)

In 2;

form

the

also even functions.


6.8.17. Hint. The derivative of the integral

~ = /(a + 7W(fl) =

(t

v
(

b)

(i=y;
7.3.6.

'

-| (2 l/"~2-

4-4-4-

2^2

(b)

!n

(1=10

-t-t^
e-j-1
2;

(c)

arc sin A~+45


3
2
1

- 283

A;

(c)

l);

7 2 5
*

'

(d)

X4

7 2 - 10
-

7.3.11.

2.

A.
15

7.2.15.

'

y.

7.3.13.9.

'

'

'

(1=^ +

(e)

1;

V& + h*

7.2.16.

7.3.16.

i-r
m-\-l

Answers and Hints

VII

to Ch.

447

y.

7.3.19.

98
7.3.23. -i-

y.

7.3.20.

7.3.24.

7.4.9.

(a)

twice at the origin at

quadrants;
in the

Hint. The curve

(b)

x=

only

2 ),

is

is

7.3.22. 0.75jt.

3).

7.3.27. -ilz

7.3.28.

oU

symmetrical about the coordinate

a, y

b.

symmetrical about the #-axis, intersecting

The loop

1.

is

situated in the second and third

Hint. The points of self-intersection ot the curve are found

tx (/),
x (t^ = x

following way: y

= x(t

x(t 1 )

15

7.3.26.

Hint. The curve

V 3)ln(2+V

axes and intersects them at the points

it

(2

7.3.25.

7.4.8. 0.75jtafr.

2ji
4

15
7.4.6.

7.3.21.

if

therefore y
(t 2 )

= 0,

= x x
^ = 0; = 2\ (c)

(t x )

i.e.

{t x

(t 2 )

at

^zt 2 and

t1

D
Hint. The curve is symmetrical with respect to both axes of
coordinates and passes twice through the origin forming two loops. Therefore, it
is sufficient to compute a quarter of the desired area corresponding to the variation
7.4.10. 0.25nab.

of

from

to

7.4.11.

and multiply the obtained result by

4.

Hint. The curve resembles an astroid extended in the vertical

^-J-.
Sab

direction.
7.5.2.

(b)

(a) 5^.;

The curve

Hint.

is

radius

circle of

through the pole and symmetrical about the polar axis,

7.5.8.(a)^;

7.5.6.2^(^1-.).

(b)

passing

^"^(p^y.
XI

JX

7.5.9.^(^-^3").

'

Tia

7.5.10.

-575-

Hint. The curve passes through the pole forming two loops located

oZ
symmetrically about the

the first and fourth quadrants. It


by one loop corresponding to variation

(/-axis in

to calculate the area enclosed

is

of

sufficient
cp

from

ji

to

and double the result thus obtained.

7.5.11.

jt<2

Hint.

The curve

passes

through the pole,

it

is

symmetrical

about the polar axis and situated in the first and fourth quadrants. It is sufficient
to calculate the area of the upper portion of the figure which corresponds to

ji

variation of

from

cp

to

and double the

result thus obtained.

(,+_).

zia^

7.5.13.

(a

Hint. The

them only

curve

is

symmetrical about the coordinate axes and

the origin, forming four loops


one in each quadrant
four-leaved rose). Therefore, it is sufficient to find the area of one loop corres-

intersects

at

ponding to the variation of


2

cp

from

to

and multiply the result by

4.

7.5.14. Y~2 na
Hint. The curve is symmetrical about the axes of coordinates and the bisectors of the coordinate angles; it cuts off equal intercepts on
the axes. The origin is an isolated point. It is sufficient to compute the area of
.

Answers and Hints

448

one-eighth of the figure corresponding


result

by

2
9

Hint.

tiply the
7.6.2.

ji.

to variation

circle

2n 2 a 2 b.

7.6.10.

7.6.16.

(a)

7.6.18.

(a)

are:

= ~
3

x x1

and mul-

to

*2

^ abk

64
(c)

jx;

therefore the cross-sectio-

(d)

-=-ji;

ji

5jx

64

(f)

11

-e

\na*\ e

+na c =
2

no*.

-r-

_ 2A

a3

~ji;

(e)

|-a 3 tana.

7.6.17.

7i.

^
3

^+

sinh

Hint. The abscissas of the points of intersex

}/" 3).

ji.

7.6.23.

7.6.24.

48

Ifinr 6

127

1Q

tt
.

7.6.14.

7.3.9).

(c)

triangle situated at a distance x from

|a;

7.6.21.

^(6ji-|-5

7.6.22.

j/"l6

Problem

(see

2L.

7.6.20.

r=

h Y^a 2

to

(b)

16

10

(b)

12ji;

tt

tion

7.6.11.

2
-f jra c.

equal

is

2nab(^\+^j\

5jL.

7.6.19.

from

cp

perpendicular to the *-axis at the point x will

plane

cut the sphere along a circle of radius


nal area S(#)
ji(16
x 2 ).
7.6.5. 0.5na 2 h. Hint. The area
of a
the centre of the

of

8.

7.6.25.-^^2

ji.

105a/?

//t/i/.

Represent the evolute

= ^-s\n

jra

c= y a 2 b 2

where

t,

of the ellipse

parametrically as follows: x

^-na A

7.6.26.

-^L

7.6.27.

cos 3

2 In
7.6.28.

|^ 2)

-f-

Jta 3 .

7.7.2.^.

(b)2ln(2-^3). //W.

|-j

7.7.4.

In

^=-i;x,
=
2
2

axes at

10^+^5^.
t

^2+ V 3)
+
7.7.9.
In

7.8.8.

7.8.9. 8na.

16a.

7.8.12.

7.9.9.

1.5jxa.

= 2 ^ 2 a cos

(
V
[

cp

7.10.18. 29.6ji.

7.10.8.

V 3/

7.10.19.

~J

is

+ r2)].

+ 3-i^V

^24-^(1+

Hint. The curve intersects the

At

= ~L

In -|-

7.9,10. 2

Hint.

the point

3a

p[|^2+ln(l

7.9.11.

2jx

f 6+

coordinates,

//m/. The curve p

7.10.15.

y.

7.9.5.

2jt

7.8.5.

8a.

(c)
7
v

'

j/"3\

2jt

7.10.5.

polar

to

7.7.8. (a)

-2-;

=iV 8. 7.8.7. 4 j/~3.


7.8.11.
7.8.10. ii^Z^l
y

and

See Fig. 79.

al

7.8.2.

over

J^

'

7.7.10.

Pass

Hint.

^2

jia.

a circle.

(a)

7.10.3.

-.

Jii

7.10.14.

(34

(b)^.

V~\j-2)?-.
9

1^2)].

4n 2 a 2

7.10.16.

y^

2
-

jta 2 .

7.10.20.

7.10.17.

7.11.7.

2).

16a 2 where a

is

--y-^

jx (<?*

Answers and Hints

radius

the

<

b
>

cylinders

the

of

(0 l(53i
o

+ 6 V 3).

7.11.13.

3 1).

na 2 /"p?.

7.11.21.

+ 21n ^fL3)
M

7.13.7.

ft

fl

rt ,
7.13.11.

7.13.19.

(c)2n ,fc

nabhd.

7.12.11.

M y =-j

2
;

aft

= f2

Xc

= y c = -j.
0.2 (2e

*r

7.13.31.

x,

(a

0.2a

if

both

and

is

7.11.23.

nR * H

(e

=x

m-\-n

+ ln(l+
/i

dx

equal to

/"2).

Mx =

7.13.10.0.15.

f/

7.13.28.^ = ^;

= 0.

7.13.8.

7.13.16. x c

= 0.4a.
yr

= 0.

* 2e*)

m n
if

m-\-n
x m have two
figure

4.5na y

and n are

and n are

of different

if

m-\-n

and y =
The area of the
n

jta 3

0.25:rc/? 3 .

7.13.3.

'

7.13.30.

are odd;

/z

2).

=f\ 7.12.9.^.

mn
7.14.1.

m-\-n

3b)

yc

71

evenness. Hint. The curves y


and (1, 1) in the first quadrant.

quadrant

')^fl.

12

4/?

12

7.13.26.
2

/"2

7.13.9.

0r

mn
even;

j2

7.13.12.

/jf

7.12.4.

-f^.

a3b

J/5-2 V

^51

7.12.13.

V'a 2Jt-b^

/*

7.13.29.

^-

(5^ 5-1); M y = ^V 5 + lln(2+^ 5).

7.12.2.

nrdh'K

7.12.12.

+^

7.U.22.

I2ji.

(^5-0) + (^2+l)

jx

(5

Y~2)\

7.11.19.
/

nabh

(a)

V2

7.11.14.

-;

(a)

(3 + 4

(b)

15

7.11.24.

7.11.10.

1.5jx.

(a)

'

(2 In

7.11.18.

2jx

7.11.20.

449

^3

7.11.17.

7.11.8.

base.

71,n

^~T arctan T-

VII

to Ch.

both

common
situated

points (0, 0)
in the first

Depending on evenness and oddness

m and n this figure is mapped symmetrically either about the coordinate


and n are of different
axes (m, n even) or about the origin (m, n odd). If
evenness, then the curves enclose only the area lying in the first quadrant.
formula for computing the area in
7.14.3. Hint. Take advantage of the
polar coordinates.
7.14.4. Hint. Since the figures are of equal area, the function S(x) appear-

of

ing in the formula for the

volume

V=

(x)

dx

is

the

same and, consequently,

the values of the integrals are also equal.


7.14.5. Hint. The formula follows directly from Simpson's formula
h

/ (x)

dx

= [f(0) + *i(j^+f(h)].

Answers and Hints

450

S(x)

sphere

for

revolution

= n(r x
2

= 2jip#

(#)

for

)\

cone

nr

(x)

x2
;

paraboloid of

for a

and so on.

Hint. Divide the curvilinear trapezoid into strips A# wide and write
an expression for the element of volume AV
2n xy Ax.
7.14.8. Hint. Use the formula for calculating the length of a curve represented parametrically.
7.14.6.

7.14.9.

In

Hint. The

tangent corresponds to

y.
V2-z.

7.14.14.

7.14.13.

the origin with a vertical

nearest to

point

7.14.16.

0.5 In

(a)

(x+y);

JX

(b)

0.5 arc sin x.

Chapter
8.1.2.

(b)

VIII

y ln2;

(c)

(d)

1;

* n

8.1.6. (a)

M^

Hint.

diverges.

(c)

8.1.17.

nX

n X

+*

dx[ } x> dx-\+

^ >

for

Represent

nX

the

integral

dx

}
1

(b) converges;

diverges.

(e)

sum

the

as

Make

4"

the substitution *

" X

that

dx=-

" X
\

dx;

(b)

_2_

(f)

(a)

(a)

diverges;

8.2.14.
(e)

9a

(a)

(b)

converges for p
8.2.7.

(e)

<

it

diverges;

and diverges

converges.

for

(a) 2ji.

mgR. Hint. The law

It

(e)

y;

8.3.8.

3jw 2

(d) converges;

(b) 2 V^ln 2;

diverges;

(c)

of attraction of a

R
= niP
-^y

6^2;

converges;

(c)

diverges;

diverges;

(b)

(d)

1.

8.2.11. (a)

converges;

It

diverges;

(c)

(b) diverges;

converges;

It
(f)

converges. 8.3.7. (a)


8.3.14.

the se-

in

'

summand and show

8.2.2.

two items:

of

-r ^
CO

cond

> Y'el;

converges;

1.

(f)

ji;

Vx

f
J

iy

> _J__

(e)

cc

f
J

Hint.

0.

(a)

CO

In 2;

^*

diverges. Hint.

It

8.3.9.

-1

(c)

y.

converges;

(d)

8.3.10.

body by the Earth

is

^p.

deter-

mined

by

the

formula

where

is

the

mass

distance between the body and the centre of the Earth,


Earth.

of the

8.3.15. e v Hint. Electric charges interact with a force -jf-

body, r

where

ex

are the magnitudes of the charges and r is the distance between them.
8.4.1. Hint. Represent the integral in the form of the sum
+co
a
+ oo

f
J
1

dx
xp \n9

_C

x~ j
1

dx

xp \n^x

J
a

dx

x?

{a

>

is

the

the radius of the

is

l)

and

e2

Answers and Hints


and apply special tests
integral In a:= ln[l

VIII

to Ch.

consideration

convergence, taking into

for

451

that in the

and in the second integral


than any power function.
the logarithmic function increases slower for q <
8.4.2. Hint. Making the substitution xQ=t, reduce the given integral to the

first

a:

as

1,

+l

form

1)]

smtdt. Represent the

integral

dt +
t*

dt,

where a =

t*

sin

as the

dt

sum

and show that the integral conver-

absolutely

ges

^ii_L

for

the

<a<2

integral

and

reduced

is

conditionally
the

to

0<a^l.

for

Note

that

converging

conditionally

at

integral

+X

-f CC

GO

f
J

pi
t

^jl-dt, and

at

to the diverging integral

~^-dt.

1/2

8.4.3.

xp

sum

Hint. Represent the given integral as the

- l (\x) (i- dxr


v

o
l

xP~

-}-

(1

x)*!-

dx and apply the special comparison

test.

1/2

T
8.4.4.

8.4.5.

Hint.

If

By

Hint.

7=

sin ax- sin fix

then

|,

substituting

=x

the

dx

integral

is

bounded.

is

reduced

to

Euler

the

gamma-function.

hM-fMdxJ fJ&dx-[ L&dxJU!!^

Hint.

8.4.6.

In

-4

h
OC

dx.

aa

that the last integral tends to zero as a


0.
8.4.7. Hint. Take the function f(x)
e~ x for the
cosx for the second and take advantage of the
f(x)
8.4.8. It converges for
< 3 and diverges for

equivalence of

cos

a:

and

integral, the function


results of Problem 8.4.6.

first

m ^ 3.

as

Hint. Take advantage

0.

ji

P
8.4.9.

aa

Applying the generalized mean value theorem, show

aa

of the

a&

Hint.

Represent

dx

as

the

sum

C
of

J (sin x)*

two integrals
5

dx

J (sin

x)

-f-

dx
;

reduce the second integral to the

first

one by making the substi-

J (sin*)*
ji

tution x

=
tl

and take advantage

of the

equivalence of sin* and x as x

0.

Answers and Hints

452

mni

8 4 .,o.
.

- cosx)

sinx{[
.

*(i-x)

dx=

dx

CC

C0S

4 ^

^'^s

^ dx

'

The

inte & rand of tne

summand on

first

the right side

is

>0. By the special comparison


an infinitely large quantity of order s
3 as x
test the first integral converges absolutely for s
3 < 1, i.e. s < 4, and diverges
for s^4. The second integral in the right side converges absolutely for s > 1,
the second insince the function sin#(l
cos a:) is bounded. But if
< s
tegral converges conditionally as the difference of two conditionally converging

<

00

cc

*-^dx

integrals

^s\nx-josx

and

dx

Pro5 em

(ee

8>I

j 3 ).

^
TT

JT

Hint.

8.4.11.

example,

Integral (2) can diverge. For

YV(x)

cp

let

2/ux<A:<(2/z4-i)ji,

1,

<

11,

(2/i

H- 1)ji

<

<

(2/z4-2)

jt.

CO

CD

The

-^^- dx converges

integral

(see

Problem

8.1.13).

But

^^-^^(x)dx =
o

CO

00

S *"

diverges (see the

<2a:

same problem). But

if

the integral

/ (x)

dx con-

o
cc

verges absolutely, then the integral

f (x) cp (#)

dx also converges absolutely:

if

a
cp
|

(#)

<

C,

then

/ (a:) cp (a:)

< C

f (x)

and

|,

it

remains

the comparison

to use

theorem.
ji

T" x
8.4.12

//m*. Transform

the

integral

(x)

into f

(x)=

In sin z

by the

dz

JT

2
Jl

substitution
the

above
8.4.13.

z.

Taking

into account that sin z

to the sum ol three integrals.


Hint. Putting w=ln cos x\ cos 2nx dx

= dv,

Z
2 sin

/,,
'

2/2

p
\

sin
sin 2/?a

sin 2/2*

= sin

(2n

cos*

dx,

2) x-cos

2a,

0.

cos

reduce

integrate by parts and get

T
=
1

the equality
J

Since

4- sin 2a -cos (2/z

2)

a:,

Answers and Hints

to

Ch. VIII

453

n_
2

sin
nx2) ^

f
/0
\ sin (2n
.

ln

~2n

a:

cos #

n
2

+ jsin(2n-2>*.sin 2

^ + U^
2

cos

(2/z

2)

a:

J
Check by

direct calculation that

equal zero. Therefore,

and

by

sin

induction,

2/i

cos

a:

sin (2/z
v

4*

2jt

n^2

for

Since /,---

for

2)
7

^2

dx

A'

= ( 1)"- ^
1

=
n

cos x

= ^ we have
4

the second and the third

/2

==

/_,.
n 1

summands

/ 3 =.-.-.
*
<j

'

T~^3^4

Das könnte Ihnen auch gefallen